...κινητή τροχαλία και ταλάντωση Στη διπλανή διάταξη η τροχαλία κέντρου Κ έχει ακτίνα R και μάζα M=4Kg ενώ η μικρή τροχαλία έχει μάζα m=1Kg και ακτίνα r. H m1=3Kg ενώ το ελατήριο έχει σταθερά Κ=400Ν/m. Τραβάμε τη μάζα m προς τα κάτω κατά d=10cm και στη συνέχεια την αφήνουμε ελεύθερη. Να βρείτε πως μεταβάλλεται με το χρόνο η δύναμη που F που ασκείται από τον τοίχο στη μικρή τροχαλία. Για την τροχαλία ισχύει Ιcm=1/2MR2 ακόμη g=10m/s2. Λύση:
Σχόλιο από τον/την Κυριακόπουλος Γιάννης στις 17 Φεβρουάριος 2012 στις 15:09
Διαγραφή σχολίου
Σχόλιο από τον/την Μιχαήλ Μιχαήλ στις 17 Φεβρουάριος 2012 στις 21:06
Διαγραφή σχολίου
Δημήτρη Αναγνώστου ευχαριστώ. Κυριακόπουλε Γιάννη ευχαριστώ για την προσομοίωση. Γιάννη πως συνδέεις το νήμα με την τροχαλία;
Σχόλιο από τον/την Κυριακόπουλος Γιάννης στις 17 Φεβρουάριος 2012 στις 21:40
Διαγραφή σχολίου
Δεν είναι νήμα. Δεν μου δουλεύει με νήμα, είναι ράβδοι. Κάνω διπλό κλικ στο σημείο επαφής και γράφω στα πεδία x και y αντίστοιχα: -body[1].radius*cos(body[1].p.r) body[1].radius*sin(body[1].p.r)΅ Στα άλλα σημεία κάνω κάτι ανάλογο. Αν κάνεις διπλό κλικ θα καταλάβεις.
Σχόλιο από τον/την Μιχαήλ Μιχαήλ στις 17 Φεβρουάριος 2012 στις 21:55
Διαγραφή σχολίου
ευχαριστώ! Γιάννη (Κυρ)
Σχόλιο από τον/την Εμμανουήλ Λαμπράκης στις 19 Φεβρουάριος 2012 στις 18:21
Διαγραφή σχολίου
Μιχαήλ επίσης ωραίο πρόβλημα στις ταλαντώσεις με κύλιση. Τελικά αν πέσει κάποιο παρόμοιο προβλημα μάλλον θα το πιάσεις!
Σχόλιο από τον/την Μιχαήλ Μιχαήλ στις 19 Φεβρουάριος 2012 στις 18:31
Διαγραφή σχολίου
Μανώλη ευχαριστώ και μακάρι μέσα από το ylikonet να βγεί κάποιο θέμα των εξετάσεων!!
Σχόλιο από τον/την ΓΙΑΝΝΗΣ ΔΟΓΡΑΜΑΤΖΑΚΗΣ στις 19 Φεβρουάριος 2012 στις 18:43
Διαγραφή σχολίου
MIXAHΛ ... ΤΟΣΕΣ ΑΝΑΡΤΗΣΕΙΣ ΕΙΝΑΙ ΔΥΣΚΟΛΟ ΝΑ ΤΙΣ ΠΑΡΑΚΟΛΟΥΘΗΣΕΙ ΚΑΙ ΝΑ ΤΙΣ ΜΕΛΕΤΗΣΕΙ ΚΑΠΟΙΟΣ...ΤΩΡΑ ΕΙΔΑ ΤΗΝ ΠΡΟΤΑΣΗ ΣΟΥ. ΕΙΝΑΙ ΠΟΛΥ ΚΑΛΗ. ΕΙΔΙΚΑ ΣΤΟ ΧΕΙΡΙΣΜΟ ΤΩΝ ΔΥΝΑΜΕΩΝ (ΣΧΕΔΙΑΣΜΟΣ-ΥΠΟΛΟΓΙΣΜΟΙ)...ΝΑ'ΣΑΙ ΚΑΛΑ.
Σχόλιο από τον/την Μιχαήλ Μιχαήλ στις 19 Φεβρουάριος 2012 στις 18:52
Διαγραφή σχολίου
Γιάννη (Δογρ) ευχαριστώ!
...κινητή τροχαλία και έργο τάσης νήματος Στη διπλανή διάταξη η τροχαλία κέντρου Κ έχει ακτίνα R και μάζα M=4Kg ενώ η μικρή τροχαλία έχει μάζα m=1Kg και ακτίνα r. Αφήνουμε τη m1=3Kg ελεύθερη να κινηθεί. Να βρείτε: α) Την επιτάχυνση με την οποία θα κινηθεί η μάζα m1 και β) τη συνολική δύναμη ΣF που ασκείται από την οριζόντια οροφή στο σύστημα των τροχαλιών, γ) Να υπολογιστούν τα έργα των τάσεων T1, T2 T3 που ασκούνται στις τροχαλίες αν η μάζα m1 μετατοπιστεί προς τα κάτω κατά h=1m. Για την τροχαλία ισχύει Ιcm=1/2MR2 ακόμη g=10m/s2.     Λύση: 7. ΤΡΟΧΑΛΙΑ ΚΑΙ … ΜΑΖΕΣ 7. ΤΡΟΧΑΛΙΑ ΚΑΙ … ΜΑΖΕΣ Η τροχαλία του σχήματος έχει μάζα Μ=6Kg και ακτίνα R=20cm και επιταχύνεται κατακόρυφα προς τα πάνω με δύναμη F=420N. Η τροχαλία μπορεί να στέφεται χωρίς τριβές γύρω από οριζόντιο άξονα που περνά από το κέντρο της. Γύρω από την τροχαλία, είναι τυλιγμένο αβαρές σχοινί, στα άκρα του οποίου είναι δεμένα σώματα με μάζες, m1=4Kg, m2=2Kg και m3=1Kg. Τότε: α) Να υπολογιστεί η επιτάχυνση α της τροχαλίας. β) Να υπολογιστούν οι τάσεις των νημάτων. γ) Για ανύψωση της τροχαλίας κατά h=15cm να υπολογιστεί η κινητική ενέργεια του συστήματος. δ) Να βρεθεί η σχέση που δίνει τη στροφορμή του συστήματος σε συνάρτηση με το χρόνο και ε) Να υπολογιστεί ο ρυθμός μεταβολής της στροφορμής του συστήματος. Δίνεται για την τροχαλία Ιcm=1/2MR2 και g=10m/s2. Λύση:
Σχόλιο από τον/την ΓΙΑΝΝΗΣ ΔΟΓΡΑΜΑΤΖΑΚΗΣ στις 22 Φεβρουάριος 2012 στις 8:07
Διαγραφή σχολίου
ΜΙΧΑΗΛ ΩΡΑΙΑ ΠΡΟΤΑΣΗ .
Σχόλιο από τον/την Τίνα Νάντσου στις 22 Φεβρουάριος 2012 στις 8:24
Διαγραφή σχολίου
Ωραίο θέμα που δίνει ιδέες και για Α' Λυκείου
Σχόλιο από τον/την Μιχαήλ Μιχαήλ στις 22 Φεβρουάριος 2012 στις 21:11
Διαγραφή σχολίου
Τίνα, Γιάννη (Δογρ), ευχαριστώ για τα σχόλιά σας!
Σχόλιο από τον/την Socrates στις 13 Απρίλιος 2016 στις 19:18
Διαγραφή σχολίου
Γεια σας! Στο ε), αν υπολογίσουμε το ρυθμό μεταβολής της στροφορμής του συστήματος ως το άθροισμα των ροπών των εξωτερικών δυνάμεων βρίσκουμε διαφορετικό αποτέλεσμα. Πού οφείλεται αυτό; Αυτό συνδέεται και με το δ), τον αναλυτικό υπολογισμό στροφορμής...
...Θ.Μ.Κ.Ε σε κινούμενο γιο-γιο Ο λεπτός σωλήνας του σχήματος έχει μάζα m=1Kg και ακτίνα R=0,1m. Τη χρονική στιγμή t=0, αφήνουμε το σωλήνα ελεύθερο να κινηθεί κατακόρυφα, ενώ το σημείο Γ παραμένει Λύση: σχολια
Σχόλιο από τον/την Μιχαήλ Μιχαήλ στις 28 Φεβρουάριος 2012 στις 15:43
Διαγραφή σχολίου
Δημήτρη (Αναγνώστου), ευχαριστώ και συγχαρητήρια και για τη δική σου προσφορά.
Σχόλιο από τον/την Εμμανουήλ Λαμπράκης στις 1 Μάρτιος 2012 στις 11:59
Διαγραφή σχολίου
Μιχαήλ ωραίο πρόβλημα που μου θύμισε κάπως ένα 4ο θέμα κάποιων πανελλήνιων.
Κύλινδρος και …ράβδος σε κεκλιμένο επίπεδο Ο κύλινδρος του σχήματος έχει μάζα Μ=3Kg και ακτίνα R=10cm. Ο κύλινδρος βρίσκεται πάνω σε κεκλιμένο επίπεδο γωνίας κλίσης φ=300 και ισορροπεί οριακά, ώστε ίσα – ίσα να μην υπερπηδά το εμπόδιο ύψους h=5cm...... Λύση: σχολια
Σχόλιο από τον/την ΜΑΝΩΛΗΣ ΔΡΑΚΑΚΗΣ στις 3 Μάρτιος 2012 στις 22:24
Διαγραφή σχολίου
Πολυ καλό ως 4ο θέμα. Συνάδελφε Μιχαήλ, με την ευκαιρία, θερμά συγχαρητήρια για τις ενδιαφέρουσες και καλά δουλεμένες αναρτήσεις σου.
Σχόλιο από τον/την Μιχαήλ Μιχαήλ στις 3 Μάρτιος 2012 στις 23:45
Διαγραφή σχολίου
Μανώλη (Δρ) σε ευχαριστώ.
Σχόλιο από τον/την Εμμανουήλ Λαμπράκης στις 4 Μάρτιος 2012 στις 14:59
Διαγραφή σχολίου
Μιχαήλ θα συμφωνήσω με το συνονόματο αγαπητό Μανώλη. Ένας μαθητής δουλεύοντας πάνω στο πρόβλημα αυτό έχει να ωφεληθεί πάρα πολύ κατά τη γνώμη μου.
Σχόλιο από τον/την Μιχαήλ Μιχαήλ στις 4 Μάρτιος 2012 στις 16:58
Διαγραφή σχολίου
Μανώλη (Λαμπράκη) ευχαριστώ. Προσπαθώντας να ταιριάξω τις ασκήσεις 4.56 και 4.57 του σχολ βιβλίου μου προέκυψε το θέμα που ανέβασα στο δίκτυο.Με βοήθησε είναι αλήθεια και η παρατήρηση του Σταύρου Πρωτογεράκη στην πρώτη μου ανάρτηση.
Τροχός και… κύλινδροι Οτροχός του σχήματος μάζας Μ=4Kg και ακτίνας R=0,2m μπορεί να περιστρέφεται ελεύθερα γύρω από οριζόντιο άξονα που περνάει από το κέντρο μάζας του Κ. Ο άξονας στηρίζεται σε ράβδο ΚΛ και αυτή στην οριζόντια ράβδο ΑΒ. Οι ράβδοι έχουν αμελητέες μάζες. Στα άκρα της ράβδου ΑΒ συνδέονται με κατάλληλο τρόπο τα κέντρα μάζας δυο όμοιων κυλίνδρων μάζας m=1/3Kg ο καθένας και ακτίνας r=5cm. Οι ράβδοι δεν εμποδίζουν την περιστροφή και δεν ασκούν τριβές. Γύρω από τον τροχό μάζας Μ, είναι τυλιγμένο αβαρές σχοινί, το ελεύθερο άκρο του οποίου το τραβάμε με σταθερή οριζόντια δύναμη F=20N. Να υπολογιστούν: α) Η μεταφορική επιτάχυνση του συστήματος. β) Οι γωνιακές επιταχύνσεις του τροχού και των δυο κυλίνδρων. γ) Η δύναμη που ασκείται από τον άξονα περιστροφής στον τροχό. δ) Το έργο της δύναμης F, και η κινητική ενέργεια του τροχού καθώς και των δυο κυλίνδρων, όταν θα ξετυλιχθεί σχοινί μήκους L=1,8m. ε) Η συνολική στροφορμή του συστήματος εκείνη τη στιγμή. Δίνεται η ροπή αδράνειας του τροχού ως προς τον άξονα περιστροφής ΙK=1/2ΜR2 και η ροπή αδράνειας του κάθε κυλίνδρου ως προς τον άξονα περιστροφής Ι=1/2mr2. Ακόμη δίνεται g=10m/s2.   Λύση: σχόλια ....Ράβδοι και κρούση Οι τρεις ράβδοι m1,m2 και m3 του σχήμα τος ίδιου μήκους L=0,6m, συγκολλούνται στο ένα άκρο τους O και σχηματίζουν ανά δύο γωνία 1200. Το σύστημα των τριών ράβδων μπορεί να περιστρέφεται ελεύθερα γύρω από οριζόντιο πείρο, κάθετο στο επίπεδό τους, που περνάει από το κοινό τους άκρο O. Κάποια στιγμή το σύστημα αφήνεται ελεύθερο να περιστραφεί από τη θέση που φαίνεται στο σχήμα. Τότε: α) Να υπολογιστεί η γωνιακή επιτάχυνση τ ου συστήματος και η αντίδραση από τον πείρο τη στιγμή της εκκίνησης, β) Πόση είναι η γωνιακή ταχύτητα του συστήματος μόλις η m3 γίνει κατακόρυφη; γ) Σε ποια θέση το σύστημα των τριών ράβδων έ χει γωνιακή επιτάχυνση ίση με μηδέν; δ) Για ποια γωνία περιστροφής σταματάει στιγμιαία η περιστροφή; ε) Αν όλες οι ράβδοι είχαν την ίδια μάζα τότε είναι σωστό να πούμε ότι το σύστημα ισορροπεί σε οποιαδή ποτε θέση; στ) Αν τη στιγμή που η m3 γίνει κατακόρυφη, το ελεύθερο άκρο της ράβδου με μάζα m2 συγκρούεται πλαστικά με σημειακή μάζα m΄, που κινείται κατακόρυφα προς τα κάτω με υ= m/s, τότε ποια είναι η γωνιακή ταχύτητα του συστήματος αμέσως μετά την κρούση; Δίνονται m1=2m, m2=m3=m, όπου m=1Kg και g=10m/s2. Ακόμη η ροπή αδράνειας ράβδου μάζας m και μήκους L γύρω από άξονα που περνάει από το ένα άκρο της είναι Ι=1/3 ML2. Λύση: Σχόλια
Σχόλιο από τον/την ΓΙΑΝΝΗΣ ΔΟΓΡΑΜΑΤΖΑΚΗΣ στις 21 Μάρτιος 2012 στις 15:02
Διαγραφή σχολίου
MIXAHΛ ...ΠΟΛΥ ΚΑΛΗ Η ΠΡΟΤΑΣΗ ΣΟΥ.
Σχόλιο από τον/την Φραγκιαδουλάκης Εμμανουήλ στις 21 Μάρτιος 2012 στις 16:50
Διαγραφή σχολίου
Βρίσκω το πρόβλημα πολύ καλό, αλλά και πολύ ενδιαφέρον για προβληματισμό, αν θα μπορούσε να είναι τέταρτο θέμα στις εξετάσεις. Με πρώτη ματιά και βλέποντας κανείς τη μαθηματική επεξεργασία που κάνεις, το απορρίπτει γιατί φαίνεται υπερβολικά φορτωμένο.Θα πρότεινε λοιπόν, να κρατήσει το πρώτο ερώτημα που είναι το καλύτερο, άντε και το δεύτερο και τέταρτο που είναι κάπως πιό κλασσικά.Ναι αλλά τώρα μου φαίνεται αρκετά ελαφρύ.Να βάλλω........ Με δεύτερη ματιά όμως μου λέει ότι χωρούν και τα έξι ερωτήματα, αφού τα (γ),(δ),(ε) έχουν πολύ απλές αποδείξεις. Και εξηγούμαι απαντώντας: γ) Γωνιακή επιτάχυνση μηδέν έχω εκεί που συνισταμένη ροπή ως προς τον άξονα περιστροφής είναι μηδέν και αυτό συμβαίνει προφανώς όταν η ράβδος m1 είναι κατακόρυφη. δ)Αφού στη θέση αυτή η γωνιακή ταχύτητα είναι μέγιστη, το σύστημα θα επιβραδύνεται στη συνέχεια μέχρις ότου όλη η κινητική του ενέργεια να μετατραπεί σε δυναμική .Άρα η ράβδος m1 θα φθάσει μέχρι την οριζοντίωσή της δηλ. η γωνία που διέγραψε είναι 180 μοίρες. ε)Αν οι μάζες είναι ίδιες τότε τα κέντρα μαζών τους θα βρίσκονται στις κορυφές ισοπλεύρου τριγώνου και επομένως η συνισταμένη τους θα ευρίσκεται στο σημείο τομής των διαμέσων (διχοτόμων και μεσοκαθέτων) του τριγώνου, που είναι το σημείο περιστροφής.Άρα η συσταμένη δεν δημιουργεί ροπή και το σύστημα ισορροπεί σε κάθε θέση.
Σχόλιο από τον/την Μιχαήλ Μιχαήλ στις 21 Μάρτιος 2012 στις 21:26
Διαγραφή σχολίου
Γιάννη Δογραματζάκη ευχαριστώ για το σχόλιό σου. Μανώλη Φραγκιαδουλάκη συμφωνώ με τις παρατηρήσεις σου. Σε ευχαριστώ.
Κύλινδροι και...μια ράβδος Το σύστημα του σχήματος αποτελείται από δυο κυλίνδρους με μάζες m1=1Kg , R1=10cm και m2=2Kg, R2=20cm. Τα κέντρα μάζας των κυλίνδρων συνδέονται με ράβδο μάζας m=1Kg η οποία δεν εμποδίζει την περιστροφή. Στο σύστημα εξασκούμε σταθερή οριζόντια δύναμη F=11N, όπως φαίνεται στο σχήμα και τότε αυτό κυλίεται σε οριζόντιο επίπεδο χωρίς να ολισθαίνει.             α) Να υπολογίσετε την μεταφορική επιτάχυνση του συστήματος β) Να υπολογιστεί ο λόγος των στροφορμών των δυο κυλίνδρων κάθε χρονική στιγμή. γ) Ποια είναι η κινητική ενέργεια του κάθε στερεού τη στιγμή που ο κύλινδρος m1 έχει κάνει Ν1=στροφές; δ) Να υπολογιστούν οι δυνάμεις που ασκεί η ράβδος σε κάθε σώμα; Δίνεται g=10m/s2 και η ροπή αδράνειας του κάθε κυλίνδρου γύρω από άξονα που περνάει από το Κ.Μ του Ι=1/2mR2. Συνοπτική λύση Σχόλια
Σχόλιο από τον/την Νίκος Ανδρεάδης στις 29 Μάρτιος 2012 στις 20:09
Διαγραφή σχολίου
Μιχαήλ μετά το θέμα του 2010 η "ράβδος" θεωρείται και επίσημα εντός ύλης. Το θέμα όμως γίνεται περίπλοκο αν η ράβδος θεωρηθεί ότι έχει μάζα. Στη τελευταία σελίδα, στη σχέση Στκ=0 ξέχασες τις οριζόντιες συνιστώσες. Προτείνω να θεωρήσεις τη ράβδο αβαρή για να λύνεται η άσκηση πιο εύκολα.
Σχόλιο από τον/την Μιχαήλ Μιχαήλ στις 29 Μάρτιος 2012 στις 20:51
Διαγραφή σχολίου
Νίκο Ανδρεάδη ευχαριστώ. Θα το ξαναδώ!
ΚΥΛΙΝΔΡΟΙ ΚΑΙ …ΜΙΑ ΣΑΝΙΔΑ   ΚΥΛΙΝΔΡΟΙ ΚΑΙ …ΜΙΑ ΣΑΝΙΔΑ Στο σχήμα που φαίνεται δίπλα ο κύλινδρος μάζας m μπορεί να κυλίεται πάνω στη σανίδα μάζας Μ=1,75 Kg. Ακόμη οι δυο κύλινδροι με μάζες m1 και m2 μπορούν να κυλίονται ελεύθερα.   Κάποια στιγμή ασκούμε στον κύλινδρο μάζας m, οριζόντια σταθερή δύναμη F=17N, όπως φαίνεται στο σχήμα. Αν δεν παρατηρείται σε καμία περίπτωση ολίσθηση τότε: α) Να υπολογιστεί η επιτάχυνση του Κ.Μ του κυλίνδρου μάζας m, καθώς και των κυλίνδρων m1 και m2, β) Πόση είναι η γωνιακή επιτάχυνση του κάθε κυλίνδρου; γ) Να υπολογιστεί το έργο της δύναμης F για μετατόπιση του Κ.Μ του κυλίνδρου μάζας m, κατά xcm=24cm, δ) Πόση είναι τότε η κινητική ενέργεια του κάθε σώματος; Τι παρατηρείτε; Δίνονται m=m1=m2=1Kg, R=R1=R2=2cm, ενώ η ροπή αδράνειας κυλίνδρου μάζας m και ακτίνας R ως προς άξονα που διέρχεται από το Κ.Μ του δίνεται από τη σχέση του Ι=1/2mR2. Συνοπτική λύση Σχόλια
Σχόλιο από τον/την Μιχαήλ Μιχαήλ στις 28 Μάρτιος 2012 στις 21:07
Διαγραφή σχολίου
Δημήτρη σε ευχαριστώ. Επιμένω ότι επίσης κάνεις πολύ και καλή δουλειά. Μπράβο σου.
Σχόλιο από τον/την Διονύσης Μάργαρης στις 28 Μάρτιος 2012 στις 21:18
Διαγραφή σχολίου
Γεια σου Μιχαήλ. Πολύ ωραίες οι τελευταίες σου ασκήσεις. Λίγο σύνθετες μεν, με αρκετή δουλειά, αλλά ουσιαστικές. Σε ευχαριστούμε που τις μοιράζεσαι μαζί μας.
Σχόλιο από τον/την Μιχαήλ Μιχαήλ στις 28 Μάρτιος 2012 στις 22:55
Διαγραφή σχολίου
Διονύση σε ευχαριστώ. Η προπάθειά σου, μου είναι πράγματι πολύτιμη.
Σχόλιο από τον/την Φραγκιαδουλάκης Εμμανουήλ στις 29 Μάρτιος 2012 στις 15:35
Διαγραφή σχολίου
Συγχαρητήρια Μιχαήλ και για τις δυο ασκήσεις που ανάρτησες. Η άσκηση με τη σανίδα είναι αρκετά περίπλοκη. Ελπίζω μέσα σ'αυτή την πολυπλοκότητα να μη χαθεί η μελέτη του σχήματος και η σχεδίαση.Γιαυτό το λόγο και για να είναι περισσότερη προσιτή στους μαθητές θα την ήθελα λιγότερο φορτωμένη.Η δεύτερη ανάρτηση είναι περισσότερο ...μαθητική..
52. 52. Πάνω σε κεκλιμένο επίπεδο γωνίας κλίσης φ=300 βρίσκεται ένας κύλινδρος μάζας M=2Kg ακτίνας R=0,4m. Σε απόσταση r=R/2 από το κέντρο του κυλίνδρου και πάνω σε αυτόν βρίσκεται τυλιγμένο κατάλληλα ένα αβαρές σχοινί που μπορεί να ξετυλίγεται χωρίς να γλιστρά. Το σχοινί περνάει από το αυλάκι μιας σταθερής τροχαλίας μάζας m1=2Kg και ακτίνας r1=0,1m, στο ελεύθερο άκρο του οποίου είναι δεμένο σώμα μάζας m=1Kg. Αν αφήσουμε το σύστημα ελεύθερο να κινηθεί και αν ο κύλινδρος κυλίεται χωρίς να ολισθαίνει τότε να υπολογιστούν: Α) α) η επιτάχυνση της μάζας m β) η γωνιακή επιτάχυνση του κυλίνδρου και της τροχαλίας γ) Η τάση στα άκρα του σχοινιού δ) Η σταθερή στατική τριβή που δέχεται ο κύλινδρος από το κεκλιμένο επίπεδο   Β) Να υπολογιστούν: α) ο ρυθμός αύξησης της στροφορμής της τροχαλίας και του κυλίνδρου β) η ταχύτητα του σώματος μάζας m, τη στιγμή που έχει ξετυλιχθεί νήμα μήκους L=0,32m . Γ) Αν τη στιγμή εκείνη κόβεται το νήμα και η μάζα m, συγκρούεται πλαστικά με τη μάζα m2=1Kg που πραγματοποιεί α.α.τ με εξίσωση x=ημ(5t) (S.I) και εκείνη τη στιγμή βρίσκεται στη θέση ισορροπίας της, κινούμενη προς τη θετική κατεύθυνση, τότε να βρείτε την εξίσωση της α.α.τ του συσσωματώματος μετά την πλαστική κρούση. Δίνεται η ροπή αδράνειας ως προς το Κ.Μ του κυλίνδρου ΙΚ=1/2ΜR2 και της τροχαλίας Ιτ=1/2m1r12. Λύση: Σχόλια
Σχόλιο από τον/την Κυριακόπουλος Γιάννης στις 1 Απρίλιος 2012 στις 20:40
Διαγραφή σχολίου
Έχει πλάκα να συναντάς κάποιον ζωντανά στα Καμμένα Βούρλα και την άλλη μέρα διαδικτυακά. Πολύ καλή. Και ένα 4ο θέμα. Στο τέλος η Κ.Ε.Ε αν διαβάζει θα ταλαιπωρηθεί να βρει θέμα.
Σχόλιο από τον/την ΑΠΟΣΤΟΛΟΣ ΚΟΡ στις 1 Απρίλιος 2012 στις 20:44
Διαγραφή σχολίου
aderfe arketa kalh skepsh gia tetarto profanws 8a thn exw sto nou mou
Σχόλιο από τον/την Μιχαήλ Μιχαήλ στις 1 Απρίλιος 2012 στις 20:54
Διαγραφή σχολίου
Γιάννη ευχαριστώ! Πώς τα πέρασες δεν σε ξαναείδα.
Σχόλιο από τον/την Μιχαήλ Μιχαήλ στις 1 Απρίλιος 2012 στις 20:55
Διαγραφή σχολίου
Απόστολε (Κορ) ευχαριστώ
Σχόλιο από τον/την Κυριακόπουλος Γιάννης στις 1 Απρίλιος 2012 στις 21:16
Διαγραφή σχολίου
Η πεντάδα Μαργαρίτη , Φασουλόπουλου, Τζαμαλή . Κορκίζογλου και εμού αναχωρήσαμε το βράδυ.
Σχόλιο από τον/την ΜΑΝΩΛΗΣ ΔΡΑΚΑΚΗΣ στις 1 Απρίλιος 2012 στις 22:02
Διαγραφή σχολίου
Πολύ καλή κατασκευή. Γιατί στον τίτλο 52;
Σχόλιο από τον/την Μιχαήλ Μιχαήλ στις 1 Απρίλιος 2012 στις 22:29
Διαγραφή σχολίου
Μανώλη Δρακάκη είναι απλώς μια αρίθμηση που έχω στις ασκήσεις
Δυο _ράβδοι Το στερεό του σχήματος αποτελείται από δυο ράβδους. Την (ΟΒ)=L1=0,2m με μάζα m1=3Kg και την (ΒΓ)=L2=0,6m με μάζα m2=1Kg, που συγκολλούνται στο κοινό τους άκρο Β. Κάθετα στις δυο ράβδους ασκούνται οι δυνάμεις F1=16N και F2=2N με τη φορά που φαίνεται στο σχήμα. H F1 εφαρμόζεται στο άκρο Γ της ΒΓ και η F2 στο μέσο Α της ΟΒ. Λόγω τριβών με τον άξονα περιστροφής το σύστημα ισορροπεί σχηματίζοντας γωνία φ=600 με την κατακόρυφο. α) Ποια είναι η συνολική ροπή που ασκείται στη ράβδο από την τριβή με τον άξονα περιστροφής; β) Ποια είναι η συνολική δύναμη που ασκείται από τον άξονα περιστροφής στη ράβδο όταν αυτή ισορροπεί; γ) Ρίχνουμε λίγο λαδάκι και μηδενίζουμε τις τριβές με τον άξονα περιστροφής οπότε το σύστημα ξεκινά να περιστρέφεται από την ηρεμία. Τότε να υπολογιστεί το έργο όλων των δυνάμεων και η γωνιακή ταχύτητα του συστήματος μόλις αυτό γίνει κατακόρυφο για πρώτη φορά. δ) Αν εκείνη τη στιγμή που γίνεται κατακόρυφο, συγκρούεται ελαστικά στο κατώτερο άκρο του, με σημειακή μάζα m=1Kg που κινείται με σταθερή ταχύτητα υ=10m/s σχηματίζοντας με την κατακόρυφο, γωνία 300, και αν θεωρήσουμε τις επιφάνειες λείες, τότε: i) Να υπολογιστούν η γωνιακή ταχύτητα της ράβδου καθώς και η ταχύτητα της μάζας m αμέσως μετά την κρούση. ii) Πόση είναι η μέγιστη ενέργεια παροδικής παραμόρφωσης; ε) Πόση είναι η αντίδραση από τον άξονα στήριξης αμέσως μετά την κρούση; Δίνεται 3^1/2=1,7 και η ροπή αδράνειας ράβδου ως προς το Κ.Μ της είναι Ι=1/12mL2. Λύση: Σχόλια
Σχόλιο από τον/την Διονύσης Μητρόπουλος στις 3 Απρίλιος 2012 στις 4:56
Διαγραφή σχολίου
Αγαπητέ Μιχάλη καλησπέρα. Συγχαρητήρια για την ωραία ιδέα της άσκησης. Νομίζω όμως ότι υπάρχει ένα θέμα σχετικό με την ελαστική κρούση των δύο σωμάτων. Η σημειακή μάζα m έχει πριν την κρούση ταχύτητα υ, με συνιστώσες: υx = υημφ = 5m/s υy = υσυνφ ≈ 8,5m/s Κατά τη διάρκεια της κρούσης οι μέσες δυνάμεις F και που ασκούνται μεταξύ των δύο σωμάτων είναι γενικά όπως στο σχήμα (και αν υποθέσουμε ότι δεν υπάρχει τριβή, τότε είναι οριζόντιες). Τη στιγμή τώρα που υπολογίζεις τη μέγιστη ελαστική ενέργεια παραμόρφωσης (σχήμα), η σχετική ταχύτητα των δύο σωμάτων στον οριζόντιο άξονα είναι μηδέν. Επομένως, οι συνιστώσες ταχύτητας της μάζας m εκείνη τη στιγμή είναι: υκx = ωκ(L1+L2) = 2m/s υκy ≤ υy → υκy ≤ 8,5m/s (η ισότητα στην περίπτωση που δεν υπάρχει τριβή). Στον υπολογισμό της ενέργειας παραμόρφωσης θεωρείς μάλιστα ότι υκy = 0 κάτι που θα χρειαζόταν ιδιαίτερα μεγάλο συντελεστή στατικής τριβής για να συμβεί. Mετά την ολοκλήρωση της κρούσης τώρα θεωρείς ότι η μάζα m έχει κατακόρυφη ταχύτητα προς τα κάτω, δηλαδή έχει συνιστώσες: υx΄ = 0 και υy΄= υ΄≈ 9,5m/s Αλλά (αγνοώντας το βάρος) η δύναμη που ασκείται ακόμα πάνω της λόγω της παραμόρφωσης, έχει συνιστώσες με φορά προς τα αριστερά και προς τα πάνω (ή μόνο προς τα αριστερά αν δεν υπάρχει τριβή). Επομένως η τιμή υx΄ = 0 είναι ίσως δυνατή (αφού περιμένουμε να είναι υx΄< 2m/s) Δεν θα έπρεπε όμως η συνιστώσα υy΄ να ικανοποιεί τον περιορισμό υy΄ ≤ υκy ; Ποιά δύναμη ασκήθηκε προς τα κάτω ώστε να γίνει η κατακόρυφη συνιστώσα μεγαλύτερη ακόμα και από την αρχική τιμή των 8,5m/s ; Αν μάλιστα δεχτούμε ότι υκy = 0 τότε η υy΄ θα έπρεπε, εξαιτίας της στατικής τριβής να αποκτήσει τώρα φορά προς τα πάνω, κάτι που είναι πρακτικά αδύνατο σε συνήθεις επιφάνειες.
Σχόλιο από τον/την Μιχαήλ Μιχαήλ στις 3 Απρίλιος 2012 στις 14:20
Διαγραφή σχολίου
Διονύση καλημέρα. Σε ευχαριστώ για την παρατήρηση και θα το ξανακοιτάξω. Αν και νομίζω ότι δεν πρέπει να αγνοήσουμε το βάρος.Πράγματι η ενέργεια παραμόρφωσης δύσκολα να υπολογιστεί.
Σχόλιο από τον/την Διονύσης Μητρόπουλος στις 3 Απρίλιος 2012 στις 14:41
Διαγραφή σχολίου
Καλημέρα κι από μένα Μιχάλη. Μια λύση που ίσως διευκόλυνε, θα ήταν να θεωρήσεις τις επιφάνειες λείες, οπότε η κατακόρυφη συνιστώσα υσυνφ παραμένει αμετάβλητη σε όλη τη διάρκεια της κρούσης. (Λόγω της μικρής διάρκειας της κρούσης νομίζω ότι η συμβολή του βάρους είναι ασήμαντη). Ετσι, στην ενδιάμεση κατάσταση βρίσκεις τη υκx από την κοινή ωκ (που την έχεις ήδη υπολογίσει) και στην τελική κατάσταση θα έχεις ως άγνωστη μόνο την οριζόντια συνιστώσα υx΄.
Σχόλιο από τον/την Μιχαήλ Μιχαήλ στις 3 Απρίλιος 2012 στις 19:29
Διαγραφή σχολίου
Διονύση την ξαναέφτιαξα όπως μου πρότεινες. Ευχαριστώ
Σχόλιο από τον/την ΓΙΑΝΝΗΣ ΔΟΓΡΑΜΑΤΖΑΚΗΣ στις 3 Απρίλιος 2012 στις 20:58
Διαγραφή σχολίου
MIXΑΗΛ ΠΟΛΥ ΩΡΑΙΑ ΠΡΟΤΑΣΗ...
Σχόλιο από τον/την Μιχαήλ Μιχαήλ στις 3 Απρίλιος 2012 στις 23:17
Διαγραφή σχολίου
Γιάννη ευχαριστώ
Σχόλιο από τον/την Διονύσης Μητρόπουλος στις 4 Απρίλιος 2012 στις 0:45
Διαγραφή σχολίου
Πολύ ωραία Μιχάλη, νάσαι καλά :-)
ΚΥΛΙΣΗ ΣΕ ΚΙΝΟΥΜΕΝΟ ΠΑΤΩΜΑ… Το σώμα Σ2 του σχήματος έχει μάζα Μ=1Kg και βρίσκεται πάνω σε λείο οριζόντιο δάπεδο. Πάνω στο Σ2 βρίσκεται ομογενής κύλινδρος Σ3 που έχει μάζα m3=4Kg. To K.M του Σ3 δένεται μέσω αβαρούς σχοινιού το οποίο περνά από τροχαλία μάζας m2=Kg και έχει δεμένο στο άλλο του άκρο σώμα μάζας m1=2Kg. Όταν αφήσουμε το σώμα μάζας m1, ελεύθερο τότε το σώμα Σ3 κυλίεται χωρίς να ολισθαίνει πάνω στο Σ2. α) Να βρείτε την επιτάχυνση του άξονα περιστροφής του κυλίνδρου β) Να βρείτε το συνολικό έργο των εξωτερικών δυνάμεων για μετατόπιση του Κ.Μ του κυλίνδρου κατά x=1,2m, γ) Πόση είναι τότε η κινητική ενέργεια του κάθε σώματος; δ) Πόσο μετακινήθηκε τότε το Σ3 πάνω στο Σ2; ε) Πόσος είναι ο ρυθμός μεταβολής της κινητικής ενέργειας του συστήματος; Δίνεται g=10m/s2 και ακόμη Ι=1/2mR2 .   Συνοπτική λύση: Σχόλια
Σχόλιο από τον/την Εμμανουήλ Λαμπράκης στις 7 Απρίλιος 2012 στις 19:42
Διαγραφή σχολίου
Μιχαήλ μπράβο το "έστησες" πάρα πολύ όμορφα. Πολύ ενδιαφέρον θέμα.
Σχόλιο από τον/την Μιχαήλ Μιχαήλ στις 7 Απρίλιος 2012 στις 20:20
Διαγραφή σχολίου
Εμμανουήλ Λαμπράκη σε ευχαριστώ και για τα δυο σου σχόλια. Η κύλιση και ολίσθηση που μας πρότεινες μου "μυρίζει" θέμα. ΚΑΛΟ ΠΑΣΧΑ
Σχόλιο από τον/την Μιχαήλ Μιχαήλ στις 7 Απρίλιος 2012 στις 23:56
Διαγραφή σχολίου
Δημήτρη σε ευχαριστώ. Πράγματι δεν μπορώ να σε προλάβω. Συγχαρητήρια για τις όμορφες και πρωτότυπες αναρτήσεις σου.
Σχόλιο από τον/την Φραγκιαδουλάκης Εμμανουήλ στις 8 Απρίλιος 2012 στις 17:29
Διαγραφή σχολίου
Πάντα με φαντασία και καλλιγραφία Μιχαήλ, μας εκπλήττεις ευχάριστα.Μου φαίνεται και μένα πως δίνεις ιδέες στους θεματοδότες των εξετάσεων.Νάσαι καλά
Σχόλιο από τον/την Διονύσης Μάργαρης στις 8 Απρίλιος 2012 στις 18:39
Διαγραφή σχολίου
Μιχαήλ και από μένα συγχαρητήρια. Όμορφο θέμα.
Σχόλιο από τον/την Μιχαήλ Μιχαήλ στις 8 Απρίλιος 2012 στις 19:02
Διαγραφή σχολίου
Φραγκιουδάκη Μανώλη σε ευχαριστώ πολύ. Διονύση Μάργαρη η έμπνευση για αυτό το θέμα ήταν η ανάρτησή σου"Μια περισσότερο ιδιόμορφη «κρούση».".Την άσκηση στην αφιερώνω και σε ευχαριστώ πολύ.

Σχόλιο από τον/την Βαγγέλης Κουντούρης στις 8 Απρίλιος 2012 στις 20:12
Διαγραφή σχολίου
Πολύ καλή, πρωτότυπη και “πλούσια” άσκηση, Μιχάλη γι αυτό και καταθέτω μερικές παρατηρήσεις για την εκφώνηση: “Το σώμα Σ2 του σχήματος έχει μάζα Μ…” Το πρωτοεμφανιζόμενο σώμα είναι καλύτερα να χαρακτηρίζεται ως Σ1 και όχι ως Σ2, διότι ο αναγνώστης ψάχνει να βρει ποιο είναι το Σ1, το οποίο, μάλιστα, εδώ δεν υπάρχει και καθόλου (!) και ο δείκτης της μάζας του να συμφωνεί ει δυνατόν (εδώ καλύτερα “Το σώμα Σ1 του σχήματος έχει μάζα m1…”) και αντί “σώμα” να δίνεται πληροφορία για το σχήμα (πολύ σωστά είναι γραμμένο: “κύλινδρος Σ3 που έχει μάζα m3”) Δεν αναφέρεται ότι το νήμα είναι αβαρές μη εκτατό και (μεταξύ κυλίνδρου και τροχαλίας) οριζόντιο. Δεν διευκρινίζεται πώς γίνεται το νήμα να είναι δεμένο στο Κ.Μ. (που πρέπει να γραφεί κέντρο μάζας του κυλίνδρου), χωρίς να τον εμποδίζει να περιστρέφεται (μια λύση θα ήταν να υπάρχουν δύο δίσκοι ενωμένοι με άξονα) Το ξεκίνημα “Όταν αφήσουμε το σώμα μάζας m1 ελεύθερο …” δεν εξασφαλίζει ότι τα νήματα είναι αρχικά τεντωμένα, γι αυτό θα ήταν καλύτερα “Όταν αφήσουμε το σώμα Σ3 ελεύθερο …” (και μια στη λύση: γράψε Τστ=Μα΄ αντί Ταστ=Μα΄, αριστερά από τη σχέση (7))
Σχόλιο από τον/την Μιχαήλ Μιχαήλ στις 8 Απρίλιος 2012 στις 20:45
Διαγραφή σχολίου
Βαγγέλη Κουντούρη ευχαριστώ για τις παρατηρήσεις σου. Κάποιες διορθώσεις έγιναν!
ΡΑΒΔΟΣ ΚΑΙ ΕΛΑΤΗΡΙΟ… Η ομογενής ράβδος (AB) του σχήματος, έχει μάζα Μ=3Kg και μήκος L=1m. Το ένα άκρο της Α το αρθρώνουμε σε κατακόρυφο τοίχο, ενώ το άλλο άκρο της Β το δένουμε με ένα σχοινί που τεντώνεται οριζόντια και κρατά τη ράβδο σε γωνία φ με ημφ=0,6, όπως στο σχήμα. Ελατήριο σταθεράς Κ=100Ν/m είναι κολλημένο στο άκρο Β της ράβδου. Στο άλλο άκρο του ελατηρίου δένουμε σώμα μάζας m=1Kg. Το σύστημα αρχικά ισορροπεί. Στη συνέχεια μετατοπίζουμε τη μάζα m κατακόρυφα προς τα κάτω κατά d=5cm. Τότε: α) Να βρείτε πως μεταβάλλεται με το χρόνο η τάση του νήματος. Να θεωρήσετε την προς τα κάτω φορά θετική. β) Ποιες χρονικές στιγμές η τάση του νήματος γίνεται μέγιστη; γ) Να βρείτε μεταξύ ποιών τιμών μεταβάλλεται το μέτρο της αντίδρασης από την άρθρωση. δ) Κάποια στιγμή απομακρύνουμε το ελατήριο με τη μάζα m. Αν το όριο θραύσης του νήματος είναι Τθρ=40Ν, τότε ποια είναι η ελάχιστη δύναμη που πρέπει να εφαρμόσουμε στο άκρο Β της ράβδου και κάθετα σε αυτή όπως φαίνεται στο σχήμα ώστε να σπάσει το νήμα; ε) Για τη δύναμη που υπολογίσατε πόση είναι η γωνιακή επιτάχυνση της ράβδου τη στιγμή που σπάει το νήμα; στ) Να υπολογίσετε το έργο του βάρους της ράβδου μέχρι αυτή να γίνει κατακόρυφη. Δίνεται για τη ράβδο ΙΑ=1/3×Μ×L2 και g=10m/s2. Συνοπτική λύση: Σχόλια
Σχόλιο από τον/την Εμμανουήλ Λαμπράκης στις 8 Απρίλιος 2012 στις 21:20
Διαγραφή σχολίου
Δημήτρη δε σου κρύβω ότι στην αρχή μου φάνηκε περίπλοκη. Με δεύτερη ματιά όμως διαπίστωσα ότι έχει μεγάλη διδακτική αξία. Είναι μια εξαιρετική πρόταση για επανάληψη.
Σχόλιο από τον/την Εμμανουήλ Λαμπράκης στις 9 Απρίλιος 2012 στις 20:02
Διαγραφή σχολίου
Μιχαήλ το μόνο λάθος σε αυτά που είπα στο σχόλιο μου είναι ότι σε είπα Δημήτρη! Το ότι θα με συγχωρήσεις το θεωρώ αυτονόητο.
Σχόλιο από τον/την Μιχαήλ Μιχαήλ στις 9 Απρίλιος 2012 στις 22:13
Διαγραφή σχολίου
Μανώλη, εννοείται! Πάντως έτσι και αλλιώς σε ευχαριστώ για το σχόλιο!
ΡΑΒΔΟΣ ΚΑΙ ΤΑΛΑΝΤΩΣΗ… Ο κύβος Σ του σχήματος έχει μάζα m=1Kg και είναι δεμένος σε ελατήριο σταθεράς Κ=100Ν/m και μπορεί να ταλαντώνεται χωρίς τριβές πάνω σε οριζόντιο επίπεδο. Ακόμη ο κύβος Σ είναι δεμένος μέσω μη εκτατού και αβαρούς νήματος με ράβδο μάζας Μ=3Kg και μήκους L=2m που μπορεί να περιστρέφεται χωρίς τριβές γύρω από το άκρο της Α, όπως φαίνεται στο σχήμα. Αρχικά το νήμα είναι οριζόντιο το ελατήριο έχει το φυσικό του μήκος και η ράβδος σχηματίζει γωνία φ=600 με το οριζόντιο έδαφος. Εκείνη τη στιγμή (t=0), αφήνουμε τη μάζα ελεύθερη να κινηθεί και αρχίζει να ταλαντώνεται. Αν τη στιγμή t=1s, που ο κύβος απομακρύνεται μέγιστα από την αρχική του θέση, η ράβδος έχει περιστραφεί κατά 300, τότε: α) Ποια είναι η σταθερά D της ταλάντωσης του κύβου; β) Να γράψετε την εξίσωση της ταλάντωσης που πραγματοποιεί ο κύβος. γ) Αν τη στιγμή που ο κύβος μάζας m, βρίσκεται στη μέγιστη θετική του απομάκρυνση κόβεται το νήμα τότε ποια είναι η εξίσωση της νέας ταλάντωσης που θα πραγματοποιήσει ο κύβος; Θεωρείστε ότι τη στιγμή που κόβεται το νήμα είναι t=0. δ) Ποια είναι η γωνιακή ταχύτητα της ράβδου τη στιγμή που συγκρούεται με το οριζόντιο έδαφος; Δίνονται=1,7 , π2=10 και ότι η ροπή αδράνειας της ράβδου γύρω από τον άξονα περιστροφής της είναι Ι=×Μ×L2. Λύση: Σχόλια
Σχόλιο από τον/την Μιχαήλ Μιχαήλ στις 15 Απρίλιος 2012 στις 22:49
Διαγραφή σχολίου
Δημήτρη ΧΡΟΝΙΑ ΠΟΛΛΑ.
Σχόλιο από τον/την γρηγορης λα στις 19 Απρίλιος 2012 στις 15:37
Διαγραφή σχολίου
Καλησπερα σας. α)Η σταθερα D δίνεται από D=m*ω^2 οπότε αν γνωρίζουμε το ω μπορουμε να βρουμε και την σταθερα. Γνωρίζουμε πως την t=1s ο κύβος βρίσκεται την μεγιστη απομακρυνση.Επισης γνωρίζουμε ότι απο την ΘΙ->Ακραία :t=T/4 Οπότε Τ=4 sec. Άρα ω=2π/Τ <=>ω=π/2 Συνεπώς : D=1*(π^2)/4=5/2 Ν/m B)x=Aημ(πt/2) Γ)χ=Αημ(πt/2+Π/2) (ή 3π/2 αν είναι στα αρνητικά) Δ)Χρησιμοποιούμε ΑΔΜΕ για να βρούμε τη γωνιακή ταχύτητα. Ωραία άσκηση!
Σχόλιο από τον/την Διονύσης Μάργαρης στις 19 Απρίλιος 2012 στις 17:06
Διαγραφή σχολίου
Χρόνια πολλά Μιχαήλ και Γρηγόρη. Γρηγόρη η αρχική θέση, δεν είναι θέση ισορροπίας αλλά η ακραία θέση και η περίοδος είναι 2s. Αλλά, είμαστε σίγουροι Μιχαήλ ότι η κίνηση αυτή είναι ΑΑΤ; Δεν πρέπει να αποδειχτεί;
Σχόλιο από τον/την γρηγορης λα στις 24 Απρίλιος 2012 στις 22:07
Διαγραφή σχολίου
Η Θφμ συμπίπτει με την ΘΙ όταν το ελατήριο είναι οριζόντιο και το επίπεδο είναι λείο,σωστα;
Σχόλιο από τον/την Νίκος Ανδρεάδης στις 24 Απρίλιος 2012 στις 23:12
Διαγραφή σχολίου
Γρηγόρη η ΘΦΜ του ελατηρίου συμπίπτει με τη θέση ισορροπίας της ταλάντωσης όταν το ελατήριο είναι οριζόντιο, το επίπεδο λείο και δεν ασκείται στην οριζόντια διεύθυνση άλλη δύναμη εκτός της δύναμης του ελατηρίου. Αν υπάρχει και άλλη δύναμη στην οριζόντια διεύθυνση τότε η θέση ισορροπίας της ταλάντωσης θα βρεθεί από τη σχέση ΣF=0. Μιχαήλ όπως έγραψε και ο Διονύσης η λύση του α. ερωτήματος είναι αυθαίρετη. Δεν γνωρίζουμε αν η αρχική κίνηση, πριν το κόψιμο του νήματος είναι ΑΑΤ. Διόρθωσε τη λύση ή αφαίρεσε το α. ερώτημα.
Σχόλιο από τον/την Βαγγέλης Κουντούρης στις 25 Απρίλιος 2012 στις 1:28
Διαγραφή σχολίου
Μιχάλη νομίζω και εγώ ότι η κίνηση του κύβου, αρχικά, δεν είναι ΓΑΤ διότι η δύναμη που δέχεται από το νήμα είναι μεταβλητή και ως προς το μέτρο και ως προς τη διεύθυνση. (και μερικά "ψιλά" στην εκφώνηση: να φύγει το "Ακόμη" από τη δεύτερη γραμμή, αντί "αφήνουμε τη μάζα" να γίνει "αφήνουμε τον κύβο", αντί "Αν τη στιγμή...που ο κύβος ...η ράβδος..." να γίνει "Αν τη στιγμή... ο κύβος ...και η ράβδος..." αντί "=1,7" να γίνει "ρίζα 3=1,7" και να γραφεί ότι το νήμα είναι δεμένο στο μέσον της ράβδου)
Σχόλιο από τον/την Μιχαήλ Μιχαήλ στις 25 Απρίλιος 2012 στις 22:36
Διαγραφή σχολίου
Νίκο Ανδρεάδη , Βαγγέλη Κουντούρη ευχαριστώ για τις παρατηρήσεις θα κάνω τις διορθώσεις προσεχώς!
...ΣΥΝΘΕΤΗ ΚΙΝΗΣΗ ΔΙΣΚΟΥ Μια άσκηση από το βιβλίο των Beer- Jonhston. ΣΥΝΘΕΤΗ ΚΙΝΗΣΗ ΔΙΣΚΟΥ Ένας ομογενής δίσκος μάζας m=4Kg και ακτίνας r=5cm, τοποθετείται σε άξονα Οx αμελητέας μάζας. Ο δίσκος μπορεί να περιστρέφεται χωρίς τριβές γύρω από τον άξονα Οx με γωνιακή ταχύτητα ω1=6 rad/s, ενώ ταυτόχρονα περιστρέφεται και γύρω από τον κατακόρυφο άξονα Οy χωρίς να ολισθαίνει, με τη φορά των δεικτών του ρολογιού, και σε απόσταση L=12cm όπως φαίνεται στο σχήμα. Προσδιορίστε: α) τη γωνιακή ταχύτητα του δίσκου, β) τη συνολική στροφορμή του , και γ) την ολική κινητική του ενέργεια. Δίνεται η ροπή αδράνειας του δίσκου για το κέντρο μάζας του και ως προς τον άξονα Οx, Ιx=1/2mr2 , ενώ για τον άξονα Oy και για το κέντρο μάζας του είναι αντίστοιχα Ιy=1/4mr2. Λύση: Σχόλια
Σχόλιο από τον/την Κυριακόπουλος Γιάννης στις 17 Απρίλιος 2012 στις 21:06
Διαγραφή σχολίου
Χρόνια Πολλά Μιχάλη. Στο παρελθόν μα ς απασχόλησε πολύ μια συναφής ανάρτηση του Χρήστου.
Σχόλιο από τον/την Μιχαήλ Μιχαήλ στις 17 Απρίλιος 2012 στις 21:19
Διαγραφή σχολίου
Γιάννη Χρόνια Πολλά. Ευχαριστώ για την ανάρτηση, δεν την είχα υπόψη μου.
Σχόλιο από τον/την Κυριακόπουλος Γιάννης στις 17 Απρίλιος 2012 στις 21:27
Διαγραφή σχολίου
Που να την ξέρεις. Έχουμε φτάσει να ξεχνάμε και δικές μας αναρτήσεις. Αυτή μας απασχόλησε πολύ τότε.
20. Κύλιση και κρούση... Κύλιση και κρούση   Από την κορυφή κεκλιμένου επιπέδου μεγάλου μήκους, γωνίας κλίσης φ με ημφ=0,6, αφήνεται να κυλίεται χωρίς να ολισθαίνει, μια σφαίρα μάζας m=100g και ακτίνας R=1cm. Η σφαίρα αφού διανύσει διάστημα S συγκρούεται ελαστικά με το κάτω άκρο ράβδου. Η ράβδος που έχει μήκος L=0,6m και μάζα M=0,3 Kg, μπορεί να περιστρέφεται χωρίς τριβές γύρω από σταθερό οριζόντιο άξονα που διέρχεται από το σημείο Α. Aκόμη η ράβδος στο κάτω άκρο της έχει μια «μύτη» αμελητέων διαστάσεων έτσι ώστε η δύναμη που αναπτύσσεται κατά τη διάρκεια της κρούσης να περνάει από το κέντρο της σφαίρας. Η διάρκεια της κρούσης είναι αμελητέα και η ακτίνα της σφαίρας θεωρείται ασήμαντη σε σχέση με το μήκος της ράβδου. α) Να υπολογιστεί η ελάχιστη απόσταση S, ώστε να πετύχουμε οριακή ανακύκλωση της ράβδου, β)i) Αν ο συντελεστής στατικής τριβής που είναι ίσος με το συντελεστή τριβής ολίσθησης είναι μ=0,25, τότε να δείξετε ότι μετά την κρούση η σφαίρα θα ολισθαίνει, ii) σε πόσο χρόνο θα σταματήσει η ολίσθηση της σφαίρας; Δίνεται η ροπή αδράνειας της ράβδου ως προς τον άξονα περιστροφής της Ιρ=1/3Μ×L2, ακόμη δίνεται η ροπή αδράνειας της σφαίρας ως προς μια διάμετρό της Ισ=2/5m×R2και g=10m/s2. Λύση: 22.Κύματα...

ΚΥΜΑΤΑ

Πανελλήνιος Διαγωνισμός Φυσικής 2012

Στο παρακάτω σχήμα φαίνεται το στιγμιότυπο τη χρονική στιγμή t΄=0,3s αρμονικού κύματος πλάτους Α=10cm, και περιόδου Τ=0,4s, που διαδίδεται στην αντίθετη κατεύθυνση από αυτήν του ημιάξονα Οx με ταχύτητα υ=10m/s.

α. Προσδιορίστε το σημείο Κ της ευθείας x’x που αρχίζει να ταλαντεύεται τη χρονική στιγμή t=0.

β. Να γράψετε την εξίσωση της απομάκρυνσης με το χρόνο του σημείου 0, y(0)=f(t), και να την παραστήσετε γραφικά.

γ. Να γράψετε την εξίσωση του κύματος και να παραστήσετε γραφικά την απομάκρυνση του σημείου Μ με xM=-3m. δ. Να σχεδιάσετε το στιγμιότυπο του κύματος τη χρονική στιγμή t΄΄=0,5s. Λύση: Σχόλια
Σχόλιο από τον/την Σταύρος Πρωτογεράκης στις 21 Απρίλιος 2012 στις 18:55
Διαγραφή σχολίου
Καλησπέρα Μιχαήλ. Ακαταπόνητος! Έχω την εντύπωση ότι το δοσμένο στιγμιότυπο φανερώνει πως η ταλάντωση κάθε υλικού σημείου ξεκινά προς τ' αρνητικά, ίσως επομένως τη στιγμή της εκκίνησης είναι φ=πrad. Το στιγμιότυπο της λύσης για το δ θα έπρεπε επίσης να τελειώνει σε "λακούβα" αν δεν κάνω κάπου λάθος.
Σχόλιο από τον/την Νίκος Ανδρεάδης στις 21 Απρίλιος 2012 στις 19:12
Διαγραφή σχολίου
Το στιγμιότυπο της άσκησης του διαγωνισμού είναι το παρακάτω.
Σχόλιο από τον/την Μιχαήλ Μιχαήλ στις 22 Απρίλιος 2012 στις 14:09
Διαγραφή σχολίου
Σταύρε ευχαριστώ για την παρατήρηση! ..(Καλή επιτυχία στην κόρη σου στις εξετάσεις)
Σχόλιο από τον/την Σταύρος Πρωτογεράκης στις 22 Απρίλιος 2012 στις 21:56
Διαγραφή σχολίου
Ευχαριστώ πολύ!
Σύστημα τροχαλιών με ιμάντα Η διάταξη του σχήματος αποτελείται από μια διπλή και από μια απλή τροχαλία οι οποίες συνδέονται μεταξύ τους με ιμάντα αμελητέας μάζας όπως φαίνεται στο σχήμα. Η διπλή τροχαλία αποτελείται από δυο τροχαλίες που έχουν κοινό άξονα. Η μικρότερη από αυτές έχει μάζα m=1Kg και ακτίνα r=0,2m, ενώ η πιο μεγάλη έχει μάζα M=2Kg και ακτίνα R=0,4m. Η απλή τροχαλία έχει μάζα m=1Kg και ακτίνα r=0,2m. Γύρω από τη μικρότερη τροχαλία της διπλής τροχαλίας είναι τυλιγμένο αβαρές σχοινί. Στο ελεύθερο άκρο του σχοινιού είναι δεμένο ένα σώμα μάζας m1= =Kg. Κάποια στιγμή αφήνουμε το σύστημα ελεύθερο. α) Αν το νήμα ξετυλιχθεί κατά L=1m, τότε να υπολογιστεί η γωνιακή ταχύτητα ω1 της διπλής τροχαλίας β) Ποιος είναι ο ρυθμός προσφερόμενης ενέργειας εκείνη τη στιγμή και γ) Να υπολογιστεί η τάση του νήματος. Η ροπή αδράνειας κυλίνδρου ως προς τον άξονα περιστροφής του είναι Ι= 1/2m×r2. Δίνεται g=10m/s2. Η άσκηση είναι μια μικρή παραλλαγή της άσκησης του Δημήτρη Αναγνώστου στον οποίο και αφιερώνεται. Συνοπτική λύση Τροχαλία και ελατήριο Η τροχαλία του σχήματος αποτελείται από δυο συγκολλημένους δίσκους με ακτίνες R=20cm και r=10cm που έχουν κοινό άξονα. Οι δίσκοι περιστρέφονται χωρίς τριβές γύρω από τον κοινό τους άξονα και έχουν συνολική ροπή αδράνειας Ι=4×10-2Kg×m2. Τα αβαρή σχοινιά που είναι τυλιγμένα στους δίσκους, έχουν στα ελεύθερα άκρα τους δεμένα τα σώματα με μάζες m1=4Kg και m2=2Kg. Το σώμα μάζας m1, είναι επίσης δεμένο σε οριζόντιο αβαρές ελατήριο Κ=100Ν/m και μπορεί να κινείται στο οριζόντιο επίπεδο χωρίς τριβές. Κάποια στιγμή εξασκούμε στο σύστημα την οριζόντια δύναμη F=80N που φαίνεται στο σχήμα. Τότε: α) Για ποια επιμήκυνση του ελατηρίου η γωνιακή ταχύτητα περιστροφής της τροχαλίας γίνεται μέγιστη; β)i) Πόση είναι τότε η κινητική ενέργεια του κάθε σώματος; ii) Ποιος είναι τότε ο ρυθμός μεταβολής της κινητικής ενέργειας του συστήματος των μαζών; Αν εκείνη τη στιγμή που η γωνιακή ταχύτητα περιστροφής της τροχαλίας γίνεται μέγιστη κοπεί το νήμα που συνδέει τη μάζα m1 με την τροχαλία, τότε γ)i) Nα υπολογιστεί η επιτάχυνση α2 της μάζας m2 καθώς και η τάση του νήματος που συνδέει την m2 με την τροχαλία ii) Nα υπολογιστεί ο ρυθμός μεταβολής της κινητικής ενέργειας του συστήματος τροχαλία – m2; δ)i) Ποια είναι η ενέργεια ταλάντωσης της μάζας m1 και ii) Ποιος είναι ο μέγιστος ρυθμός μεταβολής της κινητικής ενέργειας της m1 καθώς αυτή ταλαντώνεται; Δίνεται g=10m/s2. Συνοπτική λύση: Σχόλια
Σχόλιο από τον/την Νίκος Ανδρεάδης στις 3 Μάιος 2012 στις 0:20
Διαγραφή σχολίου
Μιχαήλ μπράβο, πολύ καλή και πλούσια άσκηση, ότι πρέπει για επανάληψη.
Σχόλιο από τον/την ΧΡΗΣΤΟΣ ΕΛΕΥΘΕΡΙΟΥ στις 3 Μάιος 2012 στις 3:04
Διαγραφή σχολίου
(Μιχαήλ)^2 μπράβο για την πανέμορφη άσκηση.Η ταλάντωση σε συνδυασμό με την μηχανική στερεού έχει να πέσει πολλά πολλά χρόνια.Μήπως είναι η χρονιά της...
Σχόλιο από τον/την Τίνα Νάντσου στις 3 Μάιος 2012 στις 10:24
Διαγραφή σχολίου
Ωραία άσκηση για επαναληψη. Μου αρέσει το δ ερώτημα, αν και το ιι το θεωρώ αρκετά δύσκολο. Μπράβο!
Σχόλιο από τον/την ΜΑΝΩΛΗΣ ΔΡΑΚΑΚΗΣ στις 3 Μάιος 2012 στις 12:40
Διαγραφή σχολίου
Μιχαήλ πολύ καλή. Σήμερα την έθεσα στην τάξη σε δυο τμήματα , που με τίμησαν οι καλοί μαθητές με την παρουσία τους. Είδα και άλλες ωραίες λύσεις και σύντομες , που γεννάνε τα φρέσκα μυαλά των παιδιών. Η μόνη απορία που χρειάστηκε να τα μπαλώσω κάπως, είναι για το ποιες δυνάμεις ασκούνται στην τροχαλία μετά το κόψιμο του νήματος. Κάποια παιδιά θεώρησαν ότι παύει να ασκείται και η F γιατί τους δημιουργήθηκε η εντύπωση ότι, το νήμα περιβάλλει την τροχαλία. Επίσης δυσκολεύτηκαν πολύ στο τελευταίο ερώτημα, γιατί όπως μου είπαν, δεν ξέρουν από την τριγωνομετρία τον τύπο ημ2α = 2ημασυνα. Κι επειδή, δεν μπορεί να ξέρει κανείς τι μένει στα μαθηματικά με τις συνεχείς περικοπές της ύλης , καλό είναι να δίνουμε όλες τις τριγωνομετρικές ταυτότητες που χρειάζονται να τελειώνουμε...
Σχόλιο από τον/την ΘΩΜΑΣ στις 3 Μάιος 2012 στις 17:05
Διαγραφή σχολίου
Το ερώτημα Βii) πώς θα μπορούσε να λυθεί διαφορετικά;
Σχόλιο από τον/την ΜΑΝΩΛΗΣ ΔΡΑΚΑΚΗΣ στις 3 Μάιος 2012 στις 17:32
Διαγραφή σχολίου
Θωμά να σου πω τι μου είπαν σήμερα οι συμμαθητές σου: Αφού το σύστημα επιταχύνεται μέχρι τα σώματα στη συγκεκριμένη θέση να αποκτήσουν επιτάχυνση μηδέν ( βλέπε προηγούμενο ερώτημα) , η ταχύτητές τους εκείνη τη στιγμή θα είναι μέγιστες άρα και οι κινητικές τους ενέργειες, άρα ο ρυθμός μεταβολής της κινητικής ενέργειας του κάθενός είναι μηδέν έτσι και του συστήματός τους θα είναι μηδέν. ή Αφού η γωνιακή ταχύτητα της τροχαλίας είναι μέγιστη και οι ταχύτητες των μαζών θα είναι μέγιστες άρα ( υ1=ωr , υ2=ω2r) , ο ρυθμός μεταβολής της κινητικής ενέργειας του κάθενός είναι μηδέν έτσι και του συστήματός τους θα είναι μηδέν.
Σχόλιο από τον/την Μιχαήλ Μιχαήλ στις 3 Μάιος 2012 στις 20:13
Διαγραφή σχολίου
Νίκο, Χρήστο, Τίνα, Μανώλη, Δημήτρη, ευχαριστώ για τα καλά σας λόγια.
Σχόλιο από τον/την Βαγγέλης Κουντούρης στις 3 Μάιος 2012 στις 21:29
Διαγραφή σχολίου
Μιχάλη πιθανόν κάτι δεν βλέπω ή κάτι δεν πάει καλά. Υποθέτω ως εννοούμενο, επειδή δεν διευκρινίζεται, ότι μέχρι την "κάποια στιγμή" το σύστημα είναι ακίνητο, οπότε Τ2=...=20Ν, Τ1=...40Ν, άρα Δx=...=0,4m. Παρά την εν συνεχεία άσκηση της F=80N η Δx παραμένει η ίδια και ανεξάρτητη από την τιμή της F;
Σχόλιο από τον/την ΜΑΝΩΛΗΣ ΔΡΑΚΑΚΗΣ στις 3 Μάιος 2012 στις 22:00
Διαγραφή σχολίου
Μιχάλη έχει δίκιο ο Βαγγέλης. Ήταν ερώτηση και των μαθητών το πρωί , που ξέχασα να την αναφέρω στο πρώτο σχόλιό μου.
Τροχαλία_ελατήριο_ράβδος Η τροχαλία του σχήματος έχει μάζα Μ=2Κg και ακτίνα R=20cm. Σε απόσταση r= από το κέντρο της υπάρχει ένα αυλάκι το οποίο είναι τυλιγμένο με αβαρές νήμα, στο ελεύθερο άκρο του οποίου είναι δεμένο σώμα μάζας m1=1Kg. Το σώμα μάζας m1 είναι επίσης δεμένο στο κατακόρυφο ελατήριο του σχήματος σταθεράς Κ=100Ν/m. Στην περιφέρεια της τροχαλίας είναι επίσης τυλιγμένο γύρω της, αβαρές νήμα στο ελεύθερο άκρο του οποίου είναι δεμένο σώμα μάζας m2=1Kg. Το σύστημα αρχικά ισορροπεί. α) Να υπολογίσετε την επιμήκυνση του ελατηρίου, για την αρχική ισορροπία του συστήματος, β) Κάποια στιγμή κόβουμε το νήμα που συνδέει τη m1 με την τροχαλία, τότε: i) Να γράψετε την εξίσωση της ταλάντωσης που πραγματοποιεί η m1 και ii) Να υπολογίσετε την επιτάχυνση της m2, γ) Η m2 αφού διανύσει απόσταση h=0,4m σπάει το νήμα που τη συνδέει με την τροχαλία και συγκρούεται πλαστικά, χτυπώντας στο άκρο οριζόντιας ράβδου. Η ράβδος έχει μάζα mρ=3 Kg, μήκος L=1m. Ακόμη στηρίζεται στο κέντρο της Κ1,σε τριγωνική βάση ενώ μπορεί να περιστρέφεται ελεύθερα χωρίς τριβές. Τότε, i) Να υπολογίσετε τη γωνιακή επιτάχυνση όταν το σύστημα ράβδος – m2, έχει περιστραφεί κατά 1800 και ii) Να υπολογιστεί η γωνία στροφής για την οποία το σύστημα ράβδος – m2 ηρεμεί στιγμιαία. Συνοπτική λύση: Σύστημα οδοντωτών τροχών Σύστημα οδοντωτών τροχών   Η διάταξη του σχήματος αποτελείται από δυο οδοντωτούς τροχούς. Ο πιο μεγάλος έχει ροπή αδράνειας Ι1=18×10-2Κg×m2 και ακτίνα R=0,4m. Ο μικρός τροχός έχει Ι2=2×10-2Κg×m2 και ακτίνα r=0,2m. Γύρω από τo μεγαλύτερο τροχό και σε απόσταση r=0,2m υπάρχει ένα αυλάκι, γύρω από το οποίο είναι τυλιγμένο αβαρές σχοινί. Στο ελεύθερο άκρο του σχοινιού είναι δεμένο ένα σώμα μάζας m1=13/8 =Kg. Κάποια στιγμή αφήνουμε το σύστημα ελεύθερο. α) Αν το νήμα ξετυλιχθεί κατά L=1m, τότε να υπολογιστεί η γωνιακή ταχύτητα ω1 του μεγαλύτερου οδοντωτού τροχού β) Ποιος είναι ο ρυθμός προσφερόμενης ενέργειας εκείνη τη στιγμή γ) Να υπολογιστεί η τάση του νήματος. δ) Να υπολογιστεί η συνολική ροπή σε κάθε τροχό ε) Να υπολογιστεί η επιτάχυνση της μάζας m1 με τους Νόμους του Newton. Δίνεται g=10m/s2. Συνοπτική λύση: Σχόλια
Σχόλιο από τον/την Μιχαήλ Μιχαήλ στις 8 Μάιος 2012 στις 18:50
Διαγραφή σχολίου
Δημήτρη ευχαριστώ. Είναι έτοιμη εικόνα από το Cd της Corel
Σχόλιο από τον/την ΜΑΝΩΛΗΣ ΔΡΑΚΑΚΗΣ στις 8 Μάιος 2012 στις 21:31
Διαγραφή σχολίου
Μιχάλη πολύ καλή κι αυτή... Στη σελίδα τούτη δεν περνάνε κλάσματα. Γιαυτό γράψε τη μάζα m1=13/8 kg για να εμφανίζεται.
Σχόλιο από τον/την Μιχαήλ Μιχαήλ στις 8 Μάιος 2012 στις 22:19
Διαγραφή σχολίου
Μανώλη να΄σαι καλά. Σε ευχαριστώ!
Σχόλιο από τον/την Βαγγέλης Κουντούρης στις 8 Μάιος 2012 στις 22:21
Διαγραφή σχολίου
Καλή είναι Μιχάλη, αλλά το σχήμα δεν είναι. Τα δοντάκια και τα κενά ενός οδοντωτού τροχού έχουν το ίδιο εύρος και για να ταιριάζει με άλλον τροχό πρέπει να είναι ίδια και στους δύο, ανεξάρτητα από τις ακτίνες τους. Αν μπορείς άλλαξέ το.
Σχόλιο από τον/την Μιχαήλ Μιχαήλ στις 8 Μάιος 2012 στις 22:55
Διαγραφή σχολίου
Βαγγέλη ευχαριστώ. Θα δω τι μπορώ να κάνω!
Lagrange και τροχαλία Lagrange και τροχαλία   Η τροχαλία του σχήματος έχει μάζα Μ και ακτίνα R και μπορεί να περιστρέφεται χωρίς τριβές γύρω από τον άξονα συμμετρίας της. Αβαρές σχοινί είναι τυλιγμένο στην περιφέρεια της τροχαλίας και στο ελεύθερο άκρο της είναι δεμένο σώμα μάζας m. Τη χρονική στιγμή t0=0 αφήνουμε ελεύθερο το σώμα. Να βρεθεί η επιτάχυνση του σώματος.   Λύση: Τροχός_ελατήριο_ράβδος Στη διάταξη του σχήματος ο τροχός έχει μάζα Μ=0,6Κg και ακτίνα R=10cm. Στην περιφέρεια του τροχού είναι τυλιγμένο γύρω της, αβαρές νήμα στο ελεύθερο άκρο του οποίου είναι δεμένο σώμα μάζας m=0,1Kg. Το σώμα μάζας m είναι επίσης δεμένο στο οριζόντιο ελατήριο του σχήματος σταθεράς Κ=90Ν/m. Επίσης σε απόσταση r=R/2 από το κέντρο του τροχού είναι δεμένο ένα δεύτερο νήμα, το άλλο άκρο του οποίου είναι δεμένο στο άκρο ράβδου μάζας Μρ=3 Κg και μήκους L=1m. Στο μέσο της ράβδου ασκείται κάθετα σε αυτή και με τη φορά που φαίνεται στο σχήμα δύναμη F=30N και η ράβδος ισορροπεί σχηματίζοντας με την κατακόρυφο γωνία φ= rad με ημφ=0,6. α) Αν αρχικά όλο το σύστημα ισορροπεί τότε να υπολογιστεί η τάση στα άκρα των δυο νημάτων. β) Κάποια στιγμή κόβουμε το νήμα που συνδέει τη ράβδο με τον τροχό. Πόση είναι τότε η γωνιακή επιτάχυνση της ράβδου; γ) Μόλις η ράβδος γίνει κατακόρυφη συγκρούεται με κατακόρυφο τοίχο. Ποια είναι η απώλεια κινητικής ενέργειας κατά την κρούση της ράβδου με τον τοίχο αν τελικά η ράβδος σταματά να περιστρέφεται; δ) Μόλις η μάζα m τη χρονική στιγμή t0=0, φτάσει στη θέση φυσικού μήκους του ελατηρίου, κόβεται και το νήμα που τη συνδέει με τον τροχό. i) Ποια είναι η εξίσωση της α.α.τ που θα πραγματοποιήσει η μάζα m; Θεωρείστε την προς τα δεξιά φορά θετική. ii) Ποια είναι η γωνιακή ταχύτητα του τροχού όταν η μάζα m πραγματοποιήσει μια ταλάντωση; Τριβές δεν υπάρχουν. Δίνεται για τον τροχό Ι=1/2Μ×R2 και για τη ράβδο Ιρ=1/3Μρ×L2 ακόμη g=10m/s2 .   Συνοπτική λύση: Σχόλια
Σχόλιο από τον/την Πέτρος Καραπέτρος στις 18 Μάιος 2012 στις 0:21
Διαγραφή σχολίου
Μιχαήλ καλησπέρα, συγχαρητήρια για την άσκηση και για όλο το υλικό που μας προσέφερες την φετινή σχολική χρονιά. Στην παραπάνω άσκηση, νομίζω ότι η γωνιακή επιτάχυνση στο β) είναι 24rad/s2. Οι ροπές τF και τwρ είναι και οι δύο αριστερόστροφες οπότε κατά την εφαρμογή Στ=Ιαγ προστίθενται.
Σχόλιο από τον/την Μιχαήλ Μιχαήλ στις 18 Μάιος 2012 στις 0:32
Διαγραφή σχολίου
Πέτρο ευχαριστώ πολύ. Έχεις δίκιο!
Ταλάντωση και «αποχωρισμός» Τα σώματα Σ1 και Σ2 του σχήματος με m1=1Kg και m2=3Kg αντίστοιχα, είναι τοποθετημένα σε λείο κεκλιμένο επίπεδο γωνίας κλίσης φ=300 και εφάπτονται μεταξύ τους. Το Σ1 είναι δεμένο στην άκρη του ελατηρίου σταθεράς Κ=100N/m. Αρχικά το σύστημα ισορροπεί. Μετακινούμε τα σώματα ώστε το ελατήριο να συσπειρωθεί κατά Α=40cm και στη συνέχεια τα αφήνουμε ελεύθερα. Να βρείτε:   α) Πως μεταβάλλεται με το χρόνο η δύναμη ανάμεσα στις δυο μάζες, αν για t=0 είναι x=0 και υ>0, β) τη θέση στην οποία θα αποχωριστεί το Σ2 από το Σ1, γ) την ενέργεια της ταλάντωσης που εκτελεί το Σ1 αφού αποχωριστεί από το Σ2, δ) το κλάσμα της κινητικής ενέργειας του Σ2, αμέσως μετά τον αποχωρισμό, προς την αρχική ενέργεια της ταλάντωσης των δυο σωμάτων, ε) την απόσταση μεταξύ των δυο σωμάτων όταν το Σ1 πραγματοποιήσει μια ταλάντωση μετά τον αποχωρισμό. Δίνεται π=3,14, π2=10, =1,7 και g=10m/s2 .   Συνοπτική λύση: ή εδώ και εδώ ΤΑΛΑΝΤΩΣΗ ΚΑΙ ΟΛΙΣΘΗΣΗ Τα σώματα Σ1 και Σ2 του σχήματος με μάζες αντίστοιχα, m1= 1Kg και m2=4 Kg αρχικά ισορροπούν. Το Σ1 βρίσκεται πάνω στο Σ2. Το επίπεδο επαφής των δυο σωμάτων είναι οριζόντιο και ο συντελεστής τριβής μεταξύ τους είναι μ=0,5. Το Σ2 βρίσκεται πάνω σε λείο οριζόντιο επίπεδο. Ακόμη το Σ2 είναι δεμένο στο ένα άκρο οριζόντιου ελατηρίου σταθεράς Κ=400N/m, όπως φαίνεται στο σχήμα. Κάποια στιγμή και ενώ το σύστημα των δυο σωμάτων ισορροπεί, δίνουμε αρχική ταχύτητα υ0= m/s στο Σ1, οπότε και αρχίζει να ολισθαίνει πάνω στο Σ2. α) Πόσο μετακινείται το Σ1 πάνω στο Σ2 μέχρι να σταματήσει; β) Πόση είναι η κοινή ταχύτητα που αποκτούν τα δύο σώματα; γ) Πόσο μετακινήθηκε το Σ1 σε σχέση με το Σ2 μέχρι τη στιγμή αυτή; δ) Πόση είναι η θερμότητα που μεταφέρεται στο περιβάλλον; ε) Τι είδους κίνηση θα ακολουθήσει μετά από τη στιγμή αυτή; Δίνεται g=10m/s2 και π2=10. Λύση: Σχόλια

Ετικέτες: τάξη-γ, ταλαντώσεις

Σχόλιο από τον/την Χαλκιαδάκης Παναγιώτης στις 1 Ιούλιος 2012 στις 13:50
Διαγραφή σχολίου
Μιχάλη, καλό μήνα. Πολύ ενδιαφέρουσα άσκηση αλλά έχω μερικούς προβληματισμούς. Την χρονική στιγμή t=π/10 s το Σ1 ακινητοποιείται ως προς το οριζόντιο λείο επίπεδο (που είναι αδρανειακό σύστημα αναφοράς) Το Σ2 λόγω της Τ΄και της Fel αρχικά επιταχύνεται προς τα αριστερά. Γιατί την t=π/10 s το Σ2 να ειναι ακίνητο ως προς το Σ1 = άρα να έχουν κοινή ταχύτητα; Και στο δ) ερώτημα το έργο της τριβής δεν μπορεί να είναι ίσο με τη Q. Λόγω της Τ' μεταφέρεται και ενέργεια και στο Σ2.
Σχόλιο από τον/την Μιχαήλ Μιχαήλ στις 1 Ιούλιος 2012 στις 17:32
Διαγραφή σχολίου
Παναγιώτη καλό μήνα. Ευχαριστώ για τις παρατηρήσεις. Έχω και εγώ τους προβληματισμούς μου για την άσκηση και γι αυτό ήθελα να τη μοιραστώ μαζί σας.
Σχόλιο από τον/την Διονύσης Μάργαρης στις 1 Ιούλιος 2012 στις 17:41
Διαγραφή σχολίου
Καλό μήνα και από μένα Μιχάλη. Να θέσω και γω ένα διευκρινιστικό ερώτημα. Στο πρώτο ερώτημα, λες μέχρι να σταματήσει. Εννοείς ως προς το έδαφος ή να σταματήσει ως προς το κάτω σώμα; Η απάντηση που δίνεις στο α) ερώτημα είναι ως προς το έδαφος, αλλά τότε δεν θα έπρεπε και η ταχύτητα του Σ2 να είναι μηδενική, δηλαδή να βρίσκεται στην δεξιά ακραία θέση της ταλάντωσής του; Για το έργο της τριβής, νομίζω Παναγιώτη ότι είναι σωστό, αφού βάζει το Δχ, δηλαδή στο πόσο γλίστρησε και όχι την μετατόπισή του χ1.
Σχόλιο από τον/την Διονύσης Μάργαρης στις 1 Ιούλιος 2012 στις 18:34
Διαγραφή σχολίου
Ναι, αλλά τότε Μιχάλη, το σώμα Σ1 θα έχει την ταχύτητα του Σ2 και αν θέλεις να είναι στη θέση ισορροπίας θα πρέπει να έχει ταχύτητα ίση με την μέγιστη ταχύτητα του Σ2.
Σχόλιο από τον/την Μιχαήλ Μιχαήλ στις 1 Ιούλιος 2012 στις 18:34
Διαγραφή σχολίου
Διονύση καλό μήνα και από μένα και ευχαριστώ για τις παρατηρήσεις. Έχεις δίκιο εννοώ να σταματήσει η ολίσθηση!
Σχόλιο από τον/την Μιχαήλ Μιχαήλ στις 1 Ιούλιος 2012 στις 18:39
Διαγραφή σχολίου
Διονύση νομίζω ότι στο β) ερώτημα αυτή την κοινή ταχύτητα υπολογίζω!
Σχόλιο από τον/την Διονύσης Μάργαρης στις 1 Ιούλιος 2012 στις 18:47
Διαγραφή σχολίου
Ναι Μιχάλη, αλλά χρησιμοποιείς το χρόνο από το α) ερώτημα όπου έχεις μηδενίσει την ταχύτητα του Σ1. Και αυτός ο χρόνος που βρίσκεις δεν υπάρχει, αφού το Σ1 θα σταματήσει την ολίσθησή του, όταν αποκτήσει κοινή ταχύτητα με το Σ2. Αλλά τότε έχουμε μια νέα ταλάντωση του συστήματος. Θέλω δηλαδή να πω, ότι δεν θα ασκείται η τριβή ολίσθησης στο Σ1, μέχρι την θέση χ1=25cm, αφού πολύ πριν, σταματά η ολίσθηση.
Σχόλιο από τον/την Μιχαήλ Μιχαήλ στις 1 Ιούλιος 2012 στις 19:28
Διαγραφή σχολίου
Διονύση εννοείς ότι για να βρούμε την κοινή ταχύτητα θα πρέπει να πάρουμε υ1=υ0-α1t=π/2-5t και υ2=0,25συν(5t+3π/2) και να λύσουμε την εξίσωση π/2-5t=0,25συν(5t+3π/2)
Σχόλιο από τον/την Διονύσης Μάργαρης στις 1 Ιούλιος 2012 στις 19:42
Διαγραφή σχολίου
Αν μείνει η διατύπωση ως έχει, ναι Μιχάλη.
Σχόλιο από τον/την Μιχαήλ Μιχαήλ στις 1 Ιούλιος 2012 στις 19:46
Διαγραφή σχολίου
Οk Διονύση ευχαριστώ!
Σχόλιο από τον/την Χαλκιαδάκης Παναγιώτης στις 1 Ιούλιος 2012 στις 19:48
Διαγραφή σχολίου
Μιχάλη, αυτή την εξίσωση έλυσα (με γραφική μέθοδο) και βρήκα t1=0,2655 s οπότε ξ ταχύτητα του Σ1 βγαίνει υ1=1,57-5*0,2655=0,2425m/s και του Σ2 υ2=0,25*συν(5*0,2655+3π/2)=0,2425m/s Aν δεν έχω κάνει λάθος στους υπολογισμούς αυτή πρέπει να είναι και η στιγμή που θα σταματήσει το Σ1 να ολισθαίνει πάνω στο Σ2
Σχόλιο από τον/την Μιχαήλ Μιχαήλ στις 1 Ιούλιος 2012 στις 20:01
Διαγραφή σχολίου
Παναγιώτη ευχαριστώ πολύ για το χρόνο σου! Θα ξανακοιτάξω την άσκηση μήπως και γίνει κάπως πιο εύκολη η εύρεση του χρόνου και της κοινής ταχύτητας!
Σχόλιο από τον/την Διονύσης Μάργαρης στις 1 Ιούλιος 2012 στις 20:15
Διαγραφή σχολίου
Μου επιτρέπεις μια ιδέα Μιχάλη; Μη δώσεις δεδομένο τη μάζα του Σ2 ή τη σταθερά του ελατηρίου και πες ότι έχουμε ταυτόχρονο μηδενισμό των ταχυτήτων των δύο σωμάτων, οπότε θα έχεις το χρόνο, ο οποίος θα είναι ίσος με την μισή περίοδο της αρχικής ταλάντωσης του Σ2.
Σχόλιο από τον/την Χαλκιαδάκης Παναγιώτης στις 1 Ιούλιος 2012 στις 20:16
Διαγραφή σχολίου
Μιχάλη μια πρόταση , την οποία δυστυχώς λόγω χρόνου δεν μπορώ να επεεξεργαστώ. Αν αλλάξεις την σταθερά του ελατηρίου στην τιμή Κ=400Ν/m τότε Τ/2=π/10s οπότε η ταχύτητα και των δύο μηδενίζεται ταυτόχρονα.
Σχόλιο από τον/την Μιχαήλ Μιχαήλ στις 1 Ιούλιος 2012 στις 20:41
Διαγραφή σχολίου
Διονύση και Παναγιώτη έκανα τις αλλαγές με τη λύση για το χρόνο που έδωσε ο Παναγιώτης. Θα δω και τις άλλες δυο ιδέες!! Ευχαριστώ
Test στην α.α.τ On line test στην Α.Α.Τ Αφιερώνεται στο Δημήτρη Αναγνώστου για το υλικό που μας χάρισε!     Σχόλια
Σχόλιο από τον/την Γκενές Δημήτρης στις 22 Ιούλιος 2012 στις 22:48
Διαγραφή σχολίου
Μιχαήλ Συγχαρητήρια. Πολύ καλοσχεδιασμένες. α ) Με ποιο πρόγραμμα τις έφτιαξες ; β) Μερικές ερωτήσεις Ε8 η συνάρτηση τετράγωνο ημιτόνου είναι αρμονική ; Ε23 βλέπω και β, δ ως σωστές απαντήσεις ; Ε25 νομίζω β Σωστό και όχι γ . τι λέω λάθος ; Ε 29 δεν βλέπω καθόλου επιλογές,... μόνο εγώ ; ΥΓ. Είχες πρόγονο τον γνωστό πρωτοπόρο του Ελληνικού Κινηματογράφου ; Μπράβο.! όχι μόνο για τις Ιδέες αλλά και για την δημιουργικότητά σου και τη προσφορά σου εδώ
Σχόλιο από τον/την Διονύσης Μάργαρης στις 23 Ιούλιος 2012 στις 9:02
Διαγραφή σχολίου
Καλημέρα Μιχάλη. Μπράβο, πολύ καλή δουλειά!
Σχόλιο από τον/την Μιχαήλ Μιχαήλ στις 23 Ιούλιος 2012 στις 9:11
Διαγραφή σχολίου
Διονύση , καλημέρα. Ευχαριστώ πολύ.
Σχόλιο από τον/την Γκενές Δημήτρης στις 23 Ιούλιος 2012 στις 9:12
Διαγραφή σχολίου
Καλημέρα Μιχαήλ. Έχεις δίκιο και για 23 και για την 25... ( ου γαρ μόνον η ραστώνη του Καλοκαιριού) Δεν με λένε βέβαια Γιάννη, αλλά... "πες με και πάτερ, εγώ θα καταλάβω"*
Σχόλιο από τον/την Μιχαήλ Μιχαήλ στις 23 Ιούλιος 2012 στις 9:26
Διαγραφή σχολίου
Δημήτρη Γκενέ, καλημέρα και σε ευχαριστώ για τις παρατηρήσεις, τις ερωτήσεις τις έφτιαξα με <script language="JavaScript"> για html που υπάρχει free στο διαδίκτυο. Στην 23 η περίοδος και άρα το ω παραμένουν σταθερά. Άρα υmax=ωΑ και υmax(2)=ω(2Α), άρα διπλασιάζεται Στην 25 ισχύει Τ1>Τ2 άρα πρώτα φτάνει το Σ2 Στην 29 τις επιλογές τις έχω σαν εικόνα! Οπότε ......... Η 8 άλλαξε
Σχόλιο από τον/την Μιχαήλ Μιχαήλ στις 23 Ιούλιος 2012 στις 9:56
Διαγραφή σχολίου
Δημήτρη συγγνώμη για το Γιάννης. Άλλαξα την καταχώριση και μετά είδα το σχόλιό σου! Να σαι καλά!
Σχόλιο από τον/την Βασίλης Δουκατζής στις 23 Ιούλιος 2012 στις 19:07
Διαγραφή σχολίου
26. Σώμα μάζας Μ έχει προσδεθεί στο κάτω άκρο κατακόρυφου ιδανικού ελατηρίου σταθεράς Κ του οποίου το άνω άκρο είναι στερεωμένο σε ακλόνητο σημείο. Απομακρύνουμε το σώμα κατακόρυφα προς τα κάτω κατά απόσταση α από τη θέση ισορροπίας και το αφήνουμε ελεύθερο να κάνει ταλάντωση. Επαναλαμβάνουμε το πείραμα και με ένα άλλο ελατήριο σταθεράς Κ΄ = 4Κ τότε, α. τα δυο σώματα έχουν την ίδια ενέργεια ταλάντωσης β. η ενέργεια ταλάντωσης είναι τετραπλάσια στην πρώτη περίπτωση γ. η κινητική ενέργεια ταλάντωσης στη δεύτερη περίπτωση είναι τετραπλάσια της πρώτης για την ίδια απομάκρυνση από τη Θ.Ι.Τ δ. η δυναμική ενέργεια ταλάντωσης στη δεύτερη περίπτωση είναι πάντα μεγαλύτερη από ότι στην πρώτη ταλάντωση
Σχόλιο από τον/την Βασίλης Δουκατζής στις 23 Ιούλιος 2012 στις 19:16
Διαγραφή σχολίου
Στο τεστ που έκανα απάντησα γ και μου την έδωσε ως σωστή. Δεν ισχύει όμως στη θέση χ = α. Υ.Γ.1 "Επαναλαμβάνουμε το πείραμα και με ένα άλλο ελατήριο σταθεράς ...." συμπλήρωσε με το ίδιο πλάτος α, διότι επαναλαμβάνω το πείραμα δε σημαίνει ότι έχω ακριβώς τις ίδιες συνθήκες. Υ.Γ. 2 Πολύ καλή δουλειά. Υ.Γ. 3 Οι 6, 17, 25, 26, 29 θέλουν "δουλειά" άρα μάλλον καλλίτερα να μπουν σε Β θέμα
Σχόλιο από τον/την Βασίλης Δουκατζής στις 23 Ιούλιος 2012 στις 19:26
Διαγραφή σχολίου
Είδα στο Blog σου το παράδοξο με το νόμο του Charles (στη φυσική Β κατεύθυνσης και στη χημεία Α λυκείου). Επειδή η ίδια απορία είχε δημιουργηθεί και σε μένα και άρχισα να το ψάχνω λίγο, με "ξελάσπωσε" ο συνάδελφος Ανδρέας Κασσέτας (Δες εδώ http://users.sch.gr/kassetas/zzGay-Lussac.htm )
Σχόλιο από τον/την Πέτρος Καραπέτρος στις 24 Ιούλιος 2012 στις 20:41
Διαγραφή σχολίου
Γιώργο καλησπέρα, όταν λες με οποιδήποτε editor, μπορείς ακόμη και σε word και να το σώσεις ως html αρχείο. ¨Η θα πρέπει να κατεβάσεις κάποιον html editor;;
Σχόλιο από τον/την Πέτρος Καραπέτρος στις 24 Ιούλιος 2012 στις 23:07
Διαγραφή σχολίου
Γιώργο σε ευχαριστώ. Το δοκίμασα και δουλεύει.
Σχόλιο από τον/την Μιχαήλ Μιχαήλ στις 26 Ιούλιος 2012 στις 20:37
Διαγραφή σχολίου
Βασίλη Δουκάτζη, (αργοπορημένη η απάντηση μου...διακοπές γαρ) σε ευχαριστώ για τις παρατηρήσεις τις έλαβα υπόψη μου.
Σχόλιο από τον/την Βασίλης Δουκατζής στις 26 Ιούλιος 2012 στις 22:11
Διαγραφή σχολίου
Καλές διακοπές Μιχαήλ (και σε όλους τους συναδέλφους του Blog). Μια παρατήρηση Δουκατζής λέγομαι (έβαλες τον τόνο στο κά) αλλά βέβαια δεν μπορείς να ξέρεις αφού το όνομα μου είναι με κεφαλαία.
Test στις ηλεκτρικές - φθίνουσες ταλαντώσεις Test στις ηλεκτρικές ταλαντώσεις Test στις φθίνουσες ταλαντώσεις 37. ΜΕΤΑΒΛΗΤΗ ΔΥΝΑΜΗ ΚΑΙ ΤΑΛΑΝΤΩΣΗ Ένα σώμα μάζας m=4Kg ισορροπεί δεμένο σε ελατήριο σταθεράς Κ=80Ν/m πάνω σε λείο κεκλιμένο επίπεδο γωνίας φ=300. Τη χρονική στιγμή t=0 εξασκούμε στο σώμα μια μεταβλητή δύναμη F=40-20x (S.I) όπου x είναι η απομάκρυνση από την αρχική θέση ισορροπίας της μάζας m και με φορά προς τα πάνω στο κεκλιμένο επίπεδο, όπως φαίνεται στο σχήμα. α) Να αποδείξετε ότι το σύστημα πραγματοποιεί α.α.τ, β) Να γράψετε την εξίσωση x(t) της α.α.τ, και να υπολογίσετε την ενέργεια της ταλάντωσης. γ) Να γράψετε τη χρονική εξίσωση της μεταβλητής δύναμης F(t). δ) Να γράψετε την εξίσωση της δύναμης επαναφοράς σε συνάρτηση με την απόσταση από την αρχική θέση ισορροπίας της μάζας m. ε) Να υπολογίσετε το ρυθμό μεταβολής της κινητικής ενέργειας της μάζας m, τη χρονική στιγμή t= π/20 s. στ) Αν τη στιγμή που η μάζα m, μετατοπιστεί κατά +Α από τη θέση ισορροπίας της ταλάντωσης καταργηθεί η εξωτερική δύναμη F, τότε πόση είναι η προσφερόμενη ενέργεια στο σύστημα από τη δύναμη F; Θεωρείστε την προς τα πάνω φορά θετική. Δίνεται g=10m/s2   Λύση: Σχόλια
Σχόλιο από τον/την Τίνα Νάντσου στις 9 Αύγουστος 2012 στις 12:56
Διαγραφή σχολίου
Πολύ καλό θέμα Μιχαήλ. Μπράβο
Σχόλιο από τον/την Μιχαήλ Μιχαήλ στις 9 Αύγουστος 2012 στις 13:17
Διαγραφή σχολίου
Δημήτρη και Τίνα ευχαριστώ πολύ. Να περνάτε καλά!
Σχόλιο από τον/την Παναγόπουλος Γ. - Βουλδής Α. στις 9 Αύγουστος 2012 στις 14:07
Διαγραφή σχολίου
Μιχάλη ωραία άσκηση και στο κλίμα των εξετάσεων. Μια μικρή παρατήρηση μόνο, αν θες στη εκφώνηση πρόσθεσε τη λέξη ''λείο'' κεκλιμένο επίπεδο'' Πάντα φιλικά Καλή συνέχεια.... Παναγόπουλος Γ.
Σχόλιο από τον/την Μιχαήλ Μιχαήλ στις 9 Αύγουστος 2012 στις 14:16
Διαγραφή σχολίου
Φίλε Παναγόπουλε Γ. ευχαριστώ πολύ. Να΄σαι πάντα καλά!

Σχόλιο

Μια εξίσωση 3 φαινόμενα... εδώ Σχόλια
Σχόλιο από τον/την ΘΑΝΑΣΗΣ ΓΙΑΝΝΑΚΟΠΟΥΛΟΣ στις 28 Αύγουστος 2012 στις 15:07
Διαγραφή σχολίου
Ενδιαφέρον Μηχαήλ...Μπράβο
Σχόλιο από τον/την Τίνα Νάντσου στις 28 Αύγουστος 2012 στις 15:41
Διαγραφή σχολίου
Πολύ ενδιαφέρον Μιχαήλ.
Σχόλιο από τον/την Μιχαήλ Μιχαήλ στις 28 Αύγουστος 2012 στις 15:53
Διαγραφή σχολίου
Τίνα, Θανάση ευχαριστώ!
Σχόλιο από τον/την Φιορεντίνος Γιάννης στις 29 Αύγουστος 2012 στις 1:48
Διαγραφή σχολίου
Μιχάλη καλησπέρα. Πολύ όμορφη και ενδιαφέρουσα δουλειά. Μια πολύ χρήσιμη ανάρτηση για διαφορετικά φυσικά φαινόμενα που περιγράφονται από τα ίδια μαθηματικά. Στην αρχή παραξενεύτηκα λίγο που διάβασα τη λύση (με μία μόνο σταθερά) αλλά αμέσως είδα την παρατήρησή σου ότι "η εξίσωση έχει και τη λύση". Στη συνέχεια προσπάθησα να επεκτείνω στα 4 τα φαινόμενα, βάζοντας και την (χρονοανεξάρτητη) εξίσωση Klein - Gordon, αλλά βγαίνει ένα -Κ^2 (που μπορεί να θεωρηθεί σαν +Κ^2 αν το Κ είναι φανταστικός).
Σχόλιο από τον/την Διονύσης Μάργαρης στις 29 Αύγουστος 2012 στις 8:38
Διαγραφή σχολίου
Μπράβο Μιχαήλ και από μένα, ιδιαίτερα προφανώς για την τρίτη περίπτωση, που είναι και λίγο ...μακρινή!!!
Σχόλιο από τον/την Μιχαήλ Μιχαήλ στις 29 Αύγουστος 2012 στις 14:57
Διαγραφή σχολίου
Γιάννη, Διονύση ευχαριστώ! Να είστε πάντα καλά
Σχόλιο από τον/την Βαγγέλης Κουντούρης στις 29 Αύγουστος 2012 στις 15:06
Διαγραφή σχολίου
καλή και "συγκεντρωτική" δουλειά Μιχαήλ (θα πρότεινα να κάνεις d όλα τα Δ ή να προσθέσεις Δt --> 0)
Σχόλιο από τον/την Μιχαήλ Μιχαήλ στις 29 Αύγουστος 2012 στις 19:35
Διαγραφή σχολίου
Βαγγέλη σε ευχαριστώ για το σχόλιο!
41. Ταλάντωση αποχωρισμός και ανύψωση Τα σώματα Σ1, Σ2 και Σ3 του σχήματος έχουν μάζες m1=2Kg, m2=1Kg και m3=0,5Kg αντίστοιχα. Τα Σ1 και Σ2 είναι δεμένα στα άκρα του κατακόρυφου ελατηρίου σταθεράς Κ=150N/m. Αρχικά το σύστημα ισορροπεί. Να βρείτε: α) Ποια είναι η μέγιστη δύναμη F που μπορούμε να ασκήσουμε ώστε να μην αποχωριστούν τα Σ2 και Σ3 αλλά ούτε και να ανυψωθεί το Σ1 από το έδαφος; β) Για ποια τιμή της δύναμης F αποκολλάται το Σ1 από το έδαφος; γ) Αν ρίξουμε κόλλα ανάμεσα στα Σ2 και Σ3, τότε τι τιμή πρέπει να έχει η η δύναμη Fκ που ασκεί η κόλλα στα δυο σώματα, ώστε τη στιγμή που το Σ1 μόλις ανυψώνεται από το έδαφος να αποχωρίζεται και το σώμα Σ3 από το Σ2; Δίνεται g=10m/s2 .   Συνοπτική λύση: 42. ΚΥΚΛΩΜΑ E-R-L-C Α)Στο κύκλωμα του διπλανού σχήματος δίνονται: Ε=3V, R=10Ω, L=10-2H και C=10-4F. Κάποια χρονική στιγμή μεταφέρουμε το διακόπτη στη θέση (1). Να βρείτε: α) Την τελική τιμή Ι0 της έντασης του ηλεκτρικού ρεύματος στο κύκλωμα. β) Το ρυθμό μεταβολής , της έντασης του ηλεκτρικού ρεύματος στο πηνίο τη στιγμή t1, που αυτή γίνεται ίση με i=Ι0/2 γ) το ρυθμό με τον οποίο αποθηκεύεται ενέργεια στο μαγνητικό πεδίο του πηνίου τη στιγμή t1. Β) Τη χρονική στιγμή t0=0 που ένταση του ρεύματος γίνεται i=Ι0/2, μεταφέρουμε ακαριαία το διακόπτη στη θέση (2). Να βρείτε: α) Το ρυθμό μεταβολής της έντασης στο πηνίο εκείνη τη στιγμή. β) Την ένταση του ηλεκτρικού ρεύματος τη χρονική στιγμή t2, που ο ρυθμός μεταβολής της , γίνεται ίσος με αυτόν που υπολογίσατε τη χρονική στιγμή t1. γ) Το ρυθμό μεταβολής της ενέργειας στο μαγνητικό πεδίο του πηνίου τη χρονική στιγμή t2. Συνοπτική λύση: 43. ΚΥΚΛΩΜΑ L-C και 3 ρυθμοί μεταβολής Πυκνωτής χωρητικότητας C=8μF φορτίζεται με τάση V0=20V. Κατόπιν συνδέεται με πηνίο συντελεστή αυτεπαγωγής L=20mH. Ακόμη το πηνίο αποτελείται από Ν=1000 σπείρες με εμβαδό εγκάρσιας διατομής η καθεμία S=10-4m2, ενώ η απόσταση μεταξύ των οπλισμών του πυκνωτή είναι ℓ=1mm. Τη χρονική στιγμή t0=0 κλείνουμε το διακόπτη. Τότε τη στιγμή t=4π/3×10-4s, να βρείτε: α) Το ρυθμό μεταβολής της έντασης του ηλεκτρικού πεδίου του πυκνωτή. β) Το ρυθμό μεταβολής της μαγνητικής ροής του μαγνητικού πεδίου του πηνίου . γ) Το ρυθμό μεταβολής της έντασης του μαγνητικού πεδίου του πηνίου . Συνοπτική λύση στο προσωπικό blog: 48. ΕΞΑΝΑΓΚΑΣΜΕΝΗ ΤΑΛΑΝΤΩΣΗ Στη διάταξη του σχήματος δίνονται η σταθερά του ελατηρίου K=100N/m και ότι η μάζα του σώματος Σ είναι m=4Kg. Το χέρι μας ασκεί περιοδική δύναμη F, και το σώμα Σ εκτελεί εξαναγκασμένη αρμονική ταλάντωση συχνότητας f1=4/2πHz και πλάτους Α=4,4cm χωρίς αρχική φάση. Το σώμα κινούμενο δέχεται δύναμη αντίστασης Fαντ= -b×υ με σταθερά απόσβεσης b=0,4Kg×s-1. α) Να γράψετε τις σχέσεις της απομάκρυνσης και της ταχύτητας του ταλαντωτή σε συνάρτηση με το χρόνο. β) Να γράψετε την εξίσωση της δύναμης F του διεγέρτη σε συνάρτηση με το χρόνο. γ) Να υπολογίσετε τη δύναμη του διεγέρτη τη χρονική στιγμή t=π/12s, καθώς και το ρυθμό προσφερόμενης ενέργειας εκείνη τη στιγμή. δ) Να γράψετε την εξίσωση της δύναμης F του διεγέρτη σε συνάρτηση με το χρόνο όταν έχουμε συντονισμό και να υπολογίσετε το ρυθμό προσφερόμενης ενέργειας τη στιγμή t= π/15s.   Συνοπτική λύση: Σχόλια
Σχόλιο από τον/την Κορφιάτης Ευάγγελος στις 14 Οκτώβριος 2012 στις 23:44
Διαγραφή σχολίου
Καλησπέρα Μιχάλη. Πολύ καλή άσκηση, η οποία μελετά ποσοτικά της εξαναγκασμένη ταλάντωση. Κάποια σχόλια 1. Η δύναμη του διεγέρτη σου προκύπτει αρχικά ως F=F1ημ(ωt)+F2συν(ωt) Στην προσπάθειά σου να την φέρεις στην μορφή F=F0ημ(ωt+φ) χρησιμοποιείς τριγωνομετρία που οι μαθητές δεν γνωρίζουν. Οι τργωνομετρικοί αριθμοί αθροίσματος δύο τόξων είναι εκτός ύλης. Ως εναλακτική λύση θα πρότεινα το σημείο αυτό να το αντιμετωπίσεις ως σύνθεση ταλαντώσεων. F=F1ημ(ωt)+F2συν(ωt)=F1ημ(ωt)+F2ημ(ωt+π/2). Στην συνέχεια εφαρμογή των ουρανοκατέβατων σχέσεων του σχολικού. 2. Μήπως μόνοι μας ανεβάζουμε τον πήχυ πολύ ψηλά;
Σχόλιο από τον/την Σαράντος Οικονομίδης στις 14 Οκτώβριος 2012 στις 23:51
Διαγραφή σχολίου
Καλησπέρα και από μένα Μιχάλη. Ναι συμφωνώ με τον Ευάγγελο. Είναι για καθηγητές ή ΥΨΙΜ φαντάζομαι :)
Σχόλιο από τον/την Κορφιάτης Ευάγγελος στις 14 Οκτώβριος 2012 στις 23:58
Διαγραφή σχολίου
Σαράντο, επειδή δεν τα πάω καλά με τα ακρονύμια, τι σημαίνει ΥΨΙΜ;
Σχόλιο από τον/την Σαράντος Οικονομίδης στις 15 Οκτώβριος 2012 στις 0:04
Διαγραφή σχολίου
ΥΨηλών Ικανοτήτων Μάθησης (μαθητές)
Σχόλιο από τον/την Μιχαήλ Μιχαήλ στις 15 Οκτώβριος 2012 στις 21:14
Διαγραφή σχολίου
Βαγγέλη και Σαράντο ευχαριστώ κατά αρχάς για τα σχόλιά σας. Βαγγέλη έχεις δίκιο για το ότι οι τργωνομετρικοί αριθμοί αθροίσματος δύο τόξων είναι εκτός ύλης. Η άσκηση για να απαντήσω και στο Σαράντο είναι για το Ι.Τ.Α (Ίδρυμα Τριγωνομετρικής Αποκατάστασης). Να είστε πάντα καλά!
Σύνθεση ταλαντώσεων Σώμα Σ πραγματοποιεί α.α.τ με εξίσωση, x1=0,4×ημ(10t+5π/12)×συν(10t+5π/12) (S.I), γύρω από τη θέση ισορροπίας του. Τη χρονική στιγμή t=0 το Σ αρχίζει να πραγματοποιεί και μια εξαναγκασμένη ταλάντωση στην ίδια διεύθυνση γύρω από την ίδια θέση ισορροπίας με εξίσωση, x2=0,2×[συν2(10t+5π/12)- ημ2(10t+5π/12)] (S.I). Πως μεταβάλλεται με το χρόνο η επιτάχυνση του Σ1; Λύση: Σχόλια
Σχόλιο από τον/την Σταύρος Κουσίδης στις 15 Οκτώβριος 2012 στις 22:38
Διαγραφή σχολίου
Μιχάλη, νομίζω ότι η σύνθεση πρέπει να δώσει αατ με φάση ωt+13π/12. Εξάλλου την t = 0 για παράδειγμα, δε φαίνεται να ισχύει η x = x1 + x2.
Σχόλιο από τον/την Μιχαήλ Μιχαήλ στις 15 Οκτώβριος 2012 στις 23:30
Διαγραφή σχολίου
Σταύρο προφανώς και έχεις δίκιο. Σε ευχαριστώ!
ΚΥΚΛΩΜΑ L-C και πολικότητες ΚΥΚΛΩΜΑ L-C και πολικότητες Σχόλια
Σχόλιο από τον/την Πέτρος Καραπέτρος στις 27 Οκτώβριος 2012 στις 10:56
Διαγραφή σχολίου
Συγχαρητήρια Μιχαήλ. Πολύ διαφωτιστικό.
Σχόλιο από τον/την Μιχαήλ Μιχαήλ στις 27 Οκτώβριος 2012 στις 11:58
Διαγραφή σχολίου
Πέτρο καλημέρα και σ' ευχαριστώ πολύ! Συγχαρητήρια και σε σένα για τις πολύ καλές αναρτήσεις σου!
Σχόλιο από τον/την Μιχαήλ Μιχαήλ στις 27 Οκτώβριος 2012 στις 13:40
Διαγραφή σχολίου
Δημήτρη ευχαριστώ! Να' σαι πάντα καλά.
49. ΕΞΑΝΑΓΚΑΣΜΕΝΗ ΗΛΕΚΤΡΙΚΗ ΤΑΛΑΝΤΩΣΗ Η μαγνητική ροή που διέρχεται μέσα από το σωληνοειδές του σχήματος με N=100 σπείρες και L=5×10-2H καθώς ο μαγνήτης μετακινείται ως προς το πηνίο μεταβάλλεται με ρυθμό ΔΦ/Δt= -5×10-3× συν5t (S.I). Αν C=2×10-3F και R=1Ω τότε: α) Ποια είναι η περίοδος των εξαναγκασμένων ηλεκτρικών ταλαντώσεων και ποια είναι η ιδιοπερίοδός τους; β) Να γραφούν οι εξισώσεις με το χρόνο της τάσης VL στα άκρα του πηνίου και του ρυθμού μεταβολής της έντασης του ηλεκτρικού ρεύματος που διαρρέει το κύκλωμα, γ) Αν το μέγιστο ηλεκτρικό φορτίο στον πυκνωτή είναι Q=0,4C τότε να υπολογιστεί ο μέγιστος ρυθμός προσφερόμενης ενέργειας στο κύκλωμα.   Συνοπτική λύση: 51. ΑΡΜΟΝΙΚΟ ΚΥΜΑ Αρμονικό κύμα διαδίδεται προς την αρνητική κατεύθυνση του άξονα xox΄ με σταθερή ταχύτητα υ=10m/s. Τη χρονική στιγμή t=0 το κύμα βρίσκεται στη θέση xM=-0,15m. Η πηγή (Ο) του κύματος βρίσκεται στη θέση x0=+0,1m. Το πλάτος ταλάντωσης της πηγής είναι Α=5cm και η περίοδός της είναι T=0,02s. Τότε: α) Ποια είναι η εξίσωση του κύματος; β) Σε πόσο χρόνο Δt το κύμα διήνυσε την απόσταση (ΟΜ); γ) Να σχεδιάσετε το στιγμιότυπο του κύματος τη χρονική στιγμή t=0. δ) Να σχεδιάσετε τη φάση του σημείου Μ με το χρόνο t. ε) Να σχεδιάσετε τη γραφική παράσταση της φάσης του κύματος σε συνάρτηση με την απόσταση x από τη πηγή τη χρονική στιγμή t=0 και τη χρονική στιγμή t=T=0,02s. στ) Για το σημείο Μ να κάνετε τη γραφική παράσταση της ταχύτητας ταλάντωσης σε συνάρτηση με το χρόνο ζ) Αν τη χρονική στιγμή t=0, η πηγή (Ο), αρχίζει να απομακρύνεται από το σημείο (Μ) με ταχύτητα v=10m/s, τότε να βρεθεί η καινούργια εξίσωση του κύματος.   Συνοπτική λύση: Ερωτήσεις πολλαπλής επιλογής στο 1ο Κεφάλαιο (ΦΘΤΚ) Φυσική Θετικής- Τεχνολογικής κατεύθυνσης 1ο Κεφάλαιο. Ερωτήσεις Συμβολή _ Σύνθεση ταλαντώσεων Η εξίσωση συμβολής όπως και η εξίσωση του στάσιμου "κύματος" μπορεί να προκύψει σα σύνθεση δυο ταλαντώσεων Αρχή του HUYGENS Η αρχή Huygens – Fresnel εφαρμόστηκε για να εξηγήσει το φαινόμενο της περίθλασης στα φωτεινά κύματα. Μόνο που την εποχή εκείνη (1600-1700) , τα φωτεινά κύματα θεωρούνται μηχανικά κύματα μέσα σ’ έναν τελείως διαπερατό αιθέρα. Αργότερα ο Maxwell (1831 – 1879) , απέδειξε ότι το φως είναι εγκάρσια ΗΛΜ κύματα και ο Einstein ολοκλήρωσε τη σύγχρονη άποψη για το φως, παραμερίζοντας την ανάγκη για την παραδοχή του αιθέρα. Η αρχή του Huygens αναφέρεται στην κυματική και όχι τη σωματιδιακή φύση του φωτός η συνέχεια εδώ Σχόλια
Σχόλιο από τον/την Σαράντος Οικονομίδης στις 13 Νοέμβριος 2012 στις 21:37
Διαγραφή σχολίου
Μιχάλη καλησπέρα. Μου άρεσε πολύ η ανάρτησή σου. Ένα θέμα που λείπει... Να είσαι καλά.
Σχόλιο από τον/την Μιχαήλ Μιχαήλ στις 13 Νοέμβριος 2012 στις 21:49
Διαγραφή σχολίου
Σαράντο σε ευχαριστώ. Να ' σαι πάντα καλά. Συγχαρητήρια και για τις δικές σου, πολύ διαφωτιστικές αναρτήσεις.
Σχόλιο από τον/την Κορφιάτης Ευάγγελος στις 14 Νοέμβριος 2012 στις 20:05
Διαγραφή σχολίου
Καλησπέρα Μιχάλη. Πολύ καλή και κατατοπιστική η ανάρτηση. Με γύρισες μερικές δεκαετίες πίσω (Φυσική Λυκείου). Αυτό που δεν έχω ξεκαθαρίσει με την συγκεκριμένη αρχή είναι το εξής: Πρόκειται για εμπειρικό νόμο, που καταφέρνει να ερμηνεύσει με ενιαίο τρόπο ένα σύνολο από κυματικά φαινόμενα, ή συνάγεται από γενικότερες αρχές;
Σχόλιο από τον/την Φιορεντίνος Γιάννης στις 14 Νοέμβριος 2012 στις 20:31
Διαγραφή σχολίου
Καλησπέρα σε όλους. Μιχάλη πολύ διαφωτιστική η ανάρτησή σου. Συνοπτική και συνάμα εξόχως περιεκτική. Βαγγέλη, έχω την ίδια απορία με σένα. Από μια ματιά που έριξα στη Wikipedia, αναφέρει ότι η αρχή του Huygens, μπορεί να θεωρηθεί σαν συνέπεια (consequence) της ισοτροπίας του χώρου.
Σχόλιο από τον/την Διονύσης Μάργαρης στις 14 Νοέμβριος 2012 στις 20:40
Διαγραφή σχολίου
Συγχαρητήρια και από μένα Μιχάλη. Πράγματι είναι μια αρχή που την έχουμε εγκαταλείψει τα τελευταία; χρόνια και δεν την διδάσκουμε, ενώ νομίζω ότι είναι άκρως διδακτική.
Σχόλιο από τον/την Μιχαήλ Μιχαήλ στις 14 Νοέμβριος 2012 στις 20:51
Διαγραφή σχολίου
Βαγγέλη και Γιάννη ευχαριστώ για τα καλά σας σχόλια. Βαγγέλη έχω και εγώ την εντύπωση ότι η η αρχή Huygens είναι μια απλή γεωμετρική κατασκευή που δεν έχει κάποια απόδειξη. Διάβασα όμως από σχετικό βιβλίο ότι ο Fresnel τη συνδύασε με τη συμβολή των κυμάτων και αργότερα οι Fresnel και Kirchhoff (1882), έδειξαν ότι η παραπάνω αρχή είναι άμεση συνέπεια της διαφορικής εξίσωσης του κύματος.
Σχόλιο από τον/την Φιορεντίνος Γιάννης στις 14 Νοέμβριος 2012 στις 20:52
Διαγραφή σχολίου
Δεν ξέρω αν απαντάει στο ερώτημα που έθεσε ο Βαγγέλης, αλλά παλιότερα είχα κάνει μια εργασία για την εξίσωση του κύματος σε n χωρικές διαστάσεις, όπου φαίνεται ότι για n=περιττός ισχύει η αρχή ενώ για n=άρτιος δεν ισχύει (το κύμα παρουσιάζει τις λεγόμενες "ουρές")
Σχόλιο από τον/την Μιχαήλ Μιχαήλ στις 14 Νοέμβριος 2012 στις 20:59
Διαγραφή σχολίου
Διονύση ευχαριστώ για το σχόλιό σου. Γράφαμε μαζί! Πιστεύω και εγώ στο διδακτικό χαρακτήρα της αρχής και προσπαθώ κάπως να την εντάξω στο μάθημα (ειδικά για το νόμο του Snell), αλλά ... Να' σαι πάντα καλά.
Σχόλιο από τον/την Κορφιάτης Ευάγγελος στις 14 Νοέμβριος 2012 στις 23:13
Διαγραφή σχολίου
Γιάννη καλησπέρα. Ξαναδιάβασα την παλαιότερη σημαντική δουλειά σου για την εύρεση σφαιρικά συμμετρικής λύσης της κυματικής εξίσωσης σε n χωρικές διασστάσεις. Στην μελέτη αυτή συσχετίζεις την ισχύ ή όχι της αρχής του Huygens με το αν η διάδοση γίνεται η όχι στον κώνο φωτός. Από που προκύπτει αυτός ο συσχετισμός; Ο συσχετισμός αυτός απαντά στο ερώτημά μου.
Σχόλιο από τον/την Φιορεντίνος Γιάννης στις 15 Νοέμβριος 2012 στις 0:45
Διαγραφή σχολίου
Καλησπέρα Βαγγέλη. Θα προσπαθήσω να δώσω μια απάντηση στο ερώτημά σου. Σύμφωνα με την αρχή του Huygens, κάθε σημείο του μετώπου του κύματος (τη χρονική στιγμή t) μπορεί να θεωρηθεί σαν σημειακή πηγή δευτερεύουσας σφαιρικής διαταραχής που ταξιδεύει με την ταχύτητα του φωτός. Η περιβάλλουσα όλων αυτών των δευτερευουσών διαταραχών (που διαδίδονται με την ίδια ταχύτητα όπως και το αρχικό κύμα, δηλαδή την ταχύτητα του φωτός) μετά χρόνο Δt δίνει το νέο μέτωπο του κύματος για τη χρονική στιγμή t + Δt. Για n=3 (ή γενικώτερα περιττό), έχουμε σα λύση της κυματικής εξίσωσης μια διαταραχή που κινείται με την ταχύτητα του φωτός. Έτσι κάθε «δευτερογενής κύμανση» (κινούμενη με την ίδια ταχύτητα όπως και το αρχικό κύμα), παριστά (κατά κάποιο τρόπο) ένα κώνο φωτός στο σημείο, και η αρχή του Huygens μας λέει ότι το φως βρίσκεται σ’ αυτό τον κώνο φωτός. Τώρα αν n=2 γενικώτερα άρτιο), η λύση δεν είναι πια ένα απλό σφαιρικό κύμα που διαδίδεται με ταχύτητα c. Αντίθετα βρίσκουμε ότι υπάρχουν άπειρες λύσεις με όλες τις δυνατές ταχύτητες με τιμή από c μέχρι μηδέν. Έτσι ένας αρχικός παλμός σε κάποιο σημείο Α θα διαδίδεται με άπειρους κώνους φωτός με ταχύτητες από την τιμή c μέχρι μηδέν. Κάποιος υποθετικός παρατηρητής σε κάποιο άλλο σημείο θα έβλεπε τον αρχικό παλμό, αλλά στη συνέχεια θα εξακολουθούσε να βλέπει «φως» (διαρκώς μειούμενης έντασης)να έρχεται από το σημείο Α.
Σχόλιο από τον/την ΓΙΑΝΝΗΣ ΔΟΓΡΑΜΑΤΖΑΚΗΣ στις 19 Νοέμβριος 2012 στις 15:59
Διαγραφή σχολίου
Μιχαήλ πολύ καλή η εργασία σου... Μια σημαντική αρχή που έπρεπε να διδάσκεται στην Γ΄Λ. Να΄σαι καλά Μιχαήλ.
Σχόλιο από τον/την Μιχαήλ Μιχαήλ στις 19 Νοέμβριος 2012 στις 20:56
Διαγραφή σχολίου
Γιάννη σ΄ευχαριστώ. Να ' σαι πάντα καλά.
54. ΣΥΝΘΕΣΗ ΗΛΕΚΤΡΙΚΩΝ ΤΑΛΑΝΤΩΣΕΩΝ Το κύκλωμα RLC του σχήματος όταν τροφοδοτείται από μια πηγή εναλλασσόμενης τάσης V1(t) τότε το φορτίο του πυκνωτή μεταβάλλεται σύμφωνα με τη σχέση, q1=10-3ημ1000t ενώ όταν τροφοδοτείται από μια πηγή εναλλασσόμενης τάσης V2(t) τότε το φορτίο του πυκνωτή μεταβάλλεται σύμφωνα με τη σχέση, q2=10-3ημ(1000t+π/3) (S.I). Αν συνδέσουμε σε σειρά τις δυο πηγές V1 και V2 ώστε το ηλεκτρικό φορτίο του πυκνωτή να πραγματοποιεί ταυτόχρονα τις δυο εξαναγκασμένες ταλαντώσεις , τότε ποια είναι η εξίσωση του φορτίου του πυκνωτή στο κύκλωμα;   Συνοπτική λύση 57. Στιγμιότυπο κύματος Η πηγή Ο (x=0), μιας διαταραχής πραγματοποιεί α.α.τ με εξίσωση y=0,2×ημ10πt. Το κύμα διαδίδεται προς το +x με ταχύτητα διάδοσης υ=10m/s. Τη χρονική στιγμή t=0,2s το πλάτος της ταλάντωσης της πηγής γίνεται Α΄=0,1m. Τότε: α) να σχεδιάσετε το στιγμιότυπο του κύματος τη χρονική στιγμή t1=0,4s, β) να σχεδιάσετε την απομάκρυνση ενός σημείου Σ του μέσου που απέχει απόσταση x=4m από την πηγή Ο σε συνάρτηση με το χρόνο, γ) Αν δυο σημεία Β και Γ απέχουν από την πηγή Ο απόσταση xB=5λ/4 και xΓ=λ/4, τότε να υπολογίσετε, i) τη διαφορά φάσης ΔφΓΒ των δυο σημείων την ίδια χρονική στιγμή και εφόσον έχει φτάσει σε αυτά η διαταραχή και ii) το λόγο των ταχυτήτων ταλάντωσης τη χρονική στιγμή t1=5T/4 και τη χρονική στιγμή t2=7T/4, δ) Για μια τυχαία χρονική στιγμή t με 5T/4<t<2T, να υπολογίσετε την απόσταση d μεταξύ των δυο σημείων Β και Γ. Πόση γίνεται η απόσταση αυτή τη χρονική στιγμή t=2T; ε) Κάποια χρονική στιγμή που τη θεωρούμε πλέον αρχή των χρόνων (t=0) και ενώ η πηγή ταλαντώνεται με εξίσωση y=Α΄×ημ10πt το πλάτος της Α΄ αρχίζει να μειώνεται εκθετικά με το χρόνο σύμφωνα με τη σχέση Α΄=0,1×e-2t, τότε να σχεδιάσετε το στιγμιότυπο του κύματος τη χρονική στιγμή t=T=0,2s. Δίνεται e-0,1=0,9. Συνοπτική Λύση Σχόλια
Σχόλιο από τον/την Διονύσης Μάργαρης στις 6 Δεκέμβριος 2012 στις 13:23
Διαγραφή σχολίου
Ωραία ιδέα. Μπράβο Μιχαήλ!
Σχόλιο από τον/την ΓΙΑΝΝΗΣ ΔΟΓΡΑΜΑΤΖΑΚΗΣ στις 6 Δεκέμβριος 2012 στις 13:47
Διαγραφή σχολίου
Ναι Μιχαήλ...συμφωνώ με τον Διονύση. Μπράβο.
Σχόλιο από τον/την Νίκος Ανδρεάδης στις 6 Δεκέμβριος 2012 στις 14:55
Διαγραφή σχολίου
Πολύ καλή ιδέα Μιχαήλ. Στο ε, να γίνει διόρθωση στην εξίσωση κύματος.
Σχόλιο από τον/την Μιχαήλ Μιχαήλ στις 6 Δεκέμβριος 2012 στις 20:52
Διαγραφή σχολίου
Μανώλη , Γιάννη , Νίκο ευχαριστώ! Νίκο Χρόνια Πολλά.
Σχόλιο από τον/την Μιχαήλ Μιχαήλ στις 6 Δεκέμβριος 2012 στις 22:04
Διαγραφή σχολίου
Διονύση (Μαρ) σ' ευχαριστώ!
58. Τριβή κύλισης Κύλινδρος βάρους w=35N και ακτίνας R=0,6m βρίσκεται πάνω σε οριζόντιο δάπεδο. Να υπολογιστεί η ελάχιστη δύναμη F, που πρέπει να εφαρμόσουμε στον άξονα Κ του κυλίνδρου ώστε αυτός να κυλίεται με σταθερή ταχύτητα. Πειραματικό δεδομένο: Σε ένα σημείο Ζ της επιφάνειας του κυλίνδρου στερεώνουμε αβαρές μη εκτατό νήμα, στα άκρα του οποίου τοποθετούνται δυο ίσα βάρη Β=2Ν και ο κύλινδρος ισορροπεί. Αν στη συνέχεια στο ένα από τα δυο ίσα βάρη τοποθετήσουμε ένα επιπρόσθετο βάρος β=1Ν, τότε επιτυχαίνουμε την οριακή κατάργηση της ισορροπίας του κυλίνδρου. Συνοπτική Λύση Σχόλια
Σχόλιο από τον/την Κυριακόπουλος Γιάννης στις 22 Δεκέμβριος 2012 στις 11:02
Διαγραφή σχολίου
Μπράβο πολύ πρωτότυπη.
Σχόλιο από τον/την Μιχαήλ Μιχαήλ στις 22 Δεκέμβριος 2012 στις 13:01
Διαγραφή σχολίου
Γιάννη ευχαριστώ!
Σχόλιο από τον/την Κορφιάτης Ευάγγελος στις 23 Δεκέμβριος 2012 στις 21:50
Διαγραφή σχολίου
Καλησπέρα Μιχάλη και ΧΡΟΝΙΑ ΠΟΛΛΑ. Το μόνο που ήξερα για την τριβή κύλισης είναι ότι είναι ροπή και το πηλίκο της τριβής κύλισης προς την Ν είναι η παράλληλη μετατόπιση της Ν . Από την ανάρτησή σου έμαθα ότι το πηλίκο αυτό είναι ο συντελεστής τριβής κύλισης Προφανώς η φράση « Η τριβή κύλισης είναι μικρότερη από την οριακή στατική τριβή» δεν έχει νόημα. Η μια είναι ροπή και ή άλλη δύναμη. Για να υπερνικηθεί η οριακή στατική τριβή απαιτείται δύναμη F≥μsN Για να υπερνικήθεί η τριβή κύλισης (αναφέρομαι στο σχήμα σου) απαιτείται δύναμη F≥NL/R. Συμπεραίνω λοιπόν ότι για τα περισσότερα υλικά και συνήθεις ακτίνες ισχύει ότι μsN≥ NL/R Ας υποθέσουμε λοιπόν ότι σε ένα αρχικά ακίνητο κύλινδρο ασκούμε μια δύναμη F< NL/R. Υποθέτω ότι στην περίπτωση αυτή ο κύλινδρος θα παραμείνει ακίνητος. Από τις συνθήκες ισορροπίας έχουμε ότι Ν=w , F=T και ροπή της T (ως προς το κέντρο)=ροπή της Ν (ως προς το κέντρο). Άρα (η ροπή της Ν) < ΝL. Θέτω λοιπόν το εξής ερώτημα: Έστω Ν η κάθετη στην επιφάνεια συνιστώσα της δύναμης επαφής και L ο συντελεστής τριβής κύλισης. Μήπως εν γένει δεν ισχύει ισότητα αλλά ανισότητα; Δηλαδή αν τ η τριβή κύλισης τότε τ <ΝL. Μήπως δηλαδή NL είναι η οριακή τριβή κύλισης; Και ένα βήμα παραπέρα: Μήπως (σε αντιστοιχία με την δύναμη τριβής) όπως υπάρχει ο συντελεστής τριβής ολίσθησης και ο συντελεστής οριακής στατικής τριβής υπάρχουν και δύο συντελεστές τριβής κύλισης; Ο συντελεστής τριβής κύλισης ( όταν η κύλιση είναι γεγονός) και ο συντελεστής οριακής τριβής κύλισης ( όταν επίκειται η κύλιση αλλά δεν έχει ακόμη πραγματοποιηθεί;)
Σχόλιο από τον/την Μιχαήλ Μιχαήλ στις 23 Δεκέμβριος 2012 στις 22:56
Διαγραφή σχολίου
Βαγγέλη καλησπέρα και από μένα και Χρόνια Πολλά! Για να έχουμε ισορροπία πρέπει να ισχύει η ισότητα! Βέβαια για κάθε δύναμη F μικρότερη της ¨οριακής¨ τιμής της F δε θα έχουμε κύλιση, πάντως .......... κάτι γίνεται και με την οριζόντια συνιστώσα της αντίδρασης Α.
Σχόλιο από τον/την Διονύσης Μάργαρης στις 24 Δεκέμβριος 2012 στις 10:00
Διαγραφή σχολίου
Μιχαήλ, χρόνια πολλά, με υγεία προκοπή και διάθεση προσφοράς. Ωραίο το θέμα που αναδεικνύεις, έξω από τα συνηθισμένα. Σε ευχαριστούμε.
Σχόλιο από τον/την Μιχαήλ Μιχαήλ στις 24 Δεκέμβριος 2012 στις 10:28
Διαγραφή σχολίου
Διονύση ΧΡΟΝΙΑ ΠΟΛΛΑ με υγεία και αγάπη. Να σαι πάντα καλά και να μας εμπνέεις με την προσφορά σου!
Σχόλιο από τον/την Μιχαήλ Μιχαήλ στις 6 Ιανουάριος 2013 στις 14:00
Διαγραφή σχολίου
Γιάννη καλημέρα. Ο κύλινδρος εξαιτίας της παραμόρφωσης εφάπτεται κατά μία επιφάνεια ΕΓ και όχι μόνο κατά τη γραμμή που περνάει από το Ε. Η εφαρμογή της δύναμης F έχει ως αποτέλεσμα η αντίδραση Α του δαπέδου να εφαρμόζεται στο σημείο Γ, που δέχεται μεγαλύτερη πίεση από το Ε. Έτσι η εφαρμογή της δύναμης F στον άξονα του κυλίνδρου, έχει σαν αποτέλεσμα την ανάπτυξη τριβής κύλισης T=Αχ με τιμή ίση προς την F (T=F), που εμποδίζει την ολίσθηση του κυλίνδρου. Στον κύλινδρο λοιπόν ασκούνται η δύναμη F, το βάρος του w και η αντίδραση Α από το δάπεδο. Για την ισορροπία του κυλίνδρου ισχύει ότι η Α είναι ίση και αντίθετη προς τη συνισταμένη των δυνάμεων w και F. Έστω ότι L είναι η απόσταση ΓΕ και N είναι η κατακόρυφη συνιστώσα της αντίδρασης Α. Ισχύει Ν=w. Στην περιστροφή του κυλίνδρου αντιδρά το ζεύγος (w,N), η ροπή του οποίου N×L πρέπει να υπερνικηθεί από τη ροπή του ζεύγους (F,T) που είναι ίση με F×R. (Τ ονομάζω την οριζόντια συνιστώσα Αχ της αντίδρασης Α). Άρα για να εξασφαλιστεί η κύλιση του κυλίνδρου πρέπει να υπερνικηθεί η ροπή κύλισης δηλαδή πρέπει Ν×L=F×R.
Σχόλιο από τον/την Κορφιάτης Ευάγγελος στις 6 Ιανουάριος 2013 στις 14:33
Διαγραφή σχολίου
Καλημέρα Μιχάλη και ΚΑΛΗ ΧΡΟΝΙΑ. Νομίζω ότι στην προηγούμενη παρέμβασή σου μας μπέρδεψες. Λες ότι η η Τ=Αx ( bold γράμματα) είναι η τριβή κύλισης. Η τριβή κύλισης είναι η ροπή της Α ως προς Δ. Η Αχ είναι κανονική δύναμη τριβής, η οποία μπορεί να είναι στατική ή ολισθήσεως. Κάνω λάθος;
Σχόλιο από τον/την Μιχαήλ Μιχαήλ στις 6 Ιανουάριος 2013 στις 16:22
Διαγραφή σχολίου
Ευάγγελε (Κορφ) , στο βιβλίο των Καρακώστα - Κυριάκου (ΑΠΘ) την είδα γραμμένη σαν τριβή κύλισης, που είναι το αποτέλεσμα της μη ελαστικής παραμόρφωσης! του σώματος και της επιφάνειας.
Σχόλιο από τον/την Μιχαήλ Μιχαήλ στις 6 Ιανουάριος 2013 στις 16:44
Διαγραφή σχολίου
Ναι Γιάννη τη θεωρώ περίπου ίση με την R.
59. Ισορροπία πλάκας Η τετράγωνη πλάκα του σχήματος πλευράς α έχει βάρος w=50N. Το σχοινί ΑΖ είναι αβαρές και έχει μήκος L. Η πλάκα ισορροπεί και το σχοινί ΑΖ στην προέκτασή του περνάει από το κέντρο Κ της πλάκας ενώ τέμνει τη μια πλευρά της πλάκας σε απόσταση α/4 από το μέσο της Μ, όπως φαίνεται στο σχήμα. α) Να υπολογίσετε τη δύναμη που ασκεί ο τοίχος στην πλάκα, β) να υπολογίσετε την τάση του σχοινιού. γ) Αν θέλουμε η δύναμη από τον τοίχο να είναι μικρότερη, θα πρέπει να χρησιμοποιήσουμε κοντύτερο ή μακρύτερο σχοινί αν θέλουμε αυτό να περνάει από το σημείο Κ; δ) Να υπολογίσετε την τάση του σχοινιού αν το σχοινί ΑΖ στην προέκτασή του περνάει από το κέντρο Κ της πλάκας ενώ τέμνει τη μια πλευρά της πλάκας σε απόσταση α/8 από το μέσο της Μ. θεωρείστε ότι ο τοίχος είναι λείος. Συνοπτική Λύση Σχόλια
Σχόλιο από τον/την Μιχαήλ Μιχαήλ στις 29 Δεκέμβριος 2012 στις 0:55
Διαγραφή σχολίου
Γιάννη σ' ευχαριστώ. Σου εύχομαι χρόνια πολλά με υγεία. Έχεις δίκιο για τις ροπές ως προς το Ζ.
Σχόλιο από τον/την ΓΙΑΝΝΗΣ ΔΟΓΡΑΜΑΤΖΑΚΗΣ στις 29 Δεκέμβριος 2012 στις 8:04
Διαγραφή σχολίου
Mιχαήλ καλημέρα και Χρόνια Πολλά...Πολύ ωραία η πρότασή σου.
Σχόλιο από τον/την Κυριακόπουλος Γιάννης στις 29 Δεκέμβριος 2012 στις 14:42
Διαγραφή σχολίου
Όμορφη Μιχαήλ. Μια προσθήκη: Ο τοίχος δεν είναι λείος κατ΄ανάγκην. Θα έλεγα ότι ο τοίχος θα μπορούσε να είναι και λείος. Δια του λόγου το αληθές:
Σχόλιο από τον/την Μιχαήλ Μιχαήλ στις 29 Δεκέμβριος 2012 στις 22:17
Διαγραφή σχολίου
Γιάννη (Δογρ) και Γιάννη (Κυρ) σας ευχαριστώ! ΧΡΟΝΙΑ ΠΟΛΛΑ με υγεία.
Σχόλιο από τον/την Διονύσης Μάργαρης στις 30 Δεκέμβριος 2012 στις 0:29
Διαγραφή σχολίου
Καλησπέρα Μιχαήλ και Χρόνια Πολλά. Πρωτότυπη και ενδιαφέρουσα η ανάρτησή σου, αλλά μετά από μια μεγάλη συζήτηση, που είχαμε πριν ακριβώς ένα χρόνο, μπορείς να την δεις εδώ, διατηρώ επιφυλάξεις, για το αν το θέμα είναι τόσο απλό και αν η δύναμη στήριξης είναι οριζόντια, αλλά και αν περνά και από το κέντρο της πλάκας. Η συζήτηση αυτή Γιάννη, με έπεισε ότι και το i.p. δεν κάνει και τόσο σωστή αναπαράσταση της πραγματικότητας.... Γιατί για παράδειγμα το σημείο που ασκείται η δύναμη από τον τοίχο, να μην βρίσκεται πιο ψηλά από το μέσον της πλάκας και να ασκείται και τριβή, οπότε η συνισταμένη δύναμη να περνά από το κέντρο της πλάκας; Πώς το αποκλείουμε;
Σχόλιο από τον/την Μιχαήλ Μιχαήλ στις 30 Δεκέμβριος 2012 στις 2:13
Διαγραφή σχολίου
Διονύση, καλησπέρα και ΧΡΟΝΙΑ ΠΟΛΛΑ. Τη θυμάμαι τη συζήτηση και με αφορμή αυτή προσπάθησα να ξαναθέσω το ζήτημα της δύναμης στήριξης. Έχεις δίκιο για το ότι το θέμα δεν είναι απλό.
Σχόλιο από τον/την Κυριακόπουλος Γιάννης στις 30 Δεκέμβριος 2012 στις 11:37
Διαγραφή σχολίου
Μιχαήλ Χρόνια Πολλά. Διονύση Χρόνια Πολλά και να περνάς καλά στην Εσπερία. Έχεις δίκιο. Εδώ ειδικά αφού οι δυνάμεις είναι 3 και πρέπει να περνάνε από το ίδιο σημείο πρέπει η δύναμη του τοίχου να περνάει από το Κ. Θα μπορούσε επομένως να υπάρχει τριβή και Ν ώστε η συνισταμένη τους να περνάει από το Κ. Θα έπρεπε να δηλώνεται από την αρχή ο τοίχος ως λείος. Το i.p. Στην αρχή για μερικά δευτερόλεπτα έδειξε άνισες Ν και ισορρόπησε εκεί που βλέπεις.
Σχόλιο από τον/την Διονύσης Μάργαρης στις 30 Δεκέμβριος 2012 στις 12:38
Διαγραφή σχολίου
Χρόνια πολλά Γιάννη. Συμφωνώ. Αν πούμε ότι ο τοίχος είναι λείος, δεν υπάρχει πλέον πρόβλημα.
Σχόλιο από τον/την Μιχαήλ Μιχαήλ στις 30 Δεκέμβριος 2012 στις 12:52
Διαγραφή σχολίου
Διονύση και Γιάννη καλημέρα. Έχετε δίκιο. Είχα κάνει την αλλαγή θεωρώντας από την αρχή τον τοίχο λείο!
60. Ισορροπία πλάκας _2 1. Η τετράγωνη πλάκα του σχήματος πλευράς α έχει βάρος w=50N. Το σχοινί ΑΖ είναι αβαρές και έχει μήκος L. Η πλάκα ισορροπεί και το σχοινί ΑΖ στην προέκτασή του περνάει από το κέντρο Κ της πλάκας ενώ τέμνει τη μια πλευρά της πλάκας σε απόσταση α/4 από το μέσο της Μ, όπως φαίνεται στο σχήμα. Η πλάκα ισορροπεί στηριζόμενη πάνω σε δυο τοίχους, όπου πάνω στον ένα στηρίζεται με μήκος α/4 και πάνω στον άλλο με μήκος α/8. α) Να υπολογίσετε τις δυνάμεις που ασκούν οι δυο τοίχοι στην πλάκα, β) να υπολογίσετε την τάση του σχοινιού. Συνοπτική Λύση Σχόλια
Σχόλιο από τον/την ΜΑΝΩΛΗΣ ΔΡΑΚΑΚΗΣ στις 26 Μάρτιος 2013 στις 21:31
Διαγραφή σχολίου
Μιχάλη καλησπέρα. Πολύ καλή ιδέα αυτή με τα τέσσερα σημεία. Μπράβο!
Σχόλιο από τον/την Μιχαήλ Μιχαήλ στις 26 Μάρτιος 2013 στις 22:38
Διαγραφή σχολίου
Μανώλη σε ευχαριστώ! Να 'σαι πάντα καλά!
81. Τροχός που «πέφτει» και μάζες που ολισθαίνουν

81. Τροχός που «πέφτει» και μάζες που ολισθαίνουν

Γύρω από τον ομογενή τροχό του σχήματος μάζας Μ=4Κg και ακτίνας R, είναι τυλιγμένο πολλές φορές ένα αβαρές νήμα. Το ένα ελεύθερο άκρο του νήματος μέσω αβαρούς τροχαλίας δένεται με σώμα μάζας m1=1Kg και το άλλο ελεύθερο άκρο του νήματος μέσω αβαρούς τροχαλίας δένεται με σώμα μάζας m2=2Kg. Τα σώματα m1 και m2 μπορούν να ολισθαίνουν πάνω σε οριζόντιο επίπεδο. Αφήνουμε τον τροχό ελεύθερο να κινηθεί. Α) Αν η m1 ολισθαίνει χωρίς τριβές, τότε για ποιες τιμές της στατικής τριβής ανάμεσα στο οριζόντιο επίπεδο και τη μάζα m2, η m2 δεν ολισθαίνει; (παραμένει ακίνητη). Β) Αν ο συντελεστής τριβής ολίσθησης ανάμεσα στο οριζόντιο επίπεδο και τη μάζα m2 γίνει μ=0,5, τότε να υπολογιστεί η επιτάχυνση της m2. Γ) Αν και τα δυο σώματα m1 και m2 ολισθαίνουν χωρίς τριβές πάνω στο οριζόντιο επίπεδο, τότε να υπολογιστούν: α) Οι επιταχύνσεις των m1 και m2, β) οι τάσεις των νημάτων, γ) η κινητική ενέργεια του συστήματος όταν ο κύλινδρος «πέσει» κατά xcm=0,42m, δ) ο ρυθμός μεταβολής της κινητικής ενέργειας των m1,m2 και Μ για xcm=0,42m. Δίνεται για τον τροχό Ιcm=M×R2 και g=10m/s2.   Συνοπτική λύση: 82. Μη μετωπική ελαστική κρούση m1 και m2 -2 Μια σφαίρα μάζας m1=5Kg κινείται (ολισθαίνει) οριζόντια με ταχύτητα υ=1m/s και συγκρούεται μη μετωπικά και ελαστικά με αρχικά ακίνητη σφαίρα μάζας m2=7Kg. Αν μετά την κρούση η σφαίρα m1 κινείται με ταχύτητα υ1 που σχηματίζει με την οριζόντια διεύθυνση γωνία θ1=600, τότε να υπολογίσετε την ταχύτητα υ1 καθώς και την ταχύτητα υ2 της m2, μετά την κρούση. Τριβές δεν υπάρχουν. Συνοπτική λύση: ή και εδώ 83. Κρούση 3 σφαιρών Από Halliday - Resnick Διαθέτουμε τρεις όμοιες λείες σφαίρες Σ, Σ1 και Σ2, μάζας m η καθεμία που βρίσκονται πάνω σε λείο οριζόντιο δάπεδο. Οι σφαίρες Σ1 και Σ2 εφάπτονται αρχικά όπως φαίνεται στο σχήμα. Η σφαίρα Σ κινείται αρχικά με οριζόντια ταχύτητα υ0=5m/s, η οποία είναι κάθετη στη διάκεντρο των Σ1 και Σ2. Όλες οι κρούσεις θεωρούνται ελαστικές. Να υπολογιστούν οι ταχύτητες των τριών σφαιρών μετά την κρούση.   Συνοπτική λύση: 85. Πλάγια κρούση σε κεκλιμένο επίπεδο Σφαίρα Σ σχετικά μικρής μάζας m=0,04Kg, προσπίπτει σε λείο κεκλιμένο επίπεδο γωνίας κλίσης φ=600. Η σφαίρα αρχικά κινείται οριζόντια, παράλληλα στη βάση του κεκλιμένου επιπέδου με σταθερή ταχύτητα, μέτρου υ=100m/s. Αν η κρούση της σφαίρας με το κεκλιμένο επίπεδο είναι ελαστική και ο χρόνος επαφής της μ’ αυτό είναι Δt=10-2 s, να βρείτε: α) την ταχύτητα της σφαίρας (μέτρο – κατεύθυνση) μετά την κρούση, β) τη δύναμη που δέχτηκε η σφαίρα κατά την κρούση. Η σφαίρα εκτελεί μόνο μεταφορική κίνηση. Συνοπτική λύση: 100. Κυλινδρικό κουτάκι μπύρας Ο κύλινδρος του σχήματος είναι ένα άδειο ομογενές κουτάκι μπύρας αμελητέου πάχους μάζας m=80g ακτίνας R=4cm και ύψους h=12cm. Ο κύλινδρος αυτός μπορεί να περιστρέφεται γύρω από τον άξονα συμμετρίας του. Αβαρές σχοινί είναι τυλιγμένο στην περιφέρεια του κυλινδρικού κουτιού μπύρας και στο ελεύθερο άκρο του είναι δεμένο σώμα μάζας m1. Ακόμη μέσω ενός δεύτερου αβαρούς σχοινιού έχουμε δέσει μια ράβδο μάζας M=240g και μήκους L=0,3m με το κουτί μπύρας, όπως φαίνεται στο σχήμα. Α) Να υπολογίσετε τη μάζα m1 ώστε το σύστημα να ισορροπεί. Β) Τη χρονική στιγμή t0=0 κόβουμε το σχοινί που συνδέει τη ράβδο με το κουτί. i) Να υπολογίσετε τη γωνιακή ταχύτητα της ράβδου όταν αυτή γίνει κατακόρυφη. ii) Να υπολογίσετε την επιτάχυνση με την οποία θα κινηθεί η μάζα m1. Δίνεται για τη ροπή αδράνειας της ράβδου ΙΑ= 0,33ML2 για τη ροπή αδράνειας κυκλικού δίσκου μάζας m΄ και ακτίνας R ότι Ι= 0,5m΄R2 και g=10m/s2. Συνοπτική λύση: Σχόλια
Σχόλιο από τον/την Γεώργιος Μπανιάς στις 11 Ιούνιος 2013 στις 2:02
Διαγραφή σχολίου
ωραια ασκηση , γιατι μπορεις να ελεγξεις τις γνωσεις ενος μαθητη σε πολλες εννοιες . Επισης μπορει να συνδυαστει και με ευρεση δυναμης απο την αρθρωση στη ραβδο , οταν η ραβδος ειναι σε οριζοντια θεση. θα προτιμουσα μια περιγραφη του στερεου, που αποτελειται απο τρια μερη , γιατι συνηθως , ο πυθμενας ειναι κοιλος και οχι επιπεδος δισκος και το πανω μερος εχει ασυμετρη τρυπα , οποτε εδω πιστευω οτι θα χρειαζοταν καποια διευκρινηση ή θα προτιμουσα μια περιγραφη του στερεου, που αποτελειται απο τρια μερη. Γενικα δεν συμφωνω με τις ασκησεις , που η λυση τους στηριζεται σε ενα στριφνο σημειο , που αν δεν γινει κατανοητο , βγαζει ενα μαθητη που εχει κοπιασει εκτος παιχνιδιου.
Σχόλιο από τον/την Μιχαήλ Μιχαήλ στις 11 Ιούνιος 2013 στις 10:17
Διαγραφή σχολίου
Γιώργο σ' ευχαριστώ για τις παρατηρήσεις.Σίγουρα η φύση έχει πιο πολύ φαντασία από τα μοντέλα που χρησιμοποιούμε. Να ΄σαι καλά!
Σχόλιο από τον/την Εμμανουήλ Λαμπράκης στις 11 Ιούνιος 2013 στις 21:12
Διαγραφή σχολίου
Μιχαήλ καλησπέρα. Από το πρωί είχα δει το θέμα σου. Είναι εξαιρετικό και έχει κατά τη γνώμη μου και πρωτοτυπία. Βλέπω μετά το φετινό Δ2 να αρχίσουμε να δίνουμε περισσότερο προσοχή σε υπολογισμούς ροπής αδράνειας.
Σχόλιο από τον/την Μιχαήλ Μιχαήλ στις 12 Ιούνιος 2013 στις 11:06
Διαγραφή σχολίου
Μανώλη οι δυο τελευταίες αναρτήσεις μου είναι εμπνευσμένες από τη δική σου άσκηση. Σ' ευχαριστώ για το υλικό που μας προσφέρεις. Να' σαι πάντα καλά!
Σχόλιο από τον/την Εμμανουήλ Λαμπράκης στις 13 Ιούνιος 2013 στις 13:25
Διαγραφή σχολίου
Μιχαήλ καλό μεσημέρι. Χαίρομαι πάρα πολύ για την αποδοχή από μέρους σου. Ο χώρος αυτός εδώ προσφέρεται για αλληλοεπίδραση και για να παίρνει ό ένας ιδέες από τον άλλο. Πολύ καλές ιδέες μπορεί να πάρουμε πολλές φορές όχι μόνο από συναδέλφους αλλά και από τους μαθητές μας. Ένα παράδειγμα είναι η ιδέα της πρόσφατης ανάρτησης μου της σχετικής με την αυτεπαγωγή.
101. Ελικοειδής κίνηση μηχανικού στερεού.

Ένας κύλινδρος μάζας m=0,2 Kg, ακτίνας R=4cm και ύψους h=20 cm, περιστρέφεται με σταθερή γωνιακή ταχύτητα ω=4π rad/s, ενώ ταυτόχρονα ολισθαίνει πάνω στον άξονα περιστροφής zz΄. Η περιστροφή γίνεται έτσι ώστε το διάστημα (z) που διανύει ο κύλινδρος κατά μήκος του άξονα περιστροφής  είναι ανάλογο με τη γωνία στροφής (φ). Τότε πραγματοποιεί ομαλή ελικοειδή κίνηση.

α) Να υπολογίσετε το διάστημα που θα διανύσει ο κύλινδρος πάνω στον άξονα περιστροφής σε χρόνο t=5s.

β) Να υπολογίσετε την κινητική του ενέργεια. Τριβές δεν υπάρχουν. Ιcm = 0,5mR2

 

Συνοπτική λύση:

ή εδώ

Σχόλια
Σχόλιο από τον/την Μιχαήλ Μιχαήλ στις 9 Ιούλιος 2013 στις 17:30
Διαγραφή σχολίου
Ξεφυλίζοντας το Μάζη (Βιβλιοπωλείον της "ΕΣΤΙΑΣ" ΑΘΗΝΑΙ 1959) έπεσα και πάνω σ΄αυτό!
Σχόλιο από τον/την Μίλτος στις 9 Ιούλιος 2013 στις 19:24
Διαγραφή σχολίου
Τα βιβλία του Μάζη καθόρισαν την μεταπολεμική Ελλάδα. Μάζη είχα σαν μαθητής στο Σχολείο το 1970, Μάζη βρήκα στα Σχολεία το 1978 σαν καθηγητής. Λίγο άσχετο με την άσκηση που μας προσφέρει ο Μιχάλης, αλλά ενδιαφέροντα 1) Ο Θόδωρος Πάγκαλος θυμάται τον Αλκίνοο Μάζη «Κύριε Μάζη» του λέω, «θέλω να σας ευχαριστήσω γιατί αν έμαθα κάτι στη Φυσική, το έμαθα από σας. Γιατί όμως με τιμωρήσατε τότε; Σας διαβεβαιώ ότι εγώ δεν ήμουν εκεί». Και μου λέει: «Άκου να δεις Πάγκαλε!. Ό,τι γίνεται σε αυτή την τάξη ή το έχεις οργανώσει εσύ ή το έχεις εγκρίνει. Το αν ήσουν ή δεν ήσουν δε μ’ ενδιαφέρει καθόλου»! http://mikrokellari.blogspot.gr/2009/05/blog-post_17.html 2) Παλαιότερα στη τάξη του αδελφού μου πού ήταν μεγαλύτερος κατά 7 χρόνια από μένα, ο Μάζης εισερχόταν στη τάξη έχοντας πάντα διπλωμένη στη τσέπη την εφημερίδα «Αυγή» πού θεωρείτο «κομμουνιστική» την εποχή εκείνη! Στη τωτινή Ελλάδα το επίθετο «κομμουνιστικός, -ή, όν» ήταν ταυτόσημο «λοιμώδους νοσήματος» μιά «επάρατος αρρώστεια». Αρκούσε να πείς κάτι πού ο άλλος δεν καταλάβαινε για να σου βάλλουν την ετικέτα «κομμουνιστής» και υπήρχαν τέτοια «μαύρα μεσάνυχτα» ως προς ποιά ιδεολογία πρέσβευε ώστε το «λοιμώδες αυτό νόσημα» να προφέρεται «Κ-ου-μμουνιστής»! Οι γεννηθέντες λοιπόν το 1947-48 μαθητές, πού έτυχε να έχουν δάσκαλο το Μάζη, βλέποντας ανελλιπώς σε κάθε μάθημα την «Αυγή» στη τσέπη του, σκέφθηκαν το «λοιμώδες νόσημα»! (Τότε η ελλάδα ήταν Βασίλειο). Πάραυτα ποτέ κανένας δεν πείραξε τον Μάζη ούτε επί χούντας πού κυβερνούσε την Ελλάδα στην εποχή μου (Τότε η Ελλάδα έγινε «Δημοκρατία»). http://mikrokellari.blogspot.gr/2011/06/1971-72.html
Σχόλιο από τον/την Εμμανουήλ Λαμπράκης στις 9 Ιούλιος 2013 στις 20:07
Διαγραφή σχολίου
Μιχαήλ καλησπέρα Τα σχετικά με το βιβλίο του Μάζη αλλά και σχετικά άλλα στα είπε ο Miltos. Από ότι φαίνεται στο σημείο αυτό η δική μου διαδρομή είναι παράλληλη με αυτή του Miltos. Με Μάζη τέλειωσα το σχολείο ως μαθητής προ αμνημονεύτων χρόνων και το Μάζη δίδασκα στα πρώτα μου χρόνια ως καθηγητής....... Μνήμες...... Στο παρόν τώρα: Πολύ καλή η ιδέα σου να αναδείξεις το θέμα. Για να το λύσει όμως κάποιος πρέπει να "τσιμπήσει" ένα δεδομένο από το σχήμα αφού αυτό δεν αναφέρεται στο κείμενο. Συγκεκριμένα πρέπει να δει στο σχήμα ότι σε μια περιστροφή ο κύλινδρος μετατοπίζεται κατακόρυφα κατά h. Η ανάρτηση σου μου θύμισε το τελευταίο ερώτημα από το 3ο θέμα μιας ωριαίας γραπτής εξέτασης που έκανα όταν στην ύλη της φυσικής Β' κατεύθυνσης συμπεριλαμβάνονταν το σχετικό θέμα. Νομίζω κολλάει στην περίπτωση. Δες και συ. Σχόλιο από τον/την Μιχαήλ Μιχαήλ στις 9 Ιούλιος 2013 στις 21:05 Διαγραφή σχολίου Φίλοι Milto και Μανώλη. Τα βιβλία του Μάζη και Αλεξόπουλου - Μαρίνου είναι ένας ανεπανάληπτος θησαυρός! Είχα την τύχη να έχω πρόσβαση σ' αυτά!
Σχόλιο από τον/την Κωστας Ψυλακος στις 9 Ιούλιος 2013 στις 21:19
Διαγραφή σχολίου
Ισως λιγο παλιοτερα πρεπει να ηταν τα βιβλια Κουγιουμζελη-Περιστερακη αυτα που εχω ειναι εκδοσεως 1969. Ειναι εκπληκτικα τα σχηματα αν σκεφτει κανεις πως τοτε δεν υπηρχαν τα μεσα του σημερα!Φυσικα ειχαν και ενα πολυ πλουσιο περιεχομενο με πολυ αναλυτικο τροπο καθως και με περιγραφη πειραματων.Συνοδευονταν και απο βιβλια ασκησεων.Τα εχω κειμηλιο απο προσωπο της οικογενειας μου!
Σχόλιο από τον/την Γκενές Δημήτρης στις 9 Ιούλιος 2013 στις 23:41
Διαγραφή σχολίου
Μιχαήλ καλησπέρα. Αναμόχλευση με ιδέες και εξαιρετική αναλυτική παρουσίαση. Ευχαριστούμε, ( προΒλέπω σύντομα σχετικές συνέχειες ) Δυστυχώς όταν ήμασταν μαθητές δεν εκτιμήσαμε την "Μαζική Φυσική" ( διάλεξα τον πιο ουδέτερο όρο από τις τότε εκφράσεις μας για τα βιβλία . Μίλτο μου θύμισες ένα παλιό μαργαριτάρι μαθητή από το βιβλίο "Είτε παίδες ελλήνων είτε παίδες βαρβάρων ...και τα δυο δεν γίνεται"... "Ανεμογκάστρι λέμε συνήθως, για γυναίκα που νομίζει έγκυος, αλλά που δεν είναι. Ένοχος συνήθως ο γκαστρίμυθος που την παρέσυρε σε ακολασίες ακατανόμαστες και αυτός είναι ύποπτος τύπος με μισόκλειστα μάτια και στραβοπατημένα παπούτσια που δίνει και το κακό παράδειγμα στους νέους και που, μάλλον, και κουκουές είναι."  
Σχόλιο από τον/την Μιχαήλ Μιχαήλ στις 10 Ιούλιος 2013 στις 1:47
Διαγραφή σχολίου
Κώστα το συγκεκριμένο βιβλίο είναι έκδοση του 1959! Δημήτρη σ' ευχαριστώ! Υπάρχει μια κρυμένη γοητεία σ' αυτά τα "παλιά" βιβλία! Σχόλιο από τον/την Μιχαήλ Μιχαήλ στις 10 Ιούλιος 2013 στις 18:44 Διαγραφή σχολίου Θοδωρή έχεις δίκιο στα όσα λες!
Σχόλιο από τον/την Στέργιος Ναστόπουλος στις 10 Ιούλιος 2013 στις 23:57
Διαγραφή σχολίου
Να θυμηθούμε βέβαια οτι ο Αρχιμήδης ασχολήθηκε πρώτος με τις υπέροχες αυτές κινήσεις. Δείτε δυο κατατοπιστικές διπλωματικές εργασίες: http://www.math.uoa.gr/me/dipl/dipl_stefanaki.pdf και http://www.math.uoa.gr/me/dipl/dipl_Mystrioti.Georgia.pdf
102. Ράβδος και σφαιρίδιο

Ομογενής άκαμπτη ράβδος ΑΒ έχει μήκος L=1,2m και μάζα Μ=0,1 Kg. Το άκρο Α της ράβδου είναι ελεύθερο. Στο άκρο Β της ράβδου υπάρχει στερεωμένο σφαιρίδιο μάζας m=0,2Kg.

Α) Να βρείτε σε ποια θέση Κ πάνω στη ράβδο, πρέπει να δέσουμε ένα αβαρές νήμα το άλλο άκρο του οποίου είναι δεμένο στο ταβάνι, ώστε η ράβδος να ισορροπεί σε οριζόντια θέση όπως φαίνεται στο σχήμα.

Β) Κατόπιν κόβουμε το νήμα. Τότε η ράβδος θα εκτελέσει

i) Μεταφορική κίνηση.

ii) Στροφική κίνηση.

iii) Σύνθετη κίνηση.

Να επιλέξετε τη σωστή απάντηση και να τη δικαιολογήσετε.

Γ) Στη συνέχεια και μόλις το σύστημα ράβδος – μάζα m «πέσει» κατά h=0,2m από την αρχική οριζόντια θέση η μάζα m συγκρούεται πλαστικά με μάζα m1=0,1 Kg, που κινείται κατακόρυφα προς τα πάνω με σταθερή ταχύτητα υ1=3,25m/s. Αν η διάρκεια της κρούσης θεωρείται αμελητέα, τότε:

α) Να υπολογίσετε την κοινή ταχύτητα του συσσωματώματος μετά την κρούση.

β) Να υπολογίσετε τη γωνιακή ταχύτητα του συσσωματώματος μετά την κρούση.

γ) Σε πόσο ύψος τουλάχιστον πρέπει να βρίσκεται αρχικά η ράβδος, ώστε όταν φτάσει στο έδαφος να είναι κατακόρυφη για πρώτη φορά;

Δ) Αν κάποια στιγμή πριν κοπεί το νήμα και καθώς η ράβδος ισορροπεί οριζόντια, αποκολληθεί η μάζα m, τότε να υπολογιστεί εκείνη τη στιγμή η γωνιακή επιτάχυνση της ράβδου καθώς και η τάση του νήματος.

Ε) Αν μόλις το σύστημα ράβδος – μάζα m «πέσει» κατά h=0,2m από την αρχική οριζόντια θέση, η μάζα m συγκρούεται ελαστικά με τη σφήνα του σχήματος, και η διάρκεια της κρούσης είναι Δt=10-3s, τότε να υπολογίσετε τη γωνιακή ταχύτητα του συστήματος αμέσως μετά την κρούση.

Δίνεται για τη ράβδο Icm= 1/12ML2 και g=10m/s2 .

Συνοπτική λύση:

Σχόλια
Σχόλιο από τον/την ΑΡΙΣΤΕΙΔΗΣ ΜΑΡΚΑΝΤΩΝΑΤΟΣ στις 24 Ιούλιος 2013 στις 23:06
Διαγραφή σχολίου
ΦΙΛΕ ΜΙΧΑΗΛ ΜΗΠΩΣ ΣΤΗ 2 ΣΕΛΙΔΑ ΣΤΗΝ Α.Δ.ΣΤΡ ΘΑ ΕΠΡΕΠΕ ΣΤΟ ΠΡΩΤΟ ΜΕΛΟΣ ΝΑ ΥΠΑΡΧΕΙ ΚΑΙ Η ΣΤΡΟΦΟΡΜΗ ΤΟΥ ΣΥΣΤΗΜΑΤΟΣ ΡΑΒΔΟΣ ΣΩΜΑ ΜΑΖΑΣ Μ ΩΣ ΠΡΟΣ ΤΟ ΝΕΟ ΚΕΝΤΡΟ ΜΑΖΑΣ ΤΟΥ ΣΥΣΤΗΜΑΤΟΣ ; ΔΗΛΑΔΗ Ο ΟΡΟΣ (Μ+m) u (x2-χ2΄) ΠΟΥ ΕΙΝΑΙ Η ΣΤΡΟΦΟΡΜΗ ΤΟΥ ΑΡΧΙΚΟΥ ΚΕΝΤΡΟΥ ΜΑΖΑΣ ΤΗΣ ΡΑΒΔΟΥ ΚΑΙ ΤΟΥ ΣΩΜΑΤΟΣ Μ ΠΡΙΝ ΤΗΝ ΠΛΑΣΤΙΚΗ ΚΡΟΥΣΗ ΛΟΓΩ ΜΕΤΑΦΟΡΙΚΗΣ ΚΙΝΗΣΗΣ ΩΣ ΠΡΟΣ ΤΟ ΝΕΟ ΚΕΝΤΡΟ ΜΑΖΑΣ ΤΟΥ ΣΥΣΤΗΜΑΤΟΣ ΑΜΕΣΩΣ ΜΕΤΑ ΤΗΝ ΠΛΑΣΤΙΚΗ ΚΡΟΥΣΗ. ΠΑΡΑΤΗΡΩ ΟΤΙ Ο ΦΟΡΕΑΣ ΤΗΣ ΤΑΧΥΤΗΤΑΣ ΤΟΥ ΚΕΝΤΡΟΥ ΜΑΖΑΣ ΠΡΙΝ ΤΗΝ ΠΛΑΣΤΙΚΗ ΚΡΟΥΣΗ ΔΕΝ ΔΙΕΡΧΕΤΑΙ ΑΠΟ ΝΕΟ ΚΕΝΤΡΟ ΜΑΖΑΣ ΜΕΤΑ ΤΗΝ ΠΛΑΣΤΙΚΗ ΚΡΟΥΣΗ ΓΙΑ ΑΥΤΟ ΠΙΣΤΕΥΩ ΟΤΙ ΥΠΑΡΧΕΙ ΑΡΧΙΚΗ ΣΤΡΟΦΟΡΜΗ
Σχόλιο από τον/την Μαλακασιώτης Νικόλαος στις 24 Ιούλιος 2013 στις 23:10
Διαγραφή σχολίου
Μπράβο Μιχάλη καλή άσκηση,ωραίες σκέψεις,με μια μικρή ένσταση στη μετατόπιση του κέντρου μάζας
Σχόλιο από τον/την Μιχαήλ Μιχαήλ στις 24 Ιούλιος 2013 στις 23:43
Διαγραφή σχολίου
Αριστείδη και Νίκο σας ευχαριστώ. Αριστείδη έχεις δίκιο η αλλαγή είχε γίνει!
Σχόλιο από τον/την ΑΡΙΣΤΕΙΔΗΣ ΜΑΡΚΑΝΤΩΝΑΤΟΣ στις 25 Ιούλιος 2013 στις 0:07
Διαγραφή σχολίου
ΜΗΠΩΣ ΚΑΙ ΤΟ ΠΡΟΣΗΜΟ ΠΡΕΠΕΙ ΝΑ ΓΙΝΕΙ (+) ΚΑΙ ΟΧΙ (-) ΣΤΗΝ ΟΡΟ ΠΟΥ ΠΡΟΣΘΕΣΕΣ; ΓΙΑ ΡΙΞΕ ΠΑΛΙ ΜΙΑ ΜΑΤΙΑ. ΠΑΝΤΩΣ Η ΑΣΚΗΣΗ ΕΙΝΑΙ ΦΟΒΕΡΗ ΜΕ ΩΡΑΙΕΣ ΙΔΕΕΣ ΚΑΙ ΑΡΚΕΤΑ ΔΥΣΚΟΛΗ. ΟΣΟ ΓΙΑ ΜΙΑ ΠΡΟΗΓΟΥΜΕΝΗ ΑΣΚΗΣΗ ΜΕ ΣΤΕΡΕΟ ΠΟΥ ΜΕΤΑΤΟΠΙΖΕΤΑΙ ΚΑΙ ΠΕΡΙΣΤΡΕΦΕΤΑΙ ΕΝΑ ΚΑΛΟ ΠΑΡΑΔΕΙΓΜΑ ΕΝΑ Η ΠΕΡΙΣΤΡΟΦΗ ΦΕΛΛΟΥ ΓΙΑ ΤΗΝ ΑΠΟΜΑΚΡΥΝΣΗ ΤΟΥ ΑΠΟ ΑΝΟΙΧΤΗΡΙ ΣΧΗΜΑΤΟΣ ΤΡΥΠΑΝΙΟΥ(ΑΛΛΗ ΦΟΒΕΡΗ ΑΣΚΗΣΗ). ΜΠΡΑΒΟ ΣΟΥ
Σχόλιο από τον/την Μιχαήλ Μιχαήλ στις 25 Ιούλιος 2013 στις 0:38
Διαγραφή σχολίου
Αριστείδη ευχαριστώ. Προφανώς έχεις δίκιο!
Σχόλιο από τον/την Μιχαήλ Μιχαήλ στις 26 Ιούλιος 2013 στις 12:36
Διαγραφή σχολίου
..και το E) ερώτημα ....blog
103. Μαγνητικό έργο

Έστω ότι  θέλουμε  να μαγνητίσουμε ένα αρχικά αμαγνήτιστο υλικό. Πόσο είναι τότε το έργο μαγνήτισης του υλικού;

 

Συνοπτική λύση:

104. Απομακρύνονται ή πλησιάζουν;

Δυο σώματα πραγματοποιούν Α.Α.Τ με εξισώσεις x1=A1ημω1t και x2=A2ημω2t με Α2=2A1 και ω1=3ω2.

Τότε:

α) να βρείτε ποιες χρονικές στιγμές στη διάρκεια μιας περιόδου τα δυο σώματα συναντώνται.

β) Από τη χρονική στιγμή 0 (αρχή της ταλάντωσης των δυο σωμάτων), μέχρι και τη χρονική στιγμή της πρώτης συνάντησής τους τα σώματα πλησιάζουν ή απομακρύνονται;

 

Συνοπτική λύση:

Σχόλια
Σχόλιο από τον/την Εμμανουήλ Λαμπράκης στις 9 Αύγουστος 2013 στις 19:26
Διαγραφή σχολίου
Μιχαήλ καλησπέρα Το πρόβλημα και παρουσίαση της λύσης εξαιρετικά. Κάνω την παρακάτω σκέψη χωρίς όμως να το έχω ψάξει Θα είχε ενδιαφέρον και ίσως απλοποιούσε τη λύση μια αντιμετώπιση με στρεφόμενα διανύσματα;
Σχόλιο από τον/την Μιχαήλ Μιχαήλ στις 9 Αύγουστος 2013 στις 20:13
Διαγραφή σχολίου
Μανώλη καλησπέρα και σ' ευχαριστώ. Η πρώτη μου προσέγγιση ήταν με το περιστρεφόμενο διάνυσμα. Δεν είδα όμως να απλοποιείται η απόδειξη. Ίσως κάτι να μου ξεφεύγει!
Σχόλιο από τον/την Βαγγέλης Κουντούρης στις 9 Αύγουστος 2013 στις 21:23
Διαγραφή σχολίου
Πολύ καλή Μιχαήλ (ιδιαίτερα το ερώτημα Αβ) πρέπει, όμως, να φαίνεται στην εκφώνηση ότι είναι ίδια η Θ.Ι. και η ευθεία κίνησης.
Σχόλιο από τον/την Μιχαήλ Μιχαήλ στις 9 Αύγουστος 2013 στις 21:45
Διαγραφή σχολίου
Βαγγέλη σ' ευχαριστώ. Οι διορθώσεις έγιναν. Να' σαι πάντα καλά!
Σχόλιο από τον/την Εμμανουήλ Λαμπράκης στις 10 Αύγουστος 2013 στις 10:56
Διαγραφή σχολίου
Μιχαήλ καλημέρα Έχεις δίκιο. Τελικά τα στρεφόμενα δε δίνουν κάτι που να κάνει τη διαφορά, ουσιαστικά στις εξισώσεις των ταλαντώσεων οδηγούν επίσης.
Σχόλιο από τον/την Pantelis Lapas στις 11 Αύγουστος 2013 στις 18:56
Διαγραφή σχολίου
Αν ασχοληθεί κανείς με το διάγραμμα της συνάρτησης |x1(t) - x2(t)| τότε μπορεί επίσης να απαντήσει στα παραπάνω ερωτήματα.
Σχόλιο από τον/την Μιχαήλ Μιχαήλ στις 11 Αύγουστος 2013 στις 21:29
Διαγραφή σχολίου
Μανώλη και Παντελή σας ευχαριστώ για τα σχόλια! Έχετε δίκιο!
Σχόλιο από τον/την Μαλακασιώτης Νικόλαος στις 12 Αύγουστος 2013 στις 20:22
Διαγραφή σχολίου
Μιχάλη καλησπέρα,πολύ ωραία ερωτήματα και υποδειγματικέα λύσεις.
Σχόλιο από τον/την Μιχαήλ Μιχαήλ στις 12 Αύγουστος 2013 στις 22:42
Διαγραφή σχολίου
Νίκο σ' ευχαριστώ!
105. A.A.T και ένταση.

Σώμα πραγματοποιεί Α.Α.Τ στον άξονα xx΄ πλάτους Α και περιόδου T. Τη χρονική t το σώμα βρίσκεται στη θέση Κ με x1=-10cm κινούμενο με θετική ταχύτητα (υ>0) και τη χρονική στιγμή t+6 περνάει για πρώτη φορά από τη θέση Λ με x2=10cm.

Αν τη χρονική στιγμή t+12 περνάει για πρώτη φορά από τη θέση Μ με x1=10cm κινούμενο με αρνητική ταχύτητα (υ<0) τότε:

α) Να υπολογιστούν η περίοδος T και το πλάτος Α της ταλάντωσης.

β) Αν t=1s τότε να υπολογιστεί η αρχική φάση φ0 της ταλάντωσης.

γ) Να κατασκευάσετε το διάγραμμα x(t) για 0£t£T.

δ) Να υπολογιστεί η ένταση της ταλάντωσης.

Συνοπτική λύση:

Σχόλια
Σχόλιο από τον/την Διονύσης Μητρόπουλος στις 16 Αύγουστος 2013 στις 17:49
Διαγραφή σχολίου
Μιχάλη καλησπέρα, Μου άρεσε πολύ ο όρος "ένταση της ταλάντωσης", δεν τον είχα ξανασυναντήσει :-)
Σχόλιο από τον/την Μιχαήλ Μιχαήλ στις 16 Αύγουστος 2013 στις 18:12
Διαγραφή σχολίου
Διονύση καλησπέρα, στο βιβλίο του Μάζη τον πρωτοσυνάντησα!
Σχόλιο από τον/την Διονύσης Μητρόπουλος στις 16 Αύγουστος 2013 στις 18:18
Διαγραφή σχολίου
Εννοείς εκείνους τους ... τεράστιους τόμους; (Κάπου πρέπει να τους έχω στο πατάρι :-))
Σχόλιο από τον/την Μιχαήλ Μιχαήλ στις 16 Αύγουστος 2013 στις 18:27
Διαγραφή σχολίου
Ο συγκεκριμένος είναι μάλλον συμπαθητικός!
Σχόλιο από τον/την Διονύσης Μάργαρης στις 17 Αύγουστος 2013 στις 8:16
Διαγραφή σχολίου
Καλημέρα Μιχαήλ. Και μένα με 'πιασες αδιάβαστο με τον όρο "ένταση" ταλάντωσης...
Σχόλιο από τον/την Μιχαήλ Μιχαήλ στις 17 Αύγουστος 2013 στις 13:57
Διαγραφή σχολίου
Διονύση καλό μεσημέρι! Μου 'κανε πάντως εντύπωση και μένα πως το Α^2*f^2 δείχνει πόσο "ισχυρή" είναι μια ταλάντωση ενός υλικού σημείου!
Σχόλιο από τον/την Μιχαήλ Μιχαήλ στις 17 Αύγουστος 2013 στις 14:14
Διαγραφή σχολίου
...πάντως δε ξέρω αν θα μπορούσαμε να βρούμε λύση στην παραπάνω άσκηση αν το σημείο Λ απέχει απόσταση 10sqr2 από το Ο!!
106. Μια παραλλαγή της άσκησης ΟΕΦΕ 2013

106. Άσκηση ΟΕΦΕ

 

Ο ομογενής δακτύλιος του παρακάτω σχήματος έχει μάζα Μ και ακτίνα R=0,1m. Η σημειακή μάζα m είναι ακλόνητα συνδεδεμένη  στο σημείο Α. Το σύστημα μπορεί να περιστρέφεται χωρίς τριβές γύρω από άξονα που διέρχεται από το σημείο Β αντιδιαμετρικό του Α. Αφήνουμε το σύστημα ελεύθερο να περιστραφεί έτσι ώστε αρχικά η ΑΒ να είναι μια οριζόντια διάμετρος. Τότε

α) Να υπολογίσετε την αρχική γωνιακή επιτάχυνση του συστήματος.

β) Να υπολογίσετε την γωνιακή του ταχύτητα μόλις η διάμετρος ΑΒ γίνει κατακόρυφη.

γ) Αν εκείνη τη στιγμή η μάζα m ξεκολλήσει τότε πόσο ψηλά θα φτάσει το Κ.Μ του δακτυλίου;

δ) Αν εκείνη τη στιγμή η μάζα m συγκρουστεί πλαστικά με μια όμοια σημειακή μάζα m τότε πόσο ψηλά θα φτάσει το Κ.Μ του δακτυλίου σε αυτήν την περίπτωση;

 

Δίνεται για το δακτύλιο Ιcm=MR2 και g=10m/s2.

 

Συνοπτική λύση:

107. Ράβδος και δίσκος

Ομογενής ράβδος ΑΓ έχει μήκος L=1m, μάζα M=3Kg και ισορροπεί αρχικά  σε οριζόντια θέση. Στο άκρο Γ της ράβδου υπάρχει δίσκος μάζας m=1Kg και ακτίνας R=0,1m, που στρέφεται κατά τη φορά των δεικτών του ρολογιού, σε κατακόρυφο επίπεδο και γύρω από οριζόντιο άξονα που περνά από το κέντρο του, που ταυτίζεται με το άκρο Γ της ράβδου όπως φαίνεται στο σχήμα. H σταθερή γωνιακή ταχύτητα περιστροφής του δίσκου είναι ωδ=100rad/s. Στη συνέχεια αφήνουμε το σύστημα ελεύθερο να περιστραφεί γύρω από το άκρο Α της ράβδου. Να υπολογιστούν:

α) Η γωνιακή επιτάχυνση (αγων), του συστήματος ράβδου – δίσκου τη στιγμή που το αφήνουμε ελεύθερο.

β) Η στροφορμή του συστήματος ως προς τον άξονα περιστροφής τη στιγμή που καθώς περιστρέφεται βρίσκεται στην κατακόρυφη θέση.

γ) Η κινητική ενέργεια του συστήματος όταν η ράβδος έχει περιστραφεί κατά 300.

Δίνονται: Η ροπή αδράνειας της ράβδου για άξονα που περνά από το Κ.Μ της Ιcm=1/12*ML2, η ροπή αδράνειας του δίσκου ως προς το Κ.Μ του Ιδ=0,5mR2  και η επιτάχυνση της βαρύτητας g=10m/s2.

 

Συνοπτική λύση:

Σχόλιο
Σχόλιο από τον/την Μιχαήλ Μιχαήλ στις 18 Αύγουστος 2013 στις 13:43
Διαγραφή σχολίου
Η άσκηση αφιερώνεται στον Γιάννη τον Κυριακόπουλο. Αλήθεια Γιάννη που είσαι;
107_b. Ράβδος και δίσκος με οπή.

Ομογενής ράβδος ΑΓ έχει μήκος L=1m, μάζα M=3Kg και ισορροπεί αρχικά  σε οριζόντια θέση. Στο άκρο Γ της ράβδου υπάρχει ένας επίπεδος συμπαγής δίσκος μάζας m=1kg και ακτίνας R=10cm, στον οποίο έχουμε κάνει μια κυκλική οπή ακτίνας r=2,5cm (r<R). Το κέντρο της οπής απέχει από το κέντρο Γ του δίσκου απόσταση d=5cm με d<R. 

Ο δίσκος με την οπή περιστρέφεται ελεύθερα, γύρω από έναν οριζόντιο άξονα που περνά από το κέντρο του Γ με ωδ=100rad/s όπως φαίνεται στο σχήμα..

Στη συνέχεια αφήνουμε το σύστημα ελεύθερο να περιστραφεί γύρω από το άκρο Α της ράβδου. Να υπολογιστούν:

α) Η γωνιακή επιτάχυνση (αγων), του συστήματος ράβδου – δίσκου τη στιγμή που το αφήνουμε ελεύθερο.

β) Να υπολογιστεί η αρχική κινητική ενέργεια του δίσκου.

Δίνονται: Η ροπή αδράνειας της ράβδου για άξονα που περνά από το Κ.Μ της Ιcm=ML2, η ροπή αδράνειας  δίσκου μάζας m και ακτίνας R ως προς το Κ.Μ του Ιδ=mR2  και η επιτάχυνση της βαρύτητας g=10m/s2.

 

Συνοπτική λύση:

108. Κύλινδρος και σημειακή μάζα m.

Στο εσωτερικό ενός κυλινδρικού κουτιού μάζας Μ=0,7 Kg και ακτίνας r=20 cm, έχουμε κολλήσει μια μικρή μεταλλική σφαίρα μάζας m=1 Kg. Το σύστημα τοποθετείται σε κεκλιμένο επίπεδο γωνίας κλίσης φ=300, έτσι ώστε η ΚΑ να είναι παράλληλη στο κεκλιμένο επίπεδο. Τότε:

α) Να υπολογίσετε τη ροπή του βάρους της σφαίρας ως προς το σημείο Κ,

β) Να υπολογίσετε την αρχική γωνιακή

επιτάχυνση τη στιγμή που αφήνουμε το σύστημα να κινηθεί πάνω στο κεκλιμένο επίπεδο.

γ) Αν το σύστημα ισορροπεί να υπολογιστούν η στατική τριβή και η κάθετη αντίδραση από το κεκλιμένο επίπεδο στο σύστημα των δυο σωμάτων.

δ) Αν απομακρύνουμε τη μάζα m τότε πόση γίνεται η γωνιακή επιτάχυνση του κυλίνδρου;

Δίνονται: g=10m/s2, ημ600=0,86 και ότι  η ροπή αδράνειας του κυλίνδρου ως προς άξονα που περνάει από το κέντρο μάζας του είναι Icm=0,5Mr2.

Η Άσκηση αφιερώνεται στο Διονύση Μητρόπουλο που μας  πρωτοέδωσε την απορία και τη λύση για την άσκηση.

Συνοπτική λύση:

και εδώ

Σχόλια
Σχόλιο από τον/την Θοδωρής Γκούτης στις 27 Αύγουστος 2013 στις 16:14
Διαγραφή σχολίου
Μιχαήλ καλημέρα. Πολύ ενδιαφέρουσα άσκηση, αλλά για να λυθεί το γ ερώτημα είναι απαραίτητο στην εκφώνηση να προστεθεί (στο γ ερώτημα): "Αν το σώμα ισορροπεί να υπολογιστουν...." (οταν πατώ το σύνδεσμο της λύσης προς το blog το avast βγάζει μήνυμα...)
Σχόλιο από τον/την Μιχαήλ Μιχαήλ στις 27 Αύγουστος 2013 στις 17:25
Διαγραφή σχολίου
Θοδωρή καλησπέρα και σ' ευχαριστώ για το σχόλιο. Η αλήθεια είναι ότι δεν θέλω στην άσκηση να αποκαλύψω από την αρχή ότι το σύστημα ισορροπεί. Έτσι προτείνω στην άσκηση, πρώτα να δούμε αν το σύστημα ισορροπεί που μπορεί να γίνει εύκολα. Αυτό προκύπτει στο β) ερώτημα. Έτσι στο γ) υπολογίζουμε πλέον την Τ και τη Ν για την ισορροπία του συστήματος. Σχόλιο από τον/την Θοδωρής Γκούτης στις 27 Αύγουστος 2013 στις 19:25 Διαγραφή σχολίου
Σχόλιο από τον/την Θοδωρής Γκούτης στις 27 Αύγουστος 2013 στις 19:25
Διαγραφή σχολίου
Μιχαηλ, Αν το πάρουμε εμπειρικά, αν υποθέσουμε οτι αφήνουμε το σύστημα αυτό σε λείο κεκλιμένο επίπεδο, ή σε επίπεδο με όχι αρκετή τριβή, δεν πρόκειται να ισορροπήσει, παρόλο που η συνισταμένη των ροπών ως προς το Δ είναι μηδέν. Το Στ=0 ως προς το Δ, δεν εξασφαλίζει οτι το σώμα ισορροπεί. Μπορεί Στ(ως προς Δ)=0 και Στ(ως προς άλλο σημείο)<>0 Επιμένω οτι είναι απαραίτητο να εξασφαλίσεις στην εκφώνηση οτι τουλάχιστον το Δ είναι ακίνητο είτε οτι είναι καρφωμένο είτε οτι είναι δεμένο με σχοινάκι παράλληλο στο κεκλιμένο είτε απαιτώντας οτι η στατική τριβή είναι αρκετά μεγάλη. Η άσκηση είναι πολύ καλή, και μας βάζει σε ψάξιμο. Θα επανέλθω με θεωρητική....τεκμηρίωση. Σχόλιο από τον/την Θοδωρής Γκούτης στις 27 Αύγουστος 2013 στις 22:58 Διαγραφή σχολίου Γνωρίζω ότι για ένα στερεό με 6 βαθμούς ελευθερίας ισχύει ΣF = ΔP/Δt = MαCM Στ = ΔL/Δt με την προϋπόθεση ότι ροπή και στροφορμή μετρώνται Είτε ως προς ακίνητο σταθερό σημείο Είτε ως προς το κέντρο μάζας. Το σύστημά μας εδώ κάνει επίπεδη κίνηση (3 βαθμοί) με το δεσμό ότι το κέντρο του κυλίνδρου θα απέχει πάντα r από το κεκλιμένο επίπεδο. Επίσης ΣF = 0 <=> το κέντρο μάζας ισορροπεί Στ = 0 (ως προς σταθερό σημείο) <=> ισορροπεί περιστροφικά ως προς το σταθερό σημείο. Στ = 0 (ως προς το CM) <=> ισορροπεί περιστροφικά ως προς το CM ΣF = 0 και Στ=0 (ως προς οποιοδήποτε σημείο) <=> ισορροπεί και θα είναι Στ=0 ως προς οποιοδήποτε άλλο σημείο. Αν όμως ΣτΑ=0 με ΣF<>0 τότε γενικά δεν ισορροπεί, αν το σημείο Α είναι κινούμενο. Στο β ερώτημα υπολογίζω τη ροπή χωρίς να προσδιορίσω το κέντρο μάζας. Η συνισταμένη ροπή ως προς το Δ είναι (Συγνώμη για την έλλειψη σχήματος) Στ(Δ) = τΜmΝτ = Mgrημ30 – mg(rσυν30-rημ30) + 0 + 0 = 0,7 10 0,2 ½ - 1 10 0,2 (0,86-0,5) = 0. Αυτό σημαίνει ότι το σώμα δεν περιστρέφεται ως προς το Δ, καθώς και ότι το CM είναι στην ίδια κατακόρυφο με το Δ.. Με μηδενική τριβή, η Στ ως προς το κέντρο μάζας θα είναι Στ(CM) = τΜmΝ = 0(εξορισμού) + τΝ = Νr1 <> 0 και ΣFx= Mgημ30+mgημ30<>0 (xx’ παράλληλος στο επίπεδο) Αυτό μου λέει ότι με μηδενική τριβή μόλις το αφήσω στο κεκλιμένο (παρόλο που στιγμιαία η ροπή ως προς το Δ είναι μηδέν) το στερεό θα περιστραφεί γύρω από το κέντρο μάζας δεξιόστροφα ενώ ταυτόχρονα θα επιταχυνθεί κατά τον άξονα xx’ Αν απαιτήσω η τριβή να είναι Τ=mgημ30+Μgημ30= 8,5Ν τότε θα είναι ΣFx = 0 και το σύστημα θα ισορροπήσει ως προς τον xx’. Με ενδιάμεση τριβή (0=<Τ<8,5) το σύστημα θα επιταχύνεται κατά τον xx’ και τότε θα είναι και πάλι Στ<>0 (ως προς το κέντρο μάζας) που λέει ότι το σύστημα θα περιστρέφεται ως προς το CM. Παρόμοιο με τη μηδενική τριβή. Ξανά τώρα στην περίπτωση που υποθέσουμε ότι Τ=8,5Ν, το CM ισορροπεί ως προς τον xx’ . Λόγω του δεσμού το CM θα ισορροπεί και ως προς τον yy’ Επειδή το Δ θα είναι ακίνητο και Στ=0 (ως προς το Δ) τότε θα είναι και Στ=0 (ως προς το CM). Προσδιορίζω το CM και από τη σχέση Στ=0 ως προς το CM υπολογίζω τη Ν Στ(CM)= 0 ó Tr-Nr1=0 ó Ν= 8,5 ρίζα(3) Εναλλακτικά ΣFy= 0 ó Ν = Μgσυν30+mgσυν30 = 8,5 ρίζα(3) Γενικώς για να υπολογίσουμε τις επιταχύνσεις στην περίπτωση που Τ<8,5 είναι λίγο δύσκολο με τους θεμελιώδεις νόμους. Ίσως οι εξισώσεις Λαγρανζ και η επιλογή κατάλληλων μεταβλητών για τους 2 βαθμούς ελευθερίας βοηθήσουν Ίσως κάποιος συνάδελφος συμμετάσχει. Αξιόλογη άσκηση . Ομολογώ ότι ξενύχτησα λίγο να τη σκέφτομαι.
Σχόλιο από τον/την Μιχαήλ Μιχαήλ στις 28 Αύγουστος 2013 στις 8:15
Διαγραφή σχολίου
Θοδωρή έχεις δίκιο για την ισορροπία. Πρόσθεσα στο β) ερώτημα ότι το σύστημα ισορροπεί! Να ' σαι καλά.
Ασκήσεις Φυσικής Γ΄ Λυκείου

1. Ένα σώμα μάζας m=1Kg ισορροπεί δεμένο πάνω σε κατακόρυφο ελατήριο σταθεράς Κ=400Ν/m. Τη χρονική στιγμή t=0 εξασκούμε στο σώμα μια σταθερή κατακόρυφη δύναμη F=90N με φορά προς τα κάτω όπως φαίνεται στο σχήμα.

α) Να αποδείξετε ότι το σύστημα πραγματοποιεί α.α.τ και να υπολογιστεί η ενέργεια της ταλάντωσης.

β) Να γράψετε την εξίσωση x(t) της α.α.τ. θεωρείστε την προς τα πάνω φορά θετική.

γ) Να υπολογιστεί το έργο της δύναμης F (WF), το έργο του βάρους (Ww) καθώς και το έργο της δύναμης του ελατηρίου (WFελ), σε χρόνο T/4 από τη στιγμή που άρχισε να ασκείται η δύναμη F.

δ) Να γίνει η γραφική παράσταση της απομάκρυνσης d του σώματος από τη θέση φυσικού μήκους του ελατηρίου σε συνάρτηση με το χρόνο t. Δίνεται g=10m/s2.

Ασκήσεις

Σχόλια
Σχόλιο από τον/την Μαλακασιώτης Νικόλαος στις 27 Αύγουστος 2013 στις 18:14
Διαγραφή σχολίου
Συγχαρητήρια για την δουλειά σου φίλε Μιχαήλ,έχω μείνει άναυδος.
Σχόλιο από τον/την Πέτρος Καραπέτρος στις 27 Αύγουστος 2013 στις 19:28
Διαγραφή σχολίου
Μιχαήλ και από μένα συγχαρητηρια και ένα μεγάλο ευχαριστώ για το υλικο που μας προσφέρεις.
Σχόλιο από τον/την ΧΡΗΣΤΟΣ ΕΛΕΥΘΕΡΙΟΥ στις 27 Αύγουστος 2013 στις 19:36
Διαγραφή σχολίου
Μιχάλη αξίζεις τα εύσημα όλων μας για την "αρχοντοδουλειά σου"....
Σχόλιο από τον/την Παπαδημητριου Θανασης στις 27 Αύγουστος 2013 στις 19:49
Διαγραφή σχολίου
Μπραβο Μιχαλη!!! Φοβερη δουλεια
Σχόλιο από τον/την Βαγγέλης Κουντούρης στις 27 Αύγουστος 2013 στις 20:31
Διαγραφή σχολίου
Καλή άσκηση Μιχαήλ. (καλύτερα, πάντως, να τηρείς το ίδιο πρόσωπο στα ερωτήματα, π.χ. β΄πληθυντικό: να αποδείξετε, να υπολογίσετε, όχι να υπολογισθεί, να κάνετε, όχι να γίνει...) Καλό ερώτημα το δ), αλλά έπρεπε να είσαι αναλυτικότερος στη λύση, (σχεδίαση της συνήθους, παράλληλη μετατόπιση...) διότι δεν μπορούν να την σχεδιάσουν απ' ευθείας οι μαθητές
Σχόλιο από τον/την Μαρούσης Βαγγέλης στις 27 Αύγουστος 2013 στις 21:28
Διαγραφή σχολίου
Πάρα πολύ καλή δουλειά Μιχάλη, όπως και όλες οι ασκήσεις που έχεις κατά καιρούς δημοσιεύσει!!!
Σχόλιο από τον/την Ηλίας Ζαρνάς στις 27 Αύγουστος 2013 στις 22:19
Διαγραφή σχολίου
Συγχαρητήρια Μιχαήλ και για αυτή την άσκηση και για ολες τις άλλες που υπάρχουν στο "προχωρημένα Θέματα" . Αν μου επιτρέπεις θα προτιμούσα στην απόδειξη οτι το σώμα κάνει αατ να ειχες πάρει την τυχαία θέση με θετική απομάκρυνση (πάνω απο τη Θ.Ι.) αφού στο β ερώτημα θεωρείς την θετική φορά προς τα πάνω.
Σχόλιο από τον/την Μιχαήλ Μιχαήλ στις 27 Αύγουστος 2013 στις 22:30
Διαγραφή σχολίου
Νίκο, Πέτρο , Χρήστο, Θανάση, Βαγγέλη, Δημήτρη, Βαγγέλη. Σας ευχαριστώ όλους! Όμως τα συγχαρητήρια αξίζουν στους πολύ αξιόλογους συναδέλφους του ylikonet, μεταξύ αυτών και σεις, που μας προσφέρουν τόσο απλόχερα τον κόπο τους!
Σχόλιο από τον/την Μιχαήλ Μιχαήλ στις 28 Αύγουστος 2013 στις 21:29
Διαγραφή σχολίου
Ηλία τώρα είδα το σχόλιό σου και σ' ευχαριστώ. Θα προσπαθήσω για κάποιες αλλαγές! Να΄σαι καλά!
109_b. A.A.T και μεταβλητή μάζα.

Σώμα μάζας m=9 Kg πραγματοποιεί Α.Α.Τ στον άξονα xx΄. Τη χρονική t0=0 το σώμα διέρχεται από τη θέση ισορροπίας του κινούμενο προς τη θετική κατεύθυνση και η εξίσωση της απομάκρυνσής του είναι x=8ημ(π/12t) S.I.

Αν κάθε φορά που το σώμα περνά από τις θέσεις μέγιστης απομάκρυνσης καθώς και από τη θέση ισορροπίας του, η μάζα του ελαττώνεται κατά m/4 , τότε

α) Να γράψετε την εξίσωση της ταλάντωσής του για t>t3 όπου t3 είναι η χρονική στιγμή που θα βρεθεί στη θέση x=-A για πρώτη φορά.

β) Πόση είναι τότε η ενέργεια της ταλάντωσής του; Δίνεται sqr(3)=1,7 , sqr(2)=1,4 και π2=10.

 

Συνοπτική λύση:

και εδώ

Σχόλια
Σχόλιο από τον/την Κυριακόπουλος Γιάννης στις 31 Αύγουστος 2013 στις 21:19
Διαγραφή σχολίου
Καλησπέρα Μιχάλη. Δεν μου ανοίγει η λύση και μου ζητά κωδικό.
Σχόλιο από τον/την Μιχαήλ Μιχαήλ στις 31 Αύγουστος 2013 στις 21:52
Διαγραφή σχολίου
Γιάννη καλησπέρα. Για δες και εδώ
Σχόλιο από τον/την Κυριακόπουλος Γιάννης στις 31 Αύγουστος 2013 στις 21:53
Διαγραφή σχολίου
Σε ευχαριστώ. Θα τη διαβάσω.
Σχόλιο από τον/την Διονύσης Μάργαρης στις 1 Σεπτέμβριος 2013 στις 9:21
Διαγραφή σχολίου
Καλημέρα Μιχαήλ. Πολύ ωραία ιδέα διαπραγματεύεσαι. Μια παρατήρηση-πρόταση. Να αφαιρείς μάζα, μόνο τη στιγμή της μέγιστης απομάκρυνσης, αφού τότε μόνο η ενέργεια ταλάντωσης παραμένει σταθερή. Αν αφαιρέσεις μάζα στη θέση ισορροπίας (πέρα από το ότι πρακτικά είναι δύσκολο), τότε μεταβάλλεται (μικραίνει) και η ενέργεια ταλάντωσης.
Σχόλιο από τον/την Μιχαήλ Μιχαήλ στις 1 Σεπτέμβριος 2013 στις 18:04
Διαγραφή σχολίου
Διονύση έκανα την αλλαγή σ' ευχαριστώ!
111. Ταλάντωση με δυο ελατήρια σε «σειρά».

Το σώμα Σ του σχήματος έχει μάζα m=4Kg. Το Σ είναι δεμένο στην άκρη δυο κατακόρυφων ελατηρίων συνδεδεμένων σε σειρά με σταθερές Κ1=200Ν/m και Κ2=400N/m όπως φαίνεται στο σχήμα. Αρχικά το σύστημα ισορροπεί πάνω σε οριζόντιο επίπεδο.

Στη συνέχεια ανυψώνουμε κατακόρυφα το σώμα κατά h=1m πάνω από το οριζόντιο επίπεδο, εξασκώντας στο ελεύθερο άκρο του ελατηρίου Κ1 κατάλληλη κατακόρυφη δύναμη F και το στερεώνουμε στην οροφή  στο σημείο Α.

Να βρείτε:

Α) Το ελάχιστο έργο της δύναμης F, μέχρι τη στιγμή που αναρτούμε το σώμα στο σημείο Α.

Β) Στη συνέχεια απομακρύνουμε μέγιστα  το σώμα Σ, κατακόρυφα προς τα κάτω και το ελατήριο Κ2 επιμηκύνεται επιπλέον κατά ΔL2=0,06 m ενώ τη χρονική στιγμή t=0 το αφήνουμε ελεύθερο. Να δείξετε ότι το σύστημα πραγματοποιεί α.α.τ και να γράψετε την εξίσωσή της. Θεωρείστε την προς τα κάτω φορά θετική.

Γ) Κόβουμε το ελατήριο Κ1 στη μέση και δένουμε στο ένα άκρο του το σώμα μάζας m. Στη συνέχεια αναρτούμε το σύστημα κατακόρυφα και το θέτουμε σε α.α.τ. Να συγκρίνετε την περίοδο της ταλάντωσης με αυτή του προηγούμενου ερωτήματος.

Δίνεται g=10m/s2 .

 

Συνοπτική λύση:

ή

εδώ

Σχόλια
Σχόλιο από τον/την Εμμανουήλ Λαμπράκης στις 17 Σεπτέμβριος 2013 στις 11:01
Διαγραφή σχολίου
Μιχαήλ καλημέρα Πολύ ενδιαφέρον θέμα κατά τη γνώμη μου. Μου αρέσει, ιδιαίτερα, το ότι εμπεριέχει τόσο τη σύνδεση ελατηρίων όσο και το κόψιμο ελατηρίου.
Σχόλιο από τον/την Μιχαήλ Μιχαήλ στις 17 Σεπτέμβριος 2013 στις 13:09
Διαγραφή σχολίου
Μανώλη, καλημέρα. Σ' ευχαριστώ για το σχόλιό σου. Πάντως έχω μια απορία, για το αν θα μπορούσαν τα παιδιά να αντιμετωπίσουν την περίπτωση που δεν κόβουμε το ελατήριο στη μέση! Μανώλη θα με ενδιέφερε η άποψή σου.
Σχόλιο από τον/την Εμμανουήλ Λαμπράκης στις 17 Σεπτέμβριος 2013 στις 20:42
Διαγραφή σχολίου
Καλησπέρα Μιχαήλ Αν και θεωρώ ότι το θέμα δεν είναι πιθανό να ζητηθεί - το θεωρώ απίθανο, σε πανελλαδικές, το κάνω σε καλούς μαθητές γιατί πιστεύω ότι έχει να προσφέρει αλλά και γιατί το αποτέλεσμα οδηγεί στο αναμενόμενο, όσο πιο μικρό κομμάτι ελατηρίου τόσο πιο σκληρό. Είναι σπουδαίο να συνειδητοποιήσει ο μαθητής ότι η δύναμη μεταφέρεται μέσω του ελατηρίου.
Σχόλιο από τον/την Μαλακασιώτης Νικόλαος στις 20 Σεπτέμβριος 2013 στις 14:14
Διαγραφή σχολίου
Μιχάλη έξυπνα διατυπωμένο και πολύ καλά λυμένο θέμα που δεν έχει ακόμη απασχολήσει τις εξετάσεις . Μανώλη έχεις δίκιο για τον τρόπο που αντιλαμβάνεται το φαινόμενο ο μαθητής.
Σχόλιο από τον/την Μιχαήλ Μιχαήλ στις 21 Σεπτέμβριος 2013 στις 19:53
Διαγραφή σχολίου
Νίκο, το να τεθεί θέμα με ελατήρια σε σειρά το θεωρώ και εγώ απίθανο. Το να ζητηθεί όμως το έργο της δύναμης F αυτό είναι πιθανό. Τι γίνεται όμως αν η δύναμη είναι μεταβλητή και πρέπει τα παιδιά να καταφύγουν σε γραφική παράσταση;
Σχόλιο από τον/την Μαλακασιώτης Νικόλαος στις 21 Σεπτέμβριος 2013 στις 20:59
Διαγραφή σχολίου
Εννοείς να δίνεται χώροεξάρτηση της F ή να προκύπτει απο την Fελ; Πάντως θα το κοιτάξω αλλά νομίζω ότι θα ξεφεύγει ,
112. ΜΙΑ ΑΡΜΟΝΙΚΗ ΤΑΛΑΝΤΩΣΗ ΠΟΥ ΔΕΝ ΕΙΝΑΙ ΑΠΛΗ.

Το σώμα Σ του σχήματος έχει μάζα , m= 1Kg  και αρχικά ισορροπεί δεμένο στο ένα άκρο οριζόντιου ελατηρίου σταθεράς Κ1=100N/m, ενώ απλώς ακουμπάει στο ελατήριο σταθεράς Κ2=800Ν/m. Το Σ βρίσκεται πάνω σε λείο οριζόντιο επίπεδο, ενώ τα ελατήρια έχουν το ίδιο φυσικό μήκος, όπως φαίνεται στο σχήμα. Απομακρύνουμε το σώμα Σ από τη θέση ισορροπίας του κατά d=0,6m και το αφήνουμε ελεύθερο. Να γίνει η γραφική παράσταση της δύναμης επαναφοράς που εξασκείται πάνω στο σώμα σε συνάρτηση με την απομάκρυνση x από τη Θ.Ί.Τ και μεταξύ των ακραίων θέσεων της ταλάντωσης.

β) Να γίνει η γραφική παράσταση της δυναμικής ενέργειας σε συνάρτηση με την απομάκρυνση x από τη Θ.Ί.Τ.

γ) Ποια είναι η περίοδος T της αρμονικής ταλάντωσης; Να γίνει η γραφική παράσταση της απομάκρυνσης x(t) για ......

 

Συνοπτική Λύση:

και

εδώ

Σχόλια
Σχόλιο από τον/την Κυριακόπουλος Γιάννης στις 17 Σεπτέμβριος 2013 στις 22:31
Διαγραφή σχολίου
Σχόλιο από τον/την Μιχαήλ Μιχαήλ στις 17 Σεπτέμβριος 2013 στις 22:41
Διαγραφή σχολίου
Δημήτρη σ' ευχαριστώ. Άντε με το καλό να επιστρέψεις. Γιάννη τι να πω! Μπράβο και σ' ευχαριστώ.
Σχόλιο από τον/την Γκενές Δημήτρης στις 17 Σεπτέμβριος 2013 στις 23:35
Διαγραφή σχολίου
Κορυφαία άσκηση. Μιχαήλ συγχαρητήρια.
Σχόλιο από τον/την ΜΑΝΩΛΗΣ ΔΡΑΚΑΚΗΣ στις 17 Σεπτέμβριος 2013 στις 23:51
Διαγραφή σχολίου
Μιχάλη πολύ καλό θέμα. Συγχαρητήρια. Αν και οι επί μέρους ταλαντώσεις είναι απλές , η συνολική ταλάντωση δεν είναι. Οφείλουν ασφαλώς όσοι την διδάξουν, να εξηγήσουν το γιατί.
Σχόλιο από τον/την ΧΡΗΣΤΟΣ ΕΛΕΥΘΕΡΙΟΥ στις 18 Σεπτέμβριος 2013 στις 0:19
Διαγραφή σχολίου
Καλησπέρα Μιχάλη.Πολύ όμορφη η περιπτώση.Θα μπορούσε βέβαια να γίνει γ.α.τ. αν....
Σχόλιο από τον/την Διονύσης Μάργαρης στις 18 Σεπτέμβριος 2013 στις 7:50
Διαγραφή σχολίου
Έστω και λίγο καθυστερημένα και από μένα τα συγχαρητήρια Μιχαήλ.
Σχόλιο από τον/την Μιχαήλ Μιχαήλ στις 18 Σεπτέμβριος 2013 στις 8:56
Διαγραφή σχολίου
Δημήτρη (Γκενέ), Μανώλη, Χρήστο, Διονύση σας ευχαριστώ πολύ. Χρήστο ίσως όταν το ένα ελατήριο βρίσκεται μέσα στο άλλο να είναι το πιο σωστό, αλλά είχα δυσκολία να το σχεδιάσω... Δημήτρη (Αναγνώστου) "φοβάμαι όλα αυτά που θα γίνουν για μένα χωρίς εμένα" ...ανδρομικά πάντα!
Σχόλιο από τον/την Κωστας Ψυλακος στις 18 Σεπτέμβριος 2013 στις 11:46
Διαγραφή σχολίου
Καλημερα Μ.Μ. ή Μ^2 !!! θα συμφωνησω με τον Δ.Α. οσον αφορα τα αναδρομικα συγχαρητηρια !!! Πολλες φορες μελετω τις αναρτησεις σου αλλα και το blog σου, ΜΠΡΑΒΟ ΣΟΥ!!!
Σχόλιο από τον/την Μιχαήλ Μιχαήλ στις 18 Σεπτέμβριος 2013 στις 13:06
Διαγραφή σχολίου
Κώστα σ' ευχαριστώ! Να' σαι πάντα καλά.
Σχόλιο από τον/την Βασίλης Δουκατζής στις 19 Σεπτέμβριος 2013 στις 2:29
Διαγραφή σχολίου
Μηχάλη καλημέρα. Μάλλον κάτι τέτοιο φανταζόσουν "Χρήστο ίσως όταν το ένα ελατήριο βρίσκεται μέσα στο άλλο να είναι το πιο σωστό, αλλά είχα δυσκολία να το σχεδιάσω..." Βέβαια στα έκανα Πολύχρωμα
113. Πότε ολισθαίνει;

Τα σώματα Σ1 και Σ2 του σχήματος έχουν μάζες Μ=4Kg και m=1Kg αντίστοιχα. Το Σ1 είναι δεμένο στην άκρη του ελατηρίου σταθεράς Κ=500N/m ενώ το Σ2 ακουμπά πάνω στο Σ1. Αρχικά το σύστημα ισορροπεί. Μετακινούμε τα σώματα ώστε το ελατήριο να συσπειρωθεί επιπλέον κατά A=ΔL=0,1m ενώ ασκούμε στο Σ2 μια σταθερή οριζόντια δύναμη F=2,5N όπως φαίνεται στο σχήμα.  Στη συνέχεια τη χρονική στιγμή t=0, αφήνουμε το σύστημα  ελεύθερο και αυτό πραγματοποιεί α.α.τ. Αν ο συντελεστής τριβής ολίσθησης μεταξύ των δυο σωμάτων είναι μ=0,5 να βρείτε:

α) Σε ποια θέση αρχίζει η ολίσθηση του Σ2; Ποια χρονική στιγμή γίνεται αυτό για πρώτη φορά;

β) Για πόσο χρόνο θα ολισθαίνει το Σ2 πάνω στο Σ1 μέχρι να σταματήσει η ολίσθηση για πρώτη φορά;

γ) Ποια είναι η συνολική επιτάχυνση του Σ2 στη θέση που το ελατήριο έχει το φυσικό του μήκος;

Δίνεται g=10m/s2 .

 

Συνοπτική λύση:

Σχόλια
Σχόλιο από τον/την Εμμανουήλ Λαμπράκης στις 24 Σεπτέμβριος 2013 στις 8:17
Διαγραφή σχολίου
Μιχαήλ καλημέρα Πολύ ενδιαφέρον θέμα. Από πρακτικής άποψης βέβαια πρέπει να ληφθεί πρόνοια ώστε να μην κάμπτεται το ελατήριο.
Σχόλιο από τον/την Βασίλης Δουκατζής στις 24 Σεπτέμβριος 2013 στις 8:37
Διαγραφή σχολίου
Μιχάλη ο Μανώλης έχει δίκαιο. Αν τα σώματα είναι υλικά σημεία τότε μόλις κινηθεί λίγο το ένα ως προς το άλλο θα αποχωριστούν και θα αλλάξει και η ταλάντωση. Αν πρόκειται για στερεά σώματα τότε μόλις το Σ2 κινηθεί το κέντρο βάρους δεν θα βρίσκεται πάνω στον άξονα του ελατηρίου, οπότε θα δημιουργηθεί ροπή και στην συνέχεια ανατροπή.  
Σχόλιο από τον/την Μιχαήλ Μιχαήλ στις 24 Σεπτέμβριος 2013 στις 20:13
Διαγραφή σχολίου
Μανώλη σ' ευχαριστώ. Βασίλη θα το βγάλω τελείως το ελατήριο ώστε να μην υπάρχει παρανόηση. Ευχαριστώ.
48. ΕΞΑΝΑΓΚΑΣΜΕΝΗ ΤΑΛΑΝΤΩΣΗ (ξανά)

Στη διάταξη του σχήματος δίνονται η σταθερά του ελατηρίου K=100N/m και ότι η μάζα του σώματος Σ είναι m=4Kg.

Το χέρι μας ασκεί περιοδική δύναμη F, και το σώμα Σ  εκτελεί εξαναγκασμένη  αρμονική ταλάντωση συχνότητας f1=Hz και πλάτους Α=4,4cm χωρίς αρχική φάση. Το σώμα κινούμενο δέχεται δύναμη αντίστασης Fαντ= -b×υ με σταθερά απόσβεσης b=0,4Kg×s-1.

α) Να γράψετε τις σχέσεις της απομάκρυνσης και της ταχύτητας του ταλαντωτή σε συνάρτηση με το χρόνο.

β) Να γράψετε την εξίσωση της δύναμης F του διεγέρτη σε συνάρτηση με το χρόνο.

γ) Να υπολογίσετε τη δύναμη του διεγέρτη τη χρονική στιγμή t, καθώς και το ρυθμό προσφερόμενης ενέργειας εκείνη τη στιγμή.

δ) Να γράψετε την εξίσωση της δύναμης F του διεγέρτη σε συνάρτηση με το χρόνο όταν έχουμε συντονισμό και να υπολογίσετε το ρυθμό προσφερόμενης ενέργειας τη στιγμή t.

Συνοπτική λύση:

Σχόλια
Σχόλιο από τον/την Κορκίζογλου Πρόδρομος στις 6 Οκτώβριος 2013 στις 23:36
Διαγραφή σχολίου
ΕΙΣΑΙ ΦΟΒΕΡΟΣ!!! Μελέτησες την εξαναγκασμένη ταλάντωση χωρίς διαφορικές. Μπράβο!!
Σχόλιο από τον/την Διονύσης Μάργαρης στις 7 Οκτώβριος 2013 στις 8:43
Διαγραφή σχολίου
Καλημέρα Μιχαήλ και συγχαρητήρια για τη νέα σου ανάρτηση. Διαβάζοντάς την μου ήρθε στο μυαλό, μια παλιότερη ανάρτηση του Πέτρου Καραπέτρου, τις πρώτες μέρες λειτουργίας του δικτύου μας:

Εξαναγκασμένη ταλάντωση

Νομίζω ότι αξίζει να την δουν τα μέλη που τότε δεν μας διάβαζαν. Πέτρο, βλέπω η μετάθεση στην Αθήνα, σου μείωσε την έμπνευση!!!!
Σχόλιο από τον/την Μιχαήλ Μιχαήλ στις 7 Οκτώβριος 2013 στις 11:02
Διαγραφή σχολίου
Πρόδρομε και Διονύση καλημέρα και σας ευχαριστώ. Διονύση η εξαναγκασμένη ταλάντωση μας έχει απασχολήσει αρκετά εδώ στο δίκτυο. Όπως εδώ και εδώ και και και και πολλές ακόμα!
Σχόλιο από τον/την Κυριακόπουλος Γιάννης στις 7 Οκτώβριος 2013 στις 15:09
Διαγραφή σχολίου
Πολύ καλή. Μια διευκρίνηση: Η μάζα της τροχαλίας είναι αμελητέα.
Σχόλιο από τον/την Μιχαήλ Μιχαήλ στις 7 Οκτώβριος 2013 στις 18:14
Διαγραφή σχολίου
Γιάννη έχεις δίκιο! Αλλά μη βάζεις ιδέες!!
Σχόλιο από τον/την Μαλακασιώτης Νικόλαος στις 17 Οκτώβριος 2013 στις 11:43
Διαγραφή σχολίου
Μιχαήλ καθυστερημένα συγχαρητήρια και απο εμένα. Κατάφερα να ολοκληρώσω την μελέτη της φοβερής αναρτησή σου όχι μόνο λόγω έλλειψης χρόνου αλλά και του υψηλού επιπέδου της γραφή σου. Βέβαια έκανα αρχικά ψάξιμο σε παλαιότερες αναρτήσεις συναδέλφων πάνω στο θέμα .
Σχόλιο από τον/την Μιχαήλ Μιχαήλ στις 17 Οκτώβριος 2013 στις 20:52
Διαγραφή σχολίου
Νίκο σ΄ευχαριστώ! Τα συγχαρητήρια ανήκουν σε όλα τα μέλη αυτού του δικτύου που τόσα μας προσφέρουν!
114. Σύνθεση δυο ταλαντώσεων (η μια φθίνουσα) και κρούση.

Σώμα μάζας m=1kg είναι δεμένο σε οριζόντιο ελατήριο σταθεράς K που αρχικά βρίσκεται στη θέση φυσικού του μήκους. Η μάζα m βρίσκεται σε επαφή με λεία οριζόντια επιφάνεια.

Απομακρύνουμε το σώμα από την αρχική του θέση κατά A1 και το αφήνουμε ελεύθερο να ταλαντώνεται.

α) Κάποια χρονική στιγμή που τη θεωρούμε αρχή των χρόνων (t=0) και που τότε η μάζα m βρίσκεται στη θέση x1=+A1/2 και έχει υ1>0, αρχίζει να πραγματοποιεί και μια δεύτερη Α.Α.Τ της ίδιας διεύθυνσης xx΄ γύρω από την ίδια θέση ισορροπίας, με την ίδια συχνότητα ω και με πλάτος Α2=2Α1. Τότε η εξίσωση ταλάντωσης της μάζας m, γίνεται x=2ριζα3×ημ(10t+2π/3) (S.I).Ποια είναι η εξίσωση της δεύτερης ταλάντωσης; Θεωρείστε πως για όλες τις ταλαντώσεις ισχύει η γενική εξίσωση x=A×ημ(ωt+φ).

β) Τη χρονική στιγμή t=s, η μάζα m συγκρούεται με σώμα μάζας m1=m που κινείται αντίθετα με ταχύτητα υ1=20m/s. Να υπολογιστεί η απώλεια της μηχανικής ενέργειας κατά την κρούση.  Ποια είναι τότε  η εξίσωση ταλάντωσης του συσσωματώματος μάζας m+m1;

γ) Αν κάποια χρονική στιγμή πριν από την κρούση που τη θεωρούμε πάλι  αρχή των χρόνων (t=0) η μάζα m αρχίζει να πραγματοποιεί φθίνουσα ταλάντωση με εξίσωση αυτή που υπολογίστηκε στο ερώτημα (α) όπου Α110×e-Λt, τότε να γράψετε την εξίσωση της συνισταμένης ταλάντωσης του σώματος.

Συνοπτική λύση:

Σχόλια
Σχόλιο από τον/την Κορκίζογλου Πρόδρομος στις 20 Οκτώβριος 2013 στις 17:28
Διαγραφή σχολίου
Συγχαρητήρια Μιχαήλ για την πολύ έξυπνη άσκησή σου. Πάντα τέτοια.
Σχόλιο από τον/την Μιχαήλ Μιχαήλ στις 20 Οκτώβριος 2013 στις 18:32
Διαγραφή σχολίου
Πρόδρομε σ' ευχαριστώ! Συγχαρητήρια για την προσφορά σου!
Σχόλιο από τον/την Κυριακόπουλος Γιάννης στις 20 Οκτώβριος 2013 στις 18:44
Διαγραφή σχολίου
Μιχάλη καλησπέρα. Δεν έχω καταλάβει κάτι: Πως η κίνηση της λείας επιφάνειας θα επηρεάσει την κίνηση του m ; Μιλάς για κίνηση ως προς την πλάκα;
Σχόλιο από τον/την Μιχαήλ Μιχαήλ στις 20 Οκτώβριος 2013 στις 18:50
Διαγραφή σχολίου
Γιάννη ίσως δεν έχω κάνει καλή διατύπωση άσε με να το σκεφτώ και θα επανέλθω!
Σχόλιο από τον/την Μιχαήλ Μιχαήλ στις 20 Οκτώβριος 2013 στις 20:28
Διαγραφή σχολίου
Γιάννη σ' ευχαριστώ! Ήδη έκανα την αλλαγή που μου είπες!
Σχόλιο από τον/την Μαλακασιώτης Νικόλαος στις 31 Οκτώβριος 2013 στις 13:26
Διαγραφή σχολίου
Πολύ καλή άσκηση Μιχαήλ και ωραία συνδυασμένα θέματα . Καταλυτική και η παρατήρηση του Γιάννη. Άρα η σελίδα 25 του σχολικού που μελετά την σύνθεση ταλάντωση του σώματος που οφείλεται και στη ταλάντωση της λείας βάσης είναι λανθασμένη , αφού αυτή δεν μπορεί να επηρεάζει την κίνηση του σώματος.
Σχόλιο από τον/την Μιχαήλ Μιχαήλ στις 31 Οκτώβριος 2013 στις 21:25
Διαγραφή σχολίου
Οπότε Νίκο το ερώτημα είναι πως μπορούμε να πετύχουμε σύνθεση ταλαντώσεων στο εργαστήριο;
Σύνθεση ταλαντώσεων-Γεωμετρική κατασκευή του Fresnel

Σύνθεση δύο απλών αρμονικών ταλαντώσεων της ίδιας συχνότητας, που γίνονται γύρω από το ίδιο σημείο στην ίδια διεύθυνση:

Έστω ότι ένα σώμα Σ κάνει ταυτόχρονα δυο (ή περισσότερες) απλές αρμονικές ταλαντώσεις με εξισώσεις:

x1=A1ημωt        και         x2=A2ημ(ωt+φ)

Η απομάκρυνσή, η ταχύτητά και η επιτάχυνση της συνισταμένης ταλάντωσης κάθε στιγμή είναι ίση με το αλγεβρικό άθροισμα των απομακρύνσεών, των ταχυτήτων και των επιταχύνσεων των επιμέρους (συνιστωσών) ταλαντώσεων στις οποίες μετέχει.

Δηλαδή ισχύει: x=x1+x2, υ=υ12 και α=α12.

Η μορφή της συνισταμένης ταλάντωσης εξαρτάται από τη διαφορά φάσης φ, των δυο συνιστωσών κινήσεων.

εδώ

Φυσική Γ΄ Λυκείου Γενικής Παιδείας

Περιεχόμενα

 

ΚΕΦΑΛΑΙΟ 1

ΤΟ  ΦΩΣ

 

1.1 Η ΦΥΣΗ ΤΟΥ ΦΩΤΟΣ ………………………………………………………………………1

1.2 Η ΤΑΧΥΤΗΤΑ ΤΟΥ ΦΩΤΟΣ……………………….........................................7

1.3 ΜΗΚΟΣ ΚΥΜΑΤΟΣ ΚΑΙ ΣΥΧΝΟΤΗΤΑ ΤΟΥ ΦΩΤΟΣ ΚΑΤΑ ΤΗ ΔΙΑΔΟΣΗ ΤΟΥ………………………………………………………………………………. 21

1.4 ΑΝΑΛΥΣΗ ΛΕΥΚΟΥ ΦΩΤΟΣ ΚΑΙ ΧΡΩΜΑ…………………………….47

1.5 ΠΟΛΩΣΗ ΤΟΥ ΦΩΤΟΣ…………………………………………………………………..71

 

ΚΕΦΑΛΑΙΟ 2

ΑΤΟΜΙΚΑ ΦΑΙΝΟΜΕΝΑ

 

2.1 ΕΝΕΡΓΕΙΑ ΤΟΥ ΗΛΕΚΤΡΟΝΙΟΥ ΣΤΟ ΑΤΟΜΟ ΤΟΥ ΥΔΡΟΓΟΝΟΥ…………………………………………………………………………………………..99

2.2 ΔΙΑΚΡΙΤΕΣ ΕΝΕΡΓΕΙΑΚΕΣ ΣΤΑΘΜΕΣ………………………………….124

2.3 ΜΗΧΑΝΙΣΜΟΣ ΠΑΡΑΓΩΓΗΣ ΚΑΙ ΑΠΟΡΡΟΦΗΣΗΣ ΦΩΤΟΝΙΩΝ…………………………………………………………………………………………….128

2.4 ΑΚΤΙΝΕΣ  Χ……………………………………………………………………………………157

 

ΚΕΦΑΛΑΙΟ 3

ΠΥΡΗΝΙΚΑ ΦΑΙΝΟΜΕΝΑ

 

3.1 ΙΔΙΟΤΗΤΕΣ ΤΩΝ ΠΥΡΗΝΩΝ…..…………………………………………………190

3.2 ΤΑ ΣΤΟΙΧΕΙΩΔΗ ΣΩΜΑΤΙΔΙΑ ………………………………………………..222

3.3 Η ΡΑΔΙΕΝΕΡΓΕΙΑ ………………………………………………………………………..242

3.4 ΠΥΡΗΝΙΚΕΣ ΑΝΤΙΔΡΑΣΕΙΣ………………………………………………………280

3.5 ΕΦΑΡΜΟΓΕΣ ΚΑΙ ΚΙΝΔΥΝΟΙ ΤΗΣ ΡΑΔΙΕΝΕΡΓΕΙΑΣ……… 309

 

ΚΕΦΑΛΑΙΟ 4

ΕΦΑΡΜΟΓΕΣ

 

4.1 ΕΙΔΗ ΛΑΜΠΤΗΡΩΝ……………………………………………………………………..321

 

ΟΙ ΑΠΑΝΤΗΣΕΙΣ ΤΩΝ ΕΡΩΤΗΣΕΩΝ ΚΑΙ ΤΩΝ ΑΣΚΗΣΕΩΝ

ΤΟΥ ΣΧΟΛΙΚΟΥ ΒΙΒΛΙΟΥ…………………………………………………………….. 335

 

ΟΙ ΑΠΑΝΤΗΣΕΙΣ ΤΩΝ ΕΡΩΤΗΣΕΩΝ ΚΑΙ ΤΩΝ ΑΣΚΗΣΕΩΝ

ΤΟΥ ΒΙΒΛΙΟΥ………………………………………………………………………………………386

Εδώ

και εδώ

Σχόλια
Σχόλιο από τον/την ΧΡΗΣΤΟΣ ΕΛΕΥΘΕΡΙΟΥ στις 5 Νοέμβριος 2013 στις 1:11
Διαγραφή σχολίου
Mιχάλη MΠΡΑΒΟ για την τεράστια προσπάθεια σου.Η φυσική γενικής παιδείας πραγματικά είναι η ομορφότερη φυσική που υπάρχει στο Λύκειο.Δυστυχώς όμως είναι και η πιο παραμελημένη...Σε ευχαριστούμε που μοιράστηκες μαζί μας την τόσο όμορφη Φυσική...
Σχόλιο από τον/την Φιορεντίνος Γιάννης στις 5 Νοέμβριος 2013 στις 10:42
Διαγραφή σχολίου
Καλημέρα σε όλους. Μιχαήλ, πολλά συγχαρητήρια για την εκπληκτική προσφορά σου! Σε ευχαριστώ πολύ!
Σχόλιο από τον/την Μιχαήλ Μιχαήλ στις 5 Νοέμβριος 2013 στις 12:27
Διαγραφή σχολίου
Χρήστο, Γιάννη, Δημήτρη σας ευχαριστώ από την καρδιά μου. Μακάρι να είμαστε γεροί όλοι μας και να προσφέρουμε όσο μπορούμε. Συγχαρητήρια για τις δουλειές σας. Σας παρακολουθώ παρόλο που δεν προλαβαίνω να σχολιάσω!
Σχόλιο από τον/την Μαλακασιώτης Νικόλαος στις 5 Νοέμβριος 2013 στις 13:47
Διαγραφή σχολίου
Συγχαρητήρια Μιχαήλ και απο εμένα , μακάρι η συγκεκριμένη ύλη να μην χαθεί στο νέο Λύκειο .
Σχόλιο από τον/την Διονύσης Μάργαρης στις 5 Νοέμβριος 2013 στις 14:23
Διαγραφή σχολίου
Μιχαήλ συγχαρητήρια για την παραπάνω ολοκληρωμένη δουλειά σου σε ένα από τα ομορφότερα κομμάτια φυσικής που "υποτίθεται" ότι διδάσκεται ακόμη στο Λύκειο... Σε ευχαριστώ που την μοιράστηκες μαζί μας. Να είσαι καλά.
Σχόλιο από τον/την Μιχαήλ Μιχαήλ στις 5 Νοέμβριος 2013 στις 20:41
Διαγραφή σχολίου
Νίκο και Διονύση σας ευχαριστώ! Δυστυχώς η Φυσική Γενικής Παιδείας της Γ΄ Λυκείου λίγους πλέον ενδιαφέρει!!!
Σχόλιο από τον/την Σαλής Αντωνης στις 30 Νοέμβριος 2013 στις 22:05
Διαγραφή σχολίου
Φέτος μετα απο αρκετα χρονια επελεξε ενας μαθητής μου να εξεταστει πανελλαδικα στη φυσική γενικής παιδείας οποτε αυτό το βιβλίο μου είναι παρα πολυ χρήσιμο στην προσπάθεια μου να τον διδάξω όσο πιο καλά μπορω.Σας ευχαριστώ παρα πολύ για την προσφορά Υ.Γ: Αυτό βέβαια που δεν μπορώ να βρώ εύκολα στα βιβλία είναι κλασικά δεύτερα θέματα. Αν κάποιος συνάδελφος γνωρίζει ή έχει καποιο υλικο θα με βοηθούσε πολυ! ευχαριστώ.
Σχόλιο από τον/την Μιχαήλ Μιχαήλ στις 2 Δεκέμβριος 2013 στις 18:18
Διαγραφή σχολίου
Αντώνη μακάρι να σου φανεί χρήσιμο. Σ' ευχαριστώ.
117. Σώμα δεμένο ανάμεσα σε δυο ελατήρια και τριβή.

Το σώμα Σ του σχήματος, μάζας m=1Kg ισορροπεί οριακά πάνω σε οριζόντιο τραπέζι δεμένο σε δυο οριζόντια ελατήρια με σταθερές Κ1=40Ν/m και Κ2=10Ν/m, (αν απομακρύνουμε ελάχιστα το σώμα προς τα δεξιά αυτό αρχίζει να ολισθαίνει). Το ελατήριο Κ1 είναι επιμηκυμένο κατά x1=12 cm από τη θέση φυσικού του μήκους ενώ το ελατήριο Κ2 είναι επιμηκυμένο κατά x2=28cm από τη θέση φυσικού του μήκους. Τότε

α) Να υπολογίσετε το συντελεστή στατικής τριβής που εδώ θεωρούμε πως είναι ίσος με το συντελεστή τριβής ολίσθησης.

β) Εξασκούμε στο σώμα μια οριζόντια δύναμη F και το μετατοπίζουμε με σταθερή ταχύτητα, κατά x=3cm προς τα δεξιά από την αρχική θέση ισορροπίας του. Να υπολογίσετε το έργο της δύναμης F.

γ) Αφήνουμε στη συνέχεια το σώμα ελεύθερο. Να υπολογίσετε την ταχύτητά του όταν περνά από την αρχική θέση ισορροπίας του.

δ) Που θα ισορροπήσει τελικά το σώμα; Ποια είναι τότε η τιμή της στατικής τριβής μεταξύ του Σ και του τραπεζιού; Δίνεται g=10m/s2.

 

Συνοπτική Λύση:

Σχόλια
Σχόλιο από τον/την Εμμανουήλ Λαμπράκης στις 11 Νοέμβριος 2013 στις 19:18
Διαγραφή σχολίου
Μιχαήλ καλησπέρα Πολύ ενδιαφέρον και πρωτότυπο θέμα.
Σχόλιο από τον/την Νίκος Δαπόντες στις 11 Νοέμβριος 2013 στις 20:00
Διαγραφή σχολίου
Μιχαήλ, μ' ενδιαφέρει το θέμα μια και θα ήθελα να επεκτείνω ένα project μου http://scratch.mit.edu/projects/11496805/ Έτσι, σ' ευχαριστώ για το ερέθισμα...
Σχόλιο από τον/την Μιχαήλ Μιχαήλ στις 11 Νοέμβριος 2013 στις 20:44
Διαγραφή σχολίου
Μανώλη σ' ευχαριστώ! Νίκο, χαίρομαι που θα επεκτείνεις την ιδέα της άσκησης και στο scratch.
Σχόλιο από τον/την Βαγγέλης Κουντούρης στις 11 Νοέμβριος 2013 στις 22:17
Διαγραφή σχολίου
καλή άσκηση Μιχαήλ, αλλά και αρκετά “μπελαλίδικη” παρατηρήσεις: στο ερώτημα β. πρέπει να δίδεται ότι η ταχύτητα να είναι περίπου μηδέν διαφωνώ με τη μεθοδολογία της λύσης στο δ., διότι επιλέγεις το Α΄δεξιά από τη Θ.Ι., επειδή εσύ το …ξέρεις νομίζω, σωστότερα είναι να γίνει διερεύνηση ως εξής: θεωρούμε ότι το σώμα φτάνει στη Θ.Ι. και υπολογίζουμε την κινητική του ενέργεια εκεί και αν βρούμε: α. Κ=0, πανηγυρίζουμε, τέλος η άσκηση β. Κ>0, ωχ, συνεχίζει κι άλλο, άρα σταματά αριστερά από τη Θ.Ι. , νέα διαδιακσία γ. Κ<0, ωχ, έχει ήδη σταματήσει πριν, δεξιά δηλαδή, άρα η διαδικασία που κάνεις
Σχόλιο από τον/την Μιχαήλ Μιχαήλ στις 11 Νοέμβριος 2013 στις 23:42
Διαγραφή σχολίου
Βαγγέλη το σώμα στη θέση ισορροπίας έχει κινητική ενέργεια, που υπολογίστηκε στο ερώτημα (γ).
Σχόλιο από τον/την Βαγγέλης Κουντούρης στις 11 Νοέμβριος 2013 στις 23:51
Διαγραφή σχολίου
Ωχ, Μιχαήλ ήθελα να γράψω ΘΦΜ1, και έγραψα ΘΙ (και δεν ...μοιάζουν κιόλας!)
Σχόλιο από τον/την Κορκίζογλου Πρόδρομος στις 13 Νοέμβριος 2013 στις 1:03
Διαγραφή σχολίου
Πολύ καλή ,μας θύμισε και λίγο τις Δέσμες!
Σχόλιο από τον/την Μιχαήλ Μιχαήλ στις 13 Νοέμβριος 2013 στις 12:29
Διαγραφή σχολίου
Πρόδρομε σε ευχαριστώ! Έχεις δίκιο. Όμως υπάρχει και το περσινό 3ο θέμα!
118. Αρμονικό κύμα.

Ένα αρμονικό κύμα διαδίδεται προς τη θετική κατεύθυνση του άξονα xox΄ και περιγράφεται από την εξίσωση y=2∙ημ2π(t/6-x/2+φ0/2π) (S.I).

α) Αν για t=0 η πηγή Ο με x=0, βρίσκεται στο y=0 και έχει ταχύτητα ταλάντωσης v>0 ποια είναι η αρχική φάση του κύματος;

β) Αν όμως για t=0 το κύμα ξεκινώντας από την αρχή Ο (x=0) έχει φτάσει στο σημείο Μ που βρίσκεται στη θέση x=+1m, τότε να βρείτε ποια είναι η αρχική φάση φ0 του κύματος και να γράψετε την εξίσωσή του, δεδομένου ότι τη στιγμή που φτάνει το κύμα στο σημείο Μ, τότε το εξαναγκάζει να κινηθεί κατακόρυφα προς τα κάτω (v<0). Για πόσο χρόνο ταλαντώνεται τότε η πηγή;

γ) Να σχεδιάσετε το στιγμιότυπο του κύματος τη χρονική στιγμή t0=0 και τη χρονική στιγμή t1=Τ/2s.

δ) Πόσο έχει μεταβληθεί η φάση της πηγής Ο από τη χρονική στιγμή t0=0 μέχρι τη χρονική στιγμή t1=Τ/2;

ε) Αν για t=0 η πηγή Ο με x=0, ξεκινά να ταλαντώνεται, με ταχύτητα ταλάντωσης v<0, τότε ποια είναι η αρχική φάση του κύματος; Να γράψετε την καινούργια εξίσωση του κύματος και να σχεδιάσετε το στιγμιότυπο του κύματος τη χρονική στιγμή t=3,5 s.

Συνοπτική Λύση:

Σχόλια
Σχόλιο από τον/την Εμμανουήλ Λαμπράκης στις 15 Νοέμβριος 2013 στις 10:06
Διαγραφή σχολίου
Μιχαήλ καλημέρα Εξαιρετικό θέμα.
Σχόλιο από τον/την Μαλακασιώτης Νικόλαος στις 15 Νοέμβριος 2013 στις 14:01
Διαγραφή σχολίου
Πολύ καλή άσκηση Μιχαήλ . Αναφέρεις σα αρχική φάση του κύματος τη φάση της ταλάντωση του χ=0 την t=0 για πιο λόγο ;
Σχόλιο από τον/την Μιχαήλ Μιχαήλ στις 15 Νοέμβριος 2013 στις 20:16
Διαγραφή σχολίου
Μανώλη και Νίκο σας ευχαριστώ! Νίκο εκείνη τη στιγμή η πηγή ξεκινά την ταλάντωσή της!
Σχόλιο από τον/την Ηλίας Ζαρνάς στις 15 Νοέμβριος 2013 στις 20:42
Διαγραφή σχολίου
Καλησπέρα Μιχαήλ .Θίγεις ένα κρίσιμο σημείο που σίγουρα υπάρχουν διάφορες απόψεις . Αν τη στιγμή τ=0 το Ο κινείται με -Vmax υπάρχουν 2 τεχνικές αντιμετώπισης Α. είναι ο τρόπος που εφαρμόζεις και συ Μιχαήλ , που θεωρείς την φάση κάθε σημείου τη στιγμή που αρχίζει να ταλαντώνεται ιση με π Β. Κάποιοι γράφουν την εξίσωση κύματος ως ψ=-Αημ2π(τ/Τ - χ/λ) και θεωρούν σαν φάση πάλι την 2π(τ/Τ - χ/λ) και παίρνουν τη φάση κάποιου σημείου ιση με μηδέν τη στιγμή που φτάνει το κύμα . ( Ουσιαστικά είναι σαν να παίρνουν θετική φορά προς τα κάτω) . Με αυτό τον τρόπο η αρχική φάση του κύματος δίνει πάντα την ίδια πληροφορία δηλ πόσο προχωρημένο είναι το κύμα τη στιγμή τ=0. Μακάρι να ήξερα τι είναι προτιμότερο απο τους μαθητές . Μάλλον να αποφύγουμε θέματα με αρχική φάση . Θα ήθελα τη γνώμη σου. Νομίζω οτι καταργείς με τον καλύτερο τρόπο τα πάσης φύσεως μαγικά για το σχεδιασμό του στιγμιότυπου πχ ξεκινάμε απ το τέλος και διάφορα τέτοια που κυκλοφορούν.
Σχόλιο από τον/την Μαλακασιώτης Νικόλαος στις 15 Νοέμβριος 2013 στις 21:01
Διαγραφή σχολίου
Δεν είδα οτι το χ=0 ήταν η πηγή του κύματος και θεώρησα οτι το κύμα έχει φορά απο τα αριστερά προς τα δεξιά. Φυσικά Μιχαήλ η φάση της ταλάντωσης του χ=0 την t=0 είναι φ=2π. Τα καλό Ηλία με το δεύτερο τρόπο που αναφέρεις δεν σου <φεύγει > το κύμα κάθε φορά κατα λ\2 όπως συμβαίνει όταν το χ=0 την t=0 έχει ψ=0 και v<0 χωρίς να πηγαίνει απευθείας προς τα αρνητικά .
Σχόλιο από τον/την Μιχαήλ Μιχαήλ στις 16 Νοέμβριος 2013 στις 11:42
Διαγραφή σχολίου
Ηλία και Νίκο, μπορείτε να δείτε και τη διαφωτιστική ανάρτηση του συναδέλφου Καραπέτρου Π. εδώ
119. Συμβολή

Δύο σύγχρονες πηγές κυμάτων Π1 και Π2 απέχουν μεταξύ τους απόσταση d=1,5m και εκτελούν απλή αρμονική ταλάντωση στην επιφάνεια υγρού με εξίσωση

y=2∙10-2ημ(40πt) (S.I).

Τα εγκάρσια κύματα που δημιουργούνται στην επιφάνεια του υγρού διαδίδονται με ταχύτητα υ=10m/s και φτάνουν στο υλικό σημείο Μ που απέχει από τις πηγές Π1 και Π2 αποστάσεις r1=2m και r2=1m, αντίστοιχα. Τότε:

 

α) Τη χρονική στιγμή t=19/80s να γίνει η γραφική παράσταση της απομάκρυνσης y για όλα τα σημεία της υπερβολής  στην οποία ανήκει το Μ και που βρίσκονται μεταξύ των Μ και Μ΄. Μ΄ είναι το σημείο στο οποίο τέμνει η υπερβολή στην οποία ανήκει το Μ το ευθύγραμμο τμήμα Π1Π2.

 

  β) Αν η αμέσως προηγούμενη υπερβολή με το ίδιο πλάτος ταλάντωσης τέμνει το Π1Π2 στο σημείο Ν΄, τότε να γίνει η γραφική παράσταση της απομάκρυνσης y για όλα τα σημεία του Π1Π2 που βρίσκονται μεταξύ των Ν΄ και Μ΄ και για τη χρονική στιγμή t=19/80s.

 

Συνοπτική Λύση:

Σχόλια
Σχόλιο από τον/την Εμμανουήλ Λαμπράκης στις 23 Νοέμβριος 2013 στις 22:22
Διαγραφή σχολίου
Μιχαήλ καλησπέρα Πολύ ενδιαφέρον και πρωτότυπο θέμα. Για την περιγραφή των απομακρύνσεων πάνω στην υπερβολή χρησιμοποιείς το s για την ανεξάρτητη μεταβλητή αλλά στο πεδίο ορισμού το κάνεις x δες το.
Σχόλιο από τον/την Μιχαήλ Μιχαήλ στις 23 Νοέμβριος 2013 στις 22:43
Διαγραφή σχολίου
Μανώλη σ' ευχαριστώ. Το άλλαξα!
122. Συμβολή II

Δυο σύγχρονες πηγές κυμάτων Π1 και Π2 βρίσκονται στην ήρεμη επιφάνεια υγρού και πραγματοποιούν α.α.τ με εξισώσεις y=0,2ημ4πt  (S.I), δημιουργώντας εγκάρσια κύματα που διαδίδονται στην επιφάνεια του. Σ’ ένα σημείο Μ της επιφάνειας του υγρού συμβάλλουν τα δυο κύματα. Αν το σημείο Μ βρίσκεται πάνω στην υπερβολή ενισχυτικής συμβολής, που περιγράφεται από την εξίσωση x2/4-y2/5=1 (x,y είναι οι συντεταγμένες του σημείου Μ (m) ως προς το ορθογώνιο σύστημα συντεταγμένων με αρχή το μέσο Ο του ευθυγράμμου τμήματος Π1Π2) τότε:

α) Να υπολογίσετε την ταχύτητα διάδοσης των κυμάτων θεωρώντας ότι το Μ βρίσκεται στη δεύτερη υπερβολή ενισχυτικής συμβολής μετρώντας από το Ο, καθώς και την απόσταση d των δύο πηγών.

β) Να υπολογίσετε τον αριθμό των σημείων του ευθυγράμμου τμήματος Π1Π2  που έχουν πλάτος ταλάντωσης |Α΄|=0,2sqr3m.

γ) Να γράψετε την εξίσωση της υπερβολής στην οποία ανήκει το πιο κοντινό σημείο στο σημείο Ο του ευθυγράμμου τμήματος Π1Π2 που ταλαντώνεται με πλάτος   |Α΄|=0,2sqr3m και βρίσκεται στον άξονα +x.

Συνοπτική λύση:

Σχόλια
Σχόλιο από τον/την Εμμανουήλ Λαμπράκης στις 7 Δεκέμβριος 2013 στις 19:57
Διαγραφή σχολίου
Μιχαήλ καλησπέρα Συγχαρητήρια για το εξαιρετικό αυτό θέμα. Η ιδέα με τις εξισώσεις της παραβολής το κάνει εξαιρετικά πρωτότυπο κατά την δική μου αντίληψη.
Σχόλιο από τον/την Μιχαήλ Μιχαήλ στις 8 Δεκέμβριος 2013 στις 8:42
Διαγραφή σχολίου
Μανώλη καλημέρα και σ' ευχαριστώ πολύ. Να' σαι πάντα καλά και να μας προσφέρεις τις ωραίες σου σκέψεις.
123. Στάσιμο κύμα χωρίς δεσμούς.

Ένα τρέχον κύμα παράγεται κατά μήκος μιας τεντωμένης χορδής πεπερασμένου μήκους από πηγή που ταλαντώνεται με συχνότητα f=5Hz . Το κύμα που παράγεται κινείται με ταχύτητα υ=0,2m/s και προσπίπτει πάνω σ’ ένα εμπόδιο οπότε ανακλάται μερικώς. Θεωρούμε ότι η ανάκλαση γίνεται χωρίς μεταβολή της φάσης. Τα δυο κύματα μετά την ανάκλαση  κινούνται σε αντίθετες διευθύνσεις και έχουν διαφορετικά πλάτη Α1=30cm (προσπίπτων κύμα) και Α2=10cm (ανακλώμενο κύμα). Μετά τη συμβολή τους παράγεται στάσιμο «κύμα» (παρόλο που τα πλάτη είναι άνισα). Τότε:

α) Να γράψετε την εξίσωση του στάσιμου κύματος που παράγεται.

β) Να βρείτε μεταξύ ποιων τιμών Αmax  και Αmin μεταβάλλεται το πλάτος |Α΄| του στάσιμου κύματος και να σχεδιάσετε τη γραφική παράσταση |Α΄(x)|.

γ) Να υπολογιστεί ο λόγος του στάσιμου κύματος. Ποια είναι η τιμή του παραπάνω λόγου για 100% ανάκλαση καθώς και για καθόλου ανάκλαση;

δ) Βρείτε το % ποσοστό ανάκλασης του παραπάνω τρέχοντος κύματος πάνω στο εμπόδιο.

 

Συνοπτική λύση:

Σχόλια
Σχόλιο από τον/την Εμμανουήλ Λαμπράκης στις 21 Δεκέμβριος 2013 στις 21:37
Διαγραφή σχολίου
Μιχαήλ καλησπέρα πολύ ωραίο γενικό θέμα επί των "πραγματικών" στάσιμων. Διόρθωσε το πρόσημο της φ2 στην y2 σε "+"
Σχόλιο από τον/την Μαλακασιώτης Νικόλαος στις 22 Δεκέμβριος 2013 στις 0:10
Διαγραφή σχολίου
Μπράβο Μιχάλη μια ακόμη πολύ καλή άσκηση .
Σχόλιο από τον/την Κορκίζογλου Πρόδρομος στις 22 Δεκέμβριος 2013 στις 0:15
Διαγραφή σχολίου
Συμφωνώ με τον Μανώλη, μπράβο Μιχαήλ!!! Ανοίγεις νέα μονοπάτια.
Σχόλιο από τον/την Μιχαήλ Μιχαήλ στις 22 Δεκέμβριος 2013 στις 10:42
Διαγραφή σχολίου
Μανώλη, Νίκο, Πρόδρομε καλημέρα σας και σας ευχαριστώ. Μανώλη έκανα τη διόρθωση!
124. Ανάκλαση - διάθλαση.

Το παρακάτω σύστημα αποτελείται από πέντε επίπεδες στρώσεις γυάλινων πλακών διαφορετικού πάχους και διαφορετικών δεικτών διάθλασης όπως φαίνεται στο σχήμα.

Αν μια παράλληλη μονοχρωματική δέσμη laser πέφτει πάνω στην 1η πλάκα που έχει δείκτη διάθλασης n1 με γωνία πρόσπτωσης  θ1=300 όπως φαίνεται στο σχήμα τότε:

 

α) Να βρείτε τη γωνία που σχηματίζει η αρχική και η τελική δέσμη.

 

β) Τι τιμή πρέπει να έχει ο δείκτης διάθλασης n5 ώστε η διεύθυνση διαδόσεως της εξερχόμενης δέσμης στην πλάκα n5, να είναι παράλληλη στη διεύθυνση της προσπίπτουσας δέσμης;

 

γ) Στη συνέχεια απομακρύνουμε από το σύστημα των πλακών όλες τις άλλες πλάκες εκτός της πρώτης που έχει δείκτη διάθλασης n1. Αν η μονοχρωματική δέσμη laser πέφτει από τον αέρα πάνω στην πλάκα με γωνία πρόσπτωσης θα=300, τότε ένα μέρος της δέσμης ανακλάται από την πάνω επιφάνεια και ένα μέρος από την κάτω, ενώ στη συνέχεια εξέρχεται και αυτή από την πάνω επιφάνεια. Τότε να δείξετε ότι η ανακλώμενη δέσμη στην πάνω επιφάνεια της πλάκας, με την εξερχόμενη από την πάνω επιφάνεια της, είναι παράλληλες.

 

Συνοπτική λύση:

Σχόλια
Σχόλιο από τον/την Διονύσης Μάργαρης στις 29 Δεκέμβριος 2013 στις 8:34
Διαγραφή σχολίου
Καλημέρα και χρόνια πολλά Μιχαήλ. Ωραία η άσκηση και πάνω σε ένα θέμα, που πάντα είχα μια "αδυναμία":-) Σε ευχαριστούμε.
Σχόλιο από τον/την ΜΑΝΩΛΗΣ ΔΡΑΚΑΚΗΣ στις 29 Δεκέμβριος 2013 στις 10:49
Διαγραφή σχολίου
Καλημέρα Μιχάλη. Όταν έφτιαχνα το θέμα με τις τέσσερεις πλάκες , είχα το ενδοιασμό μήπως είναι πολλές. Εσύ έβαλες πέντε ! Πολύ καλή δουλειά. Επιλέγω το 3ο ερώτημα ως ιδιαίτερα κρίσιμο...
Σχόλιο από τον/την Ηλίας Ζαρνάς στις 29 Δεκέμβριος 2013 στις 12:10
Διαγραφή σχολίου
Καλημέρα Μιχαήλ και χρόνια πολλά .Νομίζω πως όσα στρώματα από γυάλινες πλάκες κι αν βάλεις δεν πρόκειται να συμβεί ολική ανάκλαση σε κάποια διαχωριστική επιφάνεια ,εφ όσον η αρχική ακτίνα προέρχεται από τον αέρα.
Σχόλιο από τον/την Μιχαήλ Μιχαήλ στις 29 Δεκέμβριος 2013 στις 13:39
Διαγραφή σχολίου
Διονύση καλημέρα και Χρόνια Πολλά. Τις ασκήσεις σου που διαπραγματεύονται το συγκεκριμένο θέμα, και τις έχεις ανεβάσει στο yliko τις έχω υπόψη μου και είναι όλες εξαιρετικές. Σ΄ευχαριστώ! Μανώλη σ΄ευχαριστώ. Η αλήθεια είναι πως όταν είδα την άσκησή σου υπέθεσα πως με πρόλαβες. Η δουλειά σου με εντυπωσίασε και διαπραγματεύεται κάτι πρωτότυπο που δεν έχω ξαναδεί! Χρόνια Πολλά! Ηλία Χρόνια Πολλά. Έχεις δίκιο. Όμως η αρχική ακτίνα εδώ βρίσκεται μέσα στην πλάκα n1. Σ' ευχαριστώ να' σαι πάντα καλά!
Σχόλιο από τον/την Εμμανουήλ Λαμπράκης στις 29 Δεκέμβριος 2013 στις 14:31
Διαγραφή σχολίου
Μιχαήλ καλό μεσημέρι και πάλι. Πολύ ωραίο και έξυπνα δοσμένο θέμα. Μου άρεσε ιδιαίτερα που στο δεύτερο ερώτημα απαντάς σε ένα μάλλον "κλασσικό" θέμα θεωρώντας το ως ειδική περίπτωση - φαντάζομαι καταλαβαίνεις τι εννοώ. Φαίνεται ότι με το Μανώλη Δρακάκη κάνατε παράλληλες σκέψεις επί παράλληλων πλακών!
Σχόλιο από τον/την Μιχαήλ Μιχαήλ στις 29 Δεκέμβριος 2013 στις 21:06
Διαγραφή σχολίου
Μανώλη σ' ευχαριστώ να' σαι πάντα καλά! .....ελπίζω όχι σε παράλληλα Σύμπαντα!
Σχόλιο από τον/την ΓΙΑΝΝΗΣ ΔΟΓΡΑΜΑΤΖΑΚΗΣ στις 29 Δεκέμβριος 2013 στις 21:10
Διαγραφή σχολίου
Καλησπέρα Μιχαήλ. Χρόνια πολλά. Ωραία πρόταση. Και ''κάτι'' που είχα παρατηρήσει πειραματικά. Στις περιπτώσεις των πολλών διαδοχικών διαθλάσεων παρατηρείται μια σημαντική εξασθένιση της διαθλώμενης δέσμης λόγω των πολλών ανακλάσεων. Να'σαι καλά.
Σχόλιο από τον/την Μιχαήλ Μιχαήλ στις 29 Δεκέμβριος 2013 στις 21:21
Διαγραφή σχολίου
Γιάννη καλησπέρα και Χρόνια Πολλά! Ανοίγεις μεγάλο θέμα νομίζω....
Ένα χειμωνιάτικο παραμύθι

Η ιστορία μιας μικρής χιονονιφάδας

Σχόλια
Σχόλιο από τον/την Κυριακόπουλος Γιάννης στις 31 Δεκέμβριος 2013 στις 15:54
Διαγραφή σχολίου
Ωραίο Μιχάλη. Φανερώνεις και άλλο ταλέντο.
Σχόλιο από τον/την Γκενές Δημήτρης στις 31 Δεκέμβριος 2013 στις 16:12
Διαγραφή σχολίου
Μιχαήλ Πολύ όμορφο ... Καλή χρονιά
Σχόλιο από τον/την Διονύσης Μάργαρης στις 31 Δεκέμβριος 2013 στις 16:53
Διαγραφή σχολίου
Καλή χρονιά Μιχαήλ. Πράγματι και "άλλα ταλέντα"!!! Μπράβο.
Σχόλιο από τον/την Μαλακασιώτης Νικόλαος στις 31 Δεκέμβριος 2013 στις 17:01
Διαγραφή σχολίου
Πολύ τρυφερό Μιχάλη , Χρόνια πολλά
Σχόλιο από τον/την Μιχαήλ Μιχαήλ στις 31 Δεκέμβριος 2013 στις 17:48
Διαγραφή σχολίου
Σας ευχαριστώ πολύ για τα καλά σας λόγια! Εύχομαι ΚΑΛΗ ΧΡΟΝΙΑ σε όλους!
Σχόλιο από τον/την Εμμανουήλ Λαμπράκης στις 31 Δεκέμβριος 2013 στις 18:17
Διαγραφή σχολίου
Μιχαήλ καλησπέρα Το παραμύθι σου μου αρέσει πάρα πολύ. Λίγο πριν το έστειλα σε μια Ελένη μικρή. Καλή Πρωτοχρονιά
Σχόλιο από τον/την Κορκίζογλου Πρόδρομος στις 31 Δεκέμβριος 2013 στις 18:42
Διαγραφή σχολίου
Τέτοιες μέρες στο χωριό, τις χειμωνιάτικες νύχτες, να φτιάχνεις κόσμους ακούγοντας τον παραμυθά του χωριού, γύρω από μια σόμπα να καίει ξύλα ,και η θαλπωρή της ζέστης να σε κοιμίζει μαζί με τη φωνή του, ν'ακούγεται αργόσυρτη,να δίνει έμφαση σε κάποια σημεία, να παρακολουθεί τις αντιδράσεις μας, να συνεχίζει και τέλος να χάνεται η φωνή του κι οι εικόνες που μας έδινε με το παραμύθι του,μες στην ομίχλη του δρόμου του ύπνου... Πόσο όμαρφα ήταν! Μιχαήλ δεν υπάρχει τίποτε ωραιότερο από ένα παραμύθι που το ξεχάσανε τα παιδιά μας γιατί ο ηλεκτρονικός κόσμος άνοιξε νέους ορίζοντες,κοντόθωρους, όπως μες την πόλη κι όχι σ'ένα βουνό, και στερεί την καλλιέργεια της φαντασίας τους. Να είσαι καλά και ΚΑΛΗ ΧΡΟΝΙΑ.
Σχόλιο από τον/την Μιχαήλ Μιχαήλ στις 1 Ιανουάριος 2014 στις 11:48
Διαγραφή σχολίου
Μανώλη, Πρόδρομε σας εύχομαι μια παραμυθένια ΚΑΛΗ ΧΡΟΝΙΑ!
Σχόλιο από τον/την ΓΙΑΝΝΗΣ ΔΟΓΡΑΜΑΤΖΑΚΗΣ στις 1 Ιανουάριος 2014 στις 11:55
Διαγραφή σχολίου
Καλή Χρονιά Μιχαήλ... και εύχομαι όσα έχει περάσει ο κόσμος αυτά τα χρόνια να τα θυμάται ...σαν παραμύθι.
Σχόλιο από τον/την Εμμανουήλ Λαμπράκης στις 1 Ιανουάριος 2014 στις 20:03
Διαγραφή σχολίου
Μιχαήλ καλησπέρα Της μικρής Ελένης που ανέφερα χθες και μένει στο χωριό ο Αη Βασίλης της είχε κάνει σχετικά νωρίς δώρο ένα tablet και έτσι συνδεθήκαμε μαζί της μέσω skype και της έστειλα το παραμύθι σου αυτό. Της άρεσε πάρα πολύ.
Σχόλιο από τον/την Μιχαήλ Μιχαήλ στις 2 Ιανουάριος 2014 στις 9:50
Διαγραφή σχολίου
Γιάννη (Δογρ) σου εύχομαι και εγώ Καλή Χρονιά με υγεία! Μανώλη σ' ευχαριστώ. Μακάρι η καρδιά της μικρής Ελένης να ένοιωσε μια μικρή σπίθα ζεστασιάς.
2. Ρυθμιστική ικανότητα Ρ.Δ.

Α) Ένα ρυθμιστικό διάλυμα περιέχει CΗ3COOH συγκέντρωσης [CΗ3COOH]=cοξ=0,1Μ και [CΗ3COONa] συγκέντρωσης [CΗ3COONa]=cβ.

 α) Να υπολογιστεί η μέγιστη ρυθμιστική ικανότητα του παραπάνω διαλύματος για την προσθήκη της ισχυρής βάσης NaOH και

 β) Να υπολογιστεί η μέγιστη ρυθμιστική ικανότητα του παραπάνω διαλύματος για την προσθήκη του ισχυρού οξέος ΗCl.

 Β) Ένα ρυθμιστικό διάλυμα περιέχει CΗ3COOH συγκέντρωσης [CΗ3COOH]=cοξ και CΗ3COONa συγκέντρωσης [CΗ3COONa]=cβ=0,1Μ.

 α) Να υπολογιστεί η μέγιστη ρυθμιστική ικανότητα του παραπάνω διαλύματος για την προσθήκη της ισχυρής βάσης NaOH και

 β) Να υπολογιστεί η μέγιστη ρυθμιστική ικανότητα του παραπάνω διαλύματος για την προσθήκη του ισχυρού οξέος ΗCl.

 Θεωρείστε ότι με την προσθήκη της ισχυρής βάσης ή του ισχυρού οξέος δε μεταβάλλεται ο όγκος του Ρ.Δ.

Ακόμη θεωρείστε πως 0,1Μ≤cοξ,cβ≤1Μ και για το λόγο λ=coξ/cβ πως ισχύει 0,1≤λ≤10 .

Συνοπτική λύση:

Σχόλια
Σχόλιο από τον/την Αντώνης Μπαλτζόπουλος στις 5 Ιανουάριος 2014 στις 20:29
Διαγραφή σχολίου
Μιχαήλ , ο εργαστηριακός οδηγός ( μέσα στον οποίο βρίσκεται ο ορισμός της ρυθμιστικής ικανότητας που είναι το αντικείμενο της άσκησης ) δεν έχει δοθεί φέτος - τουλάχιστον στην πόλη που μένω - στους μαθητές της Γ Λυκείου. ( και είναι ασυζητητί εκτός ύλης ) ( εκτός έαν η 'ασκηση απευθύνεται μονο σε καθηγητές )
Σχόλιο από τον/την Μιχαήλ Μιχαήλ στις 6 Ιανουάριος 2014 στις 1:07
Διαγραφή σχολίου
Αντώνη καλησπέρα και Χρόνια Πολλά! Είδα την παρακάτω ερώτηση των Πανελληνίων εξετάσεων του 2011 και αναρωτήθηκα τι άλλο θα μπορούσαν να ρωτήσουν στα παιδιά! Από τα παρακάτω διαλύματα, μεγαλύτερη ρυθμιστική ικανότητα έχει: α .CH3COOH 0,1M – CH3COONa 0,1M β. CH3COOH 0,01M– CH3COONa 0,01M γ. CH3COOH 0,5M – CH3COONa 0,5M δ. CH3COOH 1,0M – CH3COONa 1,0M
Σχόλιο από τον/την Αντώνης Μπαλτζόπουλος στις 6 Ιανουάριος 2014 στις 12:30
Διαγραφή σχολίου
Καλημέρα - Χρόνια Πολλά Λογική η ανησυχία σου ( μας ). Για την ρυθμιστική ικανότητα - που είναι πολύ σύνθετο θέμα - μπορεί να διαβάσεις και εδώ ( το γράφω γιατι ενδεχωμένως η Ρυθμιστική ικανότητα να εκφράζεται μέσω ενός εύρους τιμών pH στο οποίο το διάλυμα την διατηρεί - την ικανότητα - ΚΑΙ μιάς τιμής PH στην οποία αυτή η ικανότητα παίρνει την μέγιστη τιμή της ) Δύσκολο θέμα ...
Σχόλιο από τον/την Μιχαήλ Μιχαήλ στις 6 Ιανουάριος 2014 στις 13:52
Διαγραφή σχολίου
Αντώνη έχεις απόλυτο δίκιο! Όμως η ανησυχία μου είναι αν θα μπορούσαν να ρωτήσουν το παρακάτω, Από τα παρακάτω διαλύματα, μεγαλύτερη ρυθμιστική ικανότητα για την προσθήκη ισχυρού οξέος έχει: α .CH3COOH 0,1M – CH3COONa 0,1M β. CH3COOH 0,1M– CH3COONa 1M;
Σχόλιο από τον/την Αντώνης Μπαλτζόπουλος στις 6 Ιανουάριος 2014 στις 16:41
Διαγραφή σχολίου
ΔΕΝ μπορεί να γίνει σύγκριση ( σε επίπεδο Λυκείου - γενικότερα έχουμε την εξίσωση Van Slyke ) στο παραδειγμα που δίνεις Το σχολικό βιβλίο γράφει ότι η ρυθμ. ικανότητα είναι μεγαλύτερη όσο μεγαλύτερες είναι οι συγκεντρώσεις των συζυγών μορφών. Δυστυχώς όμως δεν γράφει είναι ότι αυτό ισχύει για ένα δεδομένο λόγο ( πηλικο λ ) αυτών των μορφών . Επίσης δεν γράφει ότι η ρυθμ ικανότητα αυξάνει όσο το πηλικό λ είναι πιο κοντα στην μονάδα ( χωρίς όμως να μπορεί να γίνει σύγκριση στις περιπτώσεις όπου το πηλικό λ και οι συγκεντρώσεις δεν συνδέονται με κάποιον τρόπο ) πΧ Μπορεί να γίνει σύγκριση της ρυθμ ικανότητας α .CH3COOH 0,1M – CH3COONa 0,1M β. CH3COOH 1M– CH3COONa 1M; Μπορεί να γίνει σύγκριση της ρυθμ ικανότητας α .CH3COOH 0,1M – CH3COONa 0,2M β. CH3COOH 1M– CH3COONa 2M; Υγ .... ας εκτεθώ ( στην τελική όλοι οι περισσότεροι για να μάθουμε ήρθαμε στο υλικονετ ) Η αίσθηση μου είναι ότι όσο μεγαλύτερες είναι συγκεντρώσεις των συζυγών τιμών τόσο μεγαλύτερο είανι το εύρος του ph που το διάλυμα παρουσιαζει ικανότητα ρυθμισης του πεχα. Οσο πιο κοντά είναι οι τιμες των συγκεντρώσεων τόσο πιο "καλή" είναι η μέγιστη ρυθμιστική ικανότητα του .
125. ανάκλαση – διάθλαση (II)

Μια φωτεινή μονοχρωματική ακτίνα προερχόμενη από τον αέρα προσπίπτει στο σημείο Α της οριζόντιας διαμέτρου ΑΓ μιας γυάλινης σφαίρας ακτίνας R=10(sqr3)cm. Η ακτίνα βρίσκεται στο ίδιο επίπεδο με το μέγιστο κατακόρυφο κύκλο της σφαίρας, ακτίνας R που περνάει από το σημείο Α. Αν η γωνία πρόσπτωσης είναι θ=600  και ο δείκτης διάθλασης της σφαίρας είναι n=sqr(1,5), τότε:

α) Να σχεδιάσετε την πορεία της φωτεινής ακτίνας μέσα στη γυάλινη σφαίρα,

β) Να υπολογίσετε το χρόνο κίνησης της φωτεινής ακτίνας μέσα στη σφαίρα, για μια πλήρη διαδρομή. Δίνεται c=3∙108m/s.

 

Συνοπτική λύση:

και εδώ

Σχόλια
Σχόλιο από τον/την Εμμανουήλ Λαμπράκης στις 9 Ιανουάριος 2014 στις 7:34
Διαγραφή σχολίου
Μιχαήλ καλημέρα Πολύ όμορφη.
Σχόλιο από τον/την Μιχαήλ Μιχαήλ στις 9 Ιανουάριος 2014 στις 12:36
Διαγραφή σχολίου
Γιάννη καλημέρα και σ' ευχαριστώ! Θα ήταν πιο όμορφη αν μπορούσαμε να εγκλωβίσουμε τη δέσμη μέσα στη γυάλινη μπάλα!
Σχόλιο από τον/την Βασίλης Δουκατζής στις 9 Ιανουάριος 2014 στις 12:39
Διαγραφή σχολίου
Μιχάλη καλημέρα! Θα μπορούσες να την εγκλωβίσεις αν η σφαίρα ήταν βυθισμένη σε υγρό με κατάλληλο δ.δ. Η δε ακτίνα να μπαίνει από το Δ το οποίο εφαπτομενικά μόλις που βγαίνει στον αέρα.
Σχόλιο από τον/την Διονύσης Μάργαρης στις 9 Ιανουάριος 2014 στις 13:00
Διαγραφή σχολίου
Καλημέρα Μιχαήλ. Έχω την εντύπωση ότι κάπως πρέπει να τροποποιηθεί η διατύπωση, ώστε να υπάρξει η παραπάνω πορεία. Στο σημείο Β, γιατί η ακτίνα ανακλάται και δεν διαθλάται, ώστε να εξέλθει στον αέρα; μήπως πρέπει να μιλήσεις για μερική ανάκλαση;
Σχόλιο από τον/την Διονύσης Μητρόπουλος στις 9 Ιανουάριος 2014 στις 13:17
Διαγραφή σχολίου
Καλημέρα συνάδελφοι. Διονύση υποψιάζομαι ότι έχεις στο μυαλό σου τη δημιουργία του ... ουράνιου τόξου :-)
Σχόλιο από τον/την Διονύσης Μάργαρης στις 9 Ιανουάριος 2014 στις 13:21
Διαγραφή σχολίου
Καλημέρα Διονύση. Είναι και αυτό χαρακτηριστικό παράδειγμα! Αλλά απλά, αν εφαρμόσουμε στο σημείο Β το νόμο του Snell, θα βρούμε ότι η ακτίνα θα εξέλθει με γωνία θ ή πώς το λέγαμε παλιότερα "η αντίστροφη πορεία του φωτός;".
Σχόλιο από τον/την Διονύσης Μητρόπουλος στις 9 Ιανουάριος 2014 στις 13:35
Διαγραφή σχολίου
Συμφωνώ Διονύση, Απλά θυμήθηκα το θέμα Σ-Λ της γενικής παιδείας που έλεγε ότι "η δημιουργία του ουράνιου τόξου οφείλεται στο φαινόμενο της ολικής ανάκλασης"!
Σχόλιο από τον/την Διονύσης Μάργαρης στις 9 Ιανουάριος 2014 στις 13:37
Διαγραφή σχολίου
Και ... διηγώντας τα κλαις... Διονύση!
Σχόλιο από τον/την Διονύσης Μητρόπουλος στις 9 Ιανουάριος 2014 στις 13:43
Διαγραφή σχολίου
Ελλάς το μεγαλείο σου! Αλλά το μαύρο στη χώρα μας ανάλογα με τη ... γωνία θέασης γίνεται και άσπρο!
Σχόλιο από τον/την Μιχαήλ Μιχαήλ στις 9 Ιανουάριος 2014 στις 21:06
Διαγραφή σχολίου
Βασίλη ίσως να γίνεται κάπως! Διονύση (Μαρ) έχεις δίκιο εν μέρει ανακλάται! Αλλά αναφέρομαι στη δέσμη μέσα στο γυαλί. Διονύση (Μητρ) το Ουράνιο τόξο είχα στο μυαλό μου!
Σχόλιο από τον/την Μιχαήλ Μιχαήλ στις 10 Ιανουάριος 2014 στις 18:48
Διαγραφή σχολίου
Μανώλη συγγνώμη για το ......Γιάννη
Σχόλιο από τον/την Εμμανουήλ Λαμπράκης στις 10 Ιανουάριος 2014 στις 20:47
Διαγραφή σχολίου
Μιχαήλ καλησπέρα Δεν πειράζει άλλωστε και Γιάννης καλό όνομα είναι!
126. Κυκλική στεφάνη και μάζα m

Η κυκλική στεφάνη του σχήματος  έχει μάζα M=4Kg, ακτίνα R=0,3m και ισορροπεί ακουμπώντας σε κατακόρυφο τοίχο όπως φαίνεται στο σχήμα, μέσω ενός αβαρούς μη εκτατού νήματος μήκους L=0,6m.

Ακόμη μέσω ενός δεύτερου αβαρούς μη εκτατού νήματος μήκους L2=8cm κρέμεται από το ανώτερο σημείο O της στεφάνης μια μάζα m=2Kg και αυτή ισορροπεί.

 

Τότε:

α) Να υπολογιστεί η τάση των δυο σχοινιών.

 

β ) Να υπολογιστεί η συνολική δύναμη που δέχεται η κυκλική στεφάνη από τον τοίχο.

 

γ) Αν απομακρύνουμε τη μάζα m από τη θέση ισορροπίας της κατά γωνία φ/2  και την αφήσουμε ελεύθερη να ταλαντώνεται τότε να υπολογιστεί μεταξύ ποιών τιμών μεταβάλλεται η κάθετη αντίδραση από τον τοίχο στην κυκλική στεφάνη.

Δίνεται  g=10m/s2.

 

Συνοπτική λύση:

Σχόλια
Σχόλιο από τον/την Κορκίζογλου Πρόδρομος στις 12 Ιανουάριος 2014 στις 23:49
Διαγραφή σχολίου
Πολύ καλή Μιχαήλ. Μόνο που το απλό εκκρεμές δεν θα ζητηθεί,αν έβαζες στη θέση του νήματος ελατήριο να κάνει α.α.τ. θα ήταν πιο προσιτή για εξετάσεις.
Σχόλιο από τον/την Μιχαήλ Μιχαήλ στις 13 Ιανουάριος 2014 στις 0:09
Διαγραφή σχολίου
Πρόδρομε σ' ευχαριστώ! Υπάρχει ένα αίτημα φιλίας. Έλεγξε το!
Σχόλιο από τον/την Εμμανουήλ Λαμπράκης στις 13 Ιανουάριος 2014 στις 8:36
Διαγραφή σχολίου
Μιχαήλ καλημέρα. Πολύ έμπνευση! Συγχαρητήρια. Η ιδέα του Πρόδρομου με το ελατήριο, κατά τη γνώμη μου, είναι καλή για μια παραλλαγή του προβλήματος
Σχόλιο από τον/την Μιχαήλ Μιχαήλ στις 13 Ιανουάριος 2014 στις 11:13
Διαγραφή σχολίου
Μανώλη σ΄ευχαριστώ. Και μένα μου φαίνεται καλή η ιδέα του Πρόδρομου! Θα δω.......!
Σχόλιο από τον/την Παπαδάκης Παντελεήμων στις 13 Ιανουάριος 2014 στις 11:17
Διαγραφή σχολίου
Καλημέρα Μιχαήλ. Συμφωνώ και εγώ με την πρόταση του Πρόδρομου και του Εμμανουήλ για το ελατήρο. Νομίζω ότι στο β) ερώτημα θα έπρεπε να ζητήσεις «…τη δύναμη που δέχεται η κυκλική στεφάνη από τον τοίχο στο σημείο επαφής Εγώ σαν υποψήφιος θα προβληματιζόμουν στο ερώτημα «Να βρείτε τη συνολική δύναμη που δέχεται η κυκλική στεφάνη από τον τοίχο ..» αν πρέπει να βρώ την συνισταμένη Τστατ και Ν ή την συνισταμένη Τστατ ,Ν και Τ1 καθ όσον & την Τ1 κατά τη γνώμη μου την ασκεί ο τοίχος μέσω του νήματος στη στεφάνη και όχι το ( αμελητέας μάζας) νήμα στη στεφάνη, μιας και ένα απλό άμαζο νήμα δεν έχει την ικανότητα να ασκεί δύναμη παρά μόνο μέσω αυτού κάποιο άλλο σώμα (χέρι , τοίχος κ.λ.π.) Βέβαια γνωρίζω πως και αυτό που λέμε «τάση νήματος» κανονικά είναι η δύναμη που τείνουμε ένα νήμα και έχει επικρατήσει να εννοούμε την δύναμη μέσω του νήματος σε σώμα. Ευχαριστώ εκ’των προτέρων για την άποψή σας.
Σχόλιο από τον/την Μιχαήλ Μιχαήλ στις 13 Ιανουάριος 2014 στις 20:22
Διαγραφή σχολίου
Παντελεήμων αφού σ΄ευχαριστήσω, θα μου επιτρέψεις να διαφωνήσω! την Τ1 την ασκεί το νήμα στη στεφάνη!
Σχόλιο από τον/την Γιάννης Αγγελόπουλος στις 13 Ιανουάριος 2014 στις 20:28
Διαγραφή σχολίου
Για δες λίγο τα τρίγωνα, αντί για ΑΚΓ ΒΚΓ λές ΑΚΒ ΒΚΓ
Σχόλιο από τον/την Μιχαήλ Μιχαήλ στις 13 Ιανουάριος 2014 στις 21:29
Διαγραφή σχολίου
Φίλε Γιάννη Οk!
ΤΟ ΠΕΙΡΑΜΑ ΤΟΥ YOUNG (1800) ΜΕΤΡΗΣΗ ΜΗΚΟΥΣ ΚΥΜΑΤΟΣ ΜΟΝΟΧΡΩΜΑΤΙΚΗΣ ΑΚΤΙΝΟΒΟΛΙΑΣ.

Γενικά μια συσκευή που σχηματίζει κροσσούς συμβολής ονομάζεται συμβολόμετρο. Η αρχή λειτουργίας ενός συμβολόμετρου διαιρέσεως μετώπου κύματος χαρακτηρίζεται από το γεγονός ότι δυο διαφορετικές περιοχές ενός μετώπου φέρονται σε επαλληλία αφού διανύσουν διαφορετικούς οπτικούς δρόμους.

 H αρχή λειτουργίας ενός συμβολόμετρου διαιρέσεως μετώπου κύματος φαίνεται στην παρακάτω διάταξη όπου περιγράφεται το γνωστό πείραμα του Young.

 

Η διάταξη  του Young:

Οι πηγές Π1 και Π2 είναι ή μικρές οπές ή λεπτές σχισμές κάθετες στο επίπεδο του σχήματος.

 

Το κύμα που προέρχεται...............

Σχόλια
Σχόλιο από τον/την Μιχαήλ Μιχαήλ στις 18 Ιανουάριος 2014 στις 12:31
Διαγραφή σχολίου
Μετά την εισαγωγή του νέου πειράματος: ΜΕΤΡΗΣΗ ΜΗΚΟΥΣ ΚΥΜΑΤΟΣ ΜΟΝΟΧΡΩΜΑΤΙΚΗΣ ΑΚΤΙΝΟΒΟΛΙΑΣ στη Γ΄ Λυκείου Κατεύθυνσης (το οποίο κρίθηκε σαν ένα ¨κλασικό" παράδειγμα Συμβολής), προέκυψε το παραπάνω!!
Σχόλιο από τον/την Εμμανουήλ Λαμπράκης στις 18 Ιανουάριος 2014 στις 12:48
Διαγραφή σχολίου
Μιχαήλ καλημέρα Συγχαρητήρια για την εκτέλεση του πειράματος αλλά και για την εξαιρετική ανάλυση του φαινομένου. Πιστεύω τα παιδιά να το ευχαριστήθηκαν. Όσα συμμετείχαν περισσότερο ενεργά σίγουρα θα ένιωσαν όμορφα.
Σχόλιο από τον/την Κορφιάτης Ευάγγελος στις 18 Ιανουάριος 2014 στις 14:27
Διαγραφή σχολίου
Καλημέρα Μιχαήλ. Συγχαρητήρια και από εμένα. Εξαιρετική δουλειά. Κάποιοι προβληματισμοί. Έχει άραγε καμία σχέση το πείραμα και η ανάλυση Φυσικής που περιγράφεις με τις @@## που βάζουμε στις εξετάσεις; Μήπως να πηγαίναμε και στο ευθύγραμμο τμήμα ανάμεσα στις πηγές να δούμε πόσα σημεία ενίσχυσης σχηματίζονται; ( προφανώς κάνω πλάκα). Και μια προβοκατόρική ερώτηση: Μήπως κάνοντας Φυσική στο φαινόμενο Συμβολή δημιουργείς σύγχυση στους μαθητές σου, δυσκολεύοντάς τους να απαντήσουν στις α-νοησίες που εξετάζουμε; Μήπως θα έπρεπε όλα τα θέματα που έχουν τεθεί μέχρι τώρα στις εξετάσεις να τα είχαμε πετάξει στο πρόσωπο αυτών που τα έβαλαν αρνούμενοι να τα διορθώσουμε; Μήπως γινόμαστε συνεργοί στο έγκλημα;
Σχόλιο από τον/την Κυριακόπουλος Γιάννης στις 18 Ιανουάριος 2014 στις 14:33
Διαγραφή σχολίου
Μπράβο Μιχάλη. Βαγγέλη θα συμφωνήσω.
Σχόλιο από τον/την Διονύσης Μάργαρης στις 18 Ιανουάριος 2014 στις 17:08
Διαγραφή σχολίου
Καλησπέρα Μιχαήλ. Πολύ καλό θέμα και συγχαρητήρια για την πραγματοποίηση του πειράματος που παρουσιάζεις. Σε ευχαριστώ που το μοιράστηκες μαζί μας.
Σχόλιο από τον/την Μιχαήλ Μιχαήλ στις 18 Ιανουάριος 2014 στις 20:10
Διαγραφή σχολίου
Μανώλη, Βαγγέλη, Γιάννη, Διονύση σας ευχαριστώ για τα καλά σας λόγια. Βαγγέλη συμφωνώ με όσα λες. Όμως μέσα στο γενικότερο τσουβάλιασμα ψυχών και στον κυνισμό της κοινωνίας μας, η παιδεία κατάντησε ο περίγελος. Για την εξουσία, οι ψυχές των μαθητών μας έχουν αξία μόνο ως εμπόρευμα!
Σχόλιο από τον/την Μιχαήλ Μιχαήλ στις 18 Ιανουάριος 2014 στις 20:16
Διαγραφή σχολίου
Μανώλη πράγματι τα παιδειά το χάρηκαν. Το Laser είχε την τιμητική του. Στην πραγματικότητα ένας μαθητής μου είχε και δεύτερο κόκκινο Laser, οπότε μετρήσαμε ξανά το x που το βρήκαμε 35cm και υπολογίσαμε το μήκος κύματός του που βρέθηκε ίσο με 690nm!
Σχόλιο από τον/την Εμμανουήλ Λαμπράκης στις 18 Ιανουάριος 2014 στις 20:25
Διαγραφή σχολίου
Φοβερά αποτελέσματα. Το πείραμα το είχα κάνει ως τριτοετής φοιτητής προ αμνημονεύτων χρόνων.
Σχόλιο από τον/την Μιχαήλ Μιχαήλ στις 18 Ιανουάριος 2014 στις 20:48
Διαγραφή σχολίου
.....τα παιδιά....
128. Σύστημα ράβδου –m που περιστρέφεται και αντίδραση από τον άξονα περιστροφής

Λεπτή ομογενής ράβδος ΑΓ μήκους L=1m και μάζας Μ=3Kg, μπορεί να στρέφεται σε κατακόρυφο επίπεδο χωρίς τριβές γύρω από οριζόντιο άξονα που περνά από το άκρο της Α. Στο άλλο άκρο της Γ υπάρχει στερεωμένη σημειακή σφαίρα μάζας m=0,5Kg. Η ράβδος ισορροπεί σε οριζόντια θέση και τη χρονική στιγμή t0=0 αφήνουμε το σύστημα ελεύθερο να περιστραφεί.

Να υπολογιστεί η δύναμη που ασκείται στη ράβδο από τον άξονα περιστροφής όταν:

α) το σύστημα ράβδος - m είναι οριζόντιο

β) το σύστημα ράβδος - m είναι κατακόρυφο και

γ) όταν το σύστημα ράβδος - m έχει περιστραφεί κατά γωνία φ=600.

Δίνεται για τη ράβδο Ιcm= 1/12 Μ×L2 και g=10m/s2.  

 

Συνοπτική λύση:

 

Σχόλια
Σχόλιο από τον/την Μιχαήλ Μιχαήλ στις 25 Ιανουάριος 2014 στις 17:42
Διαγραφή σχολίου
Η άσκηση υπάρχει και εδώ!
Σχόλιο από τον/την Κορκίζογλου Πρόδρομος στις 26 Ιανουάριος 2014 στις 8:19
Διαγραφή σχολίου
Μιχαήλ καλημέρα. Υποδειγματική λύση,ανάδειξη εύρεσης c.m. συστήματος χωρίς να ξεφεύγεις από τους ορισμούς του σχολικού βιβλίου, ανάδειξη αλληλεπίδρασης δυο στερεών σε διάφορες φάσεις της κοινής κίνησής τους κ.λ.π. Μπράβο!!! Νομιμοποιούμαστε να διδάξουμε ένα τέτοιο θέμα για προετοιμασία ενός υποψηφίου; Εγώ προσωπικά το κάνω (1-2 ασκήσεις) και μάλιστα είχα κάνει και μια ανάρτηση εδώ .
Σχόλιο από τον/την Μιχαήλ Μιχαήλ στις 26 Ιανουάριος 2014 στις 19:29
Διαγραφή σχολίου
Πρόδρομε σ' ευχαριστώ! Πιστεύω ότι ένα τέτοιο ερώτημα που χρειάζεται υπολογισμό του c.m ακροβατεί! Όμως ο υπολογισμός της Ν μεταξύ ράβδου και μάζας m μπορεί να ζητηθεί.
Σχόλιο από τον/την Εμμανουήλ Λαμπράκης στις 27 Ιανουάριος 2014 στις 8:56
Διαγραφή σχολίου
Μιχαήλ καλημέρα Πολύ ωραίο θέμα που πρέπει να το δουν οι υποψήφιοι - συμφωνώ με τον Πρόδρομο. Να επισημάνω ότι ακόμη δεν έχει πέσει θέμα που να ζητά τη δύναμη της άρθρωσης όταν η ράβδος επιταχύνεται.
Σχόλιο από τον/την Μιχαήλ Μιχαήλ στις 27 Ιανουάριος 2014 στις 11:32
Διαγραφή σχολίου
Μανώλη καλημέρα και σ' ευχαριστώ για το σχόλιο. Θα περιμένουμε λοιπόν να δούμε...!
129. Τροχαλία με ακτίνες

Η τροχαλία του σχήματος είναι ένα λεπτό κυλινδρικό κέλυφος μάζας Μ=4Kg και ακτίνας L=0,2m.

Η τροχαλία αποτελείται ακόμη από τέσσερις ακτίνες που είναι συγκολλημένες στο εσωτερικό του κελύφους και είναι ανά δυο αντιδιαμετρικές. Η καθεμιά από αυτές έχει μάζα m=1Kg και μήκος L=0,2 m. Στην περιφέρεια του κελύφους είναι τυλιγμένο αβαρές νήμα στο ελεύθερο άκρο του οποίου είναι δεμένο σώμα μάζας m1=5Kg.

α) Να υπολογίσετε τη ροπή αδράνειας της τροχαλίας ως προς άξονα κάθετο στο επίπεδό της που διέρχεται από το κέντρο της.

β) Αν αφήσουμε ελεύθερη τη μάζα m1 τότε να υπολογιστεί η επιτάχυνσή της

γ) Κάποια στιγμή λόγω κακής συγκόλλησης αποκολλάται η μια ακτίνα. Να βρεθεί τότε μεταξύ ποιων τιμών μεταβάλλεται η επιτάχυνση της μάζας m1 καθώς αυτή κατέρχεται.

Δίνεται για ομογενή ράβδο Ιcm= 1/12 Μ×L2. Ακόμη για τις πράξεις ισχύει g=10m/s2.  

Συνοπτική λύση:

Σχόλια
Σχόλιο από τον/την Μιχαήλ Μιχαήλ στις 27 Ιανουάριος 2014 στις 22:43
Διαγραφή σχολίου
Η άσκηση αφιερώνεται στον Χρήστο Ελευθερίου!
Σχόλιο από τον/την ΧΡΗΣΤΟΣ ΕΛΕΥΘΕΡΙΟΥ στις 28 Ιανουάριος 2014 στις 0:19
Διαγραφή σχολίου
Καλησπέρα Μιχάλη και σε ευχαριστώ για την αφιέρωση. Εντυπωσιακή άσκηση και όπως πάντα ολόφρεσκη ιδέα!!!! Σε ευχαριστούμε.
Σχόλιο από τον/την Εμμανουήλ Λαμπράκης στις 28 Ιανουάριος 2014 στις 8:17
Διαγραφή σχολίου
Μιχαήλ καλημέρα Πολύ ωραία άσκηση. Το τελευταίο ερώτημα είναι, κατά τη γνώμη μου, εξαιρετικό και πρωτότυπο.
Σχόλιο από τον/την Διονύσης Μάργαρης στις 28 Ιανουάριος 2014 στις 8:55
Διαγραφή σχολίου
Καλημέρα Μιχαήλ. Συγχαρητήρια για την παραπάνω μελέτη σου, όπου με "απλά υλικά" οδηγήθηκες σε πρωτότυπο και πολύ όμορφο ερώτημα.
Σχόλιο από τον/την Μαλακασιώτης Νικόλαος στις 28 Ιανουάριος 2014 στις 9:11
Διαγραφή σχολίου
Καλημέρα Μιχαήλ εξαιρετική η έπνευσή καθώς και η διερεύνηση σου.
Σχόλιο από τον/την Βασίλης Δουκατζής στις 28 Ιανουάριος 2014 στις 10:32
Διαγραφή σχολίου
Καλημέρα Μιχάλη πολύ ωραίο θέμα. Κάνε μία διόρθωση μόνο στην 3 σελίδα
Σχόλιο από τον/την Μιχαήλ Μιχαήλ στις 28 Ιανουάριος 2014 στις 11:03
Διαγραφή σχολίου
Χρήστο, Μανώλη, Διονύση, Νίκο, Βασίλη σας ευχαριστώ πολύ! Η άσκηση του Χρήστου(Ελευθερίου) στάθηκε η αφορμή- έμπνευση για να προκύψει η άσκηση αυτή!
Σχόλιο από τον/την Διονύσης Μάργαρης στις 28 Ιανουάριος 2014 στις 11:22
Διαγραφή σχολίου
Αυτό και να μην το έλεγες Μιχαήλ, ήταν φανερό!
Σχόλιο από τον/την ΓΙΑΝΝΗΣ ΔΟΓΡΑΜΑΤΖΑΚΗΣ στις 28 Ιανουάριος 2014 στις 12:32
Διαγραφή σχολίου
Kαλησπέρα Μιχαήλ ...πολύ ενδιαφέρουσα ιδέα. Να'σαι καλά.
Σχόλιο από τον/την Κορκίζογλου Πρόδρομος στις 28 Ιανουάριος 2014 στις 14:53
Διαγραφή σχολίου
Μιχαήλ πολύ καλή και με απλά υλικά άσκηση. Τα θερμά μου συγχαρητήρια!! Σχόλιο από τον/την Γρηγόρης Μαλάμης στις 28 Ιανουάριος 2014 στις 19:45 Διαγραφή σχολίου Καλησπέρα Μιχάλη ... πολύ καλή η άσκηση Μια διόρθωση στην πρώτη σελίδα Ι2=4Ι=(4/3) mL2=4/75 Kgm2 Σχόλιο από τον/την Μιχαήλ Μιχαήλ στις 28 Ιανουάριος 2014 στις 20:09 Διαγραφή σχολίου Πρόδρομε, Γρηγόρη σας ευχαριστώ πολύ. Γρηγόρη σ' ευχαριστώ για τη διόρθωση Σχόλιο από τον/την Μιχαήλ Μιχαήλ στις 28 Ιανουάριος 2014 στις 20:10 Διαγραφή σχολίου Γιάννη Δογραματζάκη σ' ευχαριστώ!
131… Ταρζάν και Τσίτα

Ο Ταρζάν προσπαθεί να διασχίσει ένα ποτάμι και πρέπει να ισορροπήσει πάνω σε μια μικρή κρεμαστή γέφυρα ΑΓ που έχει μήκος L=4m και μάζα Μ1=30Kg και η οποία κρέμεται από δυο κατακόρυφα σχοινιά που είναι δεμένα στα άκρα της. Το ένα άκρο (Α) της γέφυρας είναι δεμένο μέσω του σχοινιού σ’ ένα βράχο ενώ το άλλο άκρο της (Γ) είναι δεμένο σε σχοινί που είναι τυλιγμένο σε μια τροχαλία που το ελεύθερο άκρο του το τραβάει η Τσίτα που κρέμεται από το σχοινί και η γέφυρα ισορροπεί οριζόντια.

Όταν ο Ταρζάν βρίσκεται  ακίνητος σε μικρή οριζόντια απόσταση από το Α ίση με x0=80/3cm τότε η τσίτα ίσα – ίσα που ισορροπεί οριζόντια τη γέφυρα.

α) Αν η μάζα της Τσίτας είναι m=20 Kg τότε να υπολογιστεί η μάζα M του Ταρζάν.

β) Στη συνέχεια ο Ταρζάν αρχίζει να περπατάει προς το άκρο Γ της γέφυρας με σταθερή ταχύτητα υ=2m/s. Με πόση επιτάχυνση πρέπει να ανεβαίνει προς τα πάνω στο σχοινί η Τσίτα ώστε η γέφυρα να εξακολουθεί να ισορροπεί οριζόντια; Ποια είναι η μέγιστη επιτάχυνση της Τσίτας;

Συνοπτική λύση:

Σχόλια
Σχόλιο από τον/την Εμμανουήλ Λαμπράκης στις 9 Φεβρουάριος 2014 στις 8:50
Διαγραφή σχολίου
Καλημέρα Μιχαήλ Ευρηματικό - Ταρζάν, Τσίτα ε και για τις ανάγκες του σεναρίου και κανένας πειρατής!
Σχόλιο από τον/την Μιχαήλ Μιχαήλ στις 9 Φεβρουάριος 2014 στις 9:15
Διαγραφή σχολίου
Μανώλη καλημέρα και σ' ευχαριστώ. Οποιαδήποτε ταλάντωση της γέφυρας λόγω στατικής τριβής ανάμεσα στα πόδια του Ταρζάν και τη γέφυρα θεωρείται αμελητέα! Έτσι τα σχοινιά θεωρούνται σχεδόν κατακόρυφα.
Σχόλιο από τον/την Διονύσης Μάργαρης στις 9 Φεβρουάριος 2014 στις 9:21
Διαγραφή σχολίου
Καλημέρα και καλή Κυριακή Μιχαήλ. Ωραίο το σενάριό σου!
Σχόλιο από τον/την Εμμανουήλ Λαμπράκης στις 9 Φεβρουάριος 2014 στις 9:54
Διαγραφή σχολίου
Μιχαήλ έτσι που λες πρέπει να το θεωρήσουμε μια που η οποιαδήποτε γωνία απόκλισης δε θα μπορούσε να παραμένει σταθερή. Τώρα αν είχαμε και κάποιο επιζήσαντα στη ζούγκλα να ασκεί οριζόντια δύναμη στη γέφυρα .....μάλλον θα περιέπλεκε την κατάσταση, άσε που θα χρειαζόταν και πρόσθετη τροφή κάτι που στη ζούγκλα δεν είναι απλό πράγμα.
Σχόλιο από τον/την Μιχαήλ Μιχαήλ στις 9 Φεβρουάριος 2014 στις 11:11
Διαγραφή σχολίου
Καλή Κυριακή Διονύση. Σ' ευχαριστώ! Μανώλη έχεις απόλυτο δίκιο.Εκτός και αν το άκρο Α της ράβδου ακουμπούσε στο βράχο.Βέβαια με αυτή την επιτάχυνση η τσίτα πρέπει να ανεβεί 20 ..m σχοινί!
132. Δυο ράβδοι

Δυο πανομοιότυπες λεπτές, ισοπαχείς και ομογενείς ράβδοι ΑΒ και ΑΓ, που έχουν το ίδιο μήκος L=30cm και την ίδια μάζα m=1Kg η καθεμία, συγκολλούνται στο άκρο τους Α, ώστε να σχηματίζουν γωνία φ=  600. Το σύστημα των δυο ράβδων μπορεί να περιστρέφεται γύρω από οριζόντιο άξονα, κάθετο στο επίπεδο ΓΑΒ, που διέρχεται από το σημείο Α, όπως  φαίνεται στο σχήμα. Το σύστημα αρχικά συγκρατείται στη θέση όπου η ράβδος ΑΒ είναι οριζόντια.

Τότε:

α) Να υπολογίσετε τη γωνιακή επιτάχυνση του συστήματος των δυο ράβδων τη στιγμή  που αφήνουμε το σύστημα από την αρχική του θέση να περιστραφεί.

β) Να βρείτε την απόσταση x από το Α, του σημείου Ρ από το οποίο έπρεπε να περνάει ο οριζόντιος άξονας ώστε το σύστημα αρχικά να ισορροπούσε.

γ) Ενώνουμε τα κέντρα μάζας των δυο ράβδων με μια αβαρή ράβδο και φροντίζουμε έτσι ώστε ένας οριζόντιος άξονας περιστροφής να διέρχεται από το μέσο G του ευθυγράμμου τμήματος ΚΚ΄. Αν αφήσουμε το σύστημα να περιστραφεί από την αρχική του θέση και γύρω από τον οριζόντιο άξονα που διέρχεται από το G, να υπολογίσετε τη γωνιακή επιτάχυνση του συστήματος.

δ) Στη συνέχεια το σύστημα ισορροπεί όπως στο σχήμα γύρω από άξονα που διέρχεται από το Α. Αν το σύστημα των δυο μαζών μετατοπιστεί ελάχιστα από αυτή τη θέση ισορροπίας του τότε ταλαντώνεται. Να βρείτε τη συχνότητα ταλάντωσής του.

Δίνεται η ροπή αδράνειας ράβδου μάζας m και μήκους L ως προς το κέντρο μάζας της Icm=1/12 mL2

 

Συνοπτική λύση:

Σχόλια
Σχόλιο από τον/την Κυριακόπουλος Γιάννης στις 16 Φεβρουάριος 2014 στις 15:30
Διαγραφή σχολίου
Πολύ καλή. Δεν μπορώ να σκεφτώ τρόπο (χωρίς Δ.Ε.) να περάσει το τελευταίο ερώτημα και σε μαθητές.
Σχόλιο από τον/την Διονύσης Μητρόπουλος στις 16 Φεβρουάριος 2014 στις 18:05
Διαγραφή σχολίου
Καλησπέρα συνάδελφοι,   Μιχάλη συγχαρητήρια για τις ωραίες ιδέες που μας δίνεις συνέχεια :-)   Γιάννη υπάρχει πάντα και ο ... πλάγιος δρόμος :-) Το CM κάνει κατά προσέγγιση γραμμική ταλάντωση για μικρές γωνίες εκτροπής, με επιτρόχια επιτάχυνση αcm = αγων∙d, όπου αγων = mολ∙g∙d∙ημθ/Ιολ όπως την υπολογίζει ο Μιχάλης. Αλλά d∙ημθ είναι η απομάκρυνση x του CM από τη ΘΙ και τελικά η δύναμη επαναφοράς στο CM είναι: ΣFx = mολ∙αcm = mολ∙αγων∙d = mολ∙mολ∙g∙d∙d∙ημθ/Ιολ = (mολ∙mολ∙g∙d/Ιολ)∙x = D∙x Δηλαδή ΓΑΤ και η ω είναι: ω = sqrt(mολ∙g∙d/Ιολ) = ...  
Σχόλιο από τον/την Μιχαήλ Μιχαήλ στις 16 Φεβρουάριος 2014 στις 18:31
Διαγραφή σχολίου
Γιάννη σ΄ευχαριστώ. Διονύση σ' ευχαριστώ. Συγχαρητήρια για τον "πλάγιο" (εγώ θα έλεγα πρωτότυπο) δρόμο που προτείνεις!
Σχόλιο από τον/την Κυριακόπουλος Γιάννης στις 16 Φεβρουάριος 2014 στις 19:04
Διαγραφή σχολίου
Σωστά!
Σχόλιο από τον/την Εμμανουήλ Λαμπράκης στις 18 Φεβρουάριος 2014 στις 8:55
Διαγραφή σχολίου
Μιχαήλ καλημέρα Πολύ καλό θέμα που προσφέρεται για την εξάσκηση των υποψηφίων - πλην, ίσως, του τελευταίου ερωτήματος. Μου άρεσε πολύ το τρίτο ερώτημα. Όσον αφορά στο τελευταίο ερώτημα να πω ότι το φυσικό εκκρεμές παλαιότερα ήταν εντός ύλης (βιβλίο Α. Μάζη)
Σχόλιο από τον/την Διονύσης Μάργαρης στις 18 Φεβρουάριος 2014 στις 12:31
Διαγραφή σχολίου
Καλό μεσημέρι Μιχαήλ. Πολύ ενδιαφέρουσα άσκηση. Μπράβο! Βέβαια το τελευταίο ερώτημα, σαν φυσικό εκκρεμές είναι νομίζω εκτός...
Σχόλιο από τον/την Μιχαήλ Μιχαήλ στις 18 Φεβρουάριος 2014 στις 21:35
Διαγραφή σχολίου
Μανώλη, Διονύση σας ευχαριστώ! Πάντως Διονύση ο τρόπος που προτείνει ο έτερος Διονύσης είναι πολύ καλός!
Σχόλιο από τον/την Διονύσης Μάργαρης στις 26 Φεβρουάριος 2014 στις 12:47
Διαγραφή σχολίου
Καλημέρα Μιχαήλ. Μόλις πήρα το βιβλίο σου. Καλοτάξιδο!!! Σε ευχαριστώ για την δωρεάν. Να είσαι καλά. Από μια πρώτη ματιά, βλέπω ένα βιβλίο υψηλού επιπέδου για 4ο θέμα. Στο πνεύμα των εδώ αναρτήσεών σου. Καλή επιτυχία.
Σχόλιο από τον/την Μιχαήλ Μιχαήλ στις 26 Φεβρουάριος 2014 στις 21:40
Διαγραφή σχολίου
Διονύση σ΄ευχαριστώ πάρα πολύ!
133. Ελαστική κρούση σφαιρών που κυλίονται

Η συμπαγής σφαίρα m1=2Kg του σχήματος κυλίεται χωρίς να ολισθαίνει σε οριζόντιο επίπεδο και συγκρούεται μετωπικά και ελαστικά με αρχικά ακίνητη κούφια σφαίρα μάζας m2=1Kg της ίδιας ακτίνας R. (Οι ακτίνες θεωρούνται ίσες ώστε η κρούση να είναι  κεντρική). Η οριζόντια ταχύτητα του κέντρου μάζας της σφαίρας πριν τη σύγκρουση έχει μέτρο υ1=3m/s όπως φαίνεται στο σχήμα.

α) Να βρείτε τις ταχύτητες των δυο σφαιρών αμέσως μετά τη σύγκρουση αν οι μάζες εξακολουθούν να κινούνται οριζόντια.

β) Σε πόσο χρόνο μετά τη σύγκρουση οι μάζες m1 και m2 θα σταματήσουν να ολισθαίνουν; Ποια μάζα θα σταματήσει την ολίσθηση πρώτη;

γ) Έστω πως η m2 είναι δεμένη σε ελατήριο σταθεράς Κ όπως φαίνεται στο σχήμα. Τότε αν αμέσως μετά την κρούση απομακρύνουμε τη μάζα m1 σε πόσο χρόνο υα σταματήσει η ολίσθηση της μάζας m2;

Δίνεται ο συντελεστής τριβής ολίσθησης μεταξύ σφαίρας και επιπέδου μ=1/7. Θεωρούμε ότι μεταξύ των σφαιρών δεν αναπτύσσεται κάποια τριβή. Aκόμη δίνεται για τη σφαίρα μάζας m1 και ακτίνας R, Icm1=2/5m1×R2 και για τη σφαίρα μάζας m2 και ακτίνας R, Icm2=m2×R2. Για τις πράξεις θεωρείστε g=10m/s2.

 

Συνοπτική λύση:

και εδώ

Σχόλια
Σχόλιο από τον/την Κορκίζογλου Πρόδρομος στις 1 Μάρτιος 2014 στις 9:13
Διαγραφή σχολίου
Μιχαήλ, συγχαρητήρια, είναι έξοχη!! Μια επαλήθευσή της στο μπιλιάρδο, με σχετικά μικρές ταχύτητες και χωρίς φάλτσα. Εύγε!!!
Σχόλιο από τον/την Κυριακόπουλος Γιάννης στις 1 Μάρτιος 2014 στις 9:35
Διαγραφή σχολίου
Μιχάλη πολύ ωραία δουλειά. Ένα σημείο με παραξενεύει μόνο. Έχει δειχθεί από συναδέλφους στο παρελθόν ότι όταν ένα σώμα συνδέεται με ελατήριο και δέχεται και σταθερού μέτρου τριβή εκτελεί ταλάντωση (ένα τμήμα της) της οποίας η θέση ισορροπίας είναι μετατοπισμένη προς τη μεριά που δείχνει η τριβή. Θα περίμενα λοιπόν αρμονική λύση και όχι εκθετική.
Σχόλιο από τον/την Κυριακόπουλος Γιάννης στις 1 Μάρτιος 2014 στις 9:46
Διαγραφή σχολίου
Σχόλιο από τον/την Μιχαήλ Μιχαήλ στις 1 Μάρτιος 2014 στις 10:44
Διαγραφή σχολίου
Πρόδρομε, Γιάννη καλημέρα και σας ευχαριστώ. Γιάννη δεν ξέρω αν μου ξέφυγε κανένα πρόσημο στη δ.ε οπότε τότε έχει αρμονική λύση
Σχόλιο από τον/την Κορκίζογλου Πρόδρομος στις 1 Μάρτιος 2014 στις 17:56
Διαγραφή σχολίου
Γιάννη πολύ καλό το Ι.Ρ. ευχαριστούμε. Η ταλάντωση του σώματος με το ελατήριο είναι φθίνουσα, κάτι αναμενόμενο. Δεν ξέρω αν θα μπορούσε να επιλεγούν τέτοια δεδομένα ώστε να είναι αμείωτη.
Σχόλιο από τον/την Μιχαήλ Μιχαήλ στις 1 Μάρτιος 2014 στις 21:07
Διαγραφή σχολίου
Κώστα καλησπέρα και σ' ευχαριστώ.
Σχόλιο από τον/την Μαλακασιώτης Νικόλαος στις 2 Μάρτιος 2014 στις 11:33
Διαγραφή σχολίου
Μιχάλη συγχαρητήρια πολύ καλή η άσκηση και η καθοδήγηση της λύσης . Θα την περίμενα και εγώ αρμονική ή μάλλον θα την προτιμούσα.
Σχόλιο από τον/την Μιχαήλ Μιχαήλ στις 2 Μάρτιος 2014 στις 19:31
Διαγραφή σχολίου
Νίκο, καλησπέρα και σ' ευχαριστώ!
134…Κύλινδρος και ράβδος

Η ομογενής και ισοπαχής ράβδος ΑΒ του σχήματος έχει μάζα M=10Kg και μήκος L=5m. Η ράβδος κρέμεται από δυο κατακόρυφα αβαρή σχοινιά σταθερού μήκους που είναι δεμένα στα άκρα της Α και Β, ενώ τα πάνω άκρα των σχοινιών είναι δεμένα στην οροφή. Πάνω στη ράβδο και σε απόσταση x0=1m από το άκρο της Α ισορροπεί ομογενής κύλινδρος μάζας m=5Kg.

Τη χρονική στιγμή t=0 ασκείται στο ανώτερο σημείο του κυλίνδρου με κατάλληλο τρόπο σταθερή οριζόντια δύναμη F=150N με φορά προς το άκρο Β. Η τροχαλία τότε κυλίεται χωρίς να ολισθαίνει ενώ η ράβδος εκτρέπεται από την αρχική της θέση ισορροπίας.

 

α) Να υπολογιστεί η γωνία θ που σχηματίζουν τα σχοινιά με τη ράβδο όταν η τροχαλία κυλίεται.

 

β) Να βρείτε πως μεταβάλλεται η τάση των σχοινιών με το χρόνο.

 

γ) Να γίνουν οι παραπάνω υπολογισμοί για την περίπτωση που η δύναμη ασκείται στο κέντρο μάζας του κυλίνδρου. 

Δίνεται για τον κύλινδρο Ιcm=1/2mR2 και ακόμη g=10m/s2.

 

Συνοπτική λύση:

Σχόλια
Σχόλιο από τον/την Βασίλης Δουκατζής στις 3 Μάρτιος 2014 στις 22:55
Διαγραφή σχολίου
Μιχάλη καλησπέρα. Έτσι όπως θέτεις το πρόβλημα θα έχεις ταλάντωση!!! Κατά την γνώμη μου: Θα μπορούσες να το ξεκινήσεις την άσκηση κρατώντας τα δύο νήματα στην γωνία που δίνεις και να πεις ότι το σύστημα με την άσκηση της F ισορροπεί ή κάτι άλλο ... κτλ
Σχόλιο από τον/την Κορκίζογλου Πρόδρομος στις 4 Μάρτιος 2014 στις 7:29
Διαγραφή σχολίου
Μιχαήλ καλημέρα. Ωραία η ιδέα σου, αλλά νομίζω ότι ο Βασίλης έχει δίκιο.Με το που ασκείς δύναμη στον κύλινδρο, ασκείται η στατική τριβή στη ράβδο,με αποτέλεσμα να ξεκινήσει ταλάντωση η ράβδος κι έτσι δεν έχεις σταθερό σύστημα. Η πρόταση του Βασίλη με βρίσκει σύμφωνο, αρκει να αλλάξεις λίγο την εκφώνηση. Το πρόβλημά σου μοιάζει με σανίδα σε λείο οριζόντιο επίπεδο, που κυλίεται πάνω της κύλινδρος με δύναμη που του ασκούμε. Να είσαι καλά.
Σχόλιο από τον/την Μιχαήλ Μιχαήλ στις 4 Μάρτιος 2014 στις 9:06
Διαγραφή σχολίου
Βασίλη, Πρόδρομε καλημέρα. Οκ θα κάνω αλλαγή στην εκφώνηση!
135…και στατική τριβή και τριβή ολίσθησης

Το σύστημα του σχήματος αποτελείται από δυο κυλίνδρους με μάζες m1= m2=1Kg , ακτίνας r=20cm. Τα κέντρα μάζας των κυλίνδρων συνδέονται με ράβδο αμελητέας μάζας η οποία δεν εμποδίζει την περιστροφή. Οι κύλινδροι κυλίονται σε οριζόντιο επίπεδο χωρίς να ολισθαίνουν.

Κάποια χρονική στιγμή που το σύστημα έχει οριζόντια ταχύτητα υ0=8m/s, ο πίσω κύλινδρος (m2) μπλοκάρει και αρχίζει να ολισθαίνει ενώ ο μπροστά κύλινδρος (m1) συνεχίζει να κυλίεται χωρίς να ολισθαίνει. Αν ο συντελεστής τριβής ολίσθησης μεταξύ του κυλίνδρου και του οριζόντιου δαπέδου είναι μ=0,2 τότε:

α) Να υπολογίσετε την μεταφορική επιβράδυνση του συστήματος.

β) Να υπολογιστούν οι δυνάμεις που ασκεί η ράβδος σε κάθε σώμα.

γ) Να υπολογιστεί η συνολική στροφορμή των δυο κυλίνδρων κάθε χρονική στιγμή και ως προς σταθερό σημείο του εδάφους.

δ) Ποια είναι η κινητική ενέργεια του συστήματος τη στιγμή που ο κύλινδρος m1 έχει κάνει Ν1=στροφές; Πόσο είναι τότε το έργο της τριβής ολίσθησης;

Δίνεται g=10m/s2 και η ροπή αδράνειας του κάθε κυλίνδρου γύρω από άξονα που περνάει από το Κ.Μ του Ι=0,5m×r2.

 

Συνοπτική λύση:

Σχόλια

Ετικέτες: στερεό, τάξη-γ

Σχόλιο από τον/την ΧΡΗΣΤΟΣ ΕΛΕΥΘΕΡΙΟΥ στις 8 Μάρτιος 2014 στις 16:02
Διαγραφή σχολίου
Μπράβο Μιχάλη πολύ καλή ιδέα. Είναι "η κόντρα" που είχαν τα παλιά ποδήλατα και δεν μπορώ να καταλάβω γιατί έχει καταργηθεί στα καινούργια ποδήλατα...
Σχόλιο από τον/την ελπιδα ανδρουλιδακη στις 8 Μάρτιος 2014 στις 16:33
Διαγραφή σχολίου
Kαλησπέρα, κύριε Μιχαήλ.Αφού πριν μπλοκάρει ο πίσω τρόχος έκανε και αυτός ΚΧΟ, η γωνιακή του ταχύτητα μετα το μπλοκάρισμα γιατι μηδενίζεται και δεν αυξάνεται διαρκώς λόγω της ροπής της τριβής ολίσθήσης; Μπλοκάρισμα δηλαδή σημαίνει δράση κατάλληλου ζεύγους δυνάμεων που ακαριαία μηδενίζει το ω και διατηρεί και μετά το Στ=0 ;
Σχόλιο από τον/την Μιχαήλ Μιχαήλ στις 8 Μάρτιος 2014 στις 17:05
Διαγραφή σχολίου
Χρήστο σ' ευχαριστώ! Ελπίδα έτσι ακριβώς όπως το λες είναι!
Σχόλιο από τον/την Διονύσης Μάργαρης στις 8 Μάρτιος 2014 στις 18:19
Διαγραφή σχολίου
Καλησπέρα Μιχαήλ. Όμορφη και πλούσια άσκηση. Μπράβο!
Σχόλιο από τον/την Μιχαήλ Μιχαήλ στις 8 Μάρτιος 2014 στις 19:08
Διαγραφή σχολίου
Διονύση σ' ευχαριστώ! Να΄σαι πάντα καλά.
Σχόλιο από τον/την ελπιδα ανδρουλιδακη στις 8 Μάρτιος 2014 στις 20:51
Διαγραφή σχολίου
Ευχαριστώ πολύ
Σχόλιο από τον/την Εμμανουήλ Λαμπράκης στις 10 Μάρτιος 2014 στις 10:09
Διαγραφή σχολίου
Μιχαήλ καλημέρα Σπουδαίο θέμα. Μπράβο. Χρήστο αν δεις το σχόλιο: Πρόλαβες στα ποδήλατα το σύστημα φρεναρίσματος που λες; Εγώ έχω την αίσθηση ότι είναι πολύ παλιό.
Σχόλιο από τον/την Μιχαήλ Μιχαήλ στις 10 Μάρτιος 2014 στις 17:59
Διαγραφή σχολίου
Μανώλη καλησπέρα. Σ' ευχαριστώ!
136. Μετωπική κρούση και συντελεστής κρούσεως

Δύο σφαίρες Σ1 και Σ2 με μάζες m1 και m2 κινούνται με ταχύτητες υ1 και υ2, όπως στο σχήμα. Οι σφαίρες συγκρούονται  έτσι ώστε μετά την κρούση να κινούνται πάλι στην ίδια ευθεία. Να υπολογίσετε τις ταχύτητές υ1΄ και υ2' , των σφαιρών μετά την κρούση αν ο συντελεστής κρούσης  (e) είναι:

α) e=0

β) e=1 και

γ) e=0,5.

 

Συνοπτική λύση:

Σχόλια
Σχόλιο από τον/την Εμμανουήλ Λαμπράκης στις 9 Μάρτιος 2014 στις 13:57
Διαγραφή σχολίου
Μιχαήλ καλό μεσημέρι Πολύ καλό θέμα που με ωραίο τρόπο εισάγει και το συντελεστή κρούσης.
Σχόλιο από τον/την Μιχαήλ Μιχαήλ στις 9 Μάρτιος 2014 στις 19:48
Διαγραφή σχολίου
Μανώλη, καλησπέρα. Σ' ευχαριστώ πολύ!
137. Κρούση, παράμετρος κρούσης και ενεργός διατομή

Μια σφαίρα Σ1, μάζας m1=m και ακτίνας r1= R κινείται με ταχύτητα υ1 και συγκρούεται ελαστικά και μη μετωπικά με όμοια αρχικά ακίνητη σφαίρα Σ2. Μάζας m2=m και ακτίνας r2= R. Αν d είναι η απόσταση του κέντρου της ακίνητης σφαίρας από το φορέα της υ1, τότε:

 

α) Να αποδείξετε ότι οι ταχύτητες των δυο σφαιρών μετά την κρούση δίνονται από τις σχέσεις υ1΄=υ1× και υ2΄= υ1× .

 

β) Να υπολογίσετε τις τιμές που μπορεί να πάρει η παράμετρος της παραπάνω κρούσης.

 

γ) Ποια είναι η ενεργός διατομή της παραπάνω κρούσης;

 

Συνοπτική λύση:

φΥΣΙΚΉ Γ΄ ΛΥΚΕΙΟΥ ΚΑΤΕΥΘΥΝΣΗ

ΘΕΩΡΙΑ- ΑΣΚΗΣΕΙΣ

Σχόλια
Σχόλιο από τον/την Μιχαήλ Μιχαήλ στις 17 Μάρτιος 2014 στις 21:20
Διαγραφή σχολίου
Δημήτρη ευχαριστώ! Πότε πρόλαβες;
Σχόλιο από τον/την Γρηγόρης Μαλάμης στις 17 Μάρτιος 2014 στις 22:38
Διαγραφή σχολίου
Πολύ καλό Μιχάλη. Αύριο θα το βάλω στην τάξη ( Στερεό ).Ευχαριστώ για την προσφορά σου αυτή και όχι μόνο αυτή
Σχόλιο από τον/την Διονύσης Μάργαρης στις 17 Μάρτιος 2014 στις 22:41
Διαγραφή σχολίου
Μιχαήλ, σε ευχαριστώ για την μεγάλη προσφορά σου, προς όλους μας. Να είσαι καλά και πάντα παραγωγικός.
Σχόλιο από τον/την Γκενές Δημήτρης στις 17 Μάρτιος 2014 στις 23:19
Διαγραφή σχολίου
Υπάρχουν συνάδελφοι εκτός συναγωνισμού στην εργατικότητα, την ποιότητα του έργου τους και στην ανιδιοτελή προσφορά του μόχθου τους Μιχαήλ με την παρουσίασή σου αυτή επιβεβαιώνεις ότι σου αξίζει η θέση μέσα σε αυτήν την ομάδα συναδέλφων αλλά και η θέση που χρόνια τώρα έχεις στις αξίες της "νησίδας μας" Ευχαριστούμε.
Σχόλιο από τον/την Μιχαήλ Μιχαήλ στις 17 Μάρτιος 2014 στις 23:33
Διαγραφή σχολίου
Γρηγόρη, Διονύση, Δημήτρη, σας ευχαριστώ! Σκέφτηκα ότι για την επανάληψη θα ήταν καλό να υπάρχει μια συγκεντρωμένη θεωρία (του σχολικού βιβλίου) με ασκήσεις! Δεν μπορώ όμως να μην αναφέρω ότι η προσφορά όλων σας είναι πηγή έμπνευσης και για μένα!
Σχόλιο από τον/την Κωστας Ψυλακος στις 17 Μάρτιος 2014 στις 23:38
Διαγραφή σχολίου
Μιχαήλ ^2 Και όχι μόνο . Θα έλεγα Μιχαήλ ! Πως λέμε 100! κάτι ανάλογο . Σε ευχαριστούμε! Θα είναι και αυτο στην "εργαλειοθήκη"!
Σχόλιο από τον/την Μιχαήλ Μιχαήλ στις 17 Μάρτιος 2014 στις 23:57
Διαγραφή σχολίου
Κώστα, σε ευχαριστώ για τα καλά σου λόγια!
Σχόλιο από τον/την Παπασγουρίδης Θοδωρής στις 18 Μάρτιος 2014 στις 2:07
Διαγραφή σχολίου
Μπράβο Μιχάλη, καλοτάξιδο…   Εντυπωσιακός όγκος δουλειάς, που προσφέρεται ανιδιοτελώς!!!!   Ευχαριστούμε…   Δε σου κρύβω ότι «ζηλεύω»….Μπράβο σου, άλλη μια φορά Η ομάδα του Βορρά, Δυτικής και Ανατολικής Μακεδονίας, πολυγραφότατη !!!!!
Σχόλιο από τον/την Μαλακασιώτης Νικόλαος στις 18 Μάρτιος 2014 στις 6:55
Διαγραφή σχολίου
Μιχάλη συγχαρητήρια καταπληκτική δουλειά που δείχνει το διάθεση σου για προσφορά . Ξεχωρίζω και το βιβλίο της γενικής το οποίο μου έχει φανεί ήδη πολύτιμο .
Σχόλιο από τον/την Εμμανουήλ Λαμπράκης στις 18 Μάρτιος 2014 στις 10:19
Διαγραφή σχολίου
Μιχαήλ καλημέρα Σε ευχαριστώ θερμά και σε συγχαίρω για την απλόχερη προσφορά, από μέρους σου, του ποιοτικότατου και πολύ όμορφου αυτού δώρου προς εμάς.
Σχόλιο από τον/την Μιχαήλ Μιχαήλ στις 18 Μάρτιος 2014 στις 11:22
Διαγραφή σχολίου
Θοδωρή, Νίκο, Μανώλη σας ευχαριστώ πολύ για τα θερμά σας λόγια!
Σχόλιο από τον/την Γεώργιος Αγγελόπουλος στις 18 Μάρτιος 2014 στις 16:15
Διαγραφή σχολίου
Χίλια μπράβο και λίγα είναι, για αυτήν την απλόχερη προσφορά! Να είσαι καλά και να συνεχίσεις με την ίδια δημιουργικότητα και ανιδιοτέλεια!
Σχόλιο από τον/την Μιχαήλ Μιχαήλ στις 18 Μάρτιος 2014 στις 21:32
Διαγραφή σχολίου
Γιώργο σ' ευχαριστώ πολύ! Να' σαι πάντα καλά!
Σχόλιο από τον/την ΓΙΑΝΝΗΣ ΔΟΓΡΑΜΑΤΖΑΚΗΣ στις 18 Μάρτιος 2014 στις 21:45
Διαγραφή σχολίου
Kαλησπέρα Μιχαήλ Μοιράζεσαι μαζί μας ...το απόσταγμα της σκέψης και της καρδιά σου. Γι'αυτό σ'ευχαριστώ.
Σχόλιο από τον/την Μιχαήλ Μιχαήλ στις 18 Μάρτιος 2014 στις 23:36
Διαγραφή σχολίου
Γιάννη σ' ευχαριστώ! Σου αφιερώνω ένα απόσπασμα για τις λεύτερες ψυχές που θέλουμε (αλλά μπορούμε;) να είμαστε... ...Νίκος Καζαντζάκης: ΑΝΑΦΟΡΑ ΣΤΟΝ ΓΚΡΕΚΟ
Σχόλιο από τον/την ΧΡΗΣΤΟΣ ΕΛΕΥΘΕΡΙΟΥ στις 18 Μάρτιος 2014 στις 23:45
Διαγραφή σχολίου
Καλησπέρα Μιχάλη. Η προσφορά σου είναι ΤΕΡΑΣΤΙΑ!!!! Μακάρι να βρεθούν και άλλοι ΜΙΜΗΤΕΣ σου... Τελικά για να γυρίσει ο ΗΛΙΟΣ θέλει δουλειά πολύ.Και εσύ Μιχάλη δεν την φοβάσαι την δουλειά.... Σε ευχαριστούμε.
Σχόλιο από τον/την Μιχαήλ Μιχαήλ στις 19 Μάρτιος 2014 στις 0:07
Διαγραφή σχολίου
Χρήστο, σίγουρα είσαι από τους μπροστάρηδες! Σε ευχαριστώ και εγώ για αυτά που μας προσφέρεις συνεχώς!.
Σχόλιο από τον/την Κυριακόπουλος Γιάννης στις 19 Μάρτιος 2014 στις 14:40
Διαγραφή σχολίου
Μπράβο.
Σχόλιο από τον/την Μιχαήλ Μιχαήλ στις 19 Μάρτιος 2014 στις 16:55
Διαγραφή σχολίου
Γιάννη σ' ευχαριστώ!
Σχόλιο από τον/την ΓΙΑΝΝΗΣ ΔΟΓΡΑΜΑΤΖΑΚΗΣ στις 19 Μάρτιος 2014 στις 17:27
Διαγραφή σχολίου
Αγαπητέ φίλε Μιχαήλ Θα ήθελα να σ'ευχαριστήσω για την αφιέρωση. Ο Ν. Καζαντζάκης ήταν η συντροφιά μου στα όμορφα χρόνια της εφηβείας. Γι'αυτό με συγκίνησε ιδιαίτερα ...αφιέρωση σου. ''Μια αστραπή η ζωή μας... μα προλαβαίνουμε'' Ν. Καζαντζάκης. Στον Μιχαήλ.
Σχόλιο από τον/την Κορκίζογλου Πρόδρομος στις 20 Μάρτιος 2014 στις 15:58
Διαγραφή σχολίου
Μιχαήλ, συγχαρητήρια κι από μένα,για τη σημαντική δουλειά σου, που μοιράζεσαι μαζί μας, δικαιώνοντας τον εμπνευστή αυτού του blog Διονύση Μάργαρη, που το έβαλε και προμετωπίδα του site , το να μοιράζεσαι πράγματα είναι καλό για όλους.
Σχόλιο από τον/την Μιχαήλ Μιχαήλ στις 21 Μάρτιος 2014 στις 9:17
Διαγραφή σχολίου
Πρόδρομε σ' ευχαριστώ! Πράγματι το να μοιράζεσαι πράγματα μας κάνει πιο δυνατούς όλους!
139. Τροχαλία και μάζες

Η τροχαλία του σχήματος, έχει μάζα Μ=10Kg και ακτίνα r. Τα σώματα έχουν μάζες m1=5Kg και m2=15Kg και το σχοινί είναι αβαρές και μη εκτατό.

Αρχικά το σύστημα είναι ακίνητο. Τότε:

α) Να υπολογίσετε τη μέγιστη δύναμη F που πρέπει να ασκήσουμε στην τροχαλία όπως φαίνεται στο σχήμα, χωρίς η μάζα m2  να ξεκολλήσει από το έδαφος.

Ποια είναι η επιτάχυνση της μάζας m1 εκείνη τη στιγμή; Να θεωρήσετε ότι το σχοινί δε γλιστρά στο αυλάκι της τροχαλίας.

β) Να απαντήσετε τα προηγούμενα ερωτήματα για αβαρή τροχαλία.

Δίνεται η ροπή αδράνειας για τροχαλία μάζας Μ και ακτίνας r ως προς το Κ.Μ της

Ι=0,5Μ×r2 και g=10m/s2.

 

Συνοπτική λύση:

Σχόλια
Σχόλιο από τον/την Κυριακόπουλος Γιάννης στις 24 Μάρτιος 2014 στις 21:45
Διαγραφή σχολίου
Ωραία δουλειά. Να θυμίσω μία ανάλογη με επίσης περίεργα συμπεράσματα.
Σχόλιο από τον/την Κωστας Ψυλακος στις 25 Μάρτιος 2014 στις 7:15
Διαγραφή σχολίου
Τροχαλια με τροχαλια εχει διαφορα σιγουρα λοιπον αυτη πραγματικα την εχει ! Ακολουθωντας τα ερωτηματα σου θελησα να βρω πρωτα την Fmax και μετα το amax. Εχει δουλεια βεβαια αλλα ειχα χρονο και ειπα να ασχοληθω ριξε μια ματια .
Σχόλιο από τον/την Μιχαήλ Μιχαήλ στις 25 Μάρτιος 2014 στις 20:57
Διαγραφή σχολίου
Γιάννη σ΄ευχαριστώ. Κώστα σ' ευχαριστώ για τη μελέτη που έκανες! Αν μου επιτρέπεις θα την εντάξω στη λύση της άσκησης.
Σχόλιο από τον/την Παπαδάκης Παντελεήμων στις 25 Μάρτιος 2014 στις 22:30
Διαγραφή σχολίου
Δυνατή η άσκηση Μιχαήλ. Για να απαλύνω τον αρχικό αιφνιδιασμό ζήτησα την Ν με δεδομένη την F. Να 'σαι καλά
Σχόλιο από τον/την Μιχαήλ Μιχαήλ στις 25 Μάρτιος 2014 στις 23:08
Διαγραφή σχολίου
Παντελεήμων σ' ευχαριστώ και σου εύχομαι ταχεία ανάρρωση!
Σχόλιο από τον/την Κορκίζογλου Πρόδρομος στις 27 Μάρτιος 2014 στις 21:50
Διαγραφή σχολίου
Πολύ καλή Μιχαήλ, μπράβο!!!
Σχόλιο από τον/την Μιχαήλ Μιχαήλ στις 28 Μάρτιος 2014 στις 18:05
Διαγραφή σχολίου
Πρόδρομε σ' ευχαριστώ πολύ. Να' σαι καλά!
ΦΥΣΙΚΗ Α΄ ΛΥΚΕΙΟΥ 200 Ερωτήσεις τύπου Σ ή Λ για έλεγχο της θεωρίας (1ο - 2o θέμα)

ΦΥΣΙΚΗ Α΄ ΛΥΚΕΙΟΥ

200 Ερωτήσεις τύπου Σ ή Λ για έλεγχο της θεωρίας (1ο - 2o θέμα)

Σχόλια
Σχόλιο από τον/την Εμμανουήλ Λαμπράκης στις 30 Μάρτιος 2014 στις 20:07
Διαγραφή σχολίου
Μιχαήλ καλησπέρα Από μια γρήγορη ματιά που έριξα - σήμερα γίνεται χαμός όσον αφορά στο πλήθος και στην ποιότητα των αναρτήσεων - η δουλειά σου είναι άριστη. Αναμενόμενο. Τις ευχαριστίες μου για την προσφορά των πολύ καλά επιλεγμένων - φτιαγμένων και οργανωμένων Ερωτήσεων φυσικής Α΄Λυκείου.
Σχόλιο από τον/την Καλκίτσας Χρήστος στις 30 Μάρτιος 2014 στις 20:30
Διαγραφή σχολίου
Μιχαήλ καλησπέρα. Όπως όλες οι αναρτήσεις σου και αυτή είναι τέλεια.
Σχόλιο από τον/την Μιχαήλ Μιχαήλ στις 30 Μάρτιος 2014 στις 23:24
Διαγραφή σχολίου
Μανώλη, Δημήτρη, Χρήστο σας ευχαριστώ!
Σχόλιο από τον/την Κωστας Ψυλακος στις 30 Μάρτιος 2014 στις 23:46
Διαγραφή σχολίου
Μιχαήλ εχουν και αυτα την βαρυτητα τους μιας και η περιοδος των εξετασεων πλησιαζει ! Μεγαλη και αυτη η προσφορα σου. Καλο βραδυ και καλη εβδομαδα!
Σχόλιο από τον/την Μιχαήλ Μιχαήλ στις 31 Μάρτιος 2014 στις 18:47
Διαγραφή σχολίου
Κώστα σ΄ευχαριστώ. Ελπίζω κάποιοι μαθητές μας να βοηθηθούν. Να ' σαι πάντα καλά!
140. Ορθογώνιοι μετασχηματισμοί - Μετατόπιση στερεού σώματος

Έστω δυο ορθογώνια συστήματα συντεταγμένων με την ίδια αρχή, τα Οx1x2x3 και Οx1΄x2΄x3΄. Εδώ θεωρούμε πως οι άξονες Ox3 και Ox3΄, ταυτίζονται ενώ είναι κάθετοι στο επίπεδο x1O

x2.

Οι διανυσματικές μονάδες (μοναδιαία διανύσματα)  και των δυο συστημάτων, επαληθεύουν τη σχέση ......

η συνέχεια

Σχόλια
Σχόλιο από τον/την Κυριακόπουλος Γιάννης στις 31 Μάρτιος 2014 στις 21:39
Διαγραφή σχολίου
Μπράβο Μιχάλη.
Σχόλιο από τον/την Μιχαήλ Μιχαήλ στις 1 Απρίλιος 2014 στις 7:01
Διαγραφή σχολίου
Γιάννη καλημέρα. Σ' ευχαριστώ!
Σχόλιο από τον/την Εμμανουήλ Λαμπράκης στις 1 Απρίλιος 2014 στις 8:52
Διαγραφή σχολίου
Μιχαήλ καλημέρα Πολύ καλή δουλειά. Με την ευκαιρία της ανάρτησης σου αυτής μου έρχονται στο νου κάποιες σκέψεις που έχω κάνει αποτιμώντας την διαδρομή μου ως φυσικός. Μια από αυτές: Είχα καλές στιγμές ως διδάσκων στη δευτεροβάθμια, δε λέω καλή αλλά δε σου δίνει τη δυνατότητα να προχωρήσεις επιστημονικά πέραν ενός σημείου.
Σχόλιο από τον/την Μιχαήλ Μιχαήλ στις 2 Απρίλιος 2014 στις 9:56
Διαγραφή σχολίου
Μανώλη (αν και με καθυστέρηση) Καλημέρα. Έχεις δίκιο και θυμάμαι μάλιστα, πως έπρεπε να "ξεχάσω" πολλά από τα μαθηματικά όταν άρχισα να διδάσκω στο Λύκειο!

Σχόλιο

Εργαστηριακά θέματα Φυσικής Α΄ Λυκείου

Οι ερωτήσεις εδώ

Σχόλια
Σχόλιο από τον/την Μιχαήλ Μιχαήλ στις 5 Απρίλιος 2014 στις 18:19
Διαγραφή σχολίου
Δημήτρη καλησπέρα. Σ΄ευχαριστώ. Η αλήθεια είναι ότι φέτος εγώ έκανα το πείραμα της επιταχυνόμενης κίνησης με φωτοπύλες. Δεν υπάρχει στον εργαστηριακό οδηγό. Άρα...
Σχόλιο από τον/την Κορκίζογλου Πρόδρομος στις 5 Απρίλιος 2014 στις 18:47
Διαγραφή σχολίου
Μπράβο Μιχαήλ, πολύ κατατοπιστικά αυτά που ανέβασες!!!
Σχόλιο από τον/την Μιχαήλ Μιχαήλ στις 5 Απρίλιος 2014 στις 18:56
Διαγραφή σχολίου
Πρόδρομε σ' ευχαριστώ πάρα πολύ! Να' σαι καλά.
Σχόλιο από τον/την Διονύσης Μάργαρης στις 5 Απρίλιος 2014 στις 19:12
Διαγραφή σχολίου
Καλησπέρα Μιχαήλ. Να σε ευχαριστήσω για την παραπάνω προσφορά σου. Έρχεται μια στιγμή, που, μετά από αίτημα φίλων, είχα προτείνει να ανεβάσουμε εργαστηριακό υλικό, για την Α΄τάξη. Μια προσπάθεια που δεν είδα να βρίσκει ανταπόκριση... Οπότε θα βάλω την παραπάνω εργασία σου και στην αντίστοιχη συζήτηση που είχα ξεκινήσει πριν λίγες μέρες.
Σχόλιο από τον/την Μιχαήλ Μιχαήλ στις 5 Απρίλιος 2014 στις 19:32
Διαγραφή σχολίου
Διονύση σ' ευχαριστώ! Μακάρι να έχουμε και εργαστηριακό υλικό.Τα ΕΚΦΕ έχουν κάνει πάρα πολύ και πολύ καλή δουλειά. Βέβαια το να τεθούν εργαστηριακά θέματα στις εξετάσεις νομίζω θα μας περιορίσει ...μόνο στα του εργαστηριακού οδηγού.
141. Ράβδος και κύλινδρος

Λεπτή ομογενής ράβδος ΑΒ μήκους L=50cm και βάρους  W=5N, στηρίζεται με το άκρος της Α στο έδαφος και με το σημείο Γ πάνω σε κύλινδρο μάζας m=200g, έτσι ώστε (ΑΓ)=40cm.

Αν το σύστημα ισορροπεί έτσι ώστε για τη γωνία φ να ισχύει ημφ=0,6 και συνφ=0,8 τότε:

α) Να βρεθούν οι δυνάμεις που ασκούνται στη ράβδο και τον κύλινδρο.

β) Αν ρίξουμε λίγο λιπαντικό και μηδενίσουμε την τριβή, στο άκρο Α της ράβδου, πόση θα είναι εκείνη τη στιγμή η οριζόντια επιτάχυνση της ράβδου και πόση του κυλίνδρου;

γ) Α αυξηθεί η δύναμη στατικής τριβής (Tστ) ανάμεσα στη ράβδο και το έδαφος τότε το μήκος ΒΓ θα αυξηθεί ή θα ελαττωθεί;

Δίνεται g=10m/s2.

 

Συνοπτική λύση:

Σχόλια
Σχόλιο από τον/την Κορκίζογλου Πρόδρομος στις 12 Απρίλιος 2014 στις 7:59
Διαγραφή σχολίου
Καλημέρα Μιχαήλ. Πολύ ωραία η ιδέα σου.
Σχόλιο από τον/την Διονύσης Μάργαρης στις 12 Απρίλιος 2014 στις 8:14
Διαγραφή σχολίου
Καλημέρα Μιχαήλ. Έχω ένα πρόβλημα με την αποδεικτική διαδικασία, όταν η ισορροπία μετατρέπεται σε επιταχυνόμενη κίνηση. Για παράδειγμα, πώς είναι δυνατόν να επιταχύνεται η ράβδος και να ισορροπεί ο κύλινδρος; Είναι κάτι που δεν βλέπω;
Σχόλιο από τον/την Μιχαήλ Μιχαήλ στις 12 Απρίλιος 2014 στις 9:58
Διαγραφή σχολίου
Πρόδρομε και Διονύση καλημέρα και σας ευχαριστώ για τις παρατηρήσεις. Επειδή λοιπόν όταν καταστρέφεται η ισορροπία όπως σωστά μου επισήμανε ο Διονύσης οι δυνάμεις ...πάνε περίπατο το β) ερώτημα αφαιρείται και μπαίνει προς μελέτη!
Σχόλιο από τον/την Εμμανουήλ Λαμπράκης στις 12 Απρίλιος 2014 στις 21:54
Διαγραφή σχολίου
Μιχαήλ καλησπέρα Πολύ ωραίο θέμα ισορροπίας. Σκεπτόμενος το ερώτημα που αρχικά είχες θέσει με το λιπαντικό ζαλίστηκα. Οι διάφοροι παράγοντες που υπεισέρχονται δημιουργούν περιπλοκή.
Σχόλιο από τον/την Μιχαήλ Μιχαήλ στις 13 Απρίλιος 2014 στις 19:04
Διαγραφή σχολίου
Μανώλη καλησπέρα και σ' ευχαριστώ! Πράγματι αν αρχίσει η ολίσθηση της ράβδου το ερ ώτημα γίνεται πολύ δύσκολο!
Σχόλιο από τον/την Μιχαήλ Μιχαήλ στις 13 Απρίλιος 2014 στις 23:44
Διαγραφή σχολίου
...και η απάντηση στο ερώτημα β) όπως μελετήθηκε από τον Πρόδρομο (Κορκίζογλου) τον οποίο και ευχαριστώ για αυτή του την προσφορά! ΑΠΑΝΤΗΣΗ:
Σχόλιο από τον/την Κορκίζογλου Πρόδρομος στις 14 Απρίλιος 2014 στις 2:25
Διαγραφή σχολίου
Να'σαι καλά Μιχάλη και πάντα καλές ιδέες. Η άσκηση μπορεί να επεκταθεί καθώς και να διερευνηθεί ως προς τον ελάχιστο συντελεστή στατικής οριακής τριβής , ώστε να μην έχουμε ολίσθηση, ούτε με το δάπεδο αλλά ούτε και η ράβδος με τον κύλινδρο. Επίσης , επειδή η επιτάχυνση είναι σταθερή μέχρι να φτάσει το άκρο της ράβδου στο Β, θα μπορούσαμε να μελετήσουμε και ενεργειακά και να βρούμε ταχύτητες ράβδου ,κέντρου μάζας κυλίνδρου, καθώς και την γωνιακή ταχύτητα του κυλίνδρου.
Σχόλιο από τον/την Διονύσης Μάργαρης στις 14 Απρίλιος 2014 στις 8:04
Διαγραφή σχολίου
Μιχαήλ και Πρόδρομε Καλημέρα. μόλις έριξα μια ματιά στη λύση σου Πρόδρομε και κάτι δεν καταλαβαίνω. Γράφεις: "Η ράβδος δεν κάνει στροφική κίνηση, εφ’όσον δεχόµαστε ότι δεν ολισθαίνει στην επαφή µε τον κύλινδρο(υπόθεση), όπως και ο κύλινδρος δεν ολισθαίνει µε το οριζόντιο δάπεδο. Η γωνία θ που σχηµατίζει η ράβδος µε το οριζόντιο δάπεδο είναι συνεχώς ίση µε θ , κι αυτό µέχρι να ‘’κυλήσει’’ το τµήµα ΒΓ στον κύλινδρο, και µετά χάνουν την επαφή τους, εφόσον µέχρι τότε δεν γίνει ολίσθηση." Πολλές υποθέσεις, οι οποίες δεν βλέπω πού πατάνε, αλλά κυρίως μια κίνηση και των δύο σωμάτων προς τα αριστερά; Γιατί;
Σχόλιο από τον/την Κορκίζογλου Πρόδρομος στις 14 Απρίλιος 2014 στις 9:20
Διαγραφή σχολίου
Καλημέρα Διονύση. Και ο έτερος Διονύσης (Μητρ.) μου έγραψε κάτι στο mail μου. Πως βλέπω το πράγμα(ίσως έχω παγιδευτεί και στην αρχική μου ιδέα, δεν ξέρω): Επειδή το άκρο Α δεν έχει τριβές με το δάπεδο, και επειδή το κέντρο μάζας της ράβδου είναι αριστερά του κυλίνδρου, η ράβδος πάει να κινηθεί προς τα αριστερά, οπότε ο κύλινδρος την εμποδίζει ασκώντας στατική τριβή προς τα πάνω Έτσι , λόγω δράσης αντίδρασης και η ράβδος ασκεί στον κύλινδρο δύναμη ίση κι αντίθετη, που πάει να τον κινήσει προς τα αριστερά. Επειδή δεν δίνει ο Μιχάλης συντελεστές τριβής , δέχομαι ότι ο συντελεστής οριακής τριβής ράβδου-κυλίνδρου είναι επαρκής , όπως και με το έδαφος , ώστε να μην έχουμε ολίσθηση, κι έτσι λύνω το πρόβλημα.Αν κάνεις ένα Ι.Ρ. θα δείξει, ίσως να μη βλέπω κάτι. Αν η συλογιστική μου είναι σωστή, μπορείς να βρεις συνθήκες για την οριακή τριβή με το έδαφος και με ράβδο-κύλινδρο. Δέχθηκα ότι η ράβδος δεν κάνει στροφική κίνηση , γιατί ''παρασύρει'' στο γλυστριμά της και τον κύλινδρο, και όσο διαρκεί αυτή η επαφή τους η γωνία δεν αλλάζει. Μπορεί και να κάνω λάθος , θα το κάνω πείραμα και τα λέμε. Αν έχω λάθος, πρέπει να πάρουμε και τη στροφική κίνηση της ράβδου , και το πρόβημα περιπλέκεται ακόμη περισσότερο.
Σχόλιο από τον/την Διονύσης Μάργαρης στις 14 Απρίλιος 2014 στις 10:26
Διαγραφή σχολίου
Καλημέρα και πάλι Πρόδρομε. Τελικά έκανα το i.p. και επιβεβαιώνει τη δική σου θέση για την κίνηση προς τα αριστερά. Με μια μικρή διόρθωση. Και η ράβδος αποκτά γωνιακή επιτάχυνση, αφού και αυτή αλλάζει προσανατολισμό. Εκτελεί δηλαδή σύνθετη κίνηση. Έχω βάλει απλά μεγαλύτερες μάζες (για να έχουμε και μεγαλύτερες δυνάμεις τριβής, κρατώντας την αναλογία). Το γκρι οριζόντιο επίπεδο είναι λείο. Το αρχείο από εδώ.
Σχόλιο από τον/την Μιχαήλ Μιχαήλ στις 14 Απρίλιος 2014 στις 11:43
Διαγραφή σχολίου
Διονύση αν βάλουμε στο Ι.P να μετρήσει τη γωνιακή επιτάχυνση της ράβδου, τελικά μετράει αγων=0.
Σχόλιο από τον/την Διονύσης Μάργαρης στις 14 Απρίλιος 2014 στις 12:25
Διαγραφή σχολίου
Μιχαήλ το έχω βάλει. είναι η τιμή του ar. Δεν είναι μηδέν. Δες την εικόνα. Απλά δεν είναι σταθερή
Σχόλιο από τον/την Διονύσης Μητρόπουλος στις 14 Απρίλιος 2014 στις 15:26
Διαγραφή σχολίου
Συνάδελφοι καλημέρα, Βάζω το σχήμα, αλλά τις πράξεις ... δεν αντέχω να τις κάνω :-) Οι α1, α2 είναι οι επιταχύνσεις των κέντρων μάζας των σωμάτων και οι a1, a2 οι γωνιακές τους επιταχύνσεις. Η φ είναι η γωνία ράβδου - επιπέδου (που την ξέχασα στο σχήμα) και x η απόσταση Κ2Γ. Τη στιγμή εκκίνησης είναι μηδενικές όλες οι ταχύτητες και δεν έχουμε κεντρομόλους επιταχύνσεις. Νομίζω ότι ισχύουν τα παρακάτω. Λόγω κύλισης του τροχού:

α1=a1∙R

Επειδή η σχετική ταχύτητα του Γ είναι μηδέν λόγω μη ολίσθησης:

Συνισταμένη (α1, a1∙R) = Συνισταμένη (α2, a2∙x)

Δηλαδή:

α2x – a2∙x∙ημφ = α1 + a1∙R∙συνφ

α2y + a2∙x∙συνφ = a1∙R∙ημφ

Επειδή το σημείο επαφής Α κινείται οριζόντια:

α2y = (a2∙L/2)∙συνφ

Σχόλιο από τον/την Κορκίζογλου Πρόδρομος στις 14 Απρίλιος 2014 στις 16:24
Διαγραφή σχολίου
Ουδείς αλάνθαστος αδέρφια! Πως την έπαθα έτσι; Να μη βλέπω ότι η ράβδος στρέφεται;Γκαβομάρα είχα, που λένε στο χωριό μου. Η ειρωνία είναι ότι πήγα να διορθώσω κάτι και έπεσα σε λάθος. Το πρόβλημα λύνεται μάλλον , ένα σύστημα ..11χ11 ίσως, όποιος έχει την υπομονή ας το λύσει. Καλή Ανάσταση αδέρφια υλικονιστές και υλικονίστριες. Αύριο φεύγω και δεν θα έχω ιντερνετ, αν δεν το λύσει κανείς μέχρι την .τρίτη μέρα του Πάσχα, ίσως το επιχειρήσω...(χαρά στο κουράγιο μου) Διονύση Μητρ. όπως το θέτεις, ''εγώ βάζω το σχήμα βάλε εσύ τις πράξεις και φύγαμε..''
Σχόλιο από τον/την Διονύσης Μητρόπουλος στις 14 Απρίλιος 2014 στις 16:27
Διαγραφή σχολίου
Νάσαι καλά Πρόδρομε :-) Καλό ταξίδι!
Σχόλιο από τον/την Διονύσης Μάργαρης στις 14 Απρίλιος 2014 στις 16:38
Διαγραφή σχολίου
Γεια σου "υλικονιστή" Πρόδρομε! Καλό ταξίδι και καλό Πάσχα να έχεις. Και για το σύστημα μην ανησυχείς. Δεν θα το λύσει κανείς:-)
Σχόλιο από τον/την Μιχαήλ Μιχαήλ στις 17 Απρίλιος 2014 στις 14:48
Διαγραφή σχολίου
Ακόμη μια προσπάθεια εδώ! ΚΑΛΗ ΑΝΑΣΤΑΣΗ
Σχόλιο από τον/την ΧΡΗΣΤΟΣ ΕΛΕΥΘΕΡΙΟΥ στις 17 Απρίλιος 2014 στις 15:12
Διαγραφή σχολίου
Καλησπέρα Μιχάλη.Ομορφη αλλά πολύ μπερδευτική όπως αποδείχθηκη η περίπτωσή σου.Με βάση το σχήμα σου μου ήρθε μία άλλη ιδέα.Ο χρόνος βέβαια αυτό το χρονικό διάστημα είναι πολύ λίγος οπότε μάλλον πάει για μετά τις 10 Ιουνίου...
Σχόλιο από τον/την Μιχαήλ Μιχαήλ στις 18 Απρίλιος 2014 στις 0:40
Διαγραφή σχολίου
Χρήστο σ' ευχαριστώ. Για απλή το πήγαινα αλλά... ! Σου εύχομαι ΚΑΛΗ ΑΝΑΣΤΑΣΗ.
Σχόλιο από τον/την Γκενές Δημήτρης στις 25 Απρίλιος 2014 στις 20:45
Διαγραφή σχολίου
Εντυπωσιακή αντοχή πρωτίστως του Μιχαήλ Αλλά και σε όλους όσους κατάφεραν να το μελετήσουν και να συμβάλλουν (Πρόδρομε το σωστό διδάσκει μόνο αν προκύπτει μέσα από λάθη). Μιχαήλ καθυστερημένα να εκφράσω τον θαυμασμό μου και τα δικά μου συγχαρητήριά ... αλλά ... θα το ξανασκεφτώ αν θα μπω ξανά σε τέτοια "λούκια".
Σχόλιο από τον/την Μιχαήλ Μιχαήλ στις 25 Απρίλιος 2014 στις 22:18
Διαγραφή σχολίου
Δημήτρη αφού σου ευχηθώ ΧΡΟΝΙΑ ΠΟΛΛΑ να σ' ευχαριστήσω για τα καλά σου λόγια. Η γνώση σου και η βοήθειά σου, μας είναι απαραίτητες.
142. Ράβδος και κύλινδρος σε κεκλιμένο επίπεδο

Ο συμπαγής και ομογενής κύλινδρος του σχήματος μάζας Μ=0,4Kg και ακτίνας r ισορροπεί σε κεκλιμένο επίπεδο γωνίας κλίσης φ με ημφ=0,6, ενώ βρίσκεται σε επαφή και με τη ράβδο ΟΑ.

Η λεπτή ομογενής ράβδος ΟΑ του σχήματος μήκους L=40cm και μάζας m=0,3Kg, στηρίζεται μέσω της άρθρωσης στο σημείο Ο στο κεκλιμένο επίπεδο και με το σημείο Λ πάνω στον κύλινδρο με (ΟΛ)=d=0,3m, έτσι ώστε να ισορροπεί στην οριζόντια θέση.Αν το σύστημα ισορροπεί τότε:

α) Να υπολογίσετε τη στατική τριβή ανάμεσα στον κύλινδρο και τις επιφάνειες επαφής.

β) Για ποιες τιμές του συντελεστή στατικής τριβής ανάμεσα στον κύλινδρο και τις επιφάνειες επαφής το σύστημα ισορροπεί;

γ)i) Αν στη συνέχεια σφίξουμε την άρθρωση ώστε να ακινητοποιηθεί η ράβδος, πόση γίνεται η δύναμη από την άρθρωση (Ο) στη ράβδο; Υποθέτουμε ότι η άρθρωση συγκρατεί τη ράβδο οριζόντια.

ii) Πόση γίνεται εκείνη τη στιγμή η δύναμη της στατικής τριβής (Tστ) ανάμεσα στον κύλινδρο και το κεκλιμένο επίπεδο;

δ) Τελικά ο κύλινδρος χάνει την επαφή του με τη ράβδο. Αν στη συνέχεια χαλαρώσουμε την άρθρωση στο (Ο), τότε αυτή αρχίζει να περιστρέφεται γύρω από το (Ο). Με ποια ταχύτητα η ράβδος συγκρούεται με το κεκλιμένο επίπεδο;

Δίνεται για τον κύλινδρο ΙK=0,5Μr2 για τη ράβδο Ιcm=0,083mL2 και για τις πράξεις θεωρείστε g=10m/s2.

 

Συνοπτική λύση:

Σχόλια

Ετικέτες: στερεό, τάξη-γ

Σχόλιο από τον/την Εμμανουήλ Λαμπράκης στις 26 Απρίλιος 2014 στις 9:33
Διαγραφή σχολίου
Μιχαήλ καλημέρα Χρόνια πολλά Πολύ ωραίο θέμα που δίνει την ευκαιρία σε έναν υποψήφιο να εξασκηθεί αλλά και να αντιληφθεί καλύτερα κάποια πράγματα. Κατά τη γνώμη μου το γ1 είναι πολύ "δυνατό" ερώτημα. Τώρα βλέπω ότι κάτι άλλαξε στην εκφώνηση ή κάνω λάθος;
Σχόλιο από τον/την Μιχαήλ Μιχαήλ στις 26 Απρίλιος 2014 στις 11:31
Διαγραφή σχολίου
Μανώλη καλημέρα και ΧΡΟΝΙΑ ΠΟΛΛΑ! Σ' ευχαριστώ που μελέτησες την άσκηση. Μανώλη η εκφώνηση άλλαξε μετά από παρέμβαση του Διονύση (Μητρ.) Δεν ξέρω αν αυτή που "βλέπεις" τώρα είναι αυτή που υπάρχει και στο pdf. Να' σαι πάντα καλά.
Σχόλιο από τον/την Εμμανουήλ Λαμπράκης στις 26 Απρίλιος 2014 στις 12:31
Διαγραφή σχολίου
Εγώ διάβασα στο pdf αυτό που βλέπω τώρα πιο πάνω. Αυτό ήταν διαφορετικό από ότι στο pdf όταν το διάβασα. Τώρα Ο.Κ.
Σχόλιο από τον/την Κωστας Ψυλακος στις 26 Απρίλιος 2014 στις 14:08
Διαγραφή σχολίου
Μανολη Καλημερα ! Δεν ξερω αν ειδες παραπανω τα σχολια εχει αλλαξει η εκφωνηση της ασκησης για να αντιμετωπιστουν καποια προβληματακια σαν αυτο που αναφερεις! Ριξε μια ματια.
Σχόλιο από τον/την Μανόλης Μαργαρίτης στις 26 Απρίλιος 2014 στις 14:41
Διαγραφή σχολίου
Μιχαήλ συγχαρητήρια για την ανάρτηση . Κώστα σ ευχαριστώ , λάθος συγνώμη, άκυρο το παραπάνω μου σχόλιο
Σχόλιο από τον/την Μιχαήλ Μιχαήλ στις 26 Απρίλιος 2014 στις 19:21
Διαγραφή σχολίου
Μανόλη καλησπέρα! Σ' ευχαριστώ.
Σχόλιο από τον/την Κορφιάτης Ευάγγελος στις 26 Απρίλιος 2014 στις 22:12
Διαγραφή σχολίου
Καλησπέρα συνάδελφοι και χρόνια πολλά σε όλους. Μιχάλη ωραίο θέμα. Ευρηματικότατος όπως πάντα. Έχω μια απορία στο γi. Υποθέτουμε ότι έχουμε πραγματοποιήσει την διάταξη. Η ράβδος λόγω του βάρους της έχει στριμώξει τον κύλινδρο, οι τριβές έχουν αναλάβει τον ρόλο τους και το σύστημα ηρεμεί. Με ένα κλειδί αρχίζω σιγα - σιγα να σφίγγω την βίδα της άρθρωσης μέχρι "καργαρίσματος". Γιατί να αλλάξει η κατάσταση; Έχω την εντύπωση ότι ο κύλινδρος δεν θα κινηθεί. Νομίζω ότι ο κύλινδρος δεν θα ισορροπήσει αν πρώτα σφίξω την βίδα και μετά προσπαθήσω να τον βάλω στο κενό. Και πάλι δεν θα κινηθεί στο πλαίσιο του απολύτως στερεού σώματος. Στην πράξη και πάλι μπορώ να τον φρακάρω αξιοποιώντας την μικρή ελαστικότητα της ράβδου.
Σχόλιο από τον/την Μιχαήλ Μιχαήλ στις 26 Απρίλιος 2014 στις 22:43
Διαγραφή σχολίου
Βαγγέλη Καλό βράδυ και σ' ευχαριστώ! Η αλήθεια είναι πως όπως μου επεσήμανε και ο Διονύσης (Μητρ.) μόλις ρίξουμε λαδάκι στο σημείο επαφής και αφού έχουμε σφίξει τη βίδα τότε η Ν1 μηδενίζεται. Αλλά έτσι και αλλιώς αυτό είναι δυσνόητο, οπότε έκανα την υπόθεση πως με το σφίξιμο της βίδας η άρθρωση σηκώνει όλο το "βάρος". Συμπέρασμα: Οι ασκήσεις στο στερεό που ξεφεύγουν από τα συνηθισμένα μπορούν να γίνουν εξαιρετικά πολύπλοκες.
144. Σφαίρα και κύβος σε κεκλιμένο επίπεδο.

Η ομογενής σφαίρα του σχήματος μάζας m=0,3Kg και ακτίνας R=4cm βρίσκεται σε επαφή με κύβο μάζας Μ=m=0,3Kg όπως φαίνεται στο σχήμα. Το σύστημα των δυο σωμάτων αφήνεται να κινηθεί από ένα σημείο Α κεκλιμένου επιπέδου γωνίας κλίσης φ. Τότε ο κύβος μάζας Μ ολισθαίνει και η σφαίρα μάζας m κυλίεται χωρίς να ολισθαίνει ενώ βρίσκεται συνέχεια σε επαφή με τον κύβο. Αν ο συντελεστής τριβής ολίσθησης ανάμεσα στον κύβο και το κεκλιμένο επίπεδο είναι μ=0,5 τότε:

α) Να υπολογιστεί η κοινή επιτάχυνση των δυο σωμάτων.

β)  Να υπολογιστεί:

  1. η στατική τριβή που δέχεται η σφαίρα από το κεκλιμένο επίπεδο
  2. η τριβή ολίσθησης κύβου – κεκλιμένου επιπέδου
  3. η δύναμη επαφής μεταξύ των δυο σωμάτων.

γ) Να υπολογιστεί:

  1. η ολική κινητική ενέργεια του συστήματος (m-M) και
  2. η στροφορμή του συστήματος (m-M)  ως προς σημείο του κεκλιμένου επιπέδου όταν αυτό κατέβει κατά (ΑΒ)=x=60cm πάνω στο κεκλιμένο επίπεδο.

δ) Αν αντικαταστήσουμε τον κύβο με έναν άλλο από πάγο και αφήσουμε ξανά το σύστημα να κινηθεί κατά μήκος του κεκλιμένου επιπέδου, τότε ποια είναι η επιτάχυνση των σωμάτων του συστήματος.

ε) Έστω ότι στη βάση του κεκλιμένου επιπέδου υπάρχει ελατήριο σταθερά Κ=30Ν/m και αφήνουμε το σύστημα [m-M(πάγου)] να ισορροπήσει πάνω σ’ αυτό, οπότε το ελατήριο συσπειρώνεται κατά x1. Στη συνέχεια συμπιέζουμε το ελατήριο κατά επιπλέον x1 και το αφήνουμε ελεύθερο να κινηθεί. Τότε να αποδείξετε ότι αυτό πραγματοποιεί α.α.τ  και να υπολογίσετε τη σταθερά D και το πλάτος Α της ταλάντωσης.

 

Θεωρείστε ότι τα κέντρα μάζας βρίσκονται στην ευθεία που είναι παράλληλη στο κεκλιμένο επίπεδο και ότι μεταξύ των επιφανειών των δυο μαζών δεν ασκείται κάποια δύναμη τριβής.

Ακόμη δίνεται για τη σφαίρα Ιcm=0,4 ×m×R2, ημφ=0,6 και g=10m/s2.

 

Συνοπτική λύση:

Σχόλια
Σχόλιο από τον/την ΧΡΗΣΤΟΣ ΕΛΕΥΘΕΡΙΟΥ στις 4 Μάιος 2014 στις 17:57
Διαγραφή σχολίου
Όμορφη Μιχάλη. Κάτι ανάλογο είχε κάνει και ο φίλος μου ο Αλε Κουάλε....
Σχόλιο από τον/την Μιχαήλ Μιχαήλ στις 4 Μάιος 2014 στις 19:49
Διαγραφή σχολίου
Χρήστο σ' ευχαριστώ! Καλά έκανες και μου το υπενθύμισες! Έχεις κάνει φοβερή δουλειά που δύσκολα μπορείς να την παρακολουθήσεις όλη! Να' σαι καλά
Σχόλιο από τον/την Κωστας Ψυλακος στις 6 Μάιος 2014 στις 16:05
Διαγραφή σχολίου
Μ^2 η συγκεκριμενη τα εχει ολα απο την αρχη μεχρι στο τελος-τελος ! Να εισαι καλα !
Σχόλιο από τον/την Μιχαήλ Μιχαήλ στις 6 Μάιος 2014 στις 20:06
Διαγραφή σχολίου
Κώστα σ' ευχαριστώ! Γιγάντια και η δική σου προσπάθεια να διαβάζεις προσεχτικά όλες τις αναρτήσεις και να κάνεις εποικοδομητικά σχόλια!
143. Καλαμάκι του φραπέ και … άλλα

 

1. Ένα καλαμάκι του φραπέ, έχει μήκος L και μάζα Μ.

α) Να υπολογιστεί η ροπή αδράνειας του, ως προς άξονα που περνά από το ένα άκρο του.

β) Στη συνέχεια το καλαμάκι κάμπτεται ώστε ένα κομμάτι του α= να σχηματίζει γωνία 900 με το υπόλοιπο καλαμάκι, ώστε να μπορούμε να πιούμε το φραπέ μας.

Μικραίνει ή αυξάνεται η ροπή αδράνειας που έχει το καλαμάκι ως προς άξονα περιστροφής που περνάει από το άκαμπτο άκρο του και είναι κάθετος στο επίπεδο περιστροφής;

Πόση είναι η ροπή αδράνειάς του ως προς το άκρο του αυτό;

Θεωρούμε ότι η διάμετρος από το καλαμάκι είναι πολύ μικρότερη από το μήκος του L ώστε αυτό να θεωρείται λεπτή ομογενής ράβδος με Ιcm=ΜL2

Συνοπτική λύση:

Σχόλια
Σχόλιο από τον/την Μανόλης Μαργαρίτης στις 17 Μάιος 2014 στις 12:14
Διαγραφή σχολίου
Εύγε Μιχαήλ χρησιμο αλλα και .....δροσιστικό:) μια μικρή διόρθωση στο Ι του 1 α
Σχόλιο από τον/την Μανόλης Μαργαρίτης στις 17 Μάιος 2014 στις 12:24
Διαγραφή σχολίου
Συγχαρητήρια για τη σημαντική δουλειά σου γενικά Μιχαήλ, και για το συγγραφικό σου έργο
Σχόλιο από τον/την Μιχαήλ Μιχαήλ στις 17 Μάιος 2014 στις 13:52
Διαγραφή σχολίου
Μανόλη σ' ευχαριστώ για τα ευγενικά σου σχόλια!
146. Σφαίρα σε κεκλιμένο επίπεδο και κρούση ράβδων

Το κέντρο Ο της ομογενούς σφαίρας του σχήματος μάζας m=2Kg και ακτίνας R είναι δεμένο στο ελεύθερο άκρο ελατηρίου σταθεράς Κ=70Ν/m, το άλλο άκρο του οποίου

είναι στερεωμένο σε ακλόνητο σημείο. Το σύστημα ελατήριο – σφαίρα, ισορροπεί σε κεκλιμένο επίπεδο γωνίας κλίσης θ όπως φαίνεται στο σχήμα.

Το κέντρο Ο της σφαίρας συνδέεται επίσης με το ένα άκρο μιας λεπτούς ομογενούς ράβδου ΑΒ, μέσω λεπτού αβαρούς και μη εκτατού νήματος που είναι παράλληλο στο κεκλιμένο επίπεδο. Η ράβδος έχει μάζα Μ=3 Kg και μήκος L=0,48m και σχηματίζει τότε γωνία φ=θ με την κατακόρυφη θέση της.

α) Κάποια στιγμή κόβουμε το νήμα. Να αποδειχτεί ότι, το κέντρο μάζας της σφαίρας πραγματοποιεί α.α.τ και να γραφεί η εξίσωση της ταλάντωσης.

β) Μόλις κοπεί το νήμα η ράβδος αρχίζει να περιστρέφεται ελεύθερα χωρίς τριβές σε κατακόρυφο επίπεδο γύρω από οριζόντιο άξονα που περνά από το άκρο της Α.

Όταν η ράβδος ΑΒ γίνει κατακόρυφη συγκρούεται μετωπικά και ελαστικά  με μια άλλη πανομοιότυπη ράβδο ΓΔ, που επίσης μπορεί να περιστρέφεται χωρίς τριβές σε κατακόρυφο επίπεδο γύρω από οριζόντιο άξονα που περνά από το άκρο της Γ. Τότε να υπολογίσετε τις ταχύτητες των δυο ράβδων αμέσως μετά την κρούση.

γ) Αν μετά την κρούση και τη στιγμή που ο ρυθμός μεταβολής της στροφορμής της ράβδου ΓΔ, γίνει μέγιστος ο άξονας περιστροφής της σπάει ακαριαία, τότε να βρείτε την κινητική ενέργεια της ράβδου ΓΔ, μόλις ακουμπήσει στο έδαφος.

Δίνονται: ημφ=0,8, για τη σφαίρα Ισ=0,4mR2 , για τη κάθε ράβδο Ι=0,33ΜL2  και g=10m/s2. Ακόμη θεωρείστε πως η σφαίρα κυλίεται.

 

Συνοπτική λύση στο blog:

ή εδώ

Σχόλια
Σχόλιο από τον/την Μιχαήλ Μιχαήλ στις 23 Μάιος 2014 στις 12:22
Διαγραφή σχολίου
Η άσκηση φιερώνεται στον Μανόλη Μαργαρίτη για την ιδέα που είχε με το σπάσιμο του άξονα περιστροφής. Τον ευχαριστώ και από εδώ!
Σχόλιο από τον/την Κορκίζογλου Πρόδρομος στις 23 Μάιος 2014 στις 13:23
Διαγραφή σχολίου
Σύνθετο πρόβλημα Μιχάλη ,εντός ύλης θα έλεγα, στα όρια ... Μου άρεσε, έχει πολύ δουλειά για εξετάσεις, εκτός αν κοπεί κάτι. Εύγε!!
Σχόλιο από τον/την Μιχαήλ Μιχαήλ στις 23 Μάιος 2014 στις 17:59
Διαγραφή σχολίου
Πρόδρομε σ' ευχαριστώ!
Σχόλιο από τον/την Μιχαήλ Μιχαήλ στις 23 Μάιος 2014 στις 18:56
Διαγραφή σχολίου
Δημήτρη σ΄ευχαριστώ! Για το κυλίεται (χωρίς ολισθαίνει) έχει γίνει αρκετές φορές συζήτηση στο yliko!
Σχόλιο από τον/την Κώστας Σερβετάς στις 23 Μάιος 2014 στις 19:29
Διαγραφή σχολίου
Μιχαήλ εξαιρετική η άσκηση!
Σχόλιο από τον/την Μανόλης Μαργαρίτης στις 23 Μάιος 2014 στις 19:35
Διαγραφή σχολίου
Ο Μιχαήλ σε όλο του το μεγαλείο!!!. Τι άλλο να πω....... ένα μεγάλο ευχαριστώ Μιχαήλ σου εύχομαι να είσαι καλά και να μας προσφέρεις πάντα τέτοια.
Σχόλιο από τον/την Γκενές Δημήτρης στις 23 Μάιος 2014 στις 19:56
Διαγραφή σχολίου
Μιχαήλ καλησπέρα Συγχαρητήρια Πλοκή φανταστική και σύνθεση Εξαιρετική, μεν, θα με άγχωνε δε αν ήμουν υποψήφιος. Αν μπορούσα να την είχα φτιάξει εγώ (δεν μπορώ γι' αυτό ζηλεύω και το δείχνω ) α) θα αφαιρούσα το πρώτο α από το αατ ( ασήμαντη παρατήρηση )... ( σαν ψείρας που είμαι ... ίσως και το Α από την ΑΔΜΕ να αφαιρούσα ) αλλά β) το μέτρο του ρυθμού μεταβολής της στροφορμής γίνεται μέγιστο στη ράβδο στην ανώτερη θέση γιατί μπορεί για κάποιον να είναι ελάχιστος αρνητικός ρυθμός μεταβολής της στροφορμής ... μάλλον καλύτερα είναι να δίνεται ως θέση μέγιστης εκτροπής να μην μπερδευόμαστε ... Ίσως εξοικονομήσω λίγο χρόνο να σου φτιάξω κι ένα i.p. ... ( χρειάζεται κάποιο κόλπο στην δεύτερη ράβδο για να μην ακουμπούν οι αρθρώσεις αλλά μάλλον θα βρω λύση ... )
Σχόλιο από τον/την Αντώνης Τόλιος στις 23 Μάιος 2014 στις 21:06
Διαγραφή σχολίου
Στην περίπτωση που εξαρθρωνεται ενω εξακολουθει να κινείται πως θα μπορούσα να βρω τη νεα γωνιακη ταχυτητα και την ταχύτητα του κέντρου μαζας? Σας ευχαριστώ εκ των προτέρων διοτι η βοήθεια σας σε νεους συνάδελφους όπως εγώ είναι ανεκτίμητη. ..
Σχόλιο από τον/την Γκενές Δημήτρης στις 23 Μάιος 2014 στις 22:38
Διαγραφή σχολίου
Επειδή το έταξα ... i.p.
Σχόλιο από τον/την Μανόλης Μαργαρίτης στις 23 Μάιος 2014 στις 22:44
Διαγραφή σχολίου
Αντώνη Τόλιο μπορείς να κοιτάξεις εδώ http://api.ning.com/files/fXUwj8AcBIlXBQPcu-Y8Vv0ZnHHKvfjiw0rXcKdwP..
Σχόλιο από τον/την Μιχαήλ Μιχαήλ στις 23 Μάιος 2014 στις 23:09
Διαγραφή σχολίου
Κώστα, Μανόλη, Δημήτρη,Αντώνη σας ευχαριστώ! Αντώνη δες το σύνδεσμο που σου δίνει ο Μανόλης! Δημήρη δε μπορώ να δω το i.p
Σχόλιο από τον/την Αντώνης Τόλιος στις 24 Μάιος 2014 στις 0:23
Διαγραφή σχολίου
Μανολη,Μιχαηλ ,ευχαριστώ ακόμη μια φορά.. Ο ενεργειακός τρόπος λύσης ήταν αυτό που σκέφτηκα και εγώ το δοκίμασα βιαστικά να εξαρθρωνεται στο σημείο της κρουση αλλά ένα αριθμητικό μου έβγαζε αύξηση της κινητικης ενέργειας.. Τελικά μετά απο σωστές πράξεις αν ο άξονας σπάσει αμεςως μετά την κρουση η ράβδος θα περιστραφει με ω=5rad/sec ως προς το cm(ίδια με πριν). Ειδικά μετά την επίλυση με την ΑΔΣ διαπιστώνω εξαιρετική ομοιότητα με την περίπτωση που αποκολλαται ένα σώμα απο το άκρο της περιστρεφομενης ράβδου όταν στρέφεται γύρω απο σταθερό άξονα..
Σχόλιο από τον/την Κωστας Ψυλακος στις 24 Μάιος 2014 στις 15:19
Διαγραφή σχολίου
Μιχαλη ομορφη η πλοκη της ασκησης με αρκετη περιπετεια ! Καταλαβαινω στο β οτι ηθελες να δωσεις μια γενικοτερη λυση αλλα εφαρμοσα ΑΔΣ Ιω=Ιω1+Ιω2=> ω=ω1+ω2 (1) ΑΔΜΕ Ιω^2=Ιω1^2+Ιω2^2 => ω^2=ω1^2+ω2^2 (2) (1) και (2) 2ω1*ω2=0 => ω1=0 αποδεκτη και απο (1) ω2=ω=5r/s Στο Γ ,με εμπλεξε λιγο ο ρυθμος μεταβολης της L συμφωνω με τον Γ.Γκενε, ομως οπως πολυ σωστα λες η ραβδος κανει μονο μεταφορικη κινηση στην συνεχεια και μαλιστα ελευθερη πτωση . Ετσι το cm διανυει την ιδια αποσταση με καθε σημειο της ραβδου την οποια την βρισκω απο την αποσταση που διενυσε το Γ αρα y=L*(1-συνθ) ,L=μηκος ραβδου (υπολογιζω πιο ευκολα το μηκος που διενυσε η ραβδος) y=ucm^2/2g => ucm^2= 2gL*(1-συνθ) => K=0.5*M*2gL*(1-συνθ)=MgL(1-συνθ)=5.76j. Και μια τελευταια παρατηρηση (για να κανουμε και λιγη πλακα) απευθυνομαι σε σενα και στον Μ.Μαργαριτη βαλτε ρε παιδια πιο ανθεκτικους αξονες να μην σπανε να τελειωνουμε με αυτην την ιστορια αμαν ....!!!! ΚΑΛΗ ΣΥΝΕΧΕΙΑ ....
Σχόλιο από τον/την Μιχαήλ Μιχαήλ στις 24 Μάιος 2014 στις 15:37
Διαγραφή σχολίου
Κώστα σ' ευχαριστώ για τα σχόλιά σου και τις παρατηρήσεις σου. Να' σαι καλά.
148. Ράβδος με σφαίρες.

Στις άκρες Α και Β μιας ράβδου ΑΒ μάζας Μ=2Κg και μήκους L=3m δένουμε δυο μικρές σφαίρες με μάζες m και 2m αντίστοιχα όπου m=Kg.

Η ράβδος μπορεί να περιστρέφεται χωρίς τριβές σε κατακόρυφο επίπεδο, γύρω από οριζόντιο άξονα που περνάει από το σημείο Ο. Το σημείο Ο απέχει από τις άκρες τις

ράβδου αντίστοιχα αποστάσεις (ΟΑ)= και (ΟΒ)=2.

α) Για την αρχική οριζόντια θέση του συστήματος να υπολογιστεί η κατακόρυφη δύναμη F που πρέπει να ασκήσουμε στο μέσον Κ της ράβδου, ώστε το σύστημα να ισορροπεί.

β) Ποια είναι η δύναμη που ασκείται από τον άξονα περιστροφής στη ράβδο όταν το σύστημα ισορροπεί στην οριζόντια θέση;

γ) Κάποια στιγμή (t=0) καταργούμε τη δύναμη F και το σύστημα αφήνεται ελεύθερο να κινηθεί. Τότε για την οριζόντια θέση:

i) ποιός είναι ο ρυθμός μεταβολής της στροφορμής του συστήματος;

ii) ποια είναι η δύναμη που ασκεί η ράβδος στη σφαίρα 2m στη θέση αυτή;

δ) Κάποια στιγμή t1 η ράβδος γίνεται κατακόρυφη. Πόση γίνεται τότε η ταχύτητα της σφαίρας m;

ε) Αν τη στιγμή t1 που η ράβδος περνάει από την κατακόρυφη θέση, η σφαίρα 2m που βρίσκεται στο σημείο Β αποκολλάται ακαριαία τότε:

i) Πόση είναι η δύναμη που ασκεί η ράβδος στη σφαίρα m που βρίσκεται στο σημείο Α;

ii) Να βρείτε την ταχύτητα της σφαίρας m μόλις η ράβδος ξαναέρθει στην οριζόντια θέση.

Δίνονται: Για την ράβδο Ιcm=1/12Μ∙L2  και g=10m/s2.

Συνοπτική λύση στο blog:

ή εδώ

Σχόλια
Σχόλιο από τον/την Μανόλης Μαργαρίτης στις 28 Μάιος 2014 στις 19:38
Διαγραφή σχολίου
Μιχαήλ Μπράβο πολύ ωραία για επανάληψη!
Σχόλιο από τον/την Μιχαήλ Μιχαήλ στις 28 Μάιος 2014 στις 22:17
Διαγραφή σχολίου
Μανόλη σ' ευχαριστώ! Η αλήθεια είναι πως πράγματι ήθελα να συμμαζέψω τις περιπτώσεις που έχουμε ράβδο με σφαίρα!

Σχόλιο

149. Σφαίρα σε οριζόντια επίπεδα.

Ομογενής σφαίρα μάζας m=2Kg και ακτίνας R=10cm ηρεμεί αρχικά πάνω σε οριζόντιο επίπεδο σε σημείο Α. Κάποια στιγμή (t=0), εξασκείται στο κέντρο μάζας Κ της σφαίρας,  σταθερή οριζόντια δύναμη F=14N όπως φαίνεται στο σχήμα. Η σφαίρα τότε αρχίζει να κυλίεται χωρίς να ολισθαίνει.

Όταν η σφαίρα διανύσει απόσταση (ΑΒ)= x1=1,6m συναντάει λείο οριζόντιο επίπεδο και συνεχίζει να κινείται  πάνω σ’ αυτό. Κάποια στιγμή φτάνει στο σημείο Γ με (ΒΓ)=x2=x1=1,6m.

A) α) Να υπολογιστεί η ολική κινητική ενέργεια Κολ της σφαίρας μόλις φτάσει στο σημείο Γ.

Πόση είναι τότε η κινητική ενέργεια της σφαίρας λόγω μεταφορικής και πόση λόγω στροφικής κίνησης;

β)i) Πως μεταβάλλεται ο ρυθμός μεταβολής της στροφορμής της σφαίρας κατά τη διάρκεια της κίνησής της από το σημείο Α και μέχρι το σημείο Γ;

ii) Να υπολογιστεί ο ρυθμός μεταβολής της κινητικής ενέργειας της σφαίρας όταν αυτή βρίσκεται στο σημείο Β και όταν βρίσκεται στο σημείο Γ.

Β) Καθώς η σφαίρα κινείται μετά το σημείο Γ και μόλις η ολική κινητική της ενέργεια γίνει 106,4 J, αυτή συναντάει ένα οριζόντιο επίπεδο με συντελεστή τριβής ολίσθησης μ=0,8 και συνεχίζει να κινείται πάνω σ’ αυτό.

α) Σε πόσο χρόνο t1 από τη στιγμή που συναντάει η σφαίρα το οριζόντιο αυτό επίπεδο αρχίζει η κύλισή της;

β) Πόσος είναι,

i) ο ρυθμός μεταβολής της στροφορμής της σφαίρας και

ii) ο ρυθμός μεταβολής της κινητικής ενέργειας της σφαίρας τη χρονική στιγμή t2=t1/2;

γ) Πως μεταβάλλεται με το χρόνο η επιτάχυνση της σφαίρας καθώς μεταβαίνει από επίπεδο σε επίπεδο κίνησης;

Δίνονται: Για τη σφαίρα Ιcm=0,4m∙R2  και g=10m/s2.

Συνοπτική λύση στο blog:

ή εδώ

Σχόλια
Σχόλιο από τον/την Μανόλης Μαργαρίτης στις 31 Μάιος 2014 στις 11:47
Διαγραφή σχολίου
Μιχαήλ προσφέρεις και πάλι μια αξιόλογη ανάρτηση που κάνει φύλλο και φτερό όλη την κινηματική (και όχι μόνο) στερεού ( κατάλληλη για επανάληψη , που αξίζει να την δουν οι υποψήφιοι) .Μιχαήλ θα ήθελα να ζητάς επίσης και το ρυθμό μεταβολής της στροφορμής ως προς σημείο του οριζόντιου εκτος από αυτόν του spin που ζητάς. ΣΥΓΧΑΡΗΤΗΡΙΑ
Σχόλιο από τον/την Μιχαήλ Μιχαήλ στις 31 Μάιος 2014 στις 17:19
Διαγραφή σχολίου
Μανόλη καλησπέρα και σ' ευχαριστώ για τα σχόλιά σου. 'Οσο για τη στροφορμή ως προς σημείο του οριζόντιου επιπέδου κίνησης έγινε και αυτό! Να' σαι καλά!
Σχόλιο από τον/την Μανόλης Μαργαρίτης στις 31 Μάιος 2014 στις 17:31
Διαγραφή σχολίου
να σαι καλά!
Σχόλιο από τον/την Εμμανουήλ Λαμπράκης στις 1 Ιούνιος 2014 στις 11:50
Διαγραφή σχολίου
Μιχαήλ καλημέρα Εξαιρετικό θέμα για επανάληψη.
Σχόλιο από τον/την Μιχαήλ Μιχαήλ στις 1 Ιούνιος 2014 στις 12:22
Διαγραφή σχολίου
Μανώλη χαίρομαι που σ' άρεσε! Σ' ευχαριστώ, να' σαι πάντα καλά!
150. Ελεύθερη (Α.Α.T) – Φθίνουσα ταλάντωση

Σ

το σώμα του σχήματος και σε μια τυχαία θέση, ασκείται μια συνολική δύναμη (δύναμη επαναφοράς), της μορφής ΣF=-D∙x. Τότε από το 2ο Νόμο του Newton προκύπτει:

η συνέχεια στο blog

ή εδώ

Σχόλια
Σχόλιο από τον/την Διονύσης Μάργαρης στις 8 Αύγουστος 2014 στις 9:26
Διαγραφή σχολίου
Καλημέρα Μιχαήλ. Σε ευχαριστούμε για την πλήρη μελέτη και των δύο ταλαντώσεων που μας έδωσες.
Σχόλιο από τον/την Δημήτρης Τσάτσης στις 8 Αύγουστος 2014 στις 10:20
Διαγραφή σχολίου
Καλημέρα Μιχαήλ. Αναλυτικότατη και πλήρης η εργασία σου. Χρησιμότατη.. Ευχαριστώ και από μένα. Να είσαι καλά.
Σχόλιο από τον/την Μιχαήλ Μιχαήλ στις 8 Αύγουστος 2014 στις 14:51
Διαγραφή σχολίου
Διονύση και Δημήτρη καλό μεσημέρι! Σας ευχαριστώ πολύ για τα θετικά σας σχόλια! Η αλήθεια είναι ότι είχα στο μυαλό μου την περίπτωση που η απόσβεση είναι μεγάλη (Λ>ω0) καθώς και την περίπτωση της κρίσιμης απόσβεσης (Λ=ω0). Πραγματικά το Geogebra δίνει χρήσιμες Γ.Π για τη συμπεριφορά του ταλαντωτή!
Σχόλιο από τον/την Στέργιος Ναστόπουλος στις 8 Αύγουστος 2014 στις 15:20
Διαγραφή σχολίου
Πολύ ωραία και πλήρης η περιγραφή Μιχάλη! Μια διαπραγμάτευση σχετικά με την ΔΕ στην υπερκρίσιμη απόσβεση έχει και στο βίντεο εδώ: https://www.youtube.com/watch?v=2-5oq-igwtU
Σχόλιο από τον/την Μιχαήλ Μιχαήλ στις 8 Αύγουστος 2014 στις 20:51
Διαγραφή σχολίου
Στέργιο σ' ευχαριστώ. Πολύ κατατοπιστικό το video!
Σχόλιο από τον/την Εμμανουήλ Λαμπράκης στις 8 Αύγουστος 2014 στις 21:08
Διαγραφή σχολίου
Μιχαήλ καλησπέρα Πολύ καλή και εμπεριστατωμένη δουλειά. Μπράβο.
Σχόλιο από τον/την Μιχαήλ Μιχαήλ στις 8 Αύγουστος 2014 στις 23:27
Διαγραφή σχολίου
Μανώλη σε ευχαριστώ. Να΄σαι καλά!
Σχόλιο από τον/την Μιχαήλ Μιχαήλ στις 14 Αύγουστος 2014 στις 19:30
Διαγραφή σχολίου
Διονύση (Μαργ) στη σελίδα 9 πρόσθεσα μερικές παρατηρήσεις σου!
Οριζόντια βολή - κυκλική κίνηση

Στη λίμνη Κερκίνη το φράγμα έχειύψος h=11,25m.Αν κάποιος πετάξει οριζόντια ένα βότσαλο πάνω από το φράγμα με υ0=10m/s, μετά από πόσο χρόνο θα ακούσει τον ήχο από το βότσαλο που έπεσε στο νερό; ήχου=340m/s).


Κεφάλαιο 1. Καμπυλόγραμμες κινήσεις: Οριζόντια βολή- Κυκλική κίνηση

ή εδώ

 

Σχόλια Σχόλιο από τον/την Βαγγέλης Κουντούρης στις 7 Σεπτέμβριος 2014 στις 21:55 Διαγραφή σχολίου Πανέμορφη, ως πραγματική, διότι η Φυσική δεν είναι …εκεί, είναι εδώ. Μπράβο Μιχαήλ. (“με το νου” βρίσκω περίπου 1,6s) Σχόλιο από τον/την Μιχαήλ Μιχαήλ στις 7 Σεπτέμβριος 2014 στις 22:52 Διαγραφή σχολίου Βαγγέλη καλησπέρα και σ΄ευχαριστώ. Οι απαντήσεις όλων των ασκήσεων υπάρχουν στο ίδιο αρχείο! Σχόλιο από τον/την Παπασγουρίδης Θοδωρής στις 8 Σεπτέμβριος 2014 στις 0:39 Διαγραφή σχολίου Γεια σου Μιχάλη και καλή συνέχεια, διότι βλέπω κάτι ετοιμάζεις για τη Β' Λυκείου Ωραία ιδέα, λιτή και ουσιαστική. Μήπως όμως στην απόσταση που διανύει ο ήχος να βάλουμε και το ύψος του ανθρώπου; Δηλαδή στο 11,25m να προσθέσουμε και 1,75m και να γίνει 13m, οπότε d=19,85m Σχόλιο από τον/την Κωνσταντίνος Λουκόπουλος στις 8 Σεπτέμβριος 2014 στις 2:05 Διαγραφή σχολίου καλημέρα Μιχαήλ! συγχαρητήρια για τη δουλειά σου! Σχόλιο από τον/την Μιχαήλ Μιχαήλ στις 8 Σεπτέμβριος 2014 στις 9:45 Διαγραφή σχολίου Θοδωρή και Κωνσταντίνε καλημέρα και σας ευχαριστώ! Θοδωρή προσπάθησα να δείξω ότι ο άνθρωπος ρίχνει την πέτρα σύριζα με το φράγμα. Όμως σωστό και αυτό που λες! 151. Σύνθεση δυο ταλαντώσεων

1)      Έστω ότι ένα σώμα πραγματοποιεί ταυτόχρονα τις παρακάτω εξαναγκασμένες ταλαντώσεις x1 και x2 με

 

x1=κ∙ημωt και x2= λ∙συνωt

 

που γίνονται στην ίδια διεύθυνση και γύρω από την ίδια θέση ισορροπίας με την ίδια συχνότητα ω. Τότε σύμφωνα με την επαλληλία των κινήσεων έχουμε:

 

x=x1+x2= κ∙ημωt+λ∙συνωt=κ( ημωt+λ/κ∙συνωt).

 

Θέτουμε ............

 

και εδώ

Σχόλια
Σχόλιο από τον/την Θρασύβουλος Μαχαίρας στις 9 Σεπτέμβριος 2014 στις 0:19
Διαγραφή σχολίου
Βρε Μιχάλη συγχώρα μου το θράσος, αλλά α) δεν υπάρχει σώμα που να εκτελεί ταυτόχρονα δύο κινήσεις. Η κίνηση είναι μία και μόνο μία β) δεν υπάρχει σύνθεση απλών αρμονικών ταλαντώσεων γ) δεν απέδειξες ότι η σύνθεση δύο απλών αρμονικών ταλαντώσεων είναι απλή αρμονική ταλάντωση, γιατί μια τέτοια απόδειξη θα σε οδηγούσε στο ότι δεν υπάρχει σύνθεση α.α.τ.. Τελικά δεν απέδειξες τίποτε... Όταν οι ταλαντώσεις που συνθέτεις έχουν ίδια συχνότητα δε σημαίνει ούτε ότι είναι α.α.τ. ούτε ότι θα οδηγήσουν σε σύνθεση α.α.τ.... Μη με πάρεις για κακό... Αλλά δεν απέδειξες απολύτως τίποτε... Μάλλον μπέρδεψες πολλά... Και πάλι ζητώ συγγνώμη Μιχάλη αλλά είπα μια γνώμη...
Σχόλιο από τον/την Μιχαήλ Μιχαήλ στις 9 Σεπτέμβριος 2014 στις 11:32
Διαγραφή σχολίου
Θρασύβουλε καλημέρα! Ας μείνει η ανάρτηση όπως έχει, έτσι και αλλιώς έχει περισσότερο τεχνικό χαρακτήρα!
Σχόλιο από τον/την Θρασύβουλος Μαχαίρας στις 9 Σεπτέμβριος 2014 στις 14:33
Διαγραφή σχολίου
Μιχάλη σε ευχαριστώ που δέχεσαι το απόλυτο του χαρακτήρα μου Κάποτε έκανα μια ομιλία σε Φυσικούς όπου απέδειξα ότι δεν υπάρχει σύνθεση α.α.τ. Κοίτα εδώ. Η δουλειά μου βάζει τη σωστή διδακτική επιλογή απαραίτητη ως γνώση και πάνω από όλα ως αξία Δασκάλου. Αυτό δεν αναιρεί την εργασία σου αλλά προτείνει να αλλάξεις τα λόγια σε κάποια σημεία, εφόσον το κρίνεις και εσύ σωστό.
Σχόλιο από τον/την Κυριακόπουλος Γιάννης στις 9 Σεπτέμβριος 2014 στις 14:55
Διαγραφή σχολίου
Θέλω να δω Θρασύβουλε να διατυπώνεις την εκφώνηση παρόμοιας άσκησης.
Σχόλιο από τον/την Θρασύβουλος Μαχαίρας στις 9 Σεπτέμβριος 2014 στις 15:26
Διαγραφή σχολίου
Να το κάνω ευθύς μόλις μου το ζήτησες Γιάνη. Αλλά πρέπει ο Μιχάλης να μου δώσει το κείμενο σε word.. Μέχρι να μου δοθεί το κείμενο σε word θα σε παρακαλούσα να πάρεις και εσύ θέση σε αυτό και αυτό και αυτό για να μη χαθεί η αξία όσων σε λιγο θα διδάξουμε
Σχόλιο από τον/την Κυριακόπουλος Γιάννης στις 9 Σεπτέμβριος 2014 στις 15:35
Διαγραφή σχολίου
Δηλαδή να πάρω θέση για το πείραμα του βιβλίου; Αν υλοποιείται ή αν είναι άσχετο με εξαναγκασμένη ταλάντωση διότι η μεταβολή του μήκους του ελατηρίου οδηγεί σε χρονοεξαρτώμενες δυνάμεις. Ευχαρίστως να τοποθετηθώ (καθυστερημένα λόγω απομόνωσης στο Ρέθυμνο) αλλά γενικώς, στο θεωρητικό ή στο πειραματικό κομμάτι της ενδιαφέρουσας (αν και εκτραπείσας) συζήτησης;
Σχόλιο από τον/την Θρασύβουλος Μαχαίρας στις 9 Σεπτέμβριος 2014 στις 15:35
Διαγραφή σχολίου
Δημήτρη χαίρομαι πολύ με τη ζωή σου και με τις κινήσεις σου...Λατρεύω τη φούρια σου... ..... Στο άσχετο τώρα: Σε έναν φοιτητή το θέμα της σύνθεσης τίθεται όπως θες να τον διδάξεις για να γίνει Φυσικός σε παιδιά που μπορεί να γίνουν Φυσικοί ή να ξεχάσουν το σχολείο .
Σχόλιο από τον/την Θρασύβουλος Μαχαίρας στις 9 Σεπτέμβριος 2014 στις 15:50
Διαγραφή σχολίου
Είπες την καλύτερη φράση: " ...ο Μιχαήλ δεν χρησιμοποίησε διαφορετική ορολογία και μεθολογία από αυτή που υπάρχουν στα καθιερωμένα βιβλία Γενικής Φυσικής..." Έχετε δίκιο! Εγώ είμαι εκτός... Στο σχολικό υπάρχει σύνθεση α.α.τ. Άρα γιατί να μη κάνουμε ασκήσεις με σύνθεση α.α.τ. Ειλικρινά και χωρίς κανένα υπονοούμενο πρέπει να γίνουν ασκήσεις σύνθεσεις α.α.τ. Ετσι το μόνο που μου μένι να φωνάξω είναι κάτι σε Φυσικούς: Δεν υπάρχει σύνθεση α.α.τ., αλλά μόνο μία και μόνο μία α.α.τ.
Σχόλιο από τον/την Θρασύβουλος Μαχαίρας στις 9 Σεπτέμβριος 2014 στις 16:15
Διαγραφή σχολίου
Γιάννη θέλω να πάρεις θέση όχι μόνο στις χρονοεξαρτώμενς "δυναμικές ενέργειες" και στο αν το κούνημα του ελατηρίου είναι αυτό που πρέπει να διδ'ασκουμε στα παιδιά. Θέλω να πάρεις θέση στη διαμάχη ανάμεσα σε μένα και το Βλάχο, μόνο και μόνο γιατί τον ενθάρρυνες να μου πει τα τόσα. Τον ενθάρρυνες σε βλακείες και μου επιβάλεται τώρα να σου πω ότι πρέπει να διαλέξεις αν τα σύκα είναι σύκα...

Γιάννη δώσμου μια στάση αλλά εδώ

.
Σχόλιο από τον/την Κυριακόπουλος Γιάννης στις 9 Σεπτέμβριος 2014 στις 18:05
Διαγραφή σχολίου
Θα απαντήσω Θρασύβουλε. Όχι διότι τον ενθάρρυνα μια και δεν χρειάζεται τη δική μου στήριξη. Ούτε για να εξομαλύνω έναν διάλογο που θα ήθελα να μην περιέχει κατηγορώ και υποτιμητικές εκφράσεις. Η αξία του βιβλίου σου δεν επηρεάζεται από το αν ένα πείραμα υλοποιείται ή όχι οπότε καλύτερα να δούμε το θέμα ψύχραιμα ή αν θέλεις τεχνικά. Δεν ξέρω αν καλούμαι να χαρακτηρίσω τα σύκα ως σύκα αλλά θα πω την άποψή μου με απλά λόγια. Στη σωστή θέση και όχι στην παρούσα αλλά λίγο χρόνο ζητώ.
152. Συμβολή κυμάτων με διαφορετικά πλάτη

Δυο σύμφωνες πηγές κυμάτων Π1 και Π2 βρίσκονται στην επιφάνεια υγρού και εκτελούν  ταλαντώσεις με εξισώσεις y1=2∙10-2 ημπt και y2=10-2∙ημ(πt+π/4) (S.I).

Τα κύματα διαδίδονται με ταχύτητα υ=2m/s. Ένα υλικό σημείο Μ βρίσκεται στην επιφάνεια του υγρού και απέχει από τις πηγές αποστάσεις r1=1m και r2= 5/6 αντίστοιχα.

i)Να γράψετε τις εξισώσεις ταλάντωσης του σημείου Μ, εξαιτίας του κάθε κύματος χωριστά καθώς και την εξίσωση ταλάντωσης του Μ μετά τη συμβολή των δύο κυμάτων.

ii) Να υπολογίσετε την απομάκρυνση του σημείου Μ τη χρονική στιγμή t=0,5s.

 

Συνοπτική λύση:

και εδώ

Σχόλια
Σχόλιο από τον/την Διονύσης Μάργαρης στις 11 Οκτώβριος 2014 στις 19:40
Διαγραφή σχολίου
Καλησπέρα Μιχαήλ. Ωραίο θέμα, που αξίζει να διδαχτεί, αφού συνδέει τη συμβολή με τη σύνθεση ταλαντώσεων, που όσο και αν το λέμε, δύσκολα ο μαθητής κάνει τη σύνδεση. Υπάρχει ένα προβληματάκι βέβαια με την εφθ. Δύσκολα να αντιμετωπισθεί από μαθητή, αν δεν προκύψει μια γνωστή γωνία...
Σχόλιο από τον/την Μιχαήλ Μιχαήλ στις 11 Οκτώβριος 2014 στις 20:29
Διαγραφή σχολίου
Διονύση καλησπέρα και σε ευχαριστώ για το σχόλιο.
Σχόλιο από τον/την Γκενές Δημήτρης στις 11 Οκτώβριος 2014 στις 20:57
Διαγραφή σχολίου
Συγχαρητήρια Μιχαήλ. Ξέρεις ΤΩΡΑ πόσο πολύ μου αρέσουν αυτά ... και ίσως να έβαζα περιορισμούς του k στον υπολογισμό των r1 kai r2 ... από την άλλη μου έρχονται εικόνες από τον εαυτό μου στο Λύκειο ... πως να το πω; ... κάτι σαν τον μαθητή του Κασσέτα
Σχόλιο από τον/την Μιχαήλ Μιχαήλ στις 11 Οκτώβριος 2014 στις 23:52
Διαγραφή σχολίου
Δημήτρη ευχαριστώ! Ευτυχώς εδώ δεν είχε πολύ τριγωνομετρία!!
153. Συμβολή κυμάτων (Asqr3 )

Δυο σύγχρονες πηγές παραγωγής κυμάτων Π1 και Π2 που βρίσκονται στην επιφάνεια υγρού και απέχουν απόσταση d=6m, εκπέμπουν αρμονικά κύματα πλάτους Α και μήκους κύματος λ=3m.

i) Να βρείτε το πλήθος των σημείων του Π1Π2 που ταλαντώνονται με πλάτος Αsqr3 και να σχεδιάσετε τις αντίστοιχες υπερβολές μεταξύ των Π1 και Π2.

ii) Να βρείτε την ελάχιστη οριζόντια απόσταση ανάμεσα σε ένα σημείο του ευθυγράμμου τμήματος Π1Π2 που ταλαντώνεται με πλάτος Αsqr3 και

α) ενός σημείου ενισχυτικής συμβολής

β) ενός σημείου αποσβεστικής συμβολής.

iii) Με ποιο πλάτος ταλαντώνονται τα σημεία της ευθείας Π1Π2 που βρίσκονται εκτός του ευθυγράμμου τμήματος Π1Π2;

 

Συνοπτική λύση:

Κεφάλαιο 2 Διατήρηση της ορμής

Κεφάλαιο 2: Διατήρηση της ορμής

Σχόλια
Σχόλιο από τον/την Βασίλης Δουκατζής στις 14 Οκτώβριος 2014 στις 23:03
Διαγραφή σχολίου
Μιχάλη εξαιρετική δουλειά!!! Πολλά μπράβο. Σ΄ευχαριστούμε. Σε λίγο θα ανεβάσω και την δική μου εκδοχή αλλά σε άσκησεις μόνο και όχι τόσο πλήρες έργο όπως εσύ. Να σαι καλά!
Σχόλιο από τον/την Μιχαήλ Μιχαήλ στις 14 Οκτώβριος 2014 στις 23:37
Διαγραφή σχολίου
Βασίλη σε ευχαριστώ! Πολλά μπράβο και σε σένα για όσα μας δίνεις!
Σχόλιο από τον/την Γκενές Δημήτρης στις 16 Οκτώβριος 2014 στις 22:09
Διαγραφή σχολίου
Μιχαήλ Συγχαρητήρια για την όμορφη αυτή παρουσίαση για την προσεκτική επιλογή των σημείων της θεωρίας και την ισορροπία στην έκταση και το βάθος. Βέβαια θίγεις πολλά ζητήματα στα οποία προβληματίζομαι αν είναι ο σωστός τρόπος εισαγωγής τους. Ίσως ο λόγος να είναι ότι προσπαθείς να μην ξεφύγεις πολύ από το σχολικό εγχειρίδιο Θα δώσω επιγραμματικά μερικά από τα σημεία αυτά του προβληματισμού μου. 1. Χρειάζεται αναφορά σε επαφή; Βέβαια εσύ τονίζεις : στη Μηχανική αλλά ποιος μαθητής θα προσέξει αυτό και ποιο είναι το σημαινόμενο του όρου μηχανική ( αντικείμενα σημειακά ; ή μήπως corpuscular or rigid αλλά λίγο αργότερα εισέρχονται παραδείγματα με διαμεσολάβηση ιδανικού ελατηρίου και πιο κάτω σημείωση με παράδειγμα υπερελαστικότητας στην πυρηνική ). Θα προτιμούσα ο ορισμός της κρούσης ως φαινόμενο ανταλλαγής ορμών μεταξύ δυο αντικειμένων πολύ μικρής χρονικής διάρκειας και με εμφάνιση πολύ μεγάλων δυνάμεων. 2. Η κινητική ενέργεια Κ του συστήματος διατηρείται στις τελείως ελαστικές; Ή η Κ αμέσως μετά την κρούση είναι ίση με την Κ ελάχιστα πριν από αυτήν; Μπορούμε να απαλείψουμε εκείνο το τελείως μπροστά από την ελαστική; 3. Η Κ του συστήματος μετά την κρούση παρουσιάζει ελάττωση μόνο στις πλαστικές ή και στις ελαστικές αλλά όχι στις τελείως ελαστικές ; Τελικά εκτός από την τελείως ελαστική και την τελείως ανελαστική ( πλαστική ) υπάρχουν ελαστικές ή ανελαστικές ; Στα συγγράμματα υπάρχουν αυτές οι οι διαφορές στην ορολογία για τα αντικείμενα με συντελεστή ελαστικότητας διάφορη του 0 και του 1. 4. Είναι σκόπιμο από διδακτική άποψη να αναφερθεί η περίπτωση να δίνεται ένα από τα δυο αντικείμενα (ή κάποιο τρίτο ) να είναι ακλόνητο ... πρόκειται για κρυμμένο δεδομένο πιθανής εξαίρεσης από την αρχή της διατήρησης της ορμής κατά την κρούση ; Ή είναι απλά μια περίπτωση που καλύπτεται από τη θεωρία και ο μαθητής πρέπει να μπορεί να δει ότι το σύστημα δεν είναι απομονωμένο ; Ας σταματήσω εδώ ... Τελικά δεν είναι και τόσο εύκολη η επιλογή της έκτασης και του βάθους στη θεωρία ... (και κυρίως αν δεν είχες τον έλεγχο της θεμελίωσης και της εδαφοτεχνικής μελέτης )
Σχόλιο από τον/την Μιχαήλ Μιχαήλ στις 18 Οκτώβριος 2014 στις 12:33
Διαγραφή σχολίου
Δημήτρη σε ευχαριστώ! Οι προβληματισμοί που θέτεις είναι πολύ αξιόλογοι και πρέπει σίγουρα κάποια σημεία τους να θίγονται κατά τη μελέτη της κρούσης.
Υπολογισμός της κεντρομόλου επιτάχυνσης και της κεντρομόλου δύναμης.

Υπολογισμός της κεντρομόλου επιτάχυνσης και της κεντρομόλου δύναμης.

Σχόλια
Σχόλιο από τον/την Κυριακόπουλος Γιάννης στις 14 Οκτώβριος 2014 στις 20:43
Διαγραφή σχολίου
Ωραία ιδέα. Παραλλαγή η δύναμη σημείου ως προς κύκλο.
Σχόλιο από τον/την Μιχαήλ Μιχαήλ στις 14 Οκτώβριος 2014 στις 23:39
Διαγραφή σχολίου
Γιάννη είναι μια απόδειξη που υπάρχει στο βιβλίο του Μάζη!
Σχόλιο από τον/την Γκενές Δημήτρης στις 15 Οκτώβριος 2014 στις 22:51
Διαγραφή σχολίου
Καλησπέρα Μιχαήλ. Πραγματικά έχω κολλήσει και δεν καταλαβαίνω πως από την προτελευταία σχέση (υ.Δt)2= ½ a(Δt)2.2R προκύπτει η α=υ2/2R τι δεν καταλαβαίνω και βλέπω ένα 2 να εμφανίζεται ή να εξαφανίζεται ;
Σχόλιο από τον/την Γκενές Δημήτρης στις 15 Οκτώβριος 2014 στις 22:54
Διαγραφή σχολίου
Συγνώμη κατάλαβα απλά τυπογραφικό λάθος της τελευταίας ... πρέπει να σβήσουμε τα 2αρια στην τελευταία γραμμή.
Σχόλιο από τον/την Μιχαήλ Μιχαήλ στις 15 Οκτώβριος 2014 στις 23:03
Διαγραφή σχολίου
Δημήτρη η διόρθωση έγινε!!
Σχόλιο από τον/την Γκενές Δημήτρης στις 15 Οκτώβριος 2014 στις 23:26
Διαγραφή σχολίου
Μιχαήλ αυτό που εντυπωσιάζει είναι ότι το είχα διδαχθεί στο σχολείο, αλλά το ξέχασα ως δάσκαλος. Τελικά το μήνυμα σου δεν είναι καλό ούτε για εμένα ούτε για τη γεωμετρία.
Ισοδυναμικές Επιφάνειες

Οι επιφάνειες που έχουν το ίδιο δυναμικό ονομάζονται ισοδυναιμκές επιφάνειες.

Οι ισοδυναμικές επιφάνειες είναι ΚΑΘΕΤΕΣ στις δυναμικές γραμμές (Ε) του πεδίου

 

Η ισοδυναμικές επιφάνειες είναι ΠΑΝΤΑ ΚΛΕΙΣΤΕΣ ΕΠΙΦΑΝΕΙΕΣ, σε αντίθεση με τις γραμμές πεδίου που αρχίζουν και τελειώνουν σε φορτία.

Στο διπλανό σχήμα με πράσινο χρώμα έχουμε σχεδιάσει τις ισοδυναμικές επιφάνειες για δυο ίσα και αντίθετα ηλεκτρικά φορτία (ηλεκτρικό δίπολο)

Για ένα σημειακό θετικό ηλεκτρικό φορτίο οι ισοδυναμικές επιφάνειες για δυναμικά V1, V2 και V3 αντίστοιχα είναι αυτές που φαίνονται στο διπλανό σχήμα όπου V1>V2>V3.

συνέχεια

ή εδώ

Σχόλια
Σχόλιο από τον/την ΚΩΣΤΟΠΟΥΛΟΣ ΑΘΑΝΑΣΙΟΣ στις 3 Νοέμβριος 2014 στις 23:27
Διαγραφή σχολίου
πολύ καλό θέμα με το γεωμετρικό τόπο, μπράβο Μιχαηλ.
Σχόλιο από τον/την Διονύσης Μάργαρης στις 4 Νοέμβριος 2014 στις 8:52
Διαγραφή σχολίου
Καλημέρα Μιχαήλ. Σε ευχαριστώ που μας θυμίζεις τμήμα ύλης, που πριν λίγα χρόνια διδάσκαμε και τώρα φαντάζουν ...εξωπραγματικά.
Σχόλιο από τον/την ΓΙΑΝΝΗΣ ΔΟΓΡΑΜΑΤΖΑΚΗΣ στις 4 Νοέμβριος 2014 στις 9:14
Διαγραφή σχολίου
Καλημέρα Μιχαήλ ...καλημέρα σε όλους. Πολύ καλή η ιδέα σου. Η διδακτική αξία της αισθητοποίησης ενός πεδίου δυνάμεων είναι μεγάλη. Είναι ένα από τα πιο ''αόρατα'' σημεία ...της διδασκαλίας μας στην Β' Λυκείου. Αν προσθέσεις και τον τυπικό ορισμό του σχολικού βιβλίου...ε τότε ...σίγουρα ο μαθητής τα ''χάνει''. Να'σαι καλά Μιχαήλ για την όμορφη ιδέα που μοιράστηκες μαζί μας.
Σχόλιο από τον/την Βαγγέλης Κουντούρης στις 4 Νοέμβριος 2014 στις 12:26
Διαγραφή σχολίου
Καλή και “παλιάς φρουράς” περιοχή Μιχαήλ. Μπορούμε και απλούστερα να προσδιορίσουμε τα χαρακτηριστικά του ζητουμένου κύκλου (απόλυτα σωστά: της ζητουμένης σφαίρας) Αν x, Χ οι αποστάσεις από το δεξιά φορτίο των σημείων αριστερά και δεξιά του, όπου ο κύκλος (η σφαίρα) τέμνει την d ισχύουν: x/d-x=1/4 απ’ όπου προκύπτει x=4/5cm και Χ/d+X=1/4 απ’ όπου προκύπτει Χ=4/3cm 2ρ=x+X απ’ όπου προκύπτει ρ=16/15cm και επειδή x<ρ<Χ το κέντρο του κύκλου (σφαίρας) βρίσκεται σε σημείο δεξιά του δεξιά φορτίου και σε απόσταση d΄από αυτό: d΄=X-ρ απ’ όπου προκύπτει d΄=4/15cm (και επειδή το είδα: γράψε “οι ισοδυναμικές” στην τρίτη σειρά και μετέφερε δύο “κλικ” δεξιά το “ι” στο “ισοδυναιμκές” στη δεύτερη σειρά)
Σχόλιο από τον/την Μιχαήλ Μιχαήλ στις 4 Νοέμβριος 2014 στις 15:15
Διαγραφή σχολίου
Αθανάσιε, Διονύση, Γιάννη, Βαγγέλη σας ευχαριστώ για τα σχόλιά σας. Βαγγέλη οι διορθώσεις έγιναν. Σ' ευχαριστώ πολύ!
Χρυσή τομή και δυναμικό

Στα σημεία Α και Β που απέχουν μεταξύ τους απόσταση λ βρίσκονται αντίστοιχα τα ακίνητα ηλεκτρικά φορτία Q1>0 και Q2<0.

Αν ο λόγος των μέτρων των δυο ηλεκτρικών φορτίων ικανοποιεί το λόγο της χρυσής τομής  τότε και το σημείο Γ του ΑΒ για το οποίο είναι Vολ=0 ικανοποιεί επίσης το λόγο της χρυσής τομής.

 

 

Ο αριθμός 1,618… ονομάζεται λόγος της χρυσής τομής και συμβολίζεται διεθνώς με το γράμμα φ προς τιμή του γλύπτη Φειδία. Οι Αρχαίοι Έλληνες είχαν διαπιστώσει ότι, όπου εμφανίζεται ο λόγος της χρυσής τομής, δημιουργείται μια αίσθηση αρμονίας.

 

Το πρόβλημα διατυπώνεται ως εξής:

«Να χωριστεί ένα ευθύγραμμο τμήμα ΑΒ=λ σε δυο άνισα μέρη ΑΓ=x και ΓΒ=λ-x, ώστε ο λόγος ολόκληρου προς το μεγαλύτερο μέρος να είναι ίσος με το λόγο του μεγαλύτερου προς το υπόλοιπο τμήμα.

η συνέχεια εδώ

Σχόλιο από τον/την Κυριακόπουλος Γιάννης στις 23 Νοέμβριος 2014 στις 15:46
Διαγραφή σχολίου
Ωραίο θέμα. Ένα βήμα να κάνουμε Μιχάλη φτάνουμε σε Απολλώνιο περιφέρεια.
Σχόλιο από τον/την Μιχαήλ Μιχαήλ στις 23 Νοέμβριος 2014 στις 16:06
Διαγραφή σχολίου
Ναι Γιάννη το είχα κάνει εδώ!
Σχόλιο από τον/την Διονύσης Μάργαρης στις 23 Νοέμβριος 2014 στις 16:26
Διαγραφή σχολίου
Γεια σου Μιχαήλ. Ο αριθμός φ και η χρυσή τομή, είναι ένα θέμα που προκαλεί πάντα το ενδιαφέρον!
Σχόλιο από τον/την Κυριακόπουλος Γιάννης στις 23 Νοέμβριος 2014 στις 17:50
Διαγραφή σχολίου
Επίσης: ΕΔΩ ΕΔΩ
Σχόλιο από τον/την Εμμανουήλ Λαμπράκης στις 26 Νοέμβριος 2014 στις 11:36
Διαγραφή σχολίου
Μιχαήλ καλημέρα Πολύ καλά κάνεις και αναδεικνύεις το θέμα της χρυσής τομής εκμεταλλευόμενος ένα πρόβλημα φυσικής. Είναι αξιοσημείωτο επίσης ότι ο αριθμός φ προκύπτει και από την ακολουθία Fibonacci.

Φυσική Β΄ Λυκείου Θεωρία στο 2ο κεφάλαιο (Φυσική Γενικής παιδείας): Συνεχές ηλεκτρικό ρεύμα

2. Συνεχές ηλεκτρικό ρεύμα

 2.1. Ηλεκτρικές πηγές

 

1)Η ηλεκτρική πηγή είναι «πηγή» ηλεκτρικών φορτίων; Ποιος είναι ο ρόλος της στο κύκλωμα;

 

Ο ρόλος της ηλεκτρικής πηγής στο κύκλωμα δεν είναι να παράγει ηλεκτρικά φορτία.

Η πηγή, δεν είναι «πηγή» ηλεκτρικού φορτίου. Διατηρεί όμως το ηλεκτρικό πεδίο και τη διαφορά δυναμικού όπως μια αντλία νερού σε υδραυλικό κύκλωμα δεν παράγει νερό, αλλά χρησιμεύει για να διατηρεί κάποια διαφορά πιέσεων και να προκαλεί ροή του ήδη υπάρχοντος νερού. Αντίστοιχα:

Η  ηλεκτρική πηγή δημιουργεί διαφορά δυναμικού  και πεδίο μέσα στο οποίο γίνεται η ροή των ήδη υπαρχόντων ελεύθερων ηλεκτρονίων του μετάλλου.

η συνέχεια εδώ

και ένα προσαρμοστικό test στο ηλεκτρικό ρεύμα που δημιουργήσαμε με το συνάδελφο Μαθηματικό - Πληροφορικό Ευάγγελο Τριανταφύλλου.

εδώ

Σχόλια
Σχόλιο από τον/την Εμμανουήλ Λαμπράκης στις 1 Δεκέμβριος 2014 στις 7:02
Διαγραφή σχολίου
Μιχαήλ καλημέρα Εξαιρετική δουλειά όπως εξελίσσεται!
Σχόλιο από τον/την Μιχαήλ Μιχαήλ στις 1 Δεκέμβριος 2014 στις 20:29
Διαγραφή σχολίου
Μανώλη σ΄ευχαριστώ!
154. Ενέργεια και στάσιμο κύμα

Κατά μήκος χορδής      μήκους L=17,5cm και μάζας Μ=0,2 Kg, διαδίδονται

ταυτόχρονα δυο αρμονικά κύματα. Από τη συμβολή των δυο κυμάτων προκύπτει το στάσιμο κύμα y=0,02 συν(20πx)ημ(40πt) (S.I)  (t=0, x=0, y=0, v>0).

Να  σχεδιάσετε το στιγμιότυπο του κύματος τη χρονική στιγμή .

Τι είδους ενέργεια έχουν τα μόρια της χορδής εκείνη τη στιγμή; Να την υπολογίσετε. 

 

Δίνεται π2=10.

 

Συνοπτική λύση:

ή εδώ

Σχόλια
Σχόλιο από τον/την Διονύσης Μάργαρης στις 6 Δεκέμβριος 2014 στις 14:27
Διαγραφή σχολίου
Καλησπέρα Μιχαήλ Πολύ ωραία ιδέα. Μια ιδέα: Από τη στιγμή που υπολόγισες τη μέγιστη κινητική ενέργεια 0,32J, τόση είναι και η ολική ενέργεια των υλικών σημείων που αποτελούν τη χορδή.
Σχόλιο από τον/την Μιχαήλ Μιχαήλ στις 6 Δεκέμβριος 2014 στις 15:52
Διαγραφή σχολίου
Διονύση σε ευχαριστώ! (Πρόσθεσα και την ολική ενέργεια)
155. Οριζόντια βολή και κεκλιμένο επίπεδο

Μια σφαίρα μάζας m=1Kg και ακτίνας r=0,01 m, ρίχνεται οριζόντια από ύψος h=2,1 m από το σημείο Α του κεκλιμένου επιπέδου του σχήματος (γωνίας κλίσης φ=300), με αρχική ταχύτητα υ0=6m/s.

Αα) Να υπολογιστεί τότε  το μέτρο και η κατεύθυνση της στροφορμής της ως προς το σημείο Ο που βρίσκεται στη βάση του κεκλιμένου επιπέδου.

β) Ποιος είναι εκείνη τη στιγμή ο ρυθμός μεταβολής (μέτρο και κατεύθυνση) της στροφορμής της σφαίρας αν αυτή πραγματοποιεί οριζόντια βολή;

Β)α) Στη συνέχεια αφήνουμε τη σφαίρα να κινηθεί κατά μήκος του κεκλιμένου επιπέδου ξεκινώντας από την ηρεμία. Αν το πρώτο μισό του κεκλιμένου επιπέδου είναι τραχύ οπότε η σφαίρα κυλίεται χωρίς να ολισθαίνει ενώ το δεύτερο μισό του είναι λείο, τότε να βρείτε το μέτρο της μεταφορικής ταχύτητας της σφαίρας όταν φτάνει στη βάση του κεκλιμένου επιπέδου.

β) Πόση είναι η ιδιοστροφορμή και πόση η τροχιακή στροφορμή της σφαίρας ως προς το σημείο Ο, στη βάση του κεκλιμένου επιπέδου;

γ) Ποιος είναι ο ρυθμός μεταβολής της τροχιακής στροφορμής της σφαίρας ως προς το σημείο Ο όταν η σφαίρα βρίσκεται στο μέσο Μ του κεκλιμένου επιπέδου; (Δίνεται για τη σφαίρα Ιcm=I=mr2 , θεωρείστε ότι r<<H).

 

Συνοπτική λύση:

Σχόλια

Ετικέτες: στερεό, τάξη-γ

Σχόλιο από τον/την Γκενές Δημήτρης στις 13 Δεκέμβριος 2014 στις 0:24
Διαγραφή σχολίου
Μιχαήλ Καλησπέρα μου αρέσει ...Πλούσια άσκηση και η σειρά των ερωτημάτων καθοδηγεί τη λύση ... Πολύ καλά κάνεις που αναδεικνύεις την σημασία της εύρεσης της κατεύθυνσης της στροφορμής. Οι μαθητές διδάσκονται στροφορμή ως προς άξονα ( και όχι ως προς σημείο ) γιατί προσπαθούμε να περιοριστούμε σε κινήσεις επί επιπέδου και εθίζονται στην εύρεση μόνο του μέτρου ... Στο τελευταίο όμως ερώτημα δεν χρειάζεται τόσο δουλειά για την εύρεση της συνισταμένης των ροπών αφού αυτή είναι η ροπή της συνισταμένης mgημφ ...( ασφαλώς εννοείς αμέσως μετά το μέσον και αφού έχει περάσει στο λείο δάπεδο )
Σχόλιο από τον/την Παπαδάκης Κώστας στις 13 Δεκέμβριος 2014 στις 0:41
Διαγραφή σχολίου
Μιχαήλ την "κατέβασα". Καλή επαναληπτική άσκηση. Ευχαριστούμε.
Σχόλιο από τον/την Γκενές Δημήτρης στις 13 Δεκέμβριος 2014 στις 0:47
Διαγραφή σχολίου
Μιχαήλ και μια ερώτηση : γιατί απαιτείται r πολύ μικρότερο του Η ;
Σχόλιο από τον/την Μιχαήλ Μιχαήλ στις 13 Δεκέμβριος 2014 στις 15:05
Διαγραφή σχολίου
Δημήτρη και Κώστα καλησπέρα και σας ευχαριστώ! Δημήτρη για τον υπολογισμό της δυναμικής ενέργειας θεώρησα ότι Uτελ=0, οπότε θεώρησα την ακτίνα της σφαίρας πολύ μικρή!
Σχόλιο από τον/την Εμμανουήλ Λαμπράκης στις 14 Δεκέμβριος 2014 στις 12:49
Διαγραφή σχολίου
Μιχαήλ καλό μεσημέρι Εξαίρετο θέμα. Μου αρέσει πολύ. Συμφωνώ με το Δημήτρη που λέει Πολύ καλά κάνεις και αναδεικνύεις την σημασία της εύρεσης της κατεύθυνσης της στροφορμής
Σχόλιο από τον/την Μιχαήλ Μιχαήλ στις 14 Δεκέμβριος 2014 στις 14:41
Διαγραφή σχολίου
Μανώλη καλησπέρα και σε ευχαριστώ πολύ! Μανώλη είδα παρόμοιο θέμα στη Φυσική των Freedman Young και θεώρησα ότι είναι καλό να το μοιραστούμε, κυρίως λόγω της κατεύθυνσης της στροφορμής!
156. Ταχύτητα των σημείων της περιφέρειας τροχού

Ένας τροχός ακτίνας R κυλίεται χωρίς να ολισθαίνει πάνω σε οριζόντιο επίπεδο με σταθερή ταχύτητα υcm  (ισχύει υcm=ωR). Θεωρούμε ορθογώνιο σύστημα συντεταγμένων όπως φαίνεται στο σχήμα. Έστω ότι το κατώτερο σημείο Α του τροχού  τη χρονική στιγμή t=0 ταυτίζεται με την αρχή του συστήματος

 συντεταγμένων. Τότε:

Α) Να βρεθούν οι συντεταγμένες του σημείου Α στην περιφέρεια του τροχού σε συνάρτηση με το χρόνο.

Β) Να υπολογίσετε τις vx και vy συνιστώσες της ταχύτητας και τις αx και αy συνιστώσες της επιτάχυνσης του Α σε συνάρτηση με το χρόνο.

Γ) Ποιες χρονικές στιγμές το σημείο Α βρίσκεται σε ηρεμία; Ποιες είναι οι συνιστώσες της επιτάχυνσης τότε;

Δ) Υπολογίστε το μέτρο της συνολικής ταχύτητας και της συνολικής επιτάχυνσης του σημείου Α κάθε χρονική στιγμή. Εξαρτάται το μέτρο της επιτάχυνσης του Α από το χρόνο;

Ε) Θεωρείστε στιγμιαίο άξονα περιστροφής και υπολογίστε το μέτρο της ταχύτητας ενός σημείου της περιφέρειας του τροχού. Τι παρατηρείτε;

 

Συνοπτική λύση:

Σχόλια
Σχόλιο από τον/την Παπαδάκης Κώστας στις 20 Δεκέμβριος 2014 στις 21:48
Διαγραφή σχολίου
Μιχαήλ μπράβο, δυνατό θέμα.
Σχόλιο από τον/την Μιχαήλ Μιχαήλ στις 21 Δεκέμβριος 2014 στις 0:45
Διαγραφή σχολίου
Κώστα σ' ευχαριστώ
Σχόλιο από τον/την Φραγκιαδουλάκης Εμμανουήλ στις 22 Δεκέμβριος 2014 στις 18:14
Διαγραφή σχολίου
Πολύ καλό θέμα με φαντασία, όπως πάντα Μιχαήλ.
Σχόλιο από τον/την Μιχαήλ Μιχαήλ στις 22 Δεκέμβριος 2014 στις 20:18
Διαγραφή σχολίου
Μανώλη σ' ευχαριστώ! Να' σαι πάντα καλά
Σχόλιο από τον/την Κυριακόπουλος Γιάννης στις 22 Δεκέμβριος 2014 στις 21:29
Διαγραφή σχολίου
Μπράβο Μιχάλη. Δες εδώ. Υποκύπτω στον πειρασμό να πω ότι ένας αδρανειακός κινούμενος παρατηρητής βλέπει μόνο την (σταθερή) κεντρομόλο επιτάχυνση. Αυτή θα βλέπει και ο ακίνητος.
Σχόλιο από τον/την Φραγκιαδουλάκης Εμμανουήλ στις 22 Δεκέμβριος 2014 στις 23:16
Διαγραφή σχολίου
Δεν αφήνεις τίποτα να πέσει κάτω Γιάννη... Πως τα προλαβαίνει όλα!!! Συγχαρητήρια.
Σχόλιο από τον/την Μιχαήλ Μιχαήλ στις 22 Δεκέμβριος 2014 στις 23:19
Διαγραφή σχολίου
Γιάννη σ' ευχαριστώ! Πολύ κατατοπιστικό το ΙΡ. ....τι γίνεται αν ο τροχός επιταχύνεται ομαλά;
Σχόλιο από τον/την Κυριακόπουλος Γιάννης στις 22 Δεκέμβριος 2014 στις 23:37
Διαγραφή σχολίου
Με επιτάχυνση 1m/s^2. ΕΔΩ Πρόσεξε την μείωση της "περιόδου".
Σχόλιο από τον/την Μιχαήλ Μιχαήλ στις 23 Δεκέμβριος 2014 στις 14:46
Διαγραφή σχολίου
Ίδιο το μήκος της κυκλοειδούς καμπύλης (8R), αλλά διαγράφεται πιο γρήγορα!! Μπράβο!
157. Κύλινδρος και σημειακή μάζα m (II).

Στο εσωτερικό ενός κυλινδρικού κουτιού μάζας Μ=1 Kg και ακτίνας R=10 cm, έχουμε κολλήσει μια μικρή μεταλλική σφαίρα μάζας m=Kg. Το σύστημα τοποθετείται σε κεκλιμένο επίπεδο γωνίας κλίσης φ, έτσι ώστε η Κm να είναι παράλληλη στο κεκλιμένο επίπεδο. Τότε:

Να υπολογίσετε την αρχική γωνιακή επιτάχυνση τη στιγμή που αφήνουμε το σύστημα να κινηθεί πάνω στο κεκλιμένο επίπεδο.

Δίνονται: g=10m/s2, ημφ=0,6, συνφ=0,8 και ότι  η ροπή αδράνειας του κυλίνδρου ως προς άξονα που περνάει από το κέντρο μάζας του είναι Icm=MR2.

 

Συνοπτική λύση:

Σχόλια
Σχόλιο από τον/την Δημήτρης Τσάτσης στις 2 Ιανουάριος 2015 στις 22:08
Διαγραφή σχολίου
Μιχάλη καλησπέρα. καλή χρονιά να έχεις. Πολύ μου άρεσε ο τρόπος (οι τρόποι) που αντιμετώπισες την άσκηση. Πληρέστατος. Ομολογώ πως άνοιξα τη δημοσίευση με μεγάλη περιέργεια για το πως θα επίλυες. Αξίζει να τη δουν και μαθητές που θέλουν να εμβαθύνουν (όπως ακριβώς την έχεις ώστε να δουν και τον λανθασμένο τρόπο). Ευχαριστούμε και συγχαρητήρια.
Σχόλιο από τον/την Κυριακόπουλος Γιάννης στις 2 Ιανουάριος 2015 στις 22:36
Διαγραφή σχολίου
Πολύ όμορφη.
Σχόλιο από τον/την Μιχαήλ Μιχαήλ στις 3 Ιανουάριος 2015 στις 0:18
Διαγραφή σχολίου
Δημήτρη και Γιάννη σας ευχαριστώ!
Σχόλιο από τον/την Παπαδάκης Κώστας στις 3 Ιανουάριος 2015 στις 0:57
Διαγραφή σχολίου
Ωραίο Μιχαήλ. Μου αρέσει που γράφεις ότι οι λύσεις σου είναι συνοπτικές, και όμως, είναι αναλυτικές !.
Σχόλιο από τον/την Διονύσης Μάργαρης στις 3 Ιανουάριος 2015 στις 8:55
Διαγραφή σχολίου
Καλημέρα και καλή χρονιά Μιχαήλ. Μια πολύ όμορφη (και πολύ δύσκολη προφανώς περίπτωση) την οποία μελετάς αναδεικνύοντας ένα μεγάλο κίνδυνο που υποβόσκει στην επίλυση αντίστοιχου προβλήματος. Τα σχόλια - συμπεράσματα στο τέλος, είναι "όλα τα λεφτά"!!
Σχόλιο από τον/την Κορκίζογλου Πρόδρομος στις 3 Ιανουάριος 2015 στις 10:43
Διαγραφή σχολίου
Μιχάλη έδωσες ''τα ρέστα σου'' ιδιαίτερα με τα σχόλιά σου για το λάθος τρόπο!! Συγχαρητήρια!!
Σχόλιο από τον/την Μιχαήλ Μιχαήλ στις 3 Ιανουάριος 2015 στις 11:12
Διαγραφή σχολίου
Κώστα, καλημέρα και σ' ευχαριστώ! Διονύση, Πρόδρομε καλημέρα και σας ευχαριστώ! Έχει ξαναγίνει συζήτηση για το συγκεκριμένο θέμα με αφορμή ανάρτηση του Αναγνώστου και με τη συμβολή του Διονύση (Μητρ), τους οποίους επίσης ευχαριστώ!
Σχόλιο από τον/την Κωστας Ψυλακος στις 3 Ιανουάριος 2015 στις 20:06
Διαγραφή σχολίου
Μιχαλη καλη χρονια ! Μπηκες αρκετα δυναμικα με πολυ ωραιο θεμα και αναλυση εξαιρετικη ! Ειχε γινει μια μεγαλη συζητηση θυμαμαι με ενα τετοιο θεμα μιας και λυνονταν με τον τριτο τροπο δηλ τον λανθασμενο. Εξαιρετικο λοιπον !!!
Σχόλιο από τον/την Μιχαήλ Μιχαήλ στις 4 Ιανουάριος 2015 στις 18:51
Διαγραφή σχολίου
Κώστα, σ΄ευχαριστώ πολύ! Σου εύχομαι και εγώ Καλή Χρονιά με Υγεία!
Σχόλιο από τον/την Βασίλης Δουκατζής στις 4 Ιανουάριος 2015 στις 21:50
Διαγραφή σχολίου
Μιχάλη πολύ ωραία: Δεν έχω κάτι επιπλέον να πω με "κάλυψαν" οι παραπάνω!!!!!!
Σχόλιο από τον/την Μιχαήλ Μιχαήλ στις 5 Ιανουάριος 2015 στις 10:37
Διαγραφή σχολίου
Βασίλη καλημέρα και σ' ευχαριστώ! Να΄σα πάντα καλά!
Σχόλιο από τον/την Διονύσης Μάργαρης στις 5 Ιανουάριος 2015 στις 18:33
Διαγραφή σχολίου
Καλή χρονιά Δημήτρη. Πάνε 2-3 μέρες τώρα που προσπαθούσα να θυμηθώ πού είχαμε συζητήσει το θέμα αυτό και δεν τα κατάφερνα. Σε ευχαριστώ που μας θύμισες την ανάρτησή σου, που στάθηκε αφορμή για μια από τις πιο ουσιαστικές συζητήσεις, που έχουμε κάνει στο δίκτυο.
Σχόλιο από τον/την Φραγκιαδουλάκης Εμμανουήλ στις 6 Ιανουάριος 2015 στις 17:07
Διαγραφή σχολίου
Καλησπέρα Μιχάλη, Χρόνια πολλά και καλή χρονιά. Όπως συνήθως μας παρουσίασες μια προσεγμένη δουλειά. Ιδιαίτερα μου άρεσε η λάθος λύση κάτι που με ιντρίγκαρε από παλιά. Όταν κάποτε έκανα μάθημα στο ΠΕΚ Κρήτης και είχα αντικείμενο τα σφάλματα, είχα εντάξει και μια ενότητα για προβληματισμό, που αναφερότανε στην τεράστια διδακτική αξία της εκμετάλλευσης του λάθους και πίστεψέ με ήταν το κομμάτι με το μεγαλύτερο ενδιαφέρον.
Σχόλιο από τον/την Μιχαήλ Μιχαήλ στις 7 Ιανουάριος 2015 στις 9:43
Διαγραφή σχολίου
Μανώλη καλημέρα και Χρόνια Πολλά. Σ' ευχαριστώ για το σχόλιό σου. Μανώλη, πράγματι έχεις δίκιο για την τεράστια διδακτική αξία που έχει η προσπάθεια για την ερμηνεία του λάθους. Πόσες φορές εδώ στο ylikonet δεν έχουμε συζητήσει με τεράστιο ενδιαφέρον τέτοιες περιπτώσεις. Σ' ευχαριστώ πολύ και να σαι πάντα καλά!
Παράγραφος 1.2.8 Φυσικής Α΄ Λυκείου

...επειδή η παράγραφος 1.2.8 είναι στην ύλη ας δούμε και τον πίνακα της δραστηριότητας του σχολικού βιβλίου

Ο πίνακας εδώ

Σχόλια
Σχόλιο από τον/την Βαγγέλης Κουντούρης στις 7 Ιανουάριος 2015 στις 11:33
Διαγραφή σχολίου
καλημέρα Μιχαήλ Δεν αντιλαμβάνομαι σε τι εστιάζεις. Το θέμα είναι πειραματικό και χρειάζονται προσεγγίσεις το δε σχολικό βιβλίο αφήνει ερωτηματικά.
Σχόλιο από τον/την Φραγκιαδουλάκης Εμμανουήλ στις 7 Ιανουάριος 2015 στις 11:39
Διαγραφή σχολίου
Δεν το πιστεύω!!! Ορίζει τη μέση ταχύτητα από τη μετατόπιση!!! Δεν έχω το βιβλίο μπροστά μου, αλλά δεν θεωρώ και απίθανο και στη θεωρία να γράφει τέτοιες ανοησίες... Επίσης για τη μέση επιτάχυνση πού ορίζεται σαν μέσος όρος επιταχύνσεων; Με την ίδια λογική θα υπολόγιζε δηλαδή και τη μέση ταχύτητα για το σύνολο της διαδρομής;!!! Μπράβο Μιχάλη που το παρατήρησες.
Σχόλιο από τον/την Μιχαήλ Μιχαήλ στις 7 Ιανουάριος 2015 στις 12:00
Διαγραφή σχολίου
Βαγγέλη καλημέρα. H δραστηριότητα όπως την προτείνει το σχολικό βιβλίο βρίσκεται εδώ Μανώλη καλημέρα! Πραγματικά δεν ξέρω αν πρέπει να αλλάξουμε κάτι! Περιμένω παρατηρήσεις!!
Σχόλιο από τον/την Παπαδάκης Κώστας στις 7 Ιανουάριος 2015 στις 13:13
Διαγραφή σχολίου
Ενδιαφέρον, θα σας παρακολουθώ στα σχόλια σας.
Σχόλιο από τον/την Διονύσης Μάργαρης στις 7 Ιανουάριος 2015 στις 13:25
Διαγραφή σχολίου
Καλησπέρα συνάδελφοι. Νομίζω ότι η πειραματική επεξεργασία των μετρήσεων μιας πτώσης, αυτήν την πορεία θα ακολουθήσει. Αν έχουμε μια χρονοφωτογραφία, τι άλλο μπορούμε να κάνουμε, από το να σημειώσουμε τις διαφορετικές θέσεις, στις διάφορες χρονικές στιγμές και στη συνέχεια, να βρούμε τις πολυ μικρές μετατοπίσεις, στα πολύ μικρά χρονικά διαστήματα; Αλλά τότε αν διαιρέσουμε τα Δχ/Δt, θα βρούμε την ταχύτητα! Ποια ταχύτητα; Μα, προφανώς τη μέση. Αλλά, πείραμα κάνουμε, αυτή την μέση ταχύτητα τη βαφτίζουμε στιγμιαία, στο μέσον του αντίστοιχου χρονικού διαστήματος. Έτσι συμπληρώνουμε τιμές ταχύτητας. Με την ίδια λογική βρίσκουμε αντίστοιχες μεταβολές ταχύτητας και μέση επιτάχυνση, την οποία ξαναβαφτίζουμε στιγμιαία επιτάχυνση, στο μέσον του αντίστοιχου χρονικού διαστήματος. Και αν αυτή η επιτάχυνση παίρνει διάφορες τιμές, μεταξύ 9,76 και 9,94, στο τέλος δεν θα μιλήσουμε για τη μέση τιμή του μέτρου της επιτάχυνσης; Την περασμένη χρονιά είχα αναρτήσει μια ανάλογη επεξεργασία.Εδώ.
Σχόλιο από τον/την Σαράμπαλης Κωνσταντίνος στις 7 Ιανουάριος 2015 στις 13:49
Διαγραφή σχολίου
Συνάδελφοι συμφωνώντας απολύτως με το Διονύση αναρτώ τον τρόπο που το διδάσκω εδώ
Σχόλιο από τον/την Γκενές Δημήτρης στις 7 Ιανουάριος 2015 στις 15:11
Διαγραφή σχολίου
Δεν νομίζω ότι υπάρχει πιο πρόσφορη μέθοδος. Η μέθοδος που είχαμε δοκιμάσει το 94-95 στο PSSC στο οποίο δίναμε ιδιαίτερο βάρος στην θεωρητική επεξεργασία των κλίσεων στα διαγράμματα x-t και v-t ( ουσιαστικά γεωμετρική απεικόνιση παραγώγων ) δεν φαίνεται να απέδιδε καλύτερα αποτελέσματα Ίσως μέσα στα πλαίσια άλλης δομής όλου του προγράμματος να είχε καλύτερα αποτελέσματα ... Πάντως τότε μέναμε σε ασκήσεις του τύπου που παρακάτω δίνω και φαίνεται ότι δεν είχε τα αποτελέσματα εξοικείωσης με τη φυσική σημασία των μεγεθών που προσδοκούσαμε ... Αυτό βέβαια είναι μια μεμονωμένη προσωπική εμπειρία και όχι το γενικό καταστάλαγμα του όλου τότε πειραματισμού.
Σχόλιο από τον/την Κυριακόπουλος Γιάννης στις 7 Ιανουάριος 2015 στις 16:22
Διαγραφή σχολίου
Μια σκέψη:
Σχόλιο από τον/την Κυριακόπουλος Γιάννης στις 7 Ιανουάριος 2015 στις 16:39
Διαγραφή σχολίου
Δουλεύοντας με 1 ως 5 (Δx1) και 6 ως 10 (Δx2) βγαίνει 9,78 m/s^2. Αυτό σημαίνει ότι η εκτίμηση 9,75 m/s^2 είναι πολύ καλή.
Σχόλιο από τον/την Φραγκιαδουλάκης Εμμανουήλ στις 7 Ιανουάριος 2015 στις 17:36
Διαγραφή σχολίου
Η παρατήρησή μου Μιχάλη αναφέρεται στον ορισμό της μέσης ταχύτητας που γράφει πάνω ότι είναι η μετατόπιση προς το χρόνο. Δεν έχω επαφή με το βιβλίο και δεν κατάλαβα ότι πρόκειται για εργαστηριακή άσκηση. Επειδή όμως εδώ η αριθμητική τιμή της μετατόπισης συμπίπτει με αυτήν του διαστήματος δεν σημαίνει ότι με νομιμοποιεί να τη χρησιμοποιήσω έτσι χωρίς να τη διορθώσω. Το ότι όμως είναι εργαστηριακή άσκηση δεν σημαίνει ότι κάνουμε εκπτώσεις στους ορισμούς. Είναι έντονη δε η ένστασή μου στον τρόπο υπολογισμού της μέσης επιτάχυνσης, αφού ο μέσος όρος επιταχύνσεων δεν έχει νόημα, όπως δεν έχει και ο μέσος όρος των ταχυτήτων. Βεβαίως Διονύση την πειραματική πορεία αυτή που γράφεις θα ακολουθήσουμε, αλλά θα αντικαταστήσουμε πάνω τη λέξη μετατόπιση με τη λέξη διάστημα, και αντί για μέση επιτάχυνση θα γράψουμε μέσος όρος επιτάχυνσης, αφού εξηγήσουμε στους μαθητές ότι αυτό το μέγεθος δεν έχει φυσική σημασία.
Σχόλιο από τον/την Βαγγέλης Κουντούρης στις 7 Ιανουάριος 2015 στις 18:09
Διαγραφή σχολίου
Α, είναι η χρονοφωτογραφία σε ελεύθερη πτώση (και όχι η πειραματική με τον ηλεκτρικό καταναλωτή, όπου και η “σαλάτα”...) Βασικά συμφωνώ με τον Διονύση, διότι όλο το μυστικό είναι η αντιστοιχία των τιμών στο μέσο του αντίστοιχου χρονικού διαστήματος Γιάννη η προσέγγιση που κάνεις προϋποθέτει ότι η κίνηση είναι ομαλά επιταχυνόμενη που δεν το ξέρεις από πριν Μανώλη μπορεί να “σωθούν” οι μέσοι όροι επειδή τα χρονικά διαστήματα είναι ίσα
Σχόλιο από τον/την Διονύσης Μάργαρης στις 7 Ιανουάριος 2015 στις 19:09
Διαγραφή σχολίου
Καλησπέρα Μανώλη. Να ξεκαθαρίσουμε αρχικά τι ονομάζουμε μέση ταχύτητα. Υπάρχει η αριθμητική μέση ταχύτητα, αυτή που οι Αγγλοσάξωνες λένε speed. Αυτή που μετράει το ταχύμετρο του αυτοκινήτου. Υπάρχει και η μέση διανυσματική ταχύτητα που ορίζεται ως το πηλίκο Δx/Δt, όπου όταν πάρουμε το όριο Δt να τείνει στο μηδέν, οδηγεί στην στιγμιαία ταχύτητα. Εδώ μιλάμε για την διανυσματική μέση ταχύτητα και όχι για την αριθμητική. Είναι η μέση ταχύτητα στο χρονικό διάστημα Δt, την οποία θεωρούμε περίπου ίση με τη στιγμιαία ταχύτητα, στο μέσον του συγκεκριμένου χρονικού διαστήματος. Γίνεται σφάλμα; Προφανώς είναι μια προσέγγιση, που όμως στα πλαίσια της πειραματικής διαδικασίας νομίζω μας επιτρέπεται. Ας το δούμε με ένα αριθμητικό παράδειγμα. Ένα σώμα κινείται και στον παρακάτω πίνακα δίνεται η θέση του κάποιες στιγμές.
t(s) 1 1,4 1,8 2,2 2,6 3
x(cm) 13 29 78 160 270 415
Στο πρώτο χρονικό διάστημα (1s-1,4s) η μέση ταχύτητα είναι υ=Δx/Δt=(0,29-0,13)m/0,4s =0,4m/s την οποία θεωρούμε ταχύτητα τη χρονική στιγμή 1,2s. Με ίδια λογική μπορούμε να συμπληρώσουμε τον παρακάτω πίνακα.
t(s) 1,2 1,6 2,0 2,4 2,8
υ(m/s) 0,40 1,26 2,05 2,75 3,62
Ας προσέξουμε ότι έχουμε βάλει άλλες χρονικές στιγμές στον πρώτο πίνακα και άλλες στον 2ο . Ακολουθούμε τώρα την ίδια λογική για την επιτάχυνση: Στο πρώτο χρονικό διάστημα (1,2s-1,6s) η μέση επιτάχυνση είναι α=Δυ/Δt=(1,26-0,40)/0,4 m/s2= =2,86 m/s2 την οποία θεωρούμε ως στιγμιαία επιτάχυνση τη χρονική στιγμή 1,4s (το μέσον του χρονικού διαστήματος (1,2s-1,6s). Με ίδια λογική μπορούμε να συμπληρώσουμε τον παρακάτω πίνακα.
t(s) 1,4 1,8 2,2 2,6
α(m/s2 ) 2,15 1,98 1,75 2,18
Από εκεί και πέρα μπορούμε να δουλέψουμε με διαγράμματα και κλίσεις. Αλλά αν θέλαμε να δουλέψουμε χωρίς αυτά, μπορούμε να εκτιμήσουμε ότι το σώμα αυτό κινήθηκε με μια σταθερή επιτάχυνση. Πόση; Η μέση τιμή των παραπάνω επιταχύνσεων, η οποία θα είναι α=(2,15+1,98+1,75+2,18)/4=2,02m/s2.        
Σχόλιο από τον/την Βαγγέλης Κουντούρης στις 7 Ιανουάριος 2015 στις 19:17
Διαγραφή σχολίου
Σωστά Διονύση (ειδικά εδώ όπου η φορά κίνησης δεν αλλάζει οι δύο μέσες ταχύτητες έχουν ίσες τιμές πειραματικά το σφάλμα μειώνεται αν Δt όσο μπορούμε μικρότερο)
Σχόλιο από τον/την Διονύσης Μάργαρης στις 7 Ιανουάριος 2015 στις 19:22
Διαγραφή σχολίου
Καλησπέρα Βαγγέλη. Ακριβώς. Όσο μικραίνει το χρονικό διάστημα, τόσο πλησιάζουμε την στιγμιαία ταχύτητα. Μήπως αυτό δεν είναι το νόημα του ορίου;
Σχόλιο από τον/την Φραγκιαδουλάκης Εμμανουήλ στις 8 Ιανουάριος 2015 στις 15:31
Διαγραφή σχολίου
Νομίζω Διονύση ότι όσοι μας διαβάζουν μπερδεύονται περισσότερο με αυτά που γράφομε. Έκανα δυο σαφέστατες παρατηρήσεις και εισέπραξα κείμενο, που δεν ξεκαθαρίζει αν έχω λάθος, σε αυτόν που μας διαβάζει. Και κάτι άλλο. Όταν λέμε μέση ταχύτητα υπάρχει περίπτωση να εννοούμε μέση διανυσματική ταχύτητα; Που αναφέρεται αυτό στα διδακτικά εγχειρίδια; Γιατί λοιπόν στην συγκεκριμένη άσκηση να εννοούμε ότι έχουμε μέση διανυσματική ταχύτητα; Σε κάθε πειραματική άσκηση επί των κινήσεων όπου μετρούμε διαστήματα χωρικά και χρονικά και συμπληρώνουμε πίνακες όπως τον δοσμένο, οι ταχύτητες που υπολογίζουμε στην πειραματική προσέγγιση είναι στιγμιαίες ταχύτητες. Έχουμε το δικαίωμα να επιλέγουμε τον χρόνο dt όσο θέλουμε αφού είναι καθαρά θέμα ακρίβειας υπολογισμού. Έτσι μπορούμε να τις αποτυπώσουμε στο χαρτί και να εξάγουμε τα ανάλογα συμπεράσματα για διαστήματα και στιγμιαίες (βεβαίως) επιταχύνσεις. Ποτέ για μέσο όρο επιταχύνσεων.
Σχόλιο από τον/την Διονύσης Μάργαρης στις 8 Ιανουάριος 2015 στις 16:43
Διαγραφή σχολίου
Καλησπέρα Μανώλη. Στο σχολικό βιβλίο γράφει: Συνεπώς υπάρχει η μέση διανυσματική ταχύτητα, που οι συγγραφείς επιλέγουν να την υποβαθμίσουν. Αλλά στην επόμενη σελίδα γράφουν: Συνεπώς μικραίνουν τα χρονικά διαστήματα για να προσεγγίσουν μέσω ενός μονόμετρου μεγέθους (υμ) ένα διανυσματικό που είναι η στιγμιαία ταχύτητα!!! Στην θέση και αντιμετώπιση αυτή, έχω προσωπικά διαφωνήσει εδώ και χρόνια με αρκετές αναρτήσεις οι οποίες προκάλεσαν και συζητήσεις. Ρίξε μια ματιά. Μέση ταχύτητα. Μέση ταχύτητα. Διάνυσμα ή μονόμετρο μέγεθος; Μέση ταχύτητα. Η συζήτηση συνεχίζεται....   Παραπέρα όμως δεν καταλαβαίνω, πώς προκύπτει ότι αυτό που μετράμε πειραματικά, όπως αναφέρεις παραπάνω, είναι στιγμιαία ταχύτητα. Λες: " Έχουμε το δικαίωμα να επιλέγουμε τον χρόνο dt όσο θέλουμε αφού είναι καθαρά θέμα ακρίβειας υπολογισμού." Πειραματικά δεν μπορούμε Μανώλη, να επιλέξουμε χρονικά διαστήματα dt και να μετρήσουμε στιγμιαία ταχύτητα. Το όριο είναι μαθηματική έννοια και ένα πείραμα όπως το παραπάνω, επιτρέπει τη μέτρηση μέσης ταχύτητας σε ένα πεπερασμένο χρονικό διάστημα και όχι στιγμιαία. Είναι άλλο πράγμα τώρα, αν την τιμή αυτής της μέσης ταχύτητας την θεωρούμε στιγμιαία ταχύτητα, στο μέσο του αντίστοιχου χρονικού διαστήματος. Και αυτό προσπάθησα να δείξω παραπάνω.    
Σχόλιο από τον/την Διονύσης Μάργαρης στις 8 Ιανουάριος 2015 στις 17:15
Διαγραφή σχολίου
Ψάχνοντας στο ebooks, για να βρω τα παραπάνω αποσπάσματα, συνειδητοποιώ ότι το ηλεκτρικό ρεύμα βρίσκεται ακόμη αναρτημένο στην Α΄τάξη..., όπως επίσης ότι στη Β΄Γ.Π, υπάρχουν η οριζόντια βολή, κυκλική. ορμή. Άμεση ενημέρωση:-)
Σχόλιο από τον/την Φραγκιαδουλάκης Εμμανουήλ στις 8 Ιανουάριος 2015 στις 19:34
Διαγραφή σχολίου
Τώρα κατάλαβα Διονύση με τις παραπομπές που έδωσες που διαφωνούμε. Δεν είχα υπόψη μου τη θέση σου στον ορισμό της μέσης ταχύτητας που πραγματικά με ξάφνιασε. Από τότε που θυμούμαι τον εαυτό μου διαβάζοντας Φυσική τη μέση ταχύτητα την ξέρω σαν το μονόμετρο μέγεθος που δίδεται από το συνολικό διάστημα προς το συνολικό χρόνο και φυσικά όχι από το μέτρο του διανύσματος μετατόπισης προς το συνολικό χρόνο. Θεωρώ ότι ορισμός της μέσης ταχύτητας όπως την ξέρω έχει φυσική σημασία, αφού σου δίδει χονδρικά το ρυθμό που κινήθηκε το κινητό, σε αντίθεση με την ταχύτητα όπως την ορίζεις εσύ, που δεν σου δίδει καμιά πληροφορία σε σχέση με το ρυθμό κίνησης. Έχω συναντήσει πάμπολλες ασκήσεις με αυτό το ζητούμενο και σε όλες τις περιπτώσεις οι απαντήσεις δίδονται σύμφωνα με τον ορισμό που δίδω.Είναι πράγματι η πρώτη φορά που είδα λυμένη άσκηση όπως αυτή της πρώτης σου παραπομπής που απαντάς με βάση το μέτρο του διανύσματος μετατόπισης. Δεν αμφισβήτησα το γεγονός ότι η ταχύτητα είναι διανυσματικό μέγεθος και δεν κατάλαβα γιατί πρέπει οπωσδήποτε να την ορίσω μέσα από τη μέση ταχύτητα όπως το θέτεις. Η ταχύτητα, ας το πούμε η κανονική, έχει ένα και μοναδικό ορισμό, και δεν ορίζεται μέσα από μια άλλη ταχύτητα. Βάζουμε το σύστημα συντεταγμένων. Ορίζουμε τη θέση του κινητού με το διάνυσμα θέσης του. Ορίζουμε μια επόμενη θέση του με το νέο διάνυσμα θέσης. Βρίσκουμε τη διαφορά των δυο διανυσμάτων σαν ένα τρίτο διάνυσμα που θα μας καθορίσει τη διεύθυνση και τη φορά της ταχύτητας και σαν μέτρο την οριακή τιμή του μέτρου αυτού του διανύσματος στο χρόνο dt. Γιατί να χρησιμοποιήσω άλλο ορισμό; Όσον αφορά την πειραματική διαδικασία και ότι δεν μπορούμε να μετρήσουμε στιγμιαία έχω να πω τα εξής. Για να βρούμε σε μια καμπύλη τροχιά την ταχύτητα παίρνοντας τα δύο διανύσματα θέσης τα πλησιάζουμε σταδιακά τείνοντας να συμπέσουν το ένα προς το άλλο και βλέπουμε το διάνυσμα της διαφοράς των να πλησιάζει προς την εφαπτόμενη που θα μας δώσει τη διεύθυνση της ταχύτητας. Η γωνιακή τους απόσταση που εξαρτάται από το χρόνο μετακίνησης μπορεί να μικραίνει και όσο μικρότερη τόσο προσεγγιστικότερο το αποτέλεσμα. Κάπου όμως σταματούμε, θα μπορούσαμε να πάμε ακόμα κοντύτερα και κοντύτερα, όμως η προσέγγισή μας μας αρκεί κάπου μακρύτερα για να αντιληφθούμε καθαρά τη διεύθυνση. Το ίδιο πράγμα εννοώ λοιπόν ότι κάνουμε και στην πειραματική διαδικασία. Ασφαλώς και δεν έχουμε βρει την στιγμιαία ταχύτητα με απόλυτη ακρίβεια, είμαστε όμως πολύ κοντά, ώστε να θεωρούμε ότι δεν έχουμε κάνει λάθος. Στο επόμενο άλλωστε βήμα έτσι θα την αποτυπώσουμε στο χαρτί για να προχωρήσουμε στους υπολογισμούς... Τελικά Διονύση πιστεύω ότι συζητούμε εμείς τώρα εδώ για κάτι που έπρεπε να έχει ξεκαθαριστεί τελεσίδικα από το σχολικό βιβλίο. Ωστόσο και η βιβλιογραφία δεν βοηθά όσο θα έπρεπε προς αυτήν την κατεύθυνση αφού όντως υπάρχουν αναφορές σαν αυτές που γράφεις και στην προηγούμενη συζήτησή σου.
Σχόλιο από τον/την Διονύσης Μάργαρης στις 8 Ιανουάριος 2015 στις 22:10
Διαγραφή σχολίου
Καλησπέρα Μανώλη. Το θέμα το έχουμε συζητήσει και εδώ στο δίκτυο στο παρελθόν. Δες κάποιες συζητήσεις:

δύο ερωτήματα και η μέση ταχύτητα

Μέση ταχύτητα

Μέση διανυσματική και αριθμητική ταχύτητα

 
Σχόλιο από τον/την Διονύσης Μάργαρης στις 8 Ιανουάριος 2015 στις 22:33
Διαγραφή σχολίου
Αλλά νομίζω ότι αξίζει να δούμε, τι γράφει το βιβλίο Δαπόντε-Κασσέτα, που πριν λίγα χρόνια διδάσκαμε στο σχολείο: Το βιβλίο ήταν αυτό: Και στην κίνηση γράφει:
Σχόλιο από τον/την Φραγκιαδουλάκης Εμμανουήλ στις 9 Ιανουάριος 2015 στις 10:40
Διαγραφή σχολίου
Στην τρίτη παραπομπή που κάνεις Διονύση παραπάνω, είναι αναρτημένη μια εργασία του Θοδωρή Παπασγουρίδη με την οποία ταυτίζομαι απολύτως. Αυτή είναι η καθαρή άποψή μου, όπως επίσης συμφωνώ απολύτως και στην προηγούμενη παραπομπή, στο σχόλιο του Αντωνίου. Για να καταλάβω επί τέλους εσύ διαφωνείς με τις λύσεις αυτές; Επίσης διαφωνείς με τη λύση που δίδεται από τον Αναγνώστου στο θέμα του κινητού που πηγαίνει με μια ταχύτητα και επιστρέφει στο ίδιο σημείο με άλλη; Συμφωνούμε ότι στη βιβλιογραφία όντως υπάρχει Βαβυλωνία... στο σχετικό θέμα; Συμφωνούμε στην αναγκαιότητα στο σχολικό βιβλίο να ξεκαθαριστεί (είναι απλούστατο θέμα ορισμού) το ζήτημα; Μου είναι πράγματι ανιαρή μια συζήτηση όταν ο καθένας θεωρεί τις δικές του αφετηρίες αυθαίρετα και δεν μπορούμε να συνεννοηθούμε. Είναι σαν αυτό, που επίσης υπάρχει διχογνωμία, αν πρέπει να μιλούμε για αρχική φάση σε ένα κύμα. Αν θέλω να την ορίσω τότε σαφώς και υπάρχει που είναι το πρόβλημα;Το ζήτημα είναι τι ορίζει το ίδιο το διδακτικό βιβλίο. Θεωρώ λοιπόν ότι ο διάλογος αυτός δεν μπορεί να συνεχιστεί έτσι με κείμενα και παραπομπές σε βιβλία που είναι γνωστά και σχεδόν σε όλα υπάρχουν και εσωτερικές αντιφάσεις.
Σχόλιο από τον/την Διονύσης Μάργαρης στις 9 Ιανουάριος 2015 στις 11:13
Διαγραφή σχολίου
Καλημέρα Μανώλη. Προσωπικά υποστήριξα από την αρχή, μια παλιά μου θέση, ότι πρέπει να διδάσκουμε στο σχολείο τη μέση διανυσματική ταχύτητα. Νόμισα ότι εσύ υποστήριξες ότι τέτοιο μέγεθος στα σχολικά βιβλία δεν υπάρχει. Για το λόγο αυτό, έβαλα μια εικόνα από το σημερινό βιβλίο που την ορίζει (αν και δεν την χρησιμοποιεί) και μια από το προηγούμενο σχολικό βιβλίο του Κασσέτα-Δαπόντε που και την όριζε και την χρησιμοποιούσε. Αν έκανα λάθος στην παραπάνω εκτίμηση της θέσης σου, πες ότι δεν έβαλα τις παραπομπές. Αν συμφωνείς με την ανάρτηση του Θοδωρή, δεν υπάρχει λόγος παραπέρα διευκρινήσεων. Συμφωνώ και εγώ! Όσον αφορά τη βιβλιογραφία, απλά υπάρχουν και οι δύο εκδοχές. άλλοι επιμένουν στη μία και άλλοι στην άλλη. Όταν εγώ υποστηρίζω να διδάξουμε τη μέση διανυσματική ταχύτητα, προφανώς δεν εννοώ ότι δεν μπορεί να διδαχτεί η αριθμητική! Κάνω μια επιλογή και μια πρόταση. Δεν λέω ότι δεν υπάρχει αντίθετη άποψη.
Σχόλιο από τον/την Φραγκιαδουλάκης Εμμανουήλ στις 9 Ιανουάριος 2015 στις 11:15
Διαγραφή σχολίου
Χαίρομαι που συνεννοηθήκαμε επί τέλους Διονύση.
Σχόλιο από τον/την Παπαδάκης Κώστας στις 9 Ιανουάριος 2015 στις 11:29
Διαγραφή σχολίου
Θεωρώ το θέμα που ανέδειξε ο Μιχαήλ, εξαιρετικά χρήσιμο (Μπράβο του). Παρακολουθώ την συζήτηση και δίνω και την δική μου άποψη : Διδακτικά εμφανίζεται στη μέση ταχύτητα ένα πρόβλημα που ξεκινάει, όπως σωστά είπατε, από το διάστημα και την μετατόπιση. Εγώ θεωρώ ότι οι ευθύγραμμες κινήσεις αν και είναι απλούστερες, σε αυτό το σημείο δεν βοηθούν, το πρώτο παράδειγμα που δίνω έχει να κάνει με ένα σώμα που κινείται στο επίπεδο. Ένα σώμα που ακολουθεί τις κορυφές ενός τετραγώνου ΑΒΓΔΑ. Υπολογίζω διαδοχικά το Δχ και S στο ΑΒ, ΑΒΓ, ΑΒΓΔ, ΑΔΓΔΑ. Το αποτέλεσμα είναι άμεσο. Η μέση ταχύτητα τώρα είναι μια έννοια που αποσυντονίζει αμέσως τον μαθητή, γιατί συνδέει άμεσα το πόσο γρήγορα με την μέση τιμή, δηλαδή στο παράδειγμα : ένα αμάξι πάει στην απόσταση Ηράκλειο - Ρέθυμνο σε μία ώρα, η απόσταση είναι 76 Κm, με ποια ταχύτητα κινήθηκε ; παίρνω αμέσως την μέση αριθμητική ταχύτητα σαν απάντηση. Τα παιδιά ξέρουν από την πρώτη γυμνασίου να υπολογίζουν τους μέσους όρους των βαθμών τους. Εγώ προσπαθώ να κάνω κατανοητή την ταχύτητα, τον διανυσματικό της χαρακτήρα και την στιγμιαία της τιμή και ένα απλό παράδειγμα, κατευθύνει τα παιδιά διαφορετικά. Προσπαθώ να το σώσω με την συζήτηση : και πήγαινε διαρκώς με την ίδια ταχύτητα, στις ευθείες, στις (επικίνδυνες όντως για αυτό το δρόμο) στροφές; βοηθάει πολύ. Εκεί ακολουθώ την άποψη του Διονύση, ορίζω την μέση διανυσματική ταχύτητα, τονίζω τον διανυσματικό χαρακτήρα της ταχύτητας ξανά. Το αποτέλεσμα είναι καλό. Τα παιδιά αποκτούν μια αρχική εντύπωση ότι η μέση ταχύτητα είναι διανυσματικό μέγεθος, όπως είναι εξάλλου και η ταχύτητα και στο ερώτημα πόσο γρήγορα, στο speed των Άγγλων υπολογίζουν την μέση αριθμητική ταχύτητα. Διδακτικά η εισαγωγή της μέσης αριθμητικής ταχύτητας, δίπλα στον ορισμό της ταχύτητας είναι δύσκολη Μανώλη και θα με ενδιέφερε η άποψη σου (όπως και όλων σας). Το θέμα είναι εμείς (οι καθηγητές) τι έχουμε οι περισσότεροι στο μυαλό μας για την μέση ταχύτητα; η ποικιλία των απαντήσεων που έχω λάβει είναι πολύ μεγάλη. Με ενδιαφέρει η γνώμη σας και συνεχίζω να παρακολουθώ την συζήτηση. Σχόλιο : κάνω δοκιμή να σας γράφω χωρίς τον όρο "δάσκαλος" δίπλα και δεν μου πάει. Είμαστε όλοι συνάδελφοι, φίλοι μέλη ενός κοινωνικού δικτύου, ίση στην αντιμετώπιση του δικτύου προς εμάς, αλλά δεν είμαστε το ίδιο όσον αφορά το επίπεδο και την προσφορά μας στη διδασκαλία. Είμαι υπέρ της ισότητας και κατά της ισοπέδωσης. Δηλαδή να συνδεθεί ένας μαθητής στο δίκτυο και να μου πεί : Ρε Κώστα συμφωνώ με σένα, σε πάω.. Δεν νομίζω ότι είναι ο ρόλος μου αυτός.
Σχόλιο από τον/την Φραγκιαδουλάκης Εμμανουήλ στις 10 Ιανουάριος 2015 στις 16:13
Διαγραφή σχολίου
Κώστα γράφεις: "Διδακτικά η εισαγωγή της μέσης αριθμητικής ταχύτητας, δίπλα στον ορισμό της ταχύτητας είναι δύσκολη Μανώλη και θα με ενδιέφερε η άποψη σου". Δεν καταλαβαίνω γιατί είναι δύσκολη. Η ταχύτητα σαν φυσικό μέγεθος είναι μία και δεν χρειάζεται τον προσδιορισμό διανυσματική, ούτε και τον προσδιορισμό στιγμιαία. Όταν μας ζητούν ταχύτητα είναι αυτονόητο (ή τουλάχιστον πρέπει να είναι ) ότι μας ζητούν ένα καθαρά διανυσματικό μέγεθος. Έχει συγκεκριμένη πραγματική σημασία συνδεόμενη με το κινητό κάθε χρονική στιγμή και σε κάθε σημείο της διαδρομής. Όταν λέμε μέση ταχύτητα χρειάζεται προσδιοριστικός όρος αριθμητική ή διανυσματική. Αυτές οι ταχύτητες έχουν σχετική αξία αφού δεν εκφράζουν πραγματικές ταχύτητες του κινητού και δεν συνδέονται με την κίνηση του κινητού σε κάθε θέση και κάθε χρονική στιγμή. Από αυτές τις δυο ταχύτητες η μέση αριθμητική που εκφράζεται σαν μονόμετρο μέγεθος, με τιμή το λόγο του διαστήματος προς τον χρόνο κίνησης σου, δίνει μια πληροφορία χρήσιμη στην καθημερινότητα, συνήθως για το πόσο γρήγορος οδηγός μπορεί να είναι κάποιος ή πόσο απέχει ένα μέρος από το άλλο, αν μας δώσουν την τιμή της και το χρόνο που χρειάζεται για να καλύψει κάποιος αυτή τη διαδρομή. Πολλές φορές πράγματι που μας ρωτούν πόσο απέχει π.χ. το Ηράκλειο από τη Χερσόνησο απαντούμε ότι "εγώ που πηγαίνω εκεί γύρω στα 60 KM/h κάνω περίπου μια ώρα..." ή ότι ο ήλιος απέχει από τη γη, νομίζω 8 λεπτά φωτός κ.λ.π. Στην καθημερινότητα λοιπόν είναι ένα χρήσιμο μέγεθος που δεν πρέπει να το μπερδεύουμε με την ταχύτητα και δεν βρίσκω γιατί είναι δύσκολο να το ξεκαθαρίσουμε αμέσως από την αρχή στο μαθητή. Η μέση διανυσματική ταχύτητα που ορίζεται μέσα από το διάνυσμα μετατόπισης δεν έχει την χρησιμότητα της μέσης αριθμητικής ταχύτητας στην καθημερινότητα. Θα έλεγα είναι εντελώς άχρηστη στην καθημερινότητα. Μας δίδει πληροφόριση μόνο ως προς τα πού κινήθηκε το κινητό και τίποτε άλλο. Θα μπορούσε να χρησιμοποιηθεί στη Φυσική για τον ορισμό της ταχύτητας, και έτσι γίνεται συνήθως, κάτι που κατά την προσωπική μου άποψη δεν είναι απαραίτητο.
Σχόλιο από τον/την Παπαδάκης Κώστας στις 10 Ιανουάριος 2015 στις 16:44
Διαγραφή σχολίου
Αναλυτικότατη απάντηση. Φυσικά είναι σωστή. :-) Ευχαριστώ δάσκαλε.
159. Πόση είναι η στατική τριβή;

Κύλινδρος ακτίνας R και βάρους w=40 N, ισορροπεί πάνω σε κεκλιμένο επίπεδο γωνίας κλίσης φ=300, με τη βοήθεια σχοινιού παράλληλου στο κεκλιμένο επίπεδο, που περνά από το κέντρο του Ο. Να υπολογιστούν:

Α)α) Η στατική τριβή και,

β) η τάση του σχοινιού

Β) Αν κόψουμε το σχοινί που περνά από το κέντρο του τροχού τότε να ξαναπαντηθούν τα ερωτήματα (α) και (β).

Ποια είναι η τιμή της οριακής τριβής δίνεται μ=sqr3/2.

Θεωρείστε ότι ο κύλινδρος δεν ολισθαίνει.

Δίνονται για τον κύλινδρο Ιcm=0,5mR2 και g=10m/s2.

Συνοπτική λύση:

Σχόλια
Σχόλιο από τον/την Βαγγέλης Φινδανής στις 4 Φεβρουάριος 2015 στις 15:07
Διαγραφή σχολίου
Πολύ ωραία η ανάρτηση. Να κάνω μία ερώτηση; αν το σώμα ήταν ορθογώνιο (δηλαδή δεν είχε την ικανότητα να πραγματοποιήσει κύλιση) στο ερώτημα Α)α) ποια θα ήταν η απάντηση;
Σχόλιο από τον/την Μιχαήλ Μιχαήλ στις 4 Φεβρουάριος 2015 στις 21:50
Διαγραφή σχολίου
Βαγγέλη σ΄ευχαριστώ! Δεν ξέρω αν σε βοηθάει αυτό!
…..ας σηκώσουμε λίγο τη ράβδο!

Η ράβδος του σχήματος έχει μάζα M=20 kg και μήκος L=4m. Ο τροχός έχει μάζα επίσης m=M=20 kg και ακτίνα R=0,5m. Έστω ότι η ράβδος σχηματίζει γωνία θ με το οριζόντιο έδαφος και ότι ζητείται να υπολογιστεί η οριζόντια δύναμη FΑ που πρέπει να ασκούμε στο άκρο Α της ράβδου καθώς και η γωνία θ, ώστε αυτή να επιταχύνεται με επιτάχυνση α=2m/s2.

Ο τροχός δεν ολισθαίνει. Δίνονται Ι=0,5mR2 και g=10m/s2.

Συνοπτική λύση:

Σχόλια
Σχόλιο από τον/την Μιχαήλ Μιχαήλ στις 14 Φεβρουάριος 2015 στις 13:42
Διαγραφή σχολίου
Αφιερώνεται στον Γίαννη (Κυριακ.) για την πολύ ωραία άσκηση που μας έδωσε! Παραλλαγή της οποίας είναι η παραπάνω.
Σχόλιο από τον/την Κυριακόπουλος Γιάννης στις 14 Φεβρουάριος 2015 στις 15:40
Διαγραφή σχολίου
Ευχαριστώ Μιχάλη. Πολύ καλή παρουσίαση. Μια απόπειρα προσομοίωσης. Σε επιβεβαιώνει αλλά πρόκειται για ασταθή ισορροπία και το i.p. τα "παίζει".
Σχόλιο από τον/την Κωστας Ψυλακος στις 14 Φεβρουάριος 2015 στις 18:06
Διαγραφή σχολίου
Μιχαλη ωραιο το θεμα σου με πολυ καλη και ενδιαφερουσα αναλυση ! Θα ηθελα να αναφερω οτι και στις τεσσερεις περιπτωσεις ο τελικος τυπος που δινει την θ ειναι ο ιδιος . Δηλαδη : εφθ=g/4α =5/4 . Μαλιστα για τις περιπτωσεις 2,3,4 εχουμε εφθ=mg/(2F-mα)=mg/(5mα-mα)=> εφθ=g/4α =5/4. (οπου F=Fα)
Σχόλιο από τον/την Κορκίζογλου Πρόδρομος στις 14 Φεβρουάριος 2015 στις 18:54
Διαγραφή σχολίου
Μιχάλη πολύ ωραία πρόταση, συγχαρητήρια!
Σχόλιο από τον/την Μιχαήλ Μιχαήλ στις 14 Φεβρουάριος 2015 στις 18:55
Διαγραφή σχολίου
Κώστα σε ευχαριστώ πολύ για τις παρατηρήσεις σου! Η προσθήκη έγινε!
Σχόλιο από τον/την Μιχαήλ Μιχαήλ στις 14 Φεβρουάριος 2015 στις 18:56
Διαγραφή σχολίου
Γιάννη σε ευχαριστώ πολύ για το i.p
Σχόλιο από τον/την Μιχαήλ Μιχαήλ στις 14 Φεβρουάριος 2015 στις 18:59
Διαγραφή σχολίου
Πρόδρομε σ΄ευχαριστώ πολύ! Να' σαι πάντα καλά.
Σχόλιο από τον/την Μιχαήλ Μιχαήλ στις 16 Φεβρουάριος 2015 στις 20:44
Διαγραφή σχολίου
Δημήτρη χαίρομαι που σου άρεσε. Η αλήθεια είναι ότι είχες κάνει μια πλήρη μελέτη του θέματος. Δεν πειράζει "Επανάληψις, μήτηρ μαθήσεως".
161.Ταλάντωση δυο σωμάτων

Το σχήμα δείχνει δυο σώματα m1 και m2 που συνδέονται με ένα αβαρές ελατήριο σταθεράς Κ. Το σύστημα μπορεί να ταλαντώνεται έτσι ώστε οι δυο μάζες να κυλίονται. Έστω ότι το φυσικό μήκος του ελατηρίου είναι L0. Τότε για τη μεταβολή x(t) του μήκους του ελατηρίου σε μια τυχαία θέση κατά τη διάρκεια της συσπείρωσης ισχύει:

x=L0-L=L0-(x1-x2).

συνέχεια...

163. Κρούση σφαιρών (έργο τριβής κύλισης).

Η συμπαγής σφαίρα m1=2Kg του σχήματος κυλίεται χωρίς να ολισθαίνει σε οριζόντιο επίπεδο και συγκρούεται μετωπικά και ελαστικά με αρχικά ακίνητη κούφια σφαίρα μάζας m2=1Kg της ίδιας ακτίνας R=5cm. (Οι ακτίνες θεωρούνται ίσες ώστε η κρούση να είναι  κεντρική). Η οριζόντια ταχύτητα του κέντρου μάζας της σφαίρας πριν τη σύγκρουση έχει μέτρο υ1=3m/s όπως φαίνεται στο σχήμα.

α) Να βρείτε τις ταχύτητες των δυο σφαιρών αμέσως μετά τη σύγκρουση αν οι μάζες εξακολουθούν να κινούνται οριζόντια.

 

β) Να υπολογιστεί η συνολική στροφορμή του συστήματος των δυο μαζών ακριβώς πριν την κρούση και για σταθερό σημείο του εδάφους.

 

γ) Σε πόσο χρόνο μετά τη σύγκρουση οι μάζες m1 και m2 θα σταματήσουν να ολισθαίνουν; Ποια μάζα θα σταματήσει την ολίσθηση πρώτη;

 

δ) i) Να υπολογιστεί το έργο της τριβής ολίσθησης ακριβώς μετά την κρούση και μέχρι να αρχίσει η κύλιση της σφαίρας m2.

ii) Ποιος είναι τότε ο ρυθμός μεταβολής της στροφορμής της ;

iii) Πόσο είναι το έργο της τριβής κύλισης από τη στιγμή που αρχίζει η κύλιση και μέχρι να σταματήσει η κίνηση της σφαίρας; Θεωρείστε αμελητέο το έργο της τριβής κύλισης κατά τη διάρκεια της ολίσθησης.

 

Δίνεται ο συντελεστής τριβής ολίσθησης μεταξύ σφαίρας και επιπέδου μ=1/7. Θεωρούμε ότι μεταξύ των σφαιρών δεν αναπτύσσεται κάποια τριβή. Aκόμη δίνεται για τη σφαίρα μάζας m1 και ακτίνας R, Icm1=0,4×m1×R2 και για τη σφαίρα μάζας m2 και ακτίνας R, Icm2=m2×R2. Για τις πράξεις θεωρείστε g=10m/s2.

 

Συνοπτική λύση:

Σχόλια
Σχόλιο από τον/την Κορφιάτης Ευάγγελος στις 10 Απρίλιος 2015 στις 16:18
Διαγραφή σχολίου
Καλησπέρα Μιχάλη. Γενικά μου άρεσε η άσκηση ( αν και σε αρκετά σημεία είναι εκτός ύλης). Έχω ένα πρόβλημα με την παραδοχή ότι μεταξύ των σφαιρών δεν αναπτύσσεται τριβή (ισοδύναμα ότι οι δυνάμεις μεταξύ των σφαιρών είναι κεντρικές). Την στιγμή της σύγκρουσης τα σημεία επαφής των δύο σφαιρών βρίσκονται σε σχετική κίνηση. Επειδή οι σφαίρες δεν είναι λείες θα ασκηθεί αναγκαστικά τριβή ολισθήσεως, η οποία θα μειώσει την γωνιακή ταχύτητα της 1 και θα προσδώσει γωνιακή ταχύτητα στην 2. Νομίζω ότι μπορείς να διατηρήσεις την βασική δομή της άσκησης και τα περισσότερα από τα ερωτήματα θεωρώντας ότι κάποια από τις δύο σφαίρες είναι λεία.
Σχόλιο από τον/την Μιχαήλ Μιχαήλ στις 11 Απρίλιος 2015 στις 9:26
Διαγραφή σχολίου
Ευάγγελε, καλημέρε και σ' ευχαριστώ για τη θετική προσέγγιση της άσκησης. Ύλη είναι μόνο η έμπνευσή μας. Τελείως μονωμένο σύστημα δεν υπάρχει. Σ΄ευχαριστώ και πάλι! ΚΑΛΗ ΑΝΑΣΤΑΣΗ
Σχόλιο από τον/την Μιχαήλ Μιχαήλ στις 11 Απρίλιος 2015 στις 13:24
Διαγραφή σχολίου
Αν δεχτούμε ότι υπάρχει και τριβή ολίσθησης μεταξύ των δυο σφαιρών τότε:
Σχόλιο από τον/την Κορφιάτης Ευάγγελος στις 11 Απρίλιος 2015 στις 13:35
Διαγραφή σχολίου
Καλημέρα Μιχάλη. Άψογος. Μια συμπλήρωση. Νομίζω ότι η γνώση του χρονικού διαστήματος της κρούσης δεν είναι απαραίτητη: Διαιρώντας κατά μέλη τους νόμους μεταβολής ορμής και στροφορμής απαλείφεται τόσο το χρονικό διάστημα όσο και η δύναμη επαφής.
Σχόλιο από τον/την Κυριακόπουλος Γιάννης στις 11 Απρίλιος 2015 στις 14:02
Διαγραφή σχολίου
Το τι θα συμβεί εξαρτάται από την τιμή του συντελεστή τριβής. Με μικρό συντελεστή υφίσταται συνεχώς ολίσθηση και Τ=μ.Ν. Όμως με μεγαλύτερες τιμές του συντελεστή τριβής παύει η ολίσθηση και τα σημεία επαφής αποκτούν κοινές ταχύτητες. Ταχύτητες σχετιζόμενες με την γωνιακή τους ταχύτητα. Ισχύει ανισότητα και όχι ισότητα. Κλειδί γίνεται η ισότητα των ταχυτήτων των σημείων επαφής. Είναι δυσκολότερο πρόβλημα. Σχετικό: Δυο δαχτυλίδια συγκρούονται. Γίνεται όλη η διερεύνηση.
Σχόλιο από τον/την Μιχαήλ Μιχαήλ στις 11 Απρίλιος 2015 στις 14:13
Διαγραφή σχολίου
Πολύ σωστά! Δηλαδή
Σχόλιο από τον/την Κορφιάτης Ευάγγελος στις 11 Απρίλιος 2015 στις 14:16
Διαγραφή σχολίου
Καλημέρα Γιάννη. Διαβάζοντας για τα δαχτυλίδια μου προέκυψε η εξής απορία: Με ποιό επιχείρημα διατηρείται η κινητική ενέργεια κέντρου μάζας; ( Το ερώτημα ισχύει τόσο στην άσκηση σου όσο και στην άσκηση του Μιχάλη).
Σχόλιο από τον/την Μιχαήλ Μιχαήλ στις 11 Απρίλιος 2015 στις 14:17
Διαγραφή σχολίου
Γιάννη γράφαμε μαζί! ΚΑΛΗ ΑΝΑΣΤΑΣΗ
Σχόλιο από τον/την Μιχαήλ Μιχαήλ στις 11 Απρίλιος 2015 στις 14:19
Διαγραφή σχολίου
....Ευάγγελε δεν υπάρχει τριβή μεταξύ των σφαιρών. Αλλιώς το πρόβλημα γίνεται δύσκολο. Στην εκφώνησή μου αυτό εννοούσα με τις κεντρικές δυνάμεις!
Σχόλιο από τον/την Κυριακόπουλος Γιάννης στις 11 Απρίλιος 2015 στις 14:24
Διαγραφή σχολίου
Καλή Ανάσταση Μιχάλη και Βαγγέλη. Βαγγέλη δεν θυμάμαι που αναφέρω διατήρηση Κ.Ε. κέντρου μάζας. Πάντως όταν ένα σύστημα είναι μονωμένο (δυναμικά) διατηρείται η ταχύτητα του κέντρου μάζας του. Μένει σταθερό το 0,5*mολ*υcm*υcm. Τώρα αν αυτό το ονομάσουμε κινητική ενέργεια κέντρου μάζας....
Σχόλιο από τον/την Κορφιάτης Ευάγγελος στις 11 Απρίλιος 2015 στις 14:34
Διαγραφή σχολίου
Γιάννη αναφέρομαι στο ελαστικό της κρούσης "όσον αφορά στην μεταφορική κίνηση". Συνολική κινητική ενέργεια κέντρου μάζας πριν = Συνολική Κινητική ενέργεια κέντρου μάζας μετά.
Σχόλιο από τον/την Κυριακόπουλος Γιάννης στις 11 Απρίλιος 2015 στις 14:57
Διαγραφή σχολίου
Για κάθε μπάλα ή δαχτυλίδι ή για το σύστημά τους; Αν λες για κάθε μπάλα τότε θα πω ότι οι δυνάμεις λόγω παραμορφώσεων είναι ίδιες. Η x-ελαστικότητα υφίσταται και η y κίνηση δεν την αλλοιώνει. Δες εδώ. Αν δυο ισόμαζες μπάλες συγκρουστούν τότε θα ανταλλάξουν τις x ταχύτητες άσχετα με το αν υπάρχει τριβή ή όχι. Άσχετα με το αν το τεκμηριώνω σωστά, στην προηγούμενη ανάρτησή μου για την ελαστική κρούση, κάθε προσομοίωση επιβεβαιώνει την θέση αυτήν.
Σχόλιο από τον/την Κορφιάτης Ευάγγελος στις 11 Απρίλιος 2015 στις 15:06
Διαγραφή σχολίου
Διαισθάνομαι ότι έχεις δίκιο. Η ελαστικότητα της κρούσης έχει σχέση με την ελαστικότητα του υλικού. Απλώς ήλπιζα ότι έχεις και κάποιο επιχείρημα πιο ποσοτικό. Κάποια στιγμή θα πρέπει να ασχοληθούμε με την ελαστικότητα στο πλαίσιο της φυσικής συνεχών μέσων. Περιοριζόμαστε στον νόμο του Hooke σε ελατήρια ή το πολύ σε κανένα λάστιχο.
Σχόλιο από τον/την Κυριακόπουλος Γιάννης στις 11 Απρίλιος 2015 στις 15:07
Διαγραφή σχολίου
Με την επίκληση των διαγραμμάτων παρακάμπτουμε την διατήρηση της ενέργειας και επικαλούμαστε μια x ελαστικότητα πλήρως επιβεβαιωμένη εξ' άλλου.
Σχόλιο από τον/την Κυριακόπουλος Γιάννης στις 11 Απρίλιος 2015 στις 15:12
Διαγραφή σχολίου
Γράφαμε μαζί. Ποσοτικό είναι το επιχείρημα ότι οι x δυνάμεις είναι ίδιες άσχετα με την μεταξύ τους τριβή. Ίδιες δυνάμεις σε ίδιες μάζες θα προκαλέσουν ίδια x αποτελέσματα. Όταν (το 2010) πρωτοασχολήθηκα (μπάλα πέφτει σε τραίνο) είχα ζωηρές αμφιβολίες για το αν κάνω ή όχι λάθος. Οι προσομοιώσεις με επιβεβαίωσαν πλήρως και αποδέχθηκα την ορθότητα.
Test Πολλαπλής επιλογής γ΄ γυμνασίου Ταλαντώσεις - Κύματα

Test Πολλαπλής επιλογής γ΄ γυμνασίου

Ταλαντώσεις - Κύματα

164. Πότε πέφτει γρηγορότερα;

Μια ομογενής ράβδος ΑΒ έχει μήκος L=2m και μάζα M=3Kg. Στο σημείο Γ με (ΒΓ)=L/4, είναι δεμένη σημειακή μάζα m=1 Κg. Η ράβδος ισορροπεί με κλίση όπως φαίνεται στο σχήμα μέσω δυο ακλόνητων αρθρώσεων που το ένα τους άκρο είναι βιδωμένο στη ράβδο και το άλλο στο ταβάνι. Η ράβδος μπορεί να περιστρέφεται χωρίς τριβές γύρω από τις βίδες.

 

Α) Αν θέλαμε να στερεώσουμε το σύστημα της ράβδου ΑΒ και της μάζας m, με μια μόνο ακλόνητη άρθρωση που το ένα άκρο της να είναι στηριγμένο στο ταβάνι, να βρείτε σε ποια θέση πάνω στη ράβδο πρέπει να στερεώσουμε το άλλο άκρο της, ώστε το σύστημα να ισορροπεί στην ίδια θέση;

 

Β) i) Αν τη χρονική στιγμή t0=0 σπάσουν και οι δυο αρθρώσεις τότε να βρείτε με ποια ταχύτητα θα φτάσει το σύστημα των δύο μαζών στο έδαφος.

ii) Αν δέσουμε τη μάζα m στο σημείο Β και αφήσουμε το σύστημα να πέσει από την ίδια θέση τότε αυτό θα φτάσει γρηγορότερα στο έδαφος ή όχι; Εξηγείστε.

 

Γ) Αν τη χρονική στιγμή t0=0 σπάσει μόνο η άρθρωση στο Β, τότε πότε το σύστημα φτάνει γρηγορότερα στην κατακόρυφη θέση, όταν η μάζα m είναι δεμένη στο σημείο Γ ή όταν είναι δεμένη στο σημείο Β;

Δίνεται για τη ράβδο  Icm=1/12×Μ×L2. Για τις πράξεις θεωρείστε g=10m/s2.

 

Συνοπτική λύση:

165. Δίσκος

Ο ομογενής δίσκος του σχήματος μάζας M=0,2Kg και ακτίνας  R=0,1m κυλίεται χωρίς να ολισθαίνει πάνω  σε οριζόντιο δάπεδο. Τη χρονική στιγμή t0=0s ο δίσκος έχει αρχική ταχύτητα υ0=4m/s και εκείνη τη στιγμή, ασκείται στο δίσκο  μια σταθερή οριζόντια δύναμη F σε απόσταση r=8cm από το κέντρο μάζας του Κ,  όπως φαίνεται στο σχήμα.

 

α) Αν τη χρονική στιγμή t=2s, η γωνιακή ταχύτητα περιστροφής του δίσκου μηδενίζεται, τότε να υπολογιστεί το μέτρο της δύναμης F.

 

β) Να υπολογιστεί το έργο της δύναμης F μέχρι τη χρονική στιγμή t=2s.

 

γ) Ποιο είναι το μέτρο του μέγιστου ρυθμού μεταβολής της κινητικής ενέργειας του δίσκου για 0≤t≤2s;

 

δ) Να υπολογιστεί η κινητική ενέργεια του δίσκου τη χρονική στιγμή t=3 s.

 

Δίνεται για το δίσκο  Icm=0,5ΜR2.

 

Συνοπτική λύση (blog):

ή εδώ

Σχόλια

Ετικέτες: στερεό, τάξη-γ

Σχόλιο από τον/την Διονύσης Μάργαρης στις 25 Μάιος 2015 στις 10:32
Διαγραφή σχολίου
Καλημέρα Μιχαήλ. Δυνατή περίπτωση, αλλά ίσως θα ήταν χρησιμότερη σε άλλη χρονική στιγμή...
Σχόλιο από τον/την Μιχαήλ Μιχαήλ στις 25 Μάιος 2015 στις 11:45
Διαγραφή σχολίου
Διονύση σ' ευχαριστώ για το σχόλιο! ...για εύκολη την ανά(ή)ρτησα!!
Σχόλιο από τον/την Κωστας Ψυλακος στις 25 Μάιος 2015 στις 12:12
Διαγραφή σχολίου
Καλημερα Μιχαλη ! Εκει που θα πρεπει να εστιασει κανεις την προσοχη του σε αυτην την ασκηση ειναι στο ρυθμο μεταβολης της κινητκης ενεργειας !!! Τοσο η Τστ οσο και η F συμμετεχουν και στις δυο κινησεις ομως η Τστ οσο "δινει" στην μια κινηση ακριβως τοσο αφαιρει απο την αλλη μιας και εχουμε συνεχως Κ.Χ.Ο. Πολυ καλη !
Σχόλιο από τον/την Μιχαήλ Μιχαήλ στις 25 Μάιος 2015 στις 12:15
Διαγραφή σχολίου
Καλημέρα Κώστα και σ' ευχαριστώ!
Σχόλιο από τον/την Παπαδάκης Παντελεήμων στις 26 Μάιος 2015 στις 10:10
Διαγραφή σχολίου
Καλημέρα Μιχαήλ. Περιεργάστηκα την άσκηση και μου δημιουργήθηκαν δύο απορίες …ας πούμε τεχνικής φύσεως αφού δεν αμφισβητώ τη λύση σου. 1) Δεν καταλαβαίνω (σαν μαθητής εννοώ) πως εφαρμόζεις τη ΣF και Στ ως προς τα πρόσημα… Εγώ συνηθίζω να λέω ‘’…φορά μεταφοράς θετική , φορά περιστροφής θετική …’’ και μετά γράφω… -F-T=Mαcm TR-Fr=(MR2/2)αγ αcm= αγR και επιλύοντας,προκύπτει ….F=-1/3 N όπου το (-) μου λέει... 2) Πως θα μπορούσαμε στη πράξη να εφαρμόσουμε στο δίσκο την F στο συγκεκριμένο σημείο ώστε να μη φρενάρει μόνο, αλλά να συνεχίσει να δρα και μετά το μηδενισμό της ω; (…καλοπροαίρετα πιστεύω, θα δεχθείς τις απορίες μου)  
Σχόλιο από τον/την Διονύσης Μάργαρης στις 26 Μάιος 2015 στις 10:30
Διαγραφή σχολίου
Καλημέρα Παντελή. Τις απαντήσεις θα τις δώσει ο Μιχαήλ, αλλά θυμήθηκα κάτι από τα παλιά και λέω να το καταθέσω. "Πως θα μπορούσαμε στη πράξη να εφαρμόσουμε στο δίσκο την F στο συγκεκριμένο σημείο ώστε να μη φρενάρει μόνο, αλλά να συνεχίσει να δρα και μετά το μηδενισμό της ω;" Στο χωριό, πριν ...κάμποσα χρόνια, δεν είχαμε νερό. ...μονάχα λίγες στέρνες, άδειες και αυτές... Κουβαλάγαμε λοιπόν με βαρέλι νερό, από μια απομακρυσμένη στέρνα, σπρώχνοντας το βαρέλι. Αν ο δρόμος ήταν οριζόντιος και "καλός" γινόταν εύκολα και μας έμοιαζε παιχνίδι... Αναλαμβάναμε τα πιτσιρίκια. Στις ανηφόρες ήταν τα δύσκολα! Μπορούσες λοιπόν να σπρώχνεις, ασκώντας δύναμη, συνήθως εφαπτομενικά (δεν το ξέραμε θεωρητικά τότε...) αλλά πρακτικά σπρώχναμε λίγο πιο κάτω. Και αν το ένα πιτσιρίκι, σπρώχνει, όπως ο Αντώνης και ενώ το βαρέλι έχει κάποια ταχύτητα, ο Βασίλης, αποφασίσει να σπρώξει αντίθετα; Παντελή και μια αφιέρωση για καλημέρα:  
Σχόλιο από τον/την Διονύσης Μάργαρης στις 26 Μάιος 2015 στις 10:34
Διαγραφή σχολίου
...που ηχούν, που ηχούν και που τις προσκυνάμε... Αλήθεια μπορεί να καταλάβει ένα σημερινός νέος το νόημα του στίχου; Τι θέλει να πει ο Σεφέρης;
Σχόλιο από τον/την Γιάννης Μπατσαούρας στις 26 Μάιος 2015 στις 12:38
Διαγραφή σχολίου
Καλημέρα Μιχαήλ και συγχαρητήρια για την άσκηση. Ως προς τα πρόσημα ακολουθώ κι εγώ την τεχνική του Παντελή. Ως θετική φορά παίρνω την φορά της αρχικής ταχύτητας. Αν δεν υπάρχει αρχική ταχύτητα παίρνουμε θετική τη φορά της κίνησης(όλα ομόροπα θα είναι) αγνωστα διανύσματα , δυνάμεις , ταχύτητες , ορμές ...τα θεωρώ θετικά Την επιτάχυνση την θεωρώ α θετική Τα αναφέρω για όσους είχαν διδάξει πλάγιες βολές- Συναντήσεις σωμάτων επί δεσμών. Η λύση με το απόλυτο α που παραθέτεις νομίζω πως είναι πιό κατανοητή για μαθητές .
Σχόλιο από τον/την Μιχαήλ Μιχαήλ στις 26 Μάιος 2015 στις 19:58
Διαγραφή σχολίου
Παντελή σε ευχαριστώ για το σχόλιό σου. Πάντα καλοπροαίρετα και με αγάπη αντιμετωπίζω όλα τα σχόλια και αυτό νομίζω ότι είναι το βασικό στοιχείο του ylikonet! 1) Εγώ δουλεύω με το μέτρο των δυνάμεων και της επιτάχυνσης ( η ΣF και η α έχουν πάντα την ίδια φορά) και αποφεύγω με αυτόν τον τρόπο τις αλγεβρικές τιμές. Το αν η κίνηση είναι επιταχυνόμενη ή επιβραδυνόμενη το καθορίζουν οι αρχικές συνθήκες. 2) Όσο για τη δύναμη F, η ιδέα προέκυψε όταν είδα παιδιά να φυσάνε μια πλαστική μπάλα που έρχονταν κατά πάνω τους προσπαθώντας να τη σταματήσουν να κινείται!! Γιάννη σ' ευχαριστώ για το σχόλιό σου! Να' σαι πάντα καλά
Σχόλιο από τον/την Παπαδάκης Παντελεήμων στις 26 Μάιος 2015 στις 21:25
Διαγραφή σχολίου
Καλησπέρα Διονύση και Μιχαήλ. Πριν λίγο τέλειωσε η βάρδια του παππού… Κατανοητά τα παραδείγματά σας ,μα μ’ ενδιέφερε το σημείο εφαρμογής της F, που ομολογώ δεν το τόνισα, καθ’όσον στο στερεό έχει σημασία νομίζω. Βέβαια στη περίπτωση σου Μιχαήλ, δεν διαφοροποιείται η ροπή είτε είναι στο εσωτερικό του δίσκου είτε στην επιφάνεια του δίσκου. Όμως σε άλλη περίπτωση όπως στο δεύτερο σχήμα; Διονύση, σ’ευχαριστώ για την όμορφη μελοποίηση απ’το Μαρκόπουλο ενός τμήματος από το ‘’μυθυστόρημα’’ του Σεφέρη. Αυτές …τις ‘’συμπληγάδες ‘’ να μπορούσαμε να περνούσαμε και ίσως τότε θα δίναμε στους νέους δυνατότητες να νοιώθουν ,να στοχάζονται σαν μέλη της ίδια κοινότητας με κοινά προβλήματα…
Σχόλιο από τον/την Κωστας Ψυλακος στις 26 Μάιος 2015 στις 21:32
Διαγραφή σχολίου
Μιχαλη καλησπερα . Θα συμφωνησω με τον τροπο που περιγραφεις στο σχολιο σου σχετικα με το μετρο της ΣF και της α. Θα ηθελα να προσθεσω κατι στην ασκηση σου . Ο τροπος που υπολογισες τον ρυθμο μεταβολης της κινητικης ενεργειας "κρυβει" λιγο τα προσημα που υπαρχουν και αυτο γιατι θελουμε το μετρο του ρυθμου μεταβολης . Στο link που ακολουθει τον υπολογιζω με δυο τροπους . Βρισκω την τιμη του ,που θα ειναι αρνητικη μιας και εχουμε επιβραδυνση,στην συνεχεια βεβαια αρκει καποιος να παρει την απολυτη τιμη του αποτελεσματος μιας και αυτο ζητας . Ομως υπαρχει και ενα αλλο συμπερασμα : ο ρυθμος μεταβολης της κινητικης ενεργειας ειναι ισος με το γινομενο της δυναμης F επι την ταχυτητα του σημειου εφαρμογης της (εστω του σημειου Α) φυσικα με ενα (-) μπροστα. http://1drv.ms/1J35fG2
Σχόλιο από τον/την Μιχαήλ Μιχαήλ στις 26 Μάιος 2015 στις 22:04
Διαγραφή σχολίου
Κώστα Μπράβο σου!
Σχόλιο από τον/την Κωστας Ψυλακος στις 26 Μάιος 2015 στις 22:05
Διαγραφή σχολίου
Ευχαριστω Μιχαλη ! Χαιρομαι που σου αρεσε . Καλο βραδυ !

Σχόλιο

166. Κύλινδρος

           

Πάνω σε οριζόντιο επίπεδο βρίσκεται ένας κύλινδρος μάζας M=2Kg ακτίνας R=20cm. Σε απόσταση r1=R/4 από το κέντρο του κυλίνδρου και πάνω σε αυτόν βρίσκεται τυλιγμένο κατάλληλα ένα αβαρές σχοινί που μπορεί να ξετυλίγεται χωρίς να γλιστρά. Στο ελεύθερο άκρο αυτού του σχοινιού ασκείται σταθερή οριζόντια δύναμη F. Επίσης σε απόσταση r2=R/2 από το κέντρο του κυλίνδρου και πάνω σε αυτόν βρίσκεται τυλιγμένο κατάλληλα και ένα δεύτερο αβαρές σχοινί που επίσης μπορεί να ξετυλίγεται χωρίς να γλιστρά.

Το δεύτερο αυτό σχοινί περνάει από το αυλάκι μιας σταθερής αβαρούς τροχαλίας στο ελεύθερο άκρο του οποίου είναι δεμένο σώμα μάζας m=1Kg. Τότε να υπολογιστούν:

 

Α) Το μέτρο της σταθερής οριζόντιας δύναμης F ώστε το σύστημα να ισορροπεί.

 

Β) α) Το μέτρο της σταθερής οριζόντιας δύναμης F έτσι ώστε το κέντρο μάζας του κυλίνδρου να επιταχύνεται προς τα δεξιά με σταθερή επιτάχυνση αcm=10m/s2.

β) Για την επιταχυνόμενη κίνηση του συστήματος να υπολογιστεί η μεταβολή της κινητικής ενέργειας του κυλίνδρου καθώς και η ολική μεταβολή της κινητικής ενέργειας του συστήματος για μετατόπιση του Κ.Μ του κυλίνδρου κατά x=0,2m.

γ) Να υπολογιστεί ο ρυθμός μεταβολής της κινητικής ενέργειας του συστήματος τη χρονική στιγμή που το Κ.Μ του κυλίνδρου έχει ταχύτητα υcm=2m/s.

Θεωρείστε ότι σε κάθε περίπτωση ο κύλινδρος κυλίεται χωρίς να ολισθαίνει.

Δίνεται η ροπή αδράνειας ως προς το Κ.Μ του κυλίνδρου Ιcm=I =0,5∙Μ∙R2  και για τις πράξεις g=10m/s2.

 

Συνοπτική λύση:

Σχόλια
Σχόλιο από τον/την Νεκτάριος Πρωτοπαπάς στις 8 Ιούνιος 2015 στις 21:49
Διαγραφή σχολίου
Καλησπέρα Μιχαήλ. Πάρα πολύ καλό θέμα. Αναρωτιέμαι γιατί δεν έχουμε δει ακόμη κάτι τέτοιο στις πανελλήνιες των παιδιών (εννοώ με συνδυασμό κινήσεων και επιταχύνσεων).
Σχόλιο από τον/την Μιχαήλ Μιχαήλ στις 10 Ιούνιος 2015 στις 15:33
Διαγραφή σχολίου
Nεκτάριε σε ευχαριστώ για το σχόλιό σου!
Σχόλιο από τον/την Κωστας Ψυλακος στις 11 Ιούνιος 2015 στις 12:07
Διαγραφή σχολίου
Καλημερα Μιχαλη ! Ειχε πολλα και καλα ! Μας εδωσες ακομα μια πολυ ομορφη ασκηση. Για το ερωτημα β αφου μας εχεις δωσει το χ=0.2m εχουμε και το αcm =10m/s^2 ==>υcm=2m/s , ω=υcm/R=10r/s Aρα ΔΚκυλ=0.75Μυcm^2=1.5mυcm^2=6j (M=2m) και ΔΚm=0.5mu^2=4.5j (u=υcm+0.5ωR=1.5υcm=3m/s Tελικα ΔΚολ=10.5j. Για το γ: (dΚ/dt)κυλ= ΣFx*υcm +Στ*ω=Μ*αcm*υcm + 0.5MR^2*αγων*ω==> (dΚ/dt)κυλ = 1.5Μ*αcm*υcm=3*m*αcm*υcm= 60 j/s (dΚ/dt)(m)=ΣF*υ=m*α*u=m*1.5αcm*1.5υcm=2.25*m*αcm*υcm= 45 j/s Συνολικα: dΚ/dt = (21/4)*m*αcm*υcm= 105 j/s Προτεινω μια τετοια λυση για να μην μπουν ξανα οι δυναμεις στην αναλυση του β και γ ερωτηματος μιας και στο ερωτημα α εχει γινει η αναλυση των δυναμεων .
Σχόλιο από τον/την Μιχαήλ Μιχαήλ στις 13 Ιούνιος 2015 στις 13:04
Διαγραφή σχολίου
Κώστα, καλημέρα και συγγνώμη για την καθυστερημένη απάντησή μου. Κώστα για το (β) συμφωνώ με την προσέγγισή σου! Για το (γ) υπάρχουν και οι δυο λύσεις! Σ΄ευχαριστώ για το σχόλιό σου. Να ' σαι πάντα καλά
3. ΜΗΧΑΝΙΚΗ ΤΩΝ ΡΕΥΣΤΩΝ

3. ΜΗΧΑΝΙΚΗ ΤΩΝ ΡΕΥΣΤΩΝ

..καλή αρχή

Σχόλια
Σχόλιο από τον/την Παπασγουρίδης Θοδωρής στις 17 Ιούνιος 2015 στις 18:10
Διαγραφή σχολίου
Δε θυμάμαι αν στο είχα γράψει, αλλά στο μυαλό μου είσαι ο "μεθοδικότερος" όλων Το επιβεβαίωσες για άλλη μια φορά Σε ευχαριστούμε
Σχόλιο από τον/την Κορφιάτης Ευάγγελος στις 17 Ιούνιος 2015 στις 20:11
Διαγραφή σχολίου
Καλησπέρα Μιχαήλ Χωρίς να έχω κοιτάξει κάτι στα ρευστά, υπάρχει κάτι στην απόδειξή σου που με προβληματίζει. Γράφεις ότι: Ποια είναι η μάζα Δm και σε ποια κίνηση γίνεται ο υπολογισμός; Λογικά φαίνεται να εφαρμόζεται σε μια στοιχειώδη μάζα που πάει από το Β στο Γ. Στην περίπτωση αυτή καταλαβαίνω το έργο του βάρους αλλά δεν καταλαβαίνω το έργο των F1 και F2.
Σχόλιο από τον/την Μιχαήλ Μιχαήλ στις 17 Ιούνιος 2015 στις 20:27
Διαγραφή σχολίου
Θοδωρή σ' ευχαριστώ πολύ. Είχα κάποιες σημειώσεις από τα παλιά και είπα να τις Μ Ο Ι Ρ Α Σ Τ Ω γιατί αυτό είναι καλό για όλους! (Θοδωρή σου χρωστάω ένα βιβλίο για τον Παπαπέτρου..θα στο στείλω αυτές τις ημέρες). Να σαι πάντα καλά. Βαγγέλη καλησπέρα. (αντιγράφω από το σχολικό βιβλίο σελίδα 95) .....Θα εφαρμόσουμε το θεώρημα έργου - ενέργειας στο μικρό χρονικό διάστημα Δt. Σύμφωνα με αυτό W + WB = ΔΚ (3.10) όπου W το έργο που προσφέρεται στο τμήμα του ρευστού από το Β στο Γ από το περιβάλλον ρευστό. Το έργο αυτό θα είναι το έργο της δύναμης p1Α1 (θετικό) συν το έργο της p2Α2 (αρνητικό) W = p1A1Δs1 - p2A2Δs2 (3.11) Όμως A1Δs1 = A2Δs2 = ΔV Οπότε W =( p1 - p2 )ΔV Το έργο του βάρους στο ίδιο χρονικό διάστημα είναι WB = - Δm g(y2 - y1) = -ρΔV g(y2 - y1) (3.12) καθώς, στην ουσία, ένα τμήμα του ρευστού Am έφυγε από το ύψος y1 και βρέθηκε στο ύψος y2. Η μεταβολή της κινητικής ενέργειας θα είναι ΔΚ = Εικόνα Δmυ22 - Εικόνα Δmυ12 = Εικόνα ρΔV(υ22 - υ12) (3.13)
Σχόλιο από τον/την Κορφιάτης Ευάγγελος στις 17 Ιούνιος 2015 στις 21:32
Διαγραφή σχολίου
Μιχάλη χωρίς καμία διάθεση αντιδικίας. Προσπαθώ να καταλάβω. Μια στοιχειώδης μάζα υγρού πάει από το Β στο Γ σε ένα μικρό χρονικό διάστημα Δt; Αν τα Β και Γ απέχουν 15Km τι γίνεται; Κάτι άλλο πρέπει να συμβαίνει και οι συγγραφείς το έχουν παρανοήσει ή κάτι δεν καταλαβαίνω εγώ.
Σχόλιο από τον/την Μιχαήλ Μιχαήλ στις 17 Ιούνιος 2015 στις 21:56
Διαγραφή σχολίου
Βαγγέλη ο νόμος του Bernoulli αναφέρεται σε ιδανικό ρευστό που είναι ασυμπίεστο. Μπορούμε λοιπόν να θεωρήσουμε ότι η ροή του ρευστού σε χρόνο dt ισοδυναμεί με μεταφορά του όγκου dV από τη διατομή S1 (θέση Β), στη διατομή S2 (θέση Γ) όσο μακριά και αν βρίσκονται αυτές. Για τα πραγματικά ρευστά θα πρέπει να λάβουμε υπόψη μας και την εσωτερική τριβή, την παρουσία οποιασδήποτε γωνιακής ταχύτητας κ.λ.π.
Σχόλιο από τον/την Κόμης Γιώργος στις 18 Ιούνιος 2015 στις 2:09
Διαγραφή σχολίου
Μιχαήλ καλημέρα.Με την ευκαιρία της ανάρτησης σου άνοιξα το βιβλίο να δω τι θυμάμαι τριάντα χρόνια από ρευστά.Έριξα μια ματιά στην απόδειξη του βιβλίου της εξίσωσης bernoulli και δυσκολεύτηκα στη κατανόηση της. Σκέφτηκα ότι κάπου χάνει ή μάλλον κάτι δεν αντιλαμβάνομαι. Είδα και halliday kai serway , απογοήτευση. Ο ένας αντιγραφή από τον άλλον και το σχολικό αντιγραφή από αυτούς.Δεν καταλάβαινα πως ενώ έχουμε δυο σαφώς διαφορετικές μάζες στις περιοχές Β,Γ και ανάμεσα τους μια άλλη μαζα που κινείτε και αυτή παίρνουμε ΘΜΚΕ.Σκέφτηκα δεν είναι τα ρευστά για μένα.Μετά όμως τη παρατήρηση του Βαγγέλη πήρα στροφές και θάρρος σκέφτηκα ότι η απόδειξη τους σίγουρα έχει πρόβλημα. Μια απόδειξη μάλλον αν δεν κάνω λάθος ε'ιναι η εξής. Χωρίζουμε όλη τη μαζα ανάμεσα στα Β, Γ σε μικρές ισες στοιχειώδεις μάζες που ακουμπούν μεταξύ τους. Σημειώνουμε τις δυνάμεις αλληλεπίδρασης μεταξύ τους δράση αντιδραση. Παίρνουμε ΘΜΚΕ για κάθε μια προσέχοντας ότι τα έργα των δυνάμεων αυτών στις στοιχειώδεις μετατοπίσεις ειναι αντίθετα και οτι η τελική ταχύτητα κάθε μάζας είναι ίση με τη γειτονική της. Προσθέτουμε κατά μέλη και μετά τις απλοποιήσεις διαγραφές τέλος.
Σχόλιο από τον/την Διονύσης Μητρόπουλος στις 18 Ιούνιος 2015 στις 4:48
Διαγραφή σχολίου
Καλημέρα συνάδελφοι, Αν θέλετε να ρίξετε μια ματιά, ο νόμος Bernoulli λίγο πιο αναλυτικά, ΕΔΩ.
Σχόλιο από τον/την Κυριακόπουλος Γιάννης στις 18 Ιούνιος 2015 στις 8:33
Διαγραφή σχολίου
Συγχαρητήρια Μιχάλη.
Σχόλιο από τον/την Διονύσης Μάργαρης στις 18 Ιούνιος 2015 στις 8:50
Διαγραφή σχολίου
Μιχάλη και συ σαν "έτοιμος από καιρό"! Σε ευχαριστούμε που μοιράστηκες με όλους, τη δουλειά σου, που είναι φανερό, ότι έπεσε σαν την πρώτη βροχή στο έδαφος, μετά μια μακριά περίοδο άβροχου Καλοκαιριού... Να είσαι καλά.
Σχόλιο από τον/την Γιώργος Μαντάς στις 18 Ιούνιος 2015 στις 9:52
Διαγραφή σχολίου
Καλημέρα σας. Το σχήμα ελπίζω να βοηθήσει στο σημείο που υπάρχει το "πρόβλημα".
Σχόλιο από τον/την Βαγγέλης Κουντούρης στις 18 Ιούνιος 2015 στις 10:44
Διαγραφή σχολίου
Μπράβο Γιώργο για το "σαν", έγραψα ακριβώς το ίδιο και σε "διπλανή" συζήτηση, στην πραγματικότητα ούτε το υγρό που υπάρχει στο ενδιάμεσο τμήμα είναι το ίδιο, απλά οι μάζες και οι ενέργειες είναι ίσες
Σχόλιο από τον/την Κορφιάτης Ευάγγελος στις 18 Ιούνιος 2015 στις 11:36
Διαγραφή σχολίου
Καλημέρα σε όλους. Επομένως το θεώρημα έργου ενέργειας το εφαρμόζουμε στο γαλάζιο υγρό του Γιώργου που από την θέση του πάνω σχήματος πήγε στην θέση του κάτω. Τα δύο σχήματα νομίζω ότι ( και διδακτικά ξεκαθαρίζουν την κατάσταση). Τώρα είμαι εντάξει
Σχόλιο από τον/την Γιώργος Μαντάς στις 18 Ιούνιος 2015 στις 11:45
Διαγραφή σχολίου
Ναι Βαγγέλη (Κορ). Και κάτι πιο πλήρες εδώ.
Σχόλιο από τον/την Βαγγέλης Κουντούρης στις 18 Ιούνιος 2015 στις 12:04
Διαγραφή σχολίου
καλά ρε Γιώργο τον ...άλλο Βαγγέλη δεν τον είδες εκεί στα ψηλά;
Σχόλιο από τον/την Παπαδάκης Παντελεήμων στις 18 Ιούνιος 2015 στις 12:08
Διαγραφή σχολίου
Γιώργο δεν ανοίγει σε μένα, ούτε αυτό. Πάντως ευχαριστώ όλους που δίνουν ...στα ρευστά για να θυμηθούμε τα παροπλισμένα.
Σχόλιο από τον/την Γιώργος Μαντάς στις 18 Ιούνιος 2015 στις 12:13
Διαγραφή σχολίου
Έχεις απόλυτο δίκιο δάσκαλε. Με συγχωρείς, με απορρόφησε η όλη διαδικασία, για την οποία ακόμη δεν είμαι σίγουρος αν λειτουργεί. Πρωτάρης βλέπεις. Ένα μπράβο από εσένα είναι παράσημο, πόσω μάλλον που η αιτία για αυτό ήταν ότι πρόλαβα τη σκέψη σου.
Σχόλιο από τον/την Γιώργος Μαντάς στις 18 Ιούνιος 2015 στις 12:28
Διαγραφή σχολίου
Προσπαθώ άλλη μια φορά μήπως τα καταφέρω. Συγγνώμη για την αναστάτωση κάθε τρεις και λίγο... εδώ
Σχόλιο από τον/την Γιώργος Μαντάς στις 18 Ιούνιος 2015 στις 13:23
Διαγραφή σχολίου
Παντελή, ευχαριστώ για την πληροφόρηση, δεν πρόλαβα να σου μιλήσω πριν, γιατί ένας μικρός εξπέρ στη μηχανική ρευστών (νευτώνιων και μη, τσίσα, κακά, κρέμες και δάκρυα), απαιτούσε τη βοήθειά μου...
Σχόλιο από τον/την Παπαδάκης Παντελεήμων στις 18 Ιούνιος 2015 στις 13:31
Διαγραφή σχολίου
Ο.Κ Γιώργο άνοιξε! Το βλέπω
Σχόλιο από τον/την Μιχαήλ Μιχαήλ στις 18 Ιούνιος 2015 στις 18:07
Διαγραφή σχολίου
Γιάννη (Κυρ), Διονύση (Μαρ), σας ευχαριστώ για τα σχόλιά σας! Γιώργο (Μαν) πολύ κατατοπστικό το σχήμα. Μπράβο σου!
Σχόλιο από τον/την Γιώργος Μαντάς στις 18 Ιούνιος 2015 στις 18:13
Διαγραφή σχολίου
Ευχαριστώ Μιχάλη. Μέχρι το Σεπτέμβρη έχουμε όλο το χρόνο να βοηθήσουμε ο ένας τον άλλον να ξεκαθαρίσουμε δέκα πράγματα σχετικά με τα ρευστά, ώστε να μην μας λένε διάφοροι ότι τα μάθαμε την ώρα που τα διδάσκαμε...
Σχόλιο από τον/την Διονύσης Μητρόπουλος στις 18 Ιούνιος 2015 στις 18:16
Διαγραφή σχολίου
Μιχάλη καλησπέρα και πάλι, Παρέλειψα στο προηγούμενο σχόλιο (λόγω προχωρημένης ώρας) να σε συγχαρώ και να σ' ευχαριστήσω, για την "ετοιμότητά σου" και για την εργασία που μας προσφέρεις :-)
Σχόλιο από τον/την Μιχαήλ Μιχαήλ στις 18 Ιούνιος 2015 στις 21:17
Διαγραφή σχολίου
Διονύση (Μητρ) σ' ευχαριστώ πολύ. Μακάρι η ετοιμότητα του ylikonet να εμπνεύσει, και να βοηθήσει. Να' σαι πάντα καλά!
Σχόλιο από τον/την Βαγγέλης Κουντούρης στις 18 Ιούνιος 2015 στις 21:42
Διαγραφή σχολίου
καλή αρχή Μιχαήλ (κάλιο αργά...) Γιώργο σ' έχω μάρτυρα: δηλώνω ότι γνωρίζω τα ρευστά από παλιά, από το 2012 τουλάχιστον, ιδού και η απόδειξη... http://ylikonet.gr/group/ctaxi/forum/topics/3647795:Topic:307447?xg...  
Σχόλιο από τον/την Γιώργος Μαντάς στις 18 Ιούνιος 2015 στις 23:08
Διαγραφή σχολίου
Ωραία Βαγγέλη! Ό,τι χρειαζόμαστε, ξέρουμε που θα τρέχουμε!
Σχόλιο από τον/την Μιχαήλ Μιχαήλ στις 19 Ιούνιος 2015 στις 7:30
Διαγραφή σχολίου
Καλημέρα Βαγγέλη και σ' ευχαριστώ!
167. Κρούσεις δυο σφαιρών

A)  Δυο μικρές ελαστικές σφαίρες με μάζες m1 και m2, προσπίπτουν ταυτόχρονα στη μια πλευρά ενός οριζόντιου τραπεζιού και συγκρούονται ελαστικά και πλάγια όπως φαίνεται στο σχήμα. Οι ταχύτητες των δυο σφαιρών πριν την κρούση έχουν μέτρα υ12=4m/s και η γωνία πρόσπτωσης για την κάθε σφαίρα είναι 450. Στη συνέχεια οι σφαίρες συγκρούονται μεταξύ τους και η κρούση είναι πλάγια και ελαστική. Να υπολογιστούν οι ταχύτητες των δυο σφαιρών μετά τη μεταξύ τους σύγκρουση αν:

α)    m1= m2=0,5Kg και

β)    m1=0,5Kg και m2=1,5Kg (m2=3m1).

 

Β) Αν η ταχύτητα της m1 πριν την κρούση είναι υ1=5m/s και η γωνία πρόσπτωσης είναι θ με εφθ=4/3, ενώ η ταχύτητα της m2 πριν την κρούση είναι υ2=4m/s με γωνία πρόσπτωσης φ=450 τότε να υπολογιστούν οι ταχύτητες των δυο σφαιρών μετά τη μεταξύ τους σύγκρουση αν:

α)    m1= m2=0,5Kg και

β)    m1=0,5Kg και m2=1,5Kg (m2=3m1).

 

Συνοπτική λύση:

ή εδώ

169. Μη μετωπική ελαστική κρούση m1 και m2 .

Μια σφαίρα μάζας m1=5Kg κινείται (ολισθαίνει) οριζόντια με ταχύτητα υ1=1m/s και συγκρούεται μη μετωπικά και ελαστικά με αρχικά ακίνητη σφαίρα μάζας m2=7Kg. Αν μετά την κρούση η σφαίρα m2  κινείται με ταχύτητα υ2΄ που σχηματίζει με την οριζόντια διεύθυνση γωνία θ με εφθ=0,87,  τότε να υπολογίσετε την ταχύτητα υ1΄ καθώς και την ταχύτητα υ2΄ της m2, μετά την κρούση. Τριβές δεν υπάρχουν.

 

  Συνοπτική λύση:

ή εδώ

Σχόλια
Σχόλιο από τον/την Κορκίζογλου Πρόδρομος στις 12 Αύγουστος 2015 στις 13:50
Διαγραφή σχολίου
Μπράβο Μιχάλη για τη λύση μιας σχετικά γνωστής άσκησης, με τον τρόπο που έκανες. Κι εγώ τον ίδιο τρόπο χρησιμοποιώ, ανάλυση των ταχυτήτων στη διεύθυνση της διακέντρου και κάθετα προς αυτή, και για ελαστική κρούση, χρήση των έτοιμων τύπων του βιβλίου. Αν δεν δίνεται η γωνία κάποιας σφαίρας μετά την κρούση, αλλά μπορεί με τριγωνομετρία να υπολογισθεί η γωνία της αρχικής ταχύτητας με την διάκεντρο, ακόμη και για διαφορετικών ακτίνων σφαίρες, με τον ίδιο τρόπο αντιμετωπίζεται και υπολογίζονται και οι γωνίες κίνησης των κέντρων μαζών, με σύνθεση των ταχυτήτων μετά την κρούση. Φυσικά οι συνιστώσες ταχυτήτων που είναι κάθετες στη διάκεντρο, δεν συμμετέχουν στην κρούση. Να'σαι καλά.
Σχόλιο από τον/την Μιχαήλ Μιχαήλ στις 12 Αύγουστος 2015 στις 14:03
Διαγραφή σχολίου
Πρόδρομε σ' ευχαριστώ! Συγχαρητήρια για τις πολύ ωραίες ασκήσεις στη μηχανική συνεχών μέσων που μας δίνεις. Η αλήθεια είναι Πρόδρομε 'οτι δεν πρόλαβα να τις μελετήσω! Μόλις το κάνω θα επανέλθω με σχόλια!!! Η παραπάνω άσκηση είναι μια διαφορετική προσέγγιση (όπως και εσύ άλλωστε προτείνεις) μιας παλιότερης ανάρτησης! Πρόδρομε να' σαι πάντα καλά!
170. Εξαναγκασμένες ταλαντώσεις

170. Εξαναγκασμένες ταλαντώσεις

Σχόλια
Σχόλιο από τον/την Διονύσης Μάργαρης στις 28 Αύγουστος 2015 στις 8:12
Διαγραφή σχολίου
Καλημέρα Μιχαήλ. Πολύ καλή παρουσίαση που εξετάζει όλες τις δυνατές περιπτώσεις που μπορεί να συναντήσει κάποιος δουλεύοντας τις εξαναγκασμένες. Σε ευχαριστούμε.
Σχόλιο από τον/την Μιχαήλ Μιχαήλ στις 28 Αύγουστος 2015 στις 12:14
Διαγραφή σχολίου
Διονύση καλημέρα και σ' ευχαριστώ για το σχόλιο! Να΄σαι πάντα καλά!
Φυσική Α΄ Γυμνασίου

βιβλίο φυσικής Α΄ γυμνασίου

Σχόλια
Σχόλιο από τον/την Διονύσης Μάργαρης στις 4 Σεπτέμβριος 2015 στις 10:15
Διαγραφή σχολίου
Καλημέρα Μιχαήλ. Πολύ μεγάλη δουλειά! Σε ευχαριστώ που την μοιράστηκες μαζί μας και ελπίζω να φανεί χρήσιμη στους συναδέλφους που θα διδάξουν στην Α΄Γυμνασίου. Ξεφυλλίζοντας, διάβασα για τη μετατόπιση και τη δυνατότητα αρνητικής τιμής της και μου δημιουργήθηκε η απορία, αν αυτό μπορεί να διδαχτεί στην τάξη. Δεν νομίζω να έχουν διδαχτεί τους αρνητικούς αριθμούς στα μαθηματικά τους.
Σχόλιο από τον/την Μιχαήλ Μιχαήλ στις 4 Σεπτέμβριος 2015 στις 12:09
Διαγραφή σχολίου
Διονύση καλημέρα και σε ευχαριστώ για το σχόλιο! Τους αρνητικούς αριθμούς τους διδάσκονται στο τελευταίο κεφάλαιο της άλγεβρας (α΄ γυμν). Όμως τις ερωτήσεις της θεωρίας μπορεί ο κάθε συνάδελφος να αξιοποιήσει ανάλογα! Να΄σαι πάντα καλά!
Σχόλιο από τον/την Τίνα Νάντσου στις 6 Σεπτέμβριος 2015 στις 10:47
Διαγραφή σχολίου
Μπράβο! Ευχαριστώ πολύ για την προσφορά του βιβλίου σου!
Σχόλιο από τον/την ΚΩΝΣΤΑΝΤΙΝΟΣ ΞΥΝΟΣ στις 6 Σεπτέμβριος 2015 στις 14:20
Διαγραφή σχολίου
Καλημέρα Μιχάλη. Πιστεύω ότι θα είναι χρήσιμο σε πολλούς συναδέλφους. Εγώ πάντως θα το χρησιμοποιήσω σαν γονέας μαθητή α΄γυμνασίου και στην τάξη αν μου προκύψει.
Σχόλιο από τον/την Μιχαήλ Μιχαήλ στις 6 Σεπτέμβριος 2015 στις 20:07
Διαγραφή σχολίου
Τίνα σε ευχαριστώ! Κώστα, χαίρομαι που ανταμώνουμε έστω και διαδικτυακά μετά από τόσο καιρό! Σ' ευχαριστώ για το σχόλιο. Μακάρι να σου φανεί χρήσιμο το βιβλίο.
Φυσική Β΄ Γυμνασίου

Φυσική Β΄ Γυμνασίου

Σχόλια
Σχόλιο από τον/την Παπαδάκης Κώστας στις 8 Σεπτέμβριος 2015 στις 13:40
Διαγραφή σχολίου
Ωραία ιδέα, μπράβο. Απομένει η Γ γυμνασίου, Μιχαήλ και ολοκλήρωσες το γυμνάσιο.
Σχόλιο από τον/την Τίνα Νάντσου στις 8 Σεπτέμβριος 2015 στις 20:39
Διαγραφή σχολίου
Εξαιρετική δουλειά Μιχαήλ. Ευχαριστώ πολύ για ακόμη μία φορά. Μου αρέσει πολύ που έχεις στο τέλος και τις λύσεις τον ασκήσεων. Βοηθά πολύ τους μαθητές.
Σχόλιο από τον/την Μιχαήλ Μιχαήλ στις 8 Σεπτέμβριος 2015 στις 20:58
Διαγραφή σχολίου
Τίνα σ΄ευχαριστώ πολύ! Προσπαθώ να ετοιμάσω και τη γ΄ γυμνασίου. Να΄σαι πάντα καλά
Σχόλιο από τον/την Διονύσης Μάργαρης στις 9 Σεπτέμβριος 2015 στις 8:08
Διαγραφή σχολίου
Σε ευχαριστούμε Μιχαήλ, που το μοιράστηκες μαζί μας. Ακούραστος...
Σχόλιο από τον/την Μιχαήλ Μιχαήλ στις 9 Σεπτέμβριος 2015 στις 20:21
Διαγραφή σχολίου
Διονύση σ' ευχαριστώ για όλα όσα ακούσια και εκούσια έκανες για μένα !!
Φυσική γ΄ Γυμνασίου

Φυσική γ΄ Γυμνασίου

Σχόλια
Σχόλιο από τον/την Διονύσης Μάργαρης στις 10 Σεπτέμβριος 2015 στις 14:29
Διαγραφή σχολίου
Βλέπω Μιχαήλ ότι ολοκλήρωσες και τις τρεις τάξεις! Πολύ δουλειά... Σε ευχαριστώ που την μοιράστηκες.
Σχόλιο από τον/την Μιχαήλ Μιχαήλ στις 10 Σεπτέμβριος 2015 στις 17:48
Διαγραφή σχολίου
Διονύση σ' ευχαριστώ!
171. Διάδοση κύματος

171. Διάδοση κύματος

                

Η εξίσωση y(x,t)=f(x-υt), είναι η γενική εξίσωση που παριστάνει ένα κύμα οποιουδήποτε σχήματος που κινείται προς τον θετικό ημιάξονα +x.

 

Τη μορφή της διαταραχής, δηλαδή τη χωρική μεταβολή του κύματος, μπορούμε να τη «δούμε», αν φωτογραφήσουμε το κύμα σε μια ορισμένη χρονική στιγμή. Αυτό σημαίνει ότι θα  πρέπει να δώσουμε μια συγκεκριμένη τιμή π.χ  t= t1 στο χρόνο, οπότε έχουμε το στιγμιότυπο του κύματος που είναι μια συνάρτηση μόνο του x, για τη δεδομένη χρονική στιγμή δηλαδή  τη συνάρτηση f(x)= y(x,t1).

 

Η γραφική παράσταση (διάγραμμα) της f(x) τη δεδομένη χρονική στιγμή (έστω t1), ονομάζεται στιγμιότυπο του κύματος και έχει ένα ορισμένο σχήμα πάνω στη χορδή.

 

Tότε η f(x-υt) περιγράφει τη διάδοση αυτού το συγκεκριμένου σχήματος με ταχύτητα υ κατά τη θετική διεύθυνση x (υ>0).

 

  1. Έστω ότι η εξίσωση που περιγράφει το σχήμα – μορφή της διαταραχής τη χρονική στιγμή t=t1=0 είναι η y(x,0)=2-x2...............

συνέχεια

Σχόλια
Σχόλιο από τον/την Διονύσης Μάργαρης στις 26 Σεπτέμβριος 2015 στις 8:53
Διαγραφή σχολίου
Καλημέρα Μιχαήλ. Μου άρεσε η διαφύλαξη "των νότων" σου:-) "Βέβαια δεν ξέρω αν υπάρχει φυσικό σύστημα που να περιγράφεται με αυτή τη διαταραχή, γιατί δεν ξέρω αν υπάρχει φυσικό σύστημα που να πραγματοποιεί α.α.τ."
Σχόλιο από τον/την Μιχαήλ Μιχαήλ στις 26 Σεπτέμβριος 2015 στις 10:17
Διαγραφή σχολίου
Διονύση καλημέρα. Διονύση αναφέρεται πράγματι στη συζήτηση για τη μονοχρωματικότητα των κυμάτων.
Σχόλιο από τον/την Διονύσης Μάργαρης στις 26 Σεπτέμβριος 2015 στις 10:43
Διαγραφή σχολίου
Μιχαήλ, κατάλαβα γιατί το έγραψες... Αν ανατρέξεις σε παλιότερες τοποθετήσεις μου πάνω στο θέμα, θα δεις ότι επανειλημμένα έγραψα ότι στο σχολείο μελετάμε το «μαθηματικό κύμα», ότι πράγματι υπάρχει ένα πρόβλημα στο σημείο που φτάνει το κύμα, αφού δεν μπορεί να υπάρξει «ξαφνικά» μια μέγιστη ταχύτητα, από το μηδέν, σε μηδενικό χρόνο. Έχω μιλήσει πολλάκις για μια προσέγγιση που δεχόμαστε, την οποία προσωπικά αποδέχομαι σαν αποδεκτή προσέγγιση. Απλά δεν συμφώνησα ποτέ, ότι επειδή συμβαίνουν αυτά, θα πρέπει να μείνουμε μονότονα στη θέση ότι «όλα είναι λάθος» και γράφουμε βλακείες…  
Σχόλιο από τον/την Κυριακόπουλος Γιάννης στις 26 Σεπτέμβριος 2015 στις 11:11
Διαγραφή σχολίου
Διονύση μην μας διαφεύγει το ότι ο Μιχάλης είχε συμμετάσχει στην συζήτηση τότε και σχολίασε και το ανέφικτον της διάδοσης και την σκοπιμότητα. Η δική μου θέση τότε (παράδειγμα της οποίας σχολίασε ο Μιχάλης) ήταν ότι υλοποιήσιμος πεπερασμένος παλμός αναλύεται σε άθροισμα δύο (ή περισσοτέρων) πεπερασμένων προφανώς ανέφικτων παλμών (άπειρες επιταχύνσεις κ.λ.π.). Ως εκ τούτου οι παλμοί αυτοί καλώς μελετώνται έστω και αν δεν διαδίδονται. Το παράδειγμα: Εν κατακλείδι το γεγονός ότι δεν έχουμε διάδοση μονοχρωματικού παλμού δεν σημαίνει ότι γράφονται βλακείες. Το μαθηματικόν κύμα είναι ένα νοητικό εργαλείο που θα μας βοηθήσει να προσεγγίσουμε το πραγματικό. Οι άπειρες επιταχύνσεις που επιβάλλονται αίρονται από τις επίσης άπειρες επιταχύνσεις του άλλου μαθηματικού κύματος και το πραγματικόν κύμα καθίσταται και ρεαλιστικό και υλοποιήσιμο.
Σχόλιο από τον/την Διονύσης Μάργαρης στις 26 Σεπτέμβριος 2015 στις 11:15
Διαγραφή σχολίου
Το θυμάμαι Γιάννη...
Σχόλιο από τον/την Μιχαήλ Μιχαήλ στις 26 Σεπτέμβριος 2015 στις 11:51
Διαγραφή σχολίου
Διονύση συμφωνώ μαζί σου. Στα πλαίσια των προσεγγίσεων που κάνουμε στη φυσική θεωρώ ότι η παραπάνω εξίσωση μπορεί να είναι μια καλή προσέγγιση ενός φυσικού συστήματος. Πέραν του ότι οποιαδήποτε πολύπλοκη διαταραχή μπορεί να προέλθει από τη σύνθεση τέτοιων αρμονικών κυμάτων σαν αυτό που περιγράφει το 2ο παράδειγμα.
Σχόλιο από τον/την Μιχαήλ Μιχαήλ στις 26 Σεπτέμβριος 2015 στις 11:53
Διαγραφή σχολίου
Γιάννη συγχώρεσέ με, γιατί απάντησα στον Διονύση πριν δω το σχόλιό σου!!
172. Διαφορά φάσης και στιγμιότυπο κύματος.

Σημείο Μ του ελαστικού μέσου διάδοσης ενός κύματος βρίσκεται στη θέση xM και ταλαντώνεται με συχνότητα f=2Hz. Το Μ, έχει διαφορά φάσης κατά 2π rad μικρότερη από την «πηγή» (εφόσον το κύμα έχει «φτάσει» ήδη στο Μ).

 Α) Πόση πρέπει να γίνει η συχνότητα της πηγής ώστε η διαφορά φάσης του Μ με την «πηγή» Ο να γίνει:

i) 4π rad και

ii) π rad;

 Β) Να σχεδιάσετε τα στιγμιότυπα του κύματος τη χρονική στιγμή t=1 s για

      i)   Δφ=2π rad

      ii)  Δφ=4π rad και

     iii)  Δφ=π rad;

 

Δίνεται η ταχύτητα διάδοσης του κύματος υ=2m/s.

 

Συνοπτική λύση:

Σχόλιο
Σχόλιο από τον/την Γιάννης Μπατσαούρας στις 2 Νοέμβριος 2015 στις 10:17
Πολύ καλή δουλειά .. Πρώτη φορά βλέπω κάποιον να ξεκαθαρίζει τη σημαίνει το Δt στη σχέση Δφ=ωΔt όταν το Δφ εκφράζει τη διαφορά φάσης μεταξύ 2 σημείων.
173. Απλή αρμονική παλινδρομική κίνηση.

Υλικό σημείο πραγματοποιεί στον άξονα x΄x την παλινδρομική αρμονική κίνηση που περιγράφεται από την εξίσωση x=5ημ[2π(1+12t)t]  (S.I).

α) Ποια χρονική στιγμή η απομάκρυνση του σώματος γίνεται μέγιστη για 1η φορά;

β) Σε πόσο χρόνο η απομάκρυνση του σώματος γίνεται για πρώτη φορά x=m ενώ η ταχύτητα είναι αρνητική;

γ) Σε πόσο χρόνο πραγματοποιεί 1 «ταλάντωση»;  

 

δ) Πόσες φορές επαναλαμβάνεται η κίνηση του σώματος (υλικού σημείου) σε χρόνο t=1 s; Να γίνει η αντίστοιχη γραφική παράσταση x(t) για 0≤t≤1s.

 

Συνοπτική λύση:

174. Πιεστική δύναμη

Ένα κλειστό δοχείο έχει σχήμα κύβου ακμής α=0,4 m. Το μισό του δοχείου είναι γεμάτο με νερό κα το υπόλοιπο μισό με ελαιόλαδο. Nα υπολογίσετε τη συνολική δύναμη:

α) στον πυθμένα του δοχείου και

β) στην μια πλευρική του έδρα.

Δίνεται ρν=103 Kg/m3, ρλ=0,9∙103 Kg/m3 και g=10m/s2.

 

Συνοπτική λύση:

ή εδώ

175. Ταλάντωση και ρευστό.

Μανομετρικός σωλήνας ΑΒΓΔ σταθερής διατομής σχήματος ανεστραμμένου Π, ανοικτός στα δυο του άκρα, δένεται σε ελατήριο σταθεράς D=Κ=4Ν/m, όπως φαίνεται στο σχήμα. Το οριζόντιο τμήμα ΒΓ του σωλήνα έχει μήκος L=20cm. Ο σωλήνας περιέχει υγρό που αρχικά ισορροπεί και η ελεύθερη επιφάνεια του υγρού βρίσκεται στο ίδιο οριζόντιο επίπεδο στα δυο του μέρη ενώ απέχει από τα ελεύθερα άκρα του σωλήνα απόσταση Η=5cm. Απομακρύνουμε το σωλήνα από τη θέση ισορροπίας του και τον αφήνουμε ελεύθερο να κινηθεί ώστε να πραγματοποιεί α.α.τ. Να υπολογίσετε το μέγιστο πλάτος ώστε να μην υπερχειλίζει το νερό από το σωλήνα κατά τη διάρκεια της ταλάντωσης. Ο σωλήνας μπορεί να ολισθαίνει στο οριζόντιο επίπεδο χωρίς τριβές. Δίνεται η συνολική μάζα του συστήματος m=250 g και g=10m/s2.

 

Συνοπτική λύση:

Σχόλια
Σχόλιο από τον/την Βασίλης Δουκατζής στις 6 Νοέμβριος 2015 στις 0:33
Καλημέρα Μιχάλη! Το σύστημα που περιγράφεις δεν νομίζω να κάνει α.α.τ. Έχεις μετατόπιση φορτίου και όχι κάποιο στερεό σώμα. Την στιγμή που ακινητοποιείται το Π το υγρό έχει ακόμη ορμή με αποτέλεσμα να "χαλά" την ταλάντωση. Επίσης εξαιτίας της κρούσης των τοιχωμάτων του Π με το υγρό (στην αντιστροφή της κίνησης) μπορεί το υγρό να πεταχτεί σαν πίδακας.
Σχόλιο από τον/την Διονύσης Μάργαρης στις 6 Νοέμβριος 2015 στις 8:42
Καλημέρα και από μένα Μιχαήλ. Δύσκολη περίπτωση η παραπάνω και νομίζω ότι έχει δίκιο ο Βασίλης. Έστω ότι το δοχείο "ξεκινά" να εκτελεί αατ. Μα παρόμοια κίνηση πρέπει να εκτελέσει και το νερό, από μόνο του. Αν φανταστούμε δηλαδή ότι ανέρχεται λίγο το νερό στο ένα σκέλος του (με ακίνητο και σταθερό το δοχείο) θα εκτελούσε ταλάντωση. Δηλαδή εδώ τι έχουμε; Μια ταλάντωση του δοχείου και μια άλλη ταλάντωση του νερού; Μήπως έχουμε εξαναγκασμένη ταλάντωση του νερού; Μάλλον δεν μου κάθεται σωστή, λόγω μη σταθερής ταλάντωσης του δοχείου. Μήπως έχουμε σύζευξη ταλαντώσεων; Μήπως το νερό εκτελεί σύνθετη ταλάντωση; Τι από όλα αυτά μπορεί να γίνεται; Νομίζω ότι είναι ένα θέμα για σοβαρό προβληματισμό και καλό θα ήταν η ανάρτηση να μην περάσει απαρατήρητη. Μιχαήλ, μας έβαλες ...δύσκολα:-)
Σχόλιο από τον/την Μιχαήλ Μιχαήλ στις 6 Νοέμβριος 2015 στις 19:28
Βασίλη και Διονύση ευχαριστώ για το σχολιασμό σας και για τους προβληματισμούς που θέτετε. Μακάρι να δώσουν αφορμή για συζήτηση ώστε να εξαχθούν χρήσιμα συμπεράσματα. Το ερώτημα βέβαια που ήθελα να αναδείξω ήταν για ποια τιμή της αρχικής επιτάχυνσης άρα της αρχικής επιμήκυνσης του ελατηρίου έχουμε ή δεν έχουμε υπερχείλιση! και όχι το είδος της κίνησης του υγρού!! Υπερχείλιση έχουμε αν η επιτάχυνση του σωλήνα είναι μεγαλύτερη της α=2gH/L.
Σχόλιο από τον/την Ηλίας Χατζής στις 7 Νοέμβριος 2015 στις 15:43
Μάλλον δε θα κάνει Α.Α.Τ. διότι μετατοπίζεται το κέντρο μάζας του δοχείου και μάλιστα και πάνω - κάτω και αριστερά - δεξιά, αλλά το ερώτημα είναι πολύ ενδιαφέρον. Ας υποθέσουμε ότι το δοχείο (κι όχι το υγρό) εκτελεί ΕΞΑΝΑΓΚΑΣΜΈΝΗ Α.Α.Τ., κολλημένο π.χ. σε μία ρομποτική πλατφόρμα. Πως συμπεριφέρεται τότε το υγρό;
176. Επιταχυνόμενη κίνηση σωλήνα με υγρό.

Μανομετρικός σωλήνας ΑΒΓΔ σταθερής διατομής σχήματος ανεστραμμένου Π, είναι ανοικτός στα δυο του άκρα. Το οριζόντιο τμήμα ΒΓ του σωλήνα έχει μήκος L=20cm. Ο σωλήνας περιέχει υγρό που αρχικά ισορροπεί και η ελεύθερη επιφάνεια του υγρού βρίσκεται στο ίδιο οριζόντιο επίπεδο στα δυο του μέρη.

 

α) Να υπολογιστεί η διαφορά ύψους του υγρού στις δυο στήλες αν ο σωλήνας αρχίσει να επιταχύνεται προς τα αριστερά με επιτάχυνση α=10m/s2.

 

β) Να υπολογίσετε την γωνιακή ταχύτητα ω, με την οποία πρέπει να περιστρέφεται ο παραπάνω σωλήνας, γύρω από κατακόρυφο άξονα που συμπίπτει με έναν από τους κατακόρυφους σωλήνες, ώστε να πετύχουμε την ίδια διαφορά ύψους του υγρού στις δυο στήλες.

 

Δίνεται g=10m/s2.

Συνοπτική λύση:

Σχόλια
Σχόλιο από τον/την Μιχαήλ Μιχαήλ στις 21 Νοέμβριος 2015 στις 13:24
Παρόμοια άσκηση υπάρχει στο βιβλίο των Sears και Zemansky.
Σχόλιο από τον/την Γιάννης Μπατσαούρας στις 21 Νοέμβριος 2015 στις 13:37
Καλησπέρα Μιχάλη. Συγκοινονούντα δοχεία σε βαρυτικό πεδίο έντασης g'.
Σχόλιο από τον/την Μιχαήλ Μιχαήλ στις 21 Νοέμβριος 2015 στις 14:50
Καλησπέρα Γιάννη! Θα μπορούσε να είναι και αυτό... !
Σχόλιο από τον/την Γιάννης Μπατσαούρας στις 21 Νοέμβριος 2015 στις 14:58
Μάλλον δεν κατάλαβες τι εννοώ . .. Η επιτάχυνση α ισοδυναμει με βαρυτικό πεδίο -α . Αρα το συνολικό βαρυτικό πεδίο θα έχει ένταση g'=gV2 και η ελεύθερη επιφάνεια του υγρού θα είναι κάθετη στην ένταση του βαρυτικού πεδίου έντασης g'. ..Με απλή γεωμετρία βρίσκουμε τη διαφορά ύψους στα 2 σκέλη .
Σχόλιο από τον/την Μιχαήλ Μιχαήλ στις 21 Νοέμβριος 2015 στις 18:11
Ναι Γιάννη δεν το πολυέψαξα. Εννοείς εφφ=α/g και εφφ=Δy/L άρα Δy=αL/g
Σχόλιο από τον/την Γιάννης Μπατσαούρας στις 21 Νοέμβριος 2015 στις 18:17
Ναι Μιχάλη , απλά το πρώτο σχόλιο το έγραψα σε χρόνο dt .
Σχόλιο από τον/την Μιχαήλ Μιχαήλ στις 22 Νοέμβριος 2015 στις 18:05
Γιάννη πρόσθεσα και τη δική σου λύση
Σχόλιο από τον/την Γιάννης Μπατσαούρας στις 22 Νοέμβριος 2015 στις 18:08
Σε ευχαριστώ Μιχαήλ να είσαι καλά .
Σχόλιο από τον/την Εμμανουήλ Λαμπράκης στις 23 Νοέμβριος 2015 στις 22:13
Μιχαήλ καλησπέρα Πολύ ωραίο θέμα. Πολύ ενδιαφέρουσα και η λύση του Γιάννη.
Σχόλιο από τον/την Μιχαήλ Μιχαήλ στις 24 Νοέμβριος 2015 στις 16:55
Μανώλη σε ευχαριστώ! Κι εμένα η λύση του Γιάννη μ' εντυπωσίασε.
177. Εξίσωση Torricelli.

Κυλινδρικό κατακόρυφο δοχείο εμβαδού Α1 περιέχει υγρό πυκνότητας ρ και ηρεμεί σε οριζόντιο δάπεδο. Το δοχείο φράσσεται υδατοστεγώς με έμβολο που μπορεί να κινείται κατακόρυφα χωρίς τριβές. Στην πλευρική επιφάνεια του δοχείου και σε απόσταση Η κάτω από την ελεύθερη οριζόντια επιφάνειά του βρίσκεται στόμιο εμβαδού Α2 από το οποίο εκρέει το υγρό. Αν ασκήσουμε στο έμβολο κατακόρυφη δύναμη F τότε το υγρό εκρέει με ταχύτητα υ2, όταν η στάθμη του υγρού απέχει απόσταση h από το στόμιο εκροής.

Να υπολογίσετε:

α) Την ταχύτητα εκροής υ2 του υγρού

β) το μέτρο του ρυθμού μεταβολής του ύψους της στάθμης του νερού και

γ) το χρόνο για να κατέβει η στάθμη του υγρού κατά H και να σταματήσει η εκροή του υγρού.

 

Δίνεται η επιτάχυνση της βαρύτητας g.

 

Συνοπτική λύση:

Σχόλια

Ετικέτες: ρευστά, τάξη-γ

Σχόλιο από τον/την Γιάννης Μπατσαούρας στις 6 Δεκέμβριος 2015 στις 21:28
Πάρα πολύ καλή Μιχαήλ. Συγχαρητήρια .
Σχόλιο από τον/την Cv.Engineer στις 6 Δεκέμβριος 2015 στις 22:42
ο μη μονιμος ορος της bernoulli τωρα ειναι ασημαντος ομως?
Σχόλιο από τον/την Μιχαήλ Μιχαήλ στις 7 Δεκέμβριος 2015 στις 9:37
Γιάννη καλημέρα και σ' ευχαριστώ. Αγαπητέ Εκ. αν μπορείς κάνε λίγο πιο σαφές το ερώτημά σου!
Σχόλιο από τον/την Cv.Engineer στις 7 Δεκέμβριος 2015 στις 14:18
η ροη δεν ειναι μονιμη (το θεωρημα toricelli χωρις το εμβολο απο πανω ισχυει με πολυ καλη προσεγγιση αφου ο μη μνοιμςο ορος της bernoulli τεινει στο μηδεν ).Μηπως τωρα που ασκουμε δυναμη στο εμβολο αυξανουμε αρκετα τον ορο που ειναι μεσα στο ολοκληρωμα και δεν γινεται αμελητεο?Θα το κοιταξω λιγο και θα δω
Σχόλιο από τον/την Σαράμπαλης Κωνσταντίνος στις 30 Δεκέμβριος 2015 στις 20:57
Συνάδελφε Μιχάλη, έστω και ετεροχρονισμένα (δεν σας προλαβαίνω να σας διαβάζω στο δίκτυο) πολύ ωραίο θέμα και ειδικά η λύση σου στο τελευταίο ερώτημα (με την έξυπνη αποφυγή ολοκληρωμάτων). Νάσαι καλά.
Σχόλιο από τον/την Μιχαήλ Μιχαήλ στις 31 Δεκέμβριος 2015 στις 20:18
Κωνσταντίνε σ' ευχαριστώ πολύ! Σου εύχομαι Χρόνια Πολλά και Καλή Χρονιά!
178. Συμβολή κυμάτων

Δύο σύγχρονες σημειακές πηγές Π1 και Π2 δημιουργούν στην επιφάνεια υγρού εγκάρσια κύματα που διαδίδονται με ταχύτητα υ = 5 m/s.

Τη χρονική στιγμή t=1,4s εμφανίζονται οι ΜΜ΄ και ΝΝ΄ που είναι οι δυο πρώτες υπερβολές ενισχυτικής συμβολής μετά τη μεσοκάθετο του Π1Π2.

 Οι πηγές αρχίζουν να ταλαντώνονται τη χρονική στιγμή t = 0 και εκτελούν ταλαντώσεις της μορφής y = Α∙ημωt .

α) Αν η χρονική διάρκεια εμφάνισης της ΜΜ΄ είναι 0,6s και της ΝΝ΄ είναι 0,4s τότε να βρείτε τις αποστάσεις  r1 και  r2 του σημείου Μ καθώς και τις αποστάσεις r1΄ και  r2΄ του σημείου Ν από τις πηγές Π1 και Π2, αντίστοιχα.

 β) Να υπολογίσετε τη διαφορά φάσης των Μ και Ν μετά τη συμβολή των κυμάτων.

 γ) Να γράψετε τις εξισώσεις των υπερβολών στις οποίες ανήκουν τα Μ και Ν θεωρώντας ορθογώνιο σύστημα συντεταγμένων (xoy) με αρχή το Ο και άξονα xx΄ που ταυτίζεται με την ευθεία Π1Π2.  

 δ) Να σχεδιάσετε τη γραφική παράσταση του πλάτους ταλάντωσης των Μ και Ν σε συνάρτηση με τον χρόνο, για t ≥ 0.

Συνοπτική λύση:

ή εδώ

181. Δύναμη στην παλάμη

 Η παροχή της βρύσης του σχήματος είναι Π=10-3m3/s . Η φλέβα νερού χτυπάει την οριζόντια παλάμη του χεριού μας, ώστε μετά την πρόσπτωση το νερό να κινείται παράλληλα προς την παλάμη και αμέσως να την εγκαταλείπει. Αν η φλέβα νερού «πέφτει» από ύψος h=0,25m, να υπολογιστεί η δύναμη που ασκείται από αυτή στην παλάμη. Δίνεται το εμβαδό της εγκάρσιας διατομής της βρύσης Α1=5cm2, η πυκνότητα του νερού ρ=103Κg/m3 και g=10m/s2.

Συνοπτική λύση:

183. Το καλαμάκι του φραπέ

Ένα καλαμάκι του φραπέ, έχει μήκος L και μάζα Μ.

α) Να υπολογιστεί η ροπή αδράνειας του, ως προς άξονα που περνά από το ένα άκρο του.

β) Στη συνέχεια το καλαμάκι κάμπτεται ώστε ένα κομμάτι του α= να σχηματίζει γωνία 900 με το υπόλοιπο καλαμάκι, ώστε να μπορούμε να πιούμε το φραπέ μας.

Μικραίνει ή αυξάνεται η ροπή αδράνειας που έχει το καλαμάκι ως προς άξονα περιστροφής που περνάει από το άκαμπτο άκρο του και είναι κάθετος στο επίπεδο περιστροφής;

Πόση είναι η ροπή αδράνειάς του ως προς το άκρο του αυτό;

Θεωρούμε ότι η διάμετρος από το καλαμάκι είναι πολύ μικρότερη από το μήκος του L ώστε αυτό να θεωρείται λεπτή ομογενής ράβδος με Ιcm=1/12ΜL2.

γ. Αν κάποια στιγμή αρχίσουμε να πίνουμε τον καφέ μας ποια θα είναι η μέγιστη ταχύτητα εκροής του frappe από το καλαμάκι; Δίνεται Ρατμ=105Ν/m2 και η πυκνότητα του καφέ για τις πράξεις να θεωρηθεί ίση με ρ=103Κg/m3.

Συνοπτική λύση:

Σχόλια
Σχόλιο από τον/την Κυριακόπουλος Γιάννης στις 4 Φεβρουάριος 2016 στις 19:05
Πολύ χαριτωμένη Μιχάλη. Συνήθως είμαι επιφυλακτικός με τις συνδυαστικές ασκήσεις αλλά αυτή έχει χάρη. Θα μπορούσε να αποτελέσει θέμα χωρίς "θύματα".
Σχόλιο από τον/την Μιχαήλ Μιχαήλ στις 4 Φεβρουάριος 2016 στις 22:43
Γιάννη σε ευχαριστώ!
Σχόλιο από τον/την Παπαδάκης Παντελεήμων στις 5 Φεβρουάριος 2016 στις 9:36
Καλημέρα Μιχαήλ και ευχαριστούμε για τον ωραίο φραπέ. Ο Γιάννης είπε ..’’χωρίς θύματα’’ μόνο που εγώ λύνοντας, χωρίς να δω τη λύση σου, έψαχνα τη πίεση ΡΑ και βρήκα πως πρέπει να είναι μηδέν αφού έλυσα ως προς υΑ=√ 2(ΡatΑ)/ρ οπότε για ΡΑ=0 έχουμε υΑ ,max=√ 2Ρat/ρ. Την τεκμηρίωσή σου …ότι ΡΑ=0 από πριν τη θεωρώ δύσκολη σύλληψη με την έννοια της φυσικής (όχι μαθηματικής) ερμηνείας .
Σχόλιο από τον/την Μιχαήλ Μιχαήλ στις 6 Φεβρουάριος 2016 στις 22:40
Παντελεήμων σε ευχαριστώ και εγώ για το σχόλιο! θεώρησα ότι δεν υπάρχει αέρας οπότε PA=0...
184. Πτώση στάθμης

Κυλινδρικό κατακόρυφο δοχείο εμβαδού βάσης Α περιέχει νερό και ηρεμεί σε οριζόντιο δάπεδο. Το δοχείο γεμίζει με νερό μέχρι ύψους x0, πάνω από το στόμιο εκροής,  εμβαδού σ.

 Να υπολογίσετε:

α) την ποσότητα του νερού που εκρέει σε χρόνο Δt, τη στιγμή που η στάθμη του νερού βρίσκεται σε ύψος x πάνω από το στόμιο εκροής.

β) την πτώση στάθμης του νερού στο δοχείο σε χρόνο Δt και

γ) το χρόνο για να κατέβει η στάθμη του νερού κατά x0 και να φτάσει στο ίδιο    οριζόντιο επίπεδο που βρίσκεται και το στόμιο εκροής.

 

Δίνεται η επιτάχυνση της βαρύτητας g.

 

Συνοπτική λύση:

Σχόλια < Προηγούμενη καταχώριση Επόμενη καταχώριση > Σχόλιο από τον/την Κυριακόπουλος Γιάννης στις 10 Φεβρουάριος 2016 στις 23:38 Διαγραφή σχολίου Να διορθώσω κάτι: Σχόλιο από τον/την Γκενές Δημήτρης στις 11 Φεβρουάριος 2016 στις 0:56 Διαγραφή σχολίου Μιχαήλ συγχαρητήρια . Ο τρόπος που αντιμετώπισες το θέμα ... αναδεικνύει πολλά κρυμμένα μυστικά κρατείται προς προσεκτικότερη μελέτη και εξαγωγή συμπερασμάτων για τον τρόπο που προσεγγίζουμε τη φύση. όσο για την αντιστοιχία με την επιβραδυνόμενη ... εγώ πριν 3 χρόνια και ... αποδείχτηκε ότι ήμουν πολύ επιδερμικός και βέβαια καθόλου επίκαιρος όταν έγραφα το 3 από αυτήν την ανάρτηση Σχόλιο από τον/την Διονύσης Μάργαρης στις 11 Φεβρουάριος 2016 στις 8:48 Διαγραφή σχολίου Καλημέρα Δημήτρη. Γιατί χαρακτηρίζεις την δική σου "ανεπίκαιρη" ανάρτηση, επιδερμική; Τα είχες πει όλα, άλλο αν εμείς δεν τους είχαμε δώσει την προσοχή που τους άξιζε... Σχόλιο από τον/την Μιχαήλ Μιχαήλ στις 11 Φεβρουάριος 2016 στις 21:32 Διαγραφή σχολίου Χρήστο σε ευχαριστώ για το σχόλιο. Στην 185 προσπάθησα λίγο να περιορίσω την έκταση της άσκησης! Σχόλιο από τον/την Μιχαήλ Μιχαήλ στις 11 Φεβρουάριος 2016 στις 21:34 Διαγραφή σχολίου Δημήτρη σε ευχαριστώ πολύ! Θα συμφωνήσω όμως με τον Διονύση! Νομίζω ότι τα είχες πει όλα και μάλιστα καθόλου επιδερμικά. Ας είναι όμως και μια ακόμη υπενθύμιση... η δική μου! Σχόλιο από τον/την Νεκτάριος Πρωτοπαπάς στις 12 Φεβρουάριος 2016 στις 0:03 Διαγραφή σχολίου Καλησπέρα Μιχαήλ, τα χουν πει όλα οι συνάδελφοι, θα ταν όμως άδικο να μην σου εκφράσω και εγώ τα εύσημα για το θέμα σου! Σχόλιο από τον/την Μιχαήλ Μιχαήλ στις 12 Φεβρουάριος 2016 στις 12:42 Διαγραφή σχολίου Νεκτάριε σε ευχαριστώ!! Σχόλιο από τον/την Λεβέτας Στάθης στις 16 Φεβρουάριος 2016 στις 20:58 Διαγραφή σχολίου Maria perpou ελπίζω να βοηθήσει το παρακάτω αρχείο. Νομίζω ότι η προσέγγιση του Μιχαήλ Μιχαήλ είναι βάσιμη, όπως δείχνουν τα νούμερα, στην προσέγγιση Bernoulli. Δεξαμενή Σχόλιο από τον/την Βασίλης Δουκατζής στις 16 Φεβρουάριος 2016 στις 22:44 Διαγραφή σχολίου Συνάδελφε η Μαρία Πέρπου δεν είναι πια μέλος του δικτύου, οπότε δεν μπορεί να σου απαντήσει! Σχόλιο από τον/την Pantelis Lapas στις 5 Μάρτιος 2016 στις 11:23 Διαγραφή σχολίου Έχω μια ένσταση σε αυτά τα θέματα αδειάσματος δοχείων η οποία έχει να κάνει με το εξής: σε μία πιο αναλυτική μελέτη που έδωσε κάποιος συνάδελφος χρησιμοποιώντας διαφορικές εξισώσεις, μέσα από μία λογική ακολουθία πράξεων (η οποία θυμίζει την Νευτώνεια θεωρία των συστημάτων μεταβλητής μάζας) και με βάση τις ίδιες παραδοχές με αυτές που διέπουν την ισχύ της εξίσωσης του Bernoulli πλήν αυτής της steady state (ημιστατικής κατάστασης), διαπιστώθηκε ότι η ελεύθερη επιφάνεια του υγρού μέσα στο δοχείο αρχικώς επιταχύνεται ερχόμενη όμως γρήγορα σε μια ημιστατική κατάσταση κατά την οποίαν κινείται κατερχόμενη με τόσο μικρή ταχύτητα που, στο όριο που η διατομή του δοχείου προς τη διατομή της οπής τείνει στο μηδέν (και ίσως και λίγο πιο χαλαρά από αυτό το όριο ... αλλά λίγο μόνο) δικαιολογεί την προσέγγιση μηδενισμού της ταχύτητας καθόδου της ελεύθερης επιφάνειας που γίνεται σε ασκήσεις με εφαρμογή του νόμου του Bernoulli. Η ανάλυση εκείνη όμως προέβλεπε αρχική ασυμφωνία με το νόμο του Bernoulli (για το κομμάτι της επιταχυνόμενης καθόδου) αλλά από τη στιγμή που προσεγγίζεται η ημιστατική κατάσταση και μετά μέχρι και το πέρας του αδειάσματος το συμπέρασμα ήταν ότι οι δύο προσεγγίσεις ταυτίζονται φοβερά. Διαισθητικά μιλώντας, αρχικά, μόλις δηλαδή ανοίγει η οπή και εκρέει το υγρό αλλά και τελικά, όταν δηλαδή τείνει να στραγγίξει η δεξαμενή, δεν φαίνεται λογικό να είναι σωστή η εφαρμογή της θεωρίας του Bernoulli ... από την κοινή εμπειρία νομίζω μπορεί κανείς εύκολα να παρατηρήσει ότι όταν αδειάζει ένα δοχείο, όταν η στάθμη πλησιάζει αρκετά κοντά (σε ύψος) στην οπή, τα μόρια του υγρού δεν έχουν μηδενική ταχύτητα ή δεν φαίνεται να κάνουν κάποια στρωτή κίνηση, αντίθετα φαίνεται να επιταχύνονται, κινούμενα "άτακτα και γρήγορα", ενδεχομένως και υπό την βοήθεια των δυνάμεων επαφής με τα τοιχώματα του δοχείου, προς την οπή. Αυτό το φαινόμενο αναμένεται να είναι υπερβολικά σύντομης διάρκειας (ίσως και αμελητέας) στο όριο όπου η διατομή της δεξαμενής προς τη διατομή της οπής τείνει στο μηδέν, και προφανώς σε ένα τέτοιο όριο η προσέγγιση της μηδενικής ταχύτητας για την κατερχόμενη στάθμη και ενδεχομένως και η προσέγγιση της εξίσωσης Bernoulli "επιβιώνουν περισσότερο" (αν και όχι κατ' ανάγκη το ίδιο χρονικό διάστημα) ... "σαν να καλύπτουν" όλη τη διάρκεια του αδειάσματος! Στην πράξη όμως αν δεν ισχύει το προαναφερθέν όριο διατομών μπορεί εύκολα να παρατηρήσει κανείς αυτό που λέω περί παραβίασης της ημιστατικής κατάστασης λίγο πριν το άδειασμα παραπάνω. Ο άλλος λόγος που ενδεχομένως δεν είναι καλή η προσέγγιση της μηδενικής ταχύτητας της κατερχόμενης στάθμης (η οποία είναι ανεξάρτητη από την εφαρμογή του νόμου του Bernoulli που είναι κι αυτή μια προσέγγιση στο τέλος τέλος) είναι ότι όταν η στάθμη πλησιάζει "επικίνδυνα κοντά" στην οπή (σε ύψος) τι ταχύτητα έχουμε μηδέν ή την ταχύτητα εκροής; Το πιο λογικό είναι οι ταχύτητες στις δύο θέσεις να τείνουν να συγκλίνουν μεταξύ τους καθώς ολοκληρώνεται το άδειασμα. Τέλος πάντων, η βασική απορία που έχω είναι πώς γίνεται να ταιριάζουν τόσο καλά ο νόμος του Bernoulli με την ανάλυση που έδωσε ο κ. Κορφιάτης ακόμη και στο τελικό στάδιο του αδειάσματος παρόλο που δεν έχουμε ημιστατική (ή στρωτή) ροή που είναι απαραίτητη προϋπόθεση για την εφαρμογή του τύπου του Bernoulli; Ενδεχομένως για εκείνο το τελικό στάδιο του αδειάσματος καμία από τις δύο προσεγγίσεις να μην είναι σωστή (αλλιώς πώς συμφωνούν τόσο τέλεια όσο φαίνεται στα σχήματα εκείνης της ανάλυσης; ή ... μία άλλη απάντηση θα ήταν ότι μπορεί να πει κανείς ότι αν ο λόγος των διατομών δεν είναι υπερβολικά μικρός δεν ταιριάζουν καλά οι δύο προσεγγίσεις και σωστή είναι εκείνη της ανάλυσης του Κορφιάτη, αν όμως εξακολοθούν να ταιριάζουν καλά και πέραν της προσέγγισης του ορίου των διατομών τότε και οι δύο δεν είναι μάλλον σωστές κατά το τελικό στάδιο του αδειάσματος), ... τι από όλα αυτά ισχύει; ... όποιος είναι ειδικός επί τέτοιων θεμάτων ας ρίξει λίγο φώς! 188. Κύλινδρος και ιξώδες.

Η αρχικά ακίνητη διάταξη του σχήματος αποτελείται από δυο ομογενείς κυλινδρικές ομοαξονικές επιφάνειες ακτινών R=20cm και r=19,8cm και μήκους L=1/2π m. Το κενό μεταξύ των δυο κυλινδρικών επιφανειών είναι γεμάτο με μηχανέλαιο συντελεστού ιξώδους n=0,25N∙s/m2.

Ο εξωτερικός ομογενής κύλινδρος ακτίνας R, έχει τυλιγμένο γύρω του αβαρές μη εκτατό σχοινί και μπορεί να περιστρέφεται, ενώ ο εσωτερικός κύλινδρος είναι συνεχώς ακίνητος. Κάποια χρονική στιγμή (t0=0), αρχίζουμε να ξετυλίγουμε το σχοινί ασκώντας σταθερή δύναμη F, στο ελεύθερο άκρο του σχοινιού. Αρχικά διαπιστώνουμε ότι ο εξωτερικός κύλινδρος επιταχύνεται ενώ τη χρονική στιγμή t1 που έχει ξετυλιχθεί σχοινί μήκους d=0,32m, ο εξωτερικός κύλινδρος αποκτά σταθερή γωνιακή ταχύτητα ω=5 rad/s με την οποία και συνεχίζει να περιστρέφεται. Τότε να βρείτε:

 

α) την ταχύτητα του ελεύθερου άκρου του σχοινιού τη χρονική στιγμή t1.

 

β) Το μέτρο της σταθερής δύναμης F.

 

γ) Το έργο της δύναμης τριβής T που ασκεί το νευτώνειο υγρό στην εξωτερική κυλινδρική επιφάνεια από τη χρονική στιγμή t0 μέχρι τη χρονική στιγμή t1.

 

δ) Το ρυθμό μεταβολής της κινητικής ενέργειας του εξωτερικού κυλίνδρου:

i) τη χρονική στιγμή που περιστρέφεται με ω1=ω/2και

ii) τη χρονική στιγμή t1.

 Δίνεται η ροπή αδράνειας του εξωτερικού κυλίνδρου Ι=8∙10-2Κg∙m2 και g=10m/s2.

 

Συνοπτική λύση:

Σχόλια
Σχόλιο από τον/την Μιχαήλ Μιχαήλ στις 1 Μάρτιος 2016 στις 13:54
Αφιερώνεται στον Διονύση (Μαργ) και στον Βαγγέλη (Κορφ)
Σχόλιο από τον/την Διονύσης Μάργαρης στις 3 Μάρτιος 2016 στις 8:52
Καλημέρα Μιχαήλ. Σε ευχαριστώ για την αφιέρωση της ανάρτησης. Πολύ πρωτότυπο θέμα, που συνδυάζει σε ουσιαστική βάση, δύο κεφάλαια! Να είσαι καλά.
189. …αποκριάτικο μπαλόνι

Το αποκριάτικο μπαλόνι του σχήματος διατομής Α=2∙10-2m2 είναι γεμάτο με αέρα πυκνότητας ρ=1,2Κg/m3 με πίεση Ρ=1,24 atm. Το ακροφύσιο στη βάση του μπαλονιού έχει διατομή σ=2∙10-3m2. Κάποια στιγμή αφήνουμε ελεύθερο τον αέρα να διαφύγει από το ακροφύσιο.

α) Να υπολογιστεί η ταχύτητα με την οποία διαφεύγει ο αέρας μέσα από το ακροφύσιο του μπαλονιού και

β) Να υπολογιστεί η δύναμη που προωθεί το μπαλόνι.

 

Δίνεται η ατμοσφαιρική πίεση Ρατ=105 Ν/m2.

 

Συνοπτική λύση:

Σχόλια
Σχόλιο από τον/την Μιχαήλ Μιχαήλ στις 3 Μάρτιος 2016 στις 20:45
Διαγραφή σχολίου
ΚΑΛΕΣ ΑΠΟΚΡΙΕΣ ΚΑΙ ΚΑΛΗ ΣΑΡΑΚΟΣΤΗ...
Σχόλιο από τον/την Κορκίζογλου Πρόδρομος στις 3 Μάρτιος 2016 στις 22:58
Διαγραφή σχολίου
Καλησπέρα Μιχάλη και καλή Σαρακοστή και σε σένα! Κάτι σχετικό είχα στο πρόσφατο διαγώνισμά μου στα ρευστά, με την ταχύτητα εκροής αέρα από μια μικρή τρύπα σε διαστημικό σταθμό, και την "" είχα πατήσει"" βάζοντας την μέση ταχύτητα λόγω θερμικής κίνησης των μορίων του αέρα! Η διεύθυνδη Εδώ . Ευτυχώς το είδε και Γ. Γεωργαντάς και ο Διονύσης Μάργαρης ,και διόρθωσα τη λύση. Νομίζω ότι όταν εφαρμόζουμε Bernoulli στα αέρια,θεωρούμε μηδέν την ταχύτητα μακριά από την οπή στο εσωτερικό του δοχείου. Εδώ ,εσύ παίρνεις την εξίσωση της συνέχειας, θεωρώντας τη διατομή του μπαλονιού Α, σε ένα σχήμα που είναι οβάλ!! Δεν μικραίνει ο όγκος του μπαλονιού από όλες τις μεριές; Θα μπορούσες να πάρεις δοχείο με ανένδοτα τοιχώματα και μηδέν την ταχύτητα του αέρα μακριά από την οπή, δηλαδή: P+0+0=Pατμ.+(1/2)ρυ^2 . Μετά μπορείς να υπολογίσεις την προωστική δύναμη .Να δούμε τί θα πούν κι οι άλλοι συνσδελφοι. Καλό βράδυ.
Σχόλιο από τον/την Μιχαήλ Μιχαήλ στις 3 Μάρτιος 2016 στις 23:49
Διαγραφή σχολίου
Πρόδρομε ήθελα να έχω μια εκτίμηση της αρχικής ταχύτητας εκροής του αέρα! Θα με ενδιέφερε η δικιά σου εκτίμηση
Σχόλιο από τον/την Μιχαήλ Μιχαήλ στις 4 Μάρτιος 2016 στις 7:21
Διαγραφή σχολίου
Πρόδρομε καλημέρα και σε ευχαριστώ πολύ!
Σχόλιο από τον/την Κορκίζογλου Πρόδρομος στις 4 Μάρτιος 2016 στις 7:30
Διαγραφή σχολίου
Καλημέρα Μιχάλη. Από τη σχέση που έγραψα παραπάνω, βρίσκω την αρχική ταχύτητα εκροής προσεγγιστικά, υ=√2(P-Pατμ)/ρ=√2•0,24•10^5/1,2=200m/s. Για την προωστική δύναμη θα έκανα: F=dΡ/dt=d(mu)/dt=(dm/dt)u+dm•(du/dt)=(dm/dt)u+0=ρ(dV/dt)•u=ρ•Π•u=ρ•σ•u•u=ρ•σ•u²=
=1,2•2•(10)-³•4•(10)⁴=96N.
Καλύτερα, αν αλλάξεις το σχήμα του μπαλονιού(μακρύ), σε σχήμα κυλινδρικό που να καταλήγει στο ακροφύσιο με κωνικό σχήμα, κάτι σαν χωνί, και να κάνεις τη λύση που έκανες, για μένα ουδέν το μεμπτόν!! Κοντολογίς, άλλαξε το σχήμα, και κράτησε τη λύση που έκανες.
Σχόλιο από τον/την Διονύσης Μάργαρης στις 4 Μάρτιος 2016 στις 9:03
Διαγραφή σχολίου
Καλημέρα και καλές απόκριες Μιχαήλ. Η λύση σου με την ταχύτητα στο εσωτερικό του δοχείου, μου δίνει την εξής εικόνα: Υπάρχει μια ελεύθερη επιφάνεια αέρα μέσα στο μπαλόνι που κατέρχεται με κάποια σταθερή ταχύτητα υ1. Νομίζω ότι την έγραψες σε αναλογία με τα υγρά, πράγμα που δεν θεωρώ ότι πλησιάζει στην πραγματικότητα.
Σχόλιο από τον/την Μιχαήλ Μιχαήλ στις 4 Μάρτιος 2016 στις 12:04
Διαγραφή σχολίου
Διονύση καλημέρα και ευχαριστώ για το σχόλιο! Δεν κάνω κάποια αλλαγή στην άσκηση μέχρι να δω αν θα προκύψει και κάτι καινούργιο από τους υπόλοιπους συναδέλφους!
Σχόλιο από τον/την Pantelis Lapas στις 5 Μάρτιος 2016 στις 11:52
Διαγραφή σχολίου
Συμφωνώ με τον Δ. Μάργαρη ... κι επίσης για τα αέρια δεν ισχύει η προσέγγιση του ασυμπίεστου όπως στα υγρά συνεπώς ότι βασίζεται σε αυτή την υπόθεση φαντάζομαι καταρρέει ... αυτή η άσκηση τείνει προς συστήματα μεταβλητής μάζας και απαιτεί διαφορικές εξισώσεις ... και μη ξεχνάμε ότι η μεταβολή του σχήματος του μπαλονιού εξαρτάται μεταξύ άλλων κι από τις ελαστικές δυνάμεις επαναφοράς που έχουν την τάση να το συρρικνώσουν.
Σχόλιο από τον/την Κυριακόπουλος Γιάννης στις 5 Μάρτιος 2016 στις 12:16
Διαγραφή σχολίου
Είναι δύσκολο πρόβλημα. Μια στοιχειώδης μάζα βρίσκεται σε πίεση 1 Atm από 1,24Atm. Αδιαβατική μεταβολή, δηλαδή ψύξη της μαζούλας; Στην μαζούλα προσφέρθηκε έργο (P-Patm).A.dx. Το έργο αυτό έγινε , κινητική ενέργεια. Όμως το αέριο εκτονώθηκε. Το παραχθέν κατά την εκτόνωση έργο έγινε επίσης κινητική ενέργεια της μαζούλας κατά ένα μέρος του; Έγινε 100% κινητική ενέργεια της μαζούλας; Η μικρή διαφορά πιέσεων μας δίνει (νομίζω) το δικαίωμα να θεωρήσουμε μικρό το έργο της αδιαβατικής και να δεχθούμε την λύση του Μιχάλη ως προσεγγιστικά σωστή.
Σχόλιο από τον/την Pantelis Lapas στις 5 Μάρτιος 2016 στις 13:04
Διαγραφή σχολίου
οκ ... πάντως αν αρχικά θεωρηθεί ακίνητο αυτό το μπαλόνι θα ξεκινήσει να εκτοξεύεται κατακόρυφα προς τα πάνω με μια επιτάχυνση εκατοντάδες φορές το g αν πάρουμε μια λογική μάζα για το μπαλόνι και το περιεχόμενό του, πέραν του ότι ξεκινάει να κινείται με μια ταχύτητα Ferrari και λίγα λέω ... στέκουν αυτά σαν αποτέλεσματα;
Σχόλιο από τον/την Κυριακόπουλος Γιάννης στις 5 Μάρτιος 2016 στις 13:12
Διαγραφή σχολίου
Δεν θα πιάσει μεγάλη ταχύτητα. Οι αντιστάσεις είναι συνάρτηση της ταχύτητας. Δευτεροβάθμια (πιστεύω) αλλά και με πρωτοβάθμιο όρο λόγω ιξώδους. Η οριακή ταχύτητα είναι μικρή.
Σχόλιο από τον/την Pantelis Lapas στις 5 Μάρτιος 2016 στις 14:07
Διαγραφή σχολίου
Η δύναμη αντίστασης είναι γραμμική για τόσο χαμηλές ταχύτητες αν θυμάμαι καλά και αλλάζει ο νόμος της δύναμης σε τετράγωνο σε μεγαλύτερες ταχύτητες ... αλλά ακόμη κι αν θυμάμαι λάθος ... η δύναμη αντίστασης εξαρτάται και από την διατομή του μπαλονιού κάθετα στη διεύθυνση κίνησης (ας πούμε χοντρικά) η οποία όμως μειώνεται ταχύτατα με το ξεφούσκωμα άρα η δύναμη αντίστασης γρήγορα θα πρέπει να φθίνει κι αυτή ... πάντως αν δύναμη από ώθηση ήταν περί τα 100 Ν (όπως στη δοθείσα άσκηση) πιστεύω θα χε διαλύσει το νήμα ... το δε παιδί θα δέχονταν μια ώθηση (υποθέτοντας αρχικά το κέντρο μάζας τους ακίνητο) περί τα 100Ν ~ 1/5 του βάρους του που δεν είναι και λίγο ...
Σχόλιο από τον/την Κυριακόπουλος Γιάννης στις 5 Μάρτιος 2016 στις 14:35
Διαγραφή σχολίου
Παντελή λογικά νομίζω τα λες. Όμως για πόσο χρόνο δρα η δύναμη αυτή; Αν πιάσεις μια μπάλα που πέφτει από 5m πόση δύναμη δέχεσαι περίπου; Αν η ακινητοποίηση γίνει σε 1/10 του δευτερολέπτου; Μεγάλες δυνάμεις αλλά μικρές (λογικές) ωθήσεις. Πάντως θέλει, αν όχι υπολογισμό, εκτίμηση για το μπαλόνι.
190. περιστροφή της πλάκας.

Η τετράγωνη πλάκα του σχήματος, μάζας Μ=0,3 Kg και πλευράς α=24 cm, μπορεί να περιστρέφεται χωρίς τριβές γύρω από οριζόντιο άξονα, που περνά από το κέντρο της Κ και είναι κάθετος σε αυτήν. Η πλάκα αρχικά ισορροπεί. Κάποια χρονική στιγμή που τη θεωρούμε αρχή των χρόνων (t=0) κολλάμε στην κορυφή Α της τετράγωνης πλάκας μια σημειακή μάζα m=0,1 Kg και αφήνουμε το σύστημα ελεύθερο χωρίς τριβές να περιστραφεί. Να βρείτε:

 

α) τη μέγιστη γωνιακή ταχύτητα του συστήματος.

 

β) πόση είναι τότε η στροφορμή της τετράγωνης πλάκας και πόση της σημειακής μάζας m ως προς το κέντρο Κ;

 

γ) το ρυθμό μεταβολής της στροφορμής του συστήματος και της μάζας m, τη χρονική στιγμή t=0 και τη χρονική στιγμή που έχουμε τη μέγιστη γωνιακή ταχύτητα περιστροφής.

 

δ) τη δύναμη από την τετράγωνη πλάκα στη σημειακή μάζα m τη χρονική στιγμή t=0 και τη στιγμή που αυτή αποκτά τη μέγιστη γωνιακή ταχύτητα περιστροφής.

 

Δίνεται για την τετράγωνη πλάκα Ιcm=1/6Mα2 και για τις πράξεις θεωρείστε g=10m/s2.

 

Συνοπτική λύση:

Σχόλια
Σχόλιο από τον/την Διονύσης Μάργαρης στις 12 Μάρτιος 2016 στις 8:18
Καλημέρα Μιχαήλ. Ωραία η πλάκα, είχαμε καιρό να δούμε κάτι αντίστοιχο. Σε ευχαριστούμε.
Σχόλιο από τον/την Μιχαήλ Μιχαήλ στις 12 Μάρτιος 2016 στις 12:08
Καλημέρα Διονύση. Σε ευχαριστώ για το σχόλιο!
Σχόλιο από τον/την Παπαδάκης Παντελεήμων στις 12 Μάρτιος 2016 στις 12:59
Καλημέρα Μιχαήλ Πάρα πολύ καλή η ''τετράγωνη'' με τα σχετικά ερωτήματα L kai dL/dt συστήματος(=Στεξωτερ) και πλάκας και m αλλά και δυνάμεων ,ξεκαθαρίζεις πράγματα σημαντικά για τον υποψήφιο
Σχόλιο από τον/την Μιχαήλ Μιχαήλ στις 12 Μάρτιος 2016 στις 16:59
Παντελή καλησπέρα και σ΄ευχαριστώ πολύ για τα καλά σου λόγια!
Σχόλιο από τον/την Νεκτάριος Πρωτοπαπάς στις 12 Μάρτιος 2016 στις 18:25
Καλησπέρα Μιχαήλ. Πρωτότυπη άσκηση και εντυπωσιακή!
Σχόλιο από τον/την koτσωνας παυλος στις 12 Μάρτιος 2016 στις 20:17
Κυριε Μιχαηλ μια απορια στο δ ερωτημα.Τη t=0 παιρνοντας ως αξονες την ακτινα ΚΑ και την εφαπτομενη αναλυοντας τοW και σχεδιαζοντας της συνιστωσες της δυναμης απο την πλακα πανω στους αξονες προκυπτει τελικα απο το 2' νομο διαφορετικο αποτελεσμα.Συγκεκριμενα Νχ=΅Wx =0.7N αφου η κεντρομολος ειναι μηδεν τη στιγμη 0 και Νψ=2.8Ν ομορροπη της Wψ οποτε το μετρο προκυπτει περιπου 2,88Ν αν δεν κανω λαθος υπολογισμους.Γραφετε στη λυση σας ΣFσυν45=mαεπ.Δεν θα πρεπει ΣF=mαεπ αφου ακ=0.Ευχαριστω.
Σχόλιο από τον/την Μιχαήλ Μιχαήλ στις 13 Μάρτιος 2016 στις 0:22
Νεκτάριε καλησπέρα και σε ευχαριστώ για το σχόλιο! Παύλε σε ευχαριστώ για το σχόλιο. Έχεις δίκιο για την κεντρομόλο δύναμη! Ίσως πρέπει να μείνουμε μόνο στη λύση:
191.Υδραντλία.

Ο ηλεκτρικός κινητήρας του σχήματος μάζας m=2,5Kg και ακτίνας r=4cm στρέφεται με γωνιακή ταχύτητα ω1 και κινεί υδραντλία μάζας Μ=2Κg και ακτίνας R=10cm, η οποία αντλεί 1m3 νερού σε ύψος h=3m μέσα σε 1min. Αν δεν υπάρχουν απώλειες (ιδανική περίπτωση), τότε:

α) Να υπολογιστεί η ωφέλιμη ισχύς της αντλίας.

β) Αν η ροπή που ασκείται από τον ιμάντα στην υδραντλία είναι 5Ν∙m, τότε να υπολογιστεί η σταθερή γωνιακή ταχύτητα ω1 του κινητήρα.

γ) Πόση είναι η ροπή που ασκείται από τον ιμάντα στον ηλεκτρικό κινητήρα;

 

Τη χρονική στιγμή t=0 κλείνουμε το διακόπτη της παροχής ηλεκτρικής ενέργειας από το ηλεκτρικό δίκτυο και εξαιτίας των τριβών με τον άξονα περιστροφής το σύστημα ακινητοποιείται σε χρόνο Δt. Τότε να βρείτε:

 δ) Τη σχέση που συνδέει τις συνολικές ροπές επιβράδυνσης στον κινητήρα και την υδραντλία θεωρώντας ότι αυτές είναι σταθερές.

ε) Το έργο της συνολικής ροπής επιβράδυνσης μέχρι να ακινητοποιηθεί το σύστημα.

 Δίνεται για τον ηλεκτρικό κινητήρα και την υδραντλία ότι Ι1=mr2 και Ι2=MR2 αντίστοιχα. Για τις πράξεις θεωρείστε τη πυκνότητα του νερού ρ=103 Κg∙m3 και g=10m/s2.

 Συνοπτική λύση:

193. Μερικές θεωρητικές επισημάνσεις στα πειράματα της Γ΄ Λυκείου.

1)    Μέτρηση του συντελεστή ιξώδους του λαδιού (Πανευρωπαϊκός διαγωνισμός EUSO 2014-www.panekfe.gr)

 

Κίνηση μικρής σφαίρας στο εσωτερικό κατακόρυφου σωλήνα γεμάτου με υγρό:

Έστω ότι μια  μικρή  πλαστική  σφαίρα κινείται κατά μήκος του άξονα συμμετρίας του κατακόρυφου κυλινδρικού σωλήνα που περιέχει υγρό.

H συνέχεια εδώ

Σχόλια
Σχόλιο από τον/την Κυριακόπουλος Γιάννης στις 26 Απρίλιος 2016 στις 14:04
Διαγραφή σχολίου
Μπράβο Μιχάλη. Για όλες.
Σχόλιο από τον/την Μιχαήλ Μιχαήλ στις 26 Απρίλιος 2016 στις 15:54
Διαγραφή σχολίου
Γιάννη σ' ευχαριστώ!
Σχόλιο από τον/την Γκενές Δημήτρης στις 26 Απρίλιος 2016 στις 19:53
Διαγραφή σχολίου
Εντυπωσιακές εργασίες Συγχαρητήρια Μιχαήλ Θα μου δώσεις λίγο χρόνο για προσεκτικότερη μελέτη διότι έχει πολύ ενδιαφέρον αλλά θέλουν τον χρόνο τους ... Θα επανέλθω
Σχόλιο από τον/την Βαγγέλης Κουντούρης στις 26 Απρίλιος 2016 στις 20:45
Διαγραφή σχολίου
καλησπέρα Μιχάλη αυτή τη φορά ομολογώ ότι δεν καταλαβαίνω τα πειράματα αυτά υπάρχουν τι ακριβώς θέλεις να πεις;
Σχόλιο από τον/την Κυριακόπουλος Γιάννης στις 26 Απρίλιος 2016 στις 21:29
Διαγραφή σχολίου
Βαγγέλη υποστηρίζει θεωρητικά τα πειράματα. Για φαντάσου να κάτσει κάτι συναφές σε θέματα εξετάσεων. Οι πιτσιρικάδες μπορούν να τα δουν, πριν τις εξετάσεις, μαζεμένα εδώ.
Σχόλιο από τον/την Θοδωρής Κυριακάκης στις 26 Απρίλιος 2016 στις 22:37
Διαγραφή σχολίου
Πολύ καλή και βοηθητική δουλειά, κ. Μιχαήλ. Διαβάζοντας την μου δημιουργήθηκε η απορία αν υπάρχει εργαστηριακός οδηγός για το βιβλίο της γ' λυκείου, όπως υπάρχει π.χ. για την α' (αν και δεν χρησιμοποείται από κανέναν) Ενημερωτικά, στον σύνδεσμο μπορείτε να δείτε τη διαδικασία που ακολουθεί το τμήμα μου για τον υπολογισμό του ιξώδους της γλυκερίνης και του νερού.
Σχόλιο από τον/την Γκενές Δημήτρης στις 26 Απρίλιος 2016 στις 22:44
Διαγραφή σχολίου
Ο εργαστηριακός οδηγός υπάρχει ΕΔΩ αλλά δεν έχει προσφέρει καμιά θεωρητική στήριξη
Σχόλιο από τον/την Θοδωρής Κυριακάκης στις 26 Απρίλιος 2016 στις 23:00
Διαγραφή σχολίου
Ευχαριστώ για την άμεση απάντηση. Δηλαδή από το 1999 που - αν δεν κάνω λάθος-γράφτηκαν τα βιβλία δεν χρησιμοποιήθηκε ποτέ;
Σχόλιο από τον/την Γκενές Δημήτρης στις 26 Απρίλιος 2016 στις 23:15
Διαγραφή σχολίου
Καλησπέρα Θοδωρή Οι οδηγοί αυτοί δεν νομίζω ότι έτυχαν μεγάλης αξιοποίησης... Αλλά όλα τα ΕΚΦΕ ( ή τέλος πάντων αρκετά ) έστελναν βοηθητικά φυλλάδια και πολλοί συνάδελφοι αφιερώνουν κάποιες λίγες ώρες για την επίδειξη. Μετωπικό εργαστήριο στην Γ Λυκέιου μάλλον λίγοι μερακλήδες έχουν επιχειρήσει. Δεν ήμουν από αυτούς. Θα πρόσεξες φαντάζομαι ότι η μέτρηση του ιξώδους μέσω της οριακής ταχύτητας δεν συμπεριλαμβάνεται ...
Σχόλιο από τον/την Μιχαήλ Μιχαήλ στις 26 Απρίλιος 2016 στις 23:50
Διαγραφή σχολίου
Δημήτρη, Θοδωρή σας ευχαριστώ! Βαγγέλη μια πρόταση για την επανάληψη έκανα.
Σχόλιο από τον/την Βαγγέλης Κουντούρης στις 27 Απρίλιος 2016 στις 0:03
Διαγραφή σχολίου
καλησπέρα Γιάννη και Μήτσο κάνετε λάθος αμφότεροι και ...οι δύο διότι αυτό το "εδώ" δεν είναι ο εργαστηριακός οδηγός, είναι το Τετράδιο Εργαστηριακών ασκήσεων υπάρχει ο εργαστηριακός οδηγός Γ΄Λυκείου Κατεύθυνσης με θεωρητική προσέγγιση των πειραμάτων, στον …καιρό μου μοιραζόταν και αυτός στους μαθητές, (τον οποίο συμβαίνει να έχω κριτικάρει αυστηρά, και εδώ, διότι εξακολουθεί 16 χρόνια μετά (!) να έχει λανθασμένο τύπο για την ταχύτητα του ήχου στον αέρα, λείπει η ρίζα…)
Σχόλιο από τον/την Κυριακόπουλος Γιάννης στις 27 Απρίλιος 2016 στις 0:08
Διαγραφή σχολίου
Μοιραζόταν Βαγγέλη. Έχεις ακούσει το "αμφότεροι και οι τρεις";
Σχόλιο από τον/την Γκενές Δημήτρης στις 27 Απρίλιος 2016 στις 0:20
Διαγραφή σχολίου
Δίκιο έχεις Βαγγέλη ... Μάλιστα μόλις τον βρήκα και καταχωνιασμένο σε ένα ράφι χαμηλά ... Υπάρχει και στο ψηφιακό σχολείο ...
Σχόλιο από τον/την Νεκτάριος Πρωτοπαπάς στις 28 Απρίλιος 2016 στις 0:04
Διαγραφή σχολίου
Καλησπέρα Μιχάλη και χρόνια πολλά. Να συμφωνήσω απόλυτα με το Γιάννη. Δεν είναι κακό στα παιδιά να ρίξουν μια ματιά στην ανάρτησή σου λίγο πριν τις εξετάσεις. Εξάλλου το 2ο θέμα λέει το ΦΕΚ ότι μπορεί να εξετάζει και ''εργαστηριακές'' γνώσεις. Φαντάζομαι ότι γι αυτό έκανες την ανάρτηση και πραγματικά είναι πάρα πολύ χρήσιμη. Σε ευχαριστώ λοιπόν και καλή ανάσταση να έχεις.
Σχόλιο από τον/την Μιχαήλ Μιχαήλ στις 29 Απρίλιος 2016 στις 1:00
Διαγραφή σχολίου
Νεκτάριε σ' ευχαριστώ! Πραγματικά ήθελα να συγκεντρώσω τις εργαστηριακές ασκήσεις σ΄ενα αρχείο για μια γρήγορη επανάληψη. Εξάλλου στον εργαστηριακό οδηγό δεν αναφέρεται ούτε η μέτρηση του συντελεστη ιξώδους, ούτε αναλυτικά το πείραμα του Young αλλά ούτε και ο σωλήνας του Kundt! Σου εύχομαι ολόψυχα Καλό Πάσχα και Καλή Ανάσταση.
Σχόλιο από τον/την Βαγγέλης Κουντούρης στις 29 Απρίλιος 2016 στις 1:55
Διαγραφή σχολίου
"Εξάλλου το 2ο θέμα λέει το ΦΕΚ ότι μπορεί να εξετάζει και ''εργαστηριακές'' γνώσεις." και όμως δεν..., Νεκτάριε Δεν θυμάμαι πόσα χρόνια το "φωνάζω"... όπως π.χ. εδώ: εδώ
Σχόλιο από τον/την Νεκτάριος Πρωτοπαπάς στις 29 Απρίλιος 2016 στις 10:41
Διαγραφή σχολίου
Καλημέρα Βαγγέλη. Καλά κάνεις και μας θυμίζεις την ανάρτησή σου γιατί όπως πολύ εύστοχα λες: Με την ελπίδα να φανούν χρήσιμες στους μαθητές που μας διαβάζουν και στα νεώτερα μέλη του δικτύου Χρόνια πολλά.
195. Ισορροπία της «πλάκας»_4

Η τετράγωνη πλάκα του σχήματος βάρους w και πλευράς α, ισορροπεί έτσι ώστε η πλευρά της ΑΒ να σχηματίζει γωνία φ, με το οριζόντιο έδαφος όπως φαίνεται στο σχήμα. Αν ο κατακόρυφος τοίχος είναι λείος  ενώ ο συντελεστής στατικής τριβής μεταξύ της πλάκας και του οριζόντιου εδάφους είναι μ, τότε:

i) Να βρείτε τις δυνάμεις που δέχεται η πλάκα από τον τοίχο και από το έδαφος.

ii) Να βρείτε την ελάχιστη τιμή της γωνίας φ για την οποία η τετράγωνη πλάκα δε γλιστρά στο έδαφος και

ii) Να βρείτε τη μέγιστη γωνία φ για την οποία η πλάκα δεν ανατρέπεται.

Συνοπτική Λύση

Σχόλια
Σχόλιο από τον/την Γκενές Δημήτρης στις 27 Ιούνιος 2016 στις 13:05
Καλημέρα Μιχαήλ. Σκέφτομαι τι θα έγραφες αν δεν ήταν συνοπτική η λύση Εξαιρετική άσκηση προς τέρψιν υμών ... δυστυχώς οι πολλές ανισότητες και κυρίως οι απαιτήσεις των τριγονομετρικών πράξεων δεν αφήνουν περιθώρια της πρότασης της προς υποψηφίους. Καλό καλοκαίρι ... αν δεν σε δούμε
Σχόλιο από τον/την Μιχαήλ Μιχαήλ στις 27 Ιούνιος 2016 στις 17:36
Δημήτρη καλησπέρα και σ' ευχαριστώ! Σου εύχομαι επίσης Καλό Καλοκαίρι. ....
196. Ισορροπία ημισφαιρίου

Το στερεό ημισφαίριο του σήματος ακτίνας r, έχει βάρος w1. Στο άκρο Α του ημισφαιρίου τοποθετούμε σημειακή μάζα βάρους w2. Να υπολογίσετε τη γωνία α, όταν το σύστημα ισορροπεί. Θεωρείστε ότι το κέντρο μάζας του στερεού ημισφαιρίου  βρίσκεται στον άξονα συμμετρίας του και σε απόσταση (OG)=3/8 r από το κέντρο του Ο και ότι η σημειακή μάζα δεν ολισθαίνει.

 

Συνοπτική Λύση

Σχόλια
Σχόλιο από τον/την Κυριακόπουλος Γιάννης στις 28 Ιούνιος 2016 στις 21:16
Διαγραφή σχολίου
Πολύ καλή. Και πολύ μικρή λύση!
Σχόλιο από τον/την Μιχαήλ Μιχαήλ στις 28 Ιούνιος 2016 στις 23:08
Διαγραφή σχολίου
Γιάννη σ' ευχαριστώ!
Σχόλιο από τον/την Δημήτρης Τσάτσης στις 28 Ιούνιος 2016 στις 23:30
Διαγραφή σχολίου
Μιχαήλ καλησπέρα...Φτιάχνεις πάντα φοβερές ασκήσεις και αυξημένης δυσκολίας... έτσι και αυτή. Μία διαφωνία (κυρίως σε ότι αφορά πως θα δείχναμε την άσκηση αυτή σε μαθητές): το βάρος του υλικού σημείου ασκείται στο ίδιο και όχι στο ημισφαίριο. Σε αυτό (το ημισφαίριο) ασκείται κάθετη (στην ΟΑ) δύναμη από τη σημειακή μάζα. Η δύναμη αυτή λόγω ισορροπίας ισούται με τη wy....και λοιπά. Και έχω μία απορία: αν η σημειακή μάζα δεν ολισθαίνει αυτό συμβαίνει λόγω μίας στατικής τριβής. Η αντίδραση αυτής δεν μας δίνει μία "μονάχη" συνιστώσα οριζόντια; Μήπως εμφανίζεται και μία στατική τριβή προς τ' αριστερά στο σημείο επαφής ημισφαιρίου δαπέδου;
Σχόλιο από τον/την Δημήτρης Τσάτσης στις 29 Ιούνιος 2016 στις 0:09
Διαγραφή σχολίου
To "μονάχη" που γράφω είναι λάθος (βιάστηκα)...(όλα οκ). Καλό βράδυ
Σχόλιο από τον/την Γκενές Δημήτρης στις 29 Ιούνιος 2016 στις 0:20
Διαγραφή σχολίου
"Τα ακριβά αρώματα σε ... μικρά μπουκάλια" Συγχαρητήρια Μιχαήλ.
Σχόλιο από τον/την Χρήστος Αγριόδημας στις 29 Ιούνιος 2016 στις 0:43
Διαγραφή σχολίου
Μιχαήλ πολύ ωραία άσκηση, μπράβο.
Σχόλιο από τον/την Μιχαήλ Μιχαήλ στις 29 Ιούνιος 2016 στις 9:21
Διαγραφή σχολίου
Δημήτρη(Γκ.), Χρήστο καλημέρα. Σας ευχαριστώ για τα σχόλιά σας!
Σχόλιο από τον/την Μιχαήλ Μιχαήλ στις 29 Ιούνιος 2016 στις 9:42
Διαγραφή σχολίου
Σχόλιο από τον/την Διονύσης Μάργαρης στις 29 Ιούνιος 2016 στις 9:44
Διαγραφή σχολίου
Μπράβο Μιχαήλ. Θα συμφωνήσω με το Δημήτρη: "Τα ακριβά αρώματα σε ... μικρά μπουκάλια"
Σχόλιο από τον/την Μιχαήλ Μιχαήλ στις 29 Ιούνιος 2016 στις 11:44
Διαγραφή σχολίου
Διονύση καλημέρα. Ευχαριστώ πολύ!
Σχόλιο από τον/την Βασίλης Δουκατζής στις 29 Ιούνιος 2016 στις 14:36
Διαγραφή σχολίου
"Και έχω μία απορία: αν η σημειακή μάζα δεν ολισθαίνει αυτό συμβαίνει λόγω μίας στατικής τριβής. Η αντίδραση αυτής δεν μας δίνει μία "μονάχη" συνιστώσα οριζόντια; Μήπως εμφανίζεται και μία στατική τριβή προς τ' αριστερά στο σημείο επαφής ημισφαιρίου δαπέδου;" Δημήτρη (Τσ) καλησπέρα. Λες για το πως πρέπει να είναι οι δυνάμεις και το σκέφτεσαι πολύ σύνθετα. Τα πράγματα είναι απλά.
Το σώμα ισορροπεί άρα δέχεται από το ημικύκλιο μία κατακόρυφη δύναμη αντίθετη του βάρους οπότε ΣF = 0.Το ίδιο ασκεί στο ημικύκλιο μία κατακόρυφη δύναμη όμοια φορά με το βάρος του και ίσου μέτρου, άρα μετά κάνουμε ότι και ο Μιχαήλ, με μία δύναμη F' = F = w2. (μέτρα)
  Μετά οι δυνάμεις που δέχεται το ημικύκλιο είναι:  
Έτσι λοιπόν εφόσον οι ν-1 δυνάμεις είναι κατακόρυφες (w1, F') για να έχουμε ισορροπία θα πρέπει και η ν-οστή να είναι κατακόρυφη.Οπότε η Ν έχει την φορά που φαίνεται (και σχεδίασε σωστά ο Μιχάλης) Υ.Γ. Πολλές φορές πέφτουμε στην παγίδα των δύο δυνάμεων από το έδαφος και κάνουμε την ζωή μας δύσκολη, ενώ τα πράγματα είναι πιο απλά!  
 
Σχόλιο από τον/την Δημήτρης Τσάτσης στις 29 Ιούνιος 2016 στις 16:44
Διαγραφή σχολίου
Βασίλη δίκιο..βιάστηκα και το διόρθωσα (χωρίς να πάρω χαρτί αρχικά με προβλημάτισε αν υπάρχει καμία στατική τριβή σε επαφή με το έδαφος..τελικά δεν υπάρχει). Από την άλλη, όμως, διδακτικά θα διαφωνήσω μαζί σου...Ο μαθητής θα πρέπει να προσέχει τις επιμέρους δυνάμεις και να τις "δίνει" στα σώματα που ανήκουν και όχι στα άλλα (αντιδράσεις). Τώρα θυμήθηκα το θέμα Δ του 2014... Πόσοι μαθητές (και καθηγητές...ακόμη και οι λύσεις της ΟΕΦΕ ...κάπου τις έχω κρατήσει) εφάρμοζαν ισορροπία για τη δοκό και αναφέρανε το βάρος της σφαίρας που κυλούσε ανερχόμενη (και το σχολικό το κάνει αυτό εκεί με τους μπογιατζήδες)...και νικνδύνους ελλοχεύει και λάθος το θεωρώ (αυστηρά σκεπτόμενος έστω). ΥΓ1: Η F' στο δεύτερο σχήμα πρέπει να σχεδιαστεί με σημείο εφαρμογής το ημισφαίτιο (επί τούτου η κουβέντα). Υγ2: Ευχαριστώ για τον κόπο σου και για την κουβέντα. Έχω μείνει χωρίς Visio και άντε να δω πως θα κάνω σχήματα (το σεμινάριό σου για το word...ευκαιρία να το πιάσω).
197. Ισορροπία σφαιρών

Οι σφαίρες του σχήματος ισορροπούν. Οι σφαίρες έχουν μάζες m1 και m2. Ακόμη η σφαίρα μάζας m2 έχει ακτίνα r και το μήκος του νήματος με το οποίο είναι δεμένη από την οροφή είναι (ΟΑ)=L. Να υπολογίσετε:

Α. Τη γωνία α, που σχηματίζει το νήμα μήκους L με την κατακόρυφο, όταν το σύστημα ισορροπεί και

Β. Τις τάσεις T1,T2 και T των νημάτων. (g: γνωστό).

 

pdf

Σχόλια
Σχόλιο από τον/την Γκενές Δημήτρης στις 5 Ιούλιος 2016 στις 18:26
Διαγραφή σχολίου
Καλησπέρα Μιχαήλ ... Νομίζω πως πρέπει να την ξαναδείς λίγο την άσκηση. α) Δεν είναι αλήθεια ότι ισορροπεί με το κέντρο Κ(2) στην προέκταση του νήματος ΟΑ . ( Αν και μπορείς να το απαιτήσεις ως δεδομένη προσέγγιση δεν θα το συνιστούσα ) β) Οι τάσεις Τ(1) και Τ(2) δεν ασκούνται στο ίδιο σημείο ( το νήμα αγκαλιάζει λίγο τη σφαίρα ... προσέγγιση της εκφώνησης; ) και γ) και σπουδαιότερο η Σχέση ισορροπίας για την σφαίρα (2) ... δεν είμαι σίγουρος ότι ισχύει ακόμα και με τις παραπάνω παραδοχές ... Ίσως πάλι όλα αυτά είναι ενστάσεις σε λάθος σκέψεις μου . Θα την ξαναδώ την νύχτα ...
Σχόλιο από τον/την Μιχαήλ Μιχαήλ στις 5 Ιούλιος 2016 στις 18:54
Διαγραφή σχολίου
Καλησπέρα Δημήτρη και ευχαριστώ για το σχόλιο! Λίγο πολύ έχω τις ίδιες ενστάσεις και γι' αυτό ανέβασα την άσκηση στο yliko. Υπάρχει μια παλιότερη ανάρτηση του Πρόδρομου εδώ μόνο που δεν μπορώ να βρω την απάντησή! Για το 1ο ερώτημα περισσότερο διαισθητικά νομίζω ότι είναι σωστό το αποτέλεσμα. Για το 2ο ερώτημα επιφυλάσσομαι (ίσως πρέπει να κάνω κάποιες επιπλέον παραδοχές)!!
Σχόλιο από τον/την Διονύσης Μάργαρης στις 5 Ιούλιος 2016 στις 19:08
Διαγραφή σχολίου
Μιχαήλ και Δημήτρη καλησπέρα. Μια γνώμη και από μένα. Για το 1ο ερώτημα, αφού δεν έχουν δοθεί τιμές στις μάζες, υπάρχει ένας συνδυασμός που επιτρέπει στο νήμα να είναι προέκταση της ακτίνας. Για το 2ο ερώτημα, πράγματι η τάση Τ2 δεν ασκείται στο ίδιο σημείο με την Τ1, είναι όμως εφαπτομενική. Συνεπώς θα μπορούσαμε να πάρουμε ροπές ως προς το κέντρο και να βρούμε ότι έχει το ίδιο μέτρο με την Τ1.
Σχόλιο από τον/την Μιχαήλ Μιχαήλ στις 5 Ιούλιος 2016 στις 20:29
Διαγραφή σχολίου
Διονύση καλησπέρα και σε ευχαριστώ για τη βοήθεια. Δημήτρη και Διονύση την έγραψα έτσι
Σχόλιο από τον/την Διονύσης Μάργαρης στις 5 Ιούλιος 2016 στις 22:37
Διαγραφή σχολίου
Μιχαήλ, νομίζω ότι τώρα είναι σωστή.
Σχόλιο από τον/την Βαγγέλης Κουντούρης στις 5 Ιούλιος 2016 στις 23:39
Διαγραφή σχολίου
καλησπέρα σε όλους έχω τον ίδιο προβληματισμό με τον Μήτσο διαισθητικά δεν μου φαίνεται η Τ να περνάει από το κέντρο της σφαίρας, αλλά και αν ναι, η συνισταμένη των Τ και W περνάει από το κέντρο, ενώ η συνισταμένη των ίσων (;) Τ1 και Τ2 δεν περνάει, άρα πώς ισορροπεί η σφαίρα;
Σχόλιο από τον/την Διονύσης Μητρόπουλος στις 6 Ιούλιος 2016 στις 2:53
Διαγραφή σχολίου
Καλησπέρα κι από μένα συνάδελφοι :-) Μιχάλη συγχαρητήρια για την ιδέα. Μια σκέψη κι από μένα, ΕΔΩ
Σχόλιο από τον/την Βαγγέλης Κουντούρης στις 6 Ιούλιος 2016 στις 8:18
Διαγραφή σχολίου
Σωστά Διονύση (Μητρ) οι φορείς των Τ1 και Τ2 τέμνονται πιο πάνω από το σημείο εφαρμογής της Τ1, άρα η συνισταμένη τους περνά από το Κ, άρα, αφού και το W, θα περνά και η Κ
Σχόλιο από τον/την Γκενές Δημήτρης στις 6 Ιούλιος 2016 στις 10:25
Διαγραφή σχολίου
Καλημέρα σε όλους Τελικά είχα άδικο όσον αφορά την πρώτη μου τουλάχιστον ένσταση. Ο Διονύσης ( Μητρ )απέδειξε σε 2 γραμμές πως, αν η σφαίρα είναι λεία, πάντα το κέντρο θα βρίσκεται στην προέκταση του νήματος (! κι εγώ παιδευόμουν με ανάλυση δυνάμεων; Μπράβο Διονύση ) Μιχαήλ έτσι ή αλλιώς ήταν ωραία ιδέα και τελικά προέκυψε και όμορφη επίλυση ... Ευχαριστούμε
Σχόλιο από τον/την Διονύσης Μάργαρης στις 6 Ιούλιος 2016 στις 10:35
Διαγραφή σχολίου
Καλημέρα σε όλους. Όμορφη η απόδειξή σου Διονύση. Δημήτρη δεν απέδειξε ο Διονύσης ότι πάντα το κέντρο θα βρίσκεται στην προέκταση του νήματος. Οι δύο τάσεις είναι ίσες, μόνο αν ισχύει το παραπάνω. Θα μπορούσαμε να έχουμε το νήμα που κρέμεται η μεγάλη σφαίρα να σχηματίζει γωνία με την ακτίνα, αλλά τότε οι δύο τάσεις δεν θα είχαν το ίδιο μέτρο, αφού θα είχαμε και ροπή ως προς το κέντρο και της τρίτης τάσης.
Σχόλιο από τον/την Γκενές Δημήτρης στις 6 Ιούλιος 2016 στις 10:48
Διαγραφή σχολίου
Καλημέρα Μάργαρη ... Αν η σφαίρα είναι λεία μπορεί να είναι οι δυο τάσεις διαφορετικές ;
Σχόλιο από τον/την Βαγγέλης Κουντούρης στις 6 Ιούλιος 2016 στις 10:50
Διαγραφή σχολίου
καλημέρα σε όλους Διονύση αυτό είχα σκεφτεί και εγώ αρχικά, συμφωνώντας με τον Μήτσο, γι’ αυτό και είχα βάλλει τη σχέση Τ1=Τ2 με ερωτηματικό, αλλά αφού η σφαίρα είναι λεία σωστά γράφει ο έτερος Διονύσης, δηλαδή η σειρά είναι: επειδή Τ1=Τ2, διότι η σφαίρα (και το νήμα...) λεία, η συνισταμένη τους περνάει από το Κ, και επειδή και το W της σφαίρας περνάει από το Κ, άρα και η Τ περνάει από το Κ, αφού η σφαίρα ισορροπεί και πρέπει οι δυνάμεις που δέχεται να συντρέχουν, λέγω, αλλά δεν παίρνω και όρκο (διότι ούτως ή άλλως δεν παίρνω...)
Σχόλιο από τον/την Διονύσης Μάργαρης στις 6 Ιούλιος 2016 στις 12:08
Διαγραφή σχολίου
Καλημέρα και πάλι Δημήτρη και Βαγγέλη. Έχετε δίκιο για τις λείες σφαίρες. Το λέει στην εκφώνηση, αλλά δεν το είδα... Γι' αυτό και από την πρώτη στιγμή μίλησα για σχέση μαζών, που εξασφαλίζεται η συνθήκη το νήμα να είναι στη διεύθυνση της ακτίνας. Θέλω να πω δηλαδή, ότι ακόμη και αν οι σφαίρες δεν είναι λείες, αν δοθεί ότι το νήμα έχει τη διεύθυνση της ακτίνας, ως δεδομένο, τότε από το Στ=0 ως προς το κέντρο, προκύπτει η ισότητα μέτρων στις δύο τάσεις.
Σχόλιο από τον/την Διονύσης Μάργαρης στις 6 Ιούλιος 2016 στις 12:10
Διαγραφή σχολίου
Και διαβάζοντας ξανά την εκφώνηση παραπάνω νιώθω και λίγο δικαιωμένος:-) Δεν λέει λείες σφαίρες!!! Όχι στη λύση, αλλά στην εκφώνηση!
Σχόλιο από τον/την Μιχαήλ Μιχαήλ στις 6 Ιούλιος 2016 στις 14:05
Διαγραφή σχολίου
Βαγγέλη Διονύση Διονύση και Δημήτρη σας ευχαριστώ για τη βοήθεια που μου προσφέρατε. Είμαι εκτός και μιλάω με κινητό. Θα ενσωμστώσω τις παρατηρήσεις σσς στη λύση. Να είστε πάντα καλά
Σχόλιο από τον/την Βαγγέλης Κουντούρης στις 6 Ιούλιος 2016 στις 14:29
Διαγραφή σχολίου
Α, καλά, Διονύση, ομοιοπαθής! επίσης δεν είχα δει στην εκφώνηση για λείες σφαίρες, γι' αυτό και έγραψα Τ1=Τ2 (;), ο έτερος Διονύσης με τη λύση του με ...παρέσυρε (αν δεν έχει έλθει το Alzhiemer, νομίζω το Alz έχει έλθει...)
Σχόλιο από τον/την Γκενές Δημήτρης στις 6 Ιούλιος 2016 στις 14:57
Διαγραφή σχολίου
Εγώ πάντως διαβάζοντας το πρωΐ την λύση του Μητρόπουλου το έλεγξα ... δεν είδα λείες στην εισαγωγή ΑΛΛΑ είδα στο pdf
Σχόλιο από τον/την Διονύσης Μάργαρης στις 6 Ιούλιος 2016 στις 15:07
Διαγραφή σχολίου
Γι΄αυτό έγραψα παραπάνω Δημήτρη: "Έχετε δίκιο για τις λείες σφαίρες. Το λέει στην εκφώνηση, αλλά δεν το είδα..." Και δεν το είδα, αφού διάβασα την δημοσιευμένη εκφώνηση παραπάνω και όχι την εκφώνηση στο pdf... Το πήρα είδηση, μετά την εμπλοκή!!!
Σχόλιο από τον/την Βαγγέλης Κουντούρης στις 6 Ιούλιος 2016 στις 15:21
Διαγραφή σχολίου
σε μένα που έπαθα το ίδιο με τον Διονύση θα ...συμπαρασταθεί κανείς;
Σχόλιο από τον/την Διονύσης Μητρόπουλος στις 6 Ιούλιος 2016 στις 15:23
Διαγραφή σχολίου
Καλημέρα συνάδελφοι, Εγώ πάντως ... τη γλύτωσα τη μπανανόφλουδα, διότι από τη φούρια μου δεν διάβασα την εισαγωγή του Μιχάλη, αλλά πήγα κατευθείαν στο pdf :-)
Σχόλιο από τον/την Γκενές Δημήτρης στις 6 Ιούλιος 2016 στις 15:24
Διαγραφή σχολίου
Διονύση τωρα γύρισα πίσω και είδα το σχόλιό σου το σχετικό ... Διάβαζα από το επόμενο και μετά ... Να παρατηρήσω ότι οι άνθρωποι που κάνουν διάλογο και καταλήγουν στο σωστό αισθάνονται πάντα δικαιωμένοι όλοι ... γιατί ο καθένας συνεισφέρει στο σωστό-όχι εξ ίσου αλλά δεν κάνουμε διαγωνισμό - Να είμαστε όλοι καλά και στην επόμενη συνεστίαση μπορεί να καταφέρουμε και τον Μιχαήλ και τον Κουντούρη.... Καλό μεσημέρι.
198. Ισορροπία ράβδου σε ημισφαίριο

Η ομογενής ράβδος AΜ του σχήματος βάρους w=5N και μήκους L στηρίζεται με το ένα άκρο της σε λείο στερεό ημισφαίριο και με το άλλο σε οριζόντιο επίπεδο έτσι ώστε να σχηματίζει μ’ αυτό γωνία φ με ημφ=0,6. Ο συντελεστής στατικής τριβής μεταξύ της ράβδου και του οριζοντίου επιπέδου είναι  μ=0,5. Αν η ράβδος ισορροπεί οριακά να υπολογίσετε τις δυνάμεις που δέχεται η ράβδος από το έδαφος και το ημισφαίριο.

Συνοπτική λύση:

199. Ισορροπία ράβδου και κεκλιμένο επίπεδο

Η ομογενής ράβδος AΜ του σχήματος βάρους w=5N και μήκους L στηρίζεται με το ένα άκρο της σε κεκλιμένο επίπεδο γωνίας κλίσης φ με ημφ=0,6  και με το άλλο σε λείο οριζόντιο επίπεδο. Αν η ράβδος ισορροπεί όπως φαίνεται στο σχήμα να υπολογίσετε τις δυνάμεις που δέχεται η ράβδος από το έδαφος και από το κεκλιμένο επίπεδο.

 

Συνοπτική λύση:

Σχόλια
Σχόλιο από τον/την Διονύσης Μάργαρης στις 20 Ιούλιος 2016 στις 13:04
Καλό μεσημέρι Μιχαήλ. Ωραίες και οι δυο ισορροπίες σου!
Σχόλιο από τον/την Μιχαήλ Μιχαήλ στις 20 Ιούλιος 2016 στις 15:58
Καλησπέρα Διονύση. Σ' ευχαριστώ πολύ!
200. Ισορροπία ράβδου και κύλινδροι

Η λεπτή ομογενής ράβδος του σχήματος στηρίζεται πάνω στους δυο όμοιους ακλόνητους κυλίνδρους Α και Β, έτσι ώστε το τμήμα της ράβδου ανάμεσα σ’ αυτούς να είναι α όπως φαίνεται στο σχήμα.

Να βρεθεί τότε η ελάχιστη απόσταση x του κέντρου μάζας Κ της ράβδου από το σημείο επαφής του κυλίνδρου Α με τη ράβδο ώστε αυτή να ισορροπεί (αλλιώς η ράβδος είτε ολισθαίνει είτε ανατρέπεται).

Δίνεται ο συντελεστής στατικής τριβής μ ανάμεσα στη ράβδο και τους κυλίνδρους καθώς και η γωνία φ που σχηματίζει αυτή με το οριζόντιο επίπεδο.

 

Συνοπτική λύση:

201_4. Ενέργεια ταλάντωσης και κρούση

Στο ελεύθερο άκρο του κατακόρυφου ελατήριου του σχήματος σταθεράς Κ=12,5Ν/m, έχουμε δέσει σώμα μάζας m2=2Kg και το σύστημα αρχικά ισορροπεί. Ένα δεύτερο σώμα μάζας m1=m2 κινείται κατακόρυφα προς τα πάνω και συγκρούεται μετωπικά και πλαστικά με το σώμα m1. Αν η αρχική κινητική ενέργεια της m1 λίγο πριν την κρούση είναι Καρχ=20 J τότε η ενέργεια Ε της ταλάντωσης μετά την κρούση είναι:

α) Ε<20J

β) Ε=20J

γ) Ε>20J

δ) δε γνωρίζουμε.

 

Συνοπτική λύση:

Σχόλια
Σχόλιο από τον/την Εμμανουήλ Λαμπράκης στις 5 Αύγουστος 2016 στις 18:24
Διαγραφή σχολίου
Καλησπέρα Μιχαήλ Πολύ ωραίο θέμα
Σχόλιο από τον/την Μιχαήλ Μιχαήλ στις 6 Αύγουστος 2016 στις 2:03
Διαγραφή σχολίου
Μανώλη καλησπέρα και σ' ευχαριστώ για το σχόλιο!!
Σχόλιο από τον/την Κωστας Ψυλακος στις 7 Αύγουστος 2016 στις 13:02
Διαγραφή σχολίου
Μιχαλη Καλημερα ! Η αναλυση σου σχετικα με την Εταλ και την Καρχ ειναι πολυ ενδιαφερουσα και παρατηρω οτι εξεταζεις ολες τις πιθανες περιπτωσεις με πολυ διεξοδικό τροπο και αξιζει καποιος να το μελετησει! Το κατακορυφο ελατηριο με σωμα στο ενα του ακρο και στην συνεχεια πλαστικη κρουση, η εξελιξη της Εταλ σε σχεση με την Καρχ γινονται λιγο περιπλοκα και ειναι και το σημειο που πρεπει να προσεξει κανεις και εκει ακριβως ειναι που εστιαζεις. Θα ηθελα να δωσω μια πιο "απλη" αναλυση στο κατακορυφο ελατηριο και στην πλαστικη κρουση. Βασιζομενος βεβαια στο παραδειγμα σου στο οποιο απο το ελατηριο κρεμεται η m2 και η m1 συγκρουεται πλαστικα με αυτην . Το θεμα μας ειναι τι γινεται με την διαφορα ΔΕ=Καρχ - Εταλ=Καρχ-Κτελ-U Καρχ η αρχικη κινητικη ενεργεια της m1 Κτελ η κινητικη ενεργεια της (m1+m2) U η δυναμικη ενεργεια ταλαντωσης της (m1+m2) χρησιμοποιώντας τους τυπους σου εχουμε ΔΕ= (m2/(m1+m2))*Καρχ - U θέτω λ=m1/m2 (εσυ το εχεις παρει τον λογο αυτο αντιστροφα). Τοτε : ΔΕ=[Καρχ/(λ+1)] - U 1. Για ΔΕ>0 => λ<(Καρχ/U) -1 2. Για ΔΕ<0 => λ>(Καρχ/U) -1 3. Για ΔΕ=0 => λ=(Καρχ/U) -1 Έχοντας γνωστα τα Καρχ και U μπορει κανεις να αναζητησει για πιο λ θα εχουμε το προσημο που θελουμε για το ΔΕ=Καρχ - Εταλ . Μιχαλη καλη συνεχεια ευχομαι στις διακοπες σου ! Να εισαι καλα !
Σχόλιο από τον/την Μιχαήλ Μιχαήλ στις 7 Αύγουστος 2016 στις 14:16
Διαγραφή σχολίου
Κώστα σε ευχαριστώ πολύ! Εξάλλου η διεξοδικότερη μελέτη οφείλεται στη δικιά σου παρατήρηση που μου έστειλες με email. Καλή συνέχεια και να περνάς καλά!
Σχόλιο από τον/την Εμμανουήλ Λαμπράκης στις 8 Αύγουστος 2016 στις 12:05
Διαγραφή σχολίου
Καλημέρα Η διερεύνηση συνοψίζεται πολύ όμορφα όπως σημειώνει ο Κώστας.
203. Eξαναγκασμένη ταλάντωση: διαφορετικές συχνότητες ίδιο πλάτος.

Στη διάταξη του σχήματος η σταθερά του ιδανικού ελατηρίου είναι K=100N/m και η μάζα του σώματος είναι m=4Kg.

Το χέρι μας ασκεί περιοδική δύναμη F, και το σώμα  εκτελεί τελικά εξαναγκασμένη  αρμονική ταλάντωση συχνότητας f1=4/2π Hz και πλάτους Α=4,4cm χωρίς αρχική φάση. Το σώμα κινούμενο δέχεται δύναμη αντίστασης Fαντ= -b×υ με σταθερά απόσβεσης b=0,4Kg×s-1.

Για ποια συχνότητα f2  το πλάτος της ταλάντωσης ξαναγίνεται ίσο με 4,4 cm;

 

Συνοπτική λύση:

Σχόλια
Σχόλιο από τον/την Νίκος Ανέστης στις 17 Αύγουστος 2016 στις 13:07
Διαγραφή σχολίου
Μπράβο Μιχαήλ πολύ καλή .
Σχόλιο από τον/την Μιχαήλ Μιχαήλ στις 17 Αύγουστος 2016 στις 17:58
Διαγραφή σχολίου
Νίκο σ' ευχαριστώ.
Σχόλιο από τον/την Διονύσης Μάργαρης στις 17 Αύγουστος 2016 στις 20:05
Διαγραφή σχολίου
Καλησπέρα Μιχαήλ. Ομολογώ ότι διαβάζοντας την εκφώνηση, περίμενα λύση της διαφορικής και εύρεση πλάτους κ.λ.π. Πολύ καλή η λύση σου και "εύκολα" προσβάσιμη, ακόμη και από μαθητές. Σε ευχαριστώ.
Σχόλιο από τον/την Γκενές Δημήτρης στις 17 Αύγουστος 2016 στις 23:13
Διαγραφή σχολίου
Καλησπέρα Μιχαήλ Πολύ έξυπνο και χαρά στο κουράγιο σου Διατηρώ μερικές αμφιβολίες ότι ίσως με την διάταξή σου θα παραχθούν διακροτήματα και συνεπώς δεν μπορώ να αντικαταστήσω την θέση x με Α*ημωt και την ταχύτητα υ με ωΑσυνωt ... Πάλι μπορεί να κάνω και λάθος. Για τις αμφιβολίες μου δεν φταις εσύ αλλά ... κάπου νοιώθω να μην είμαι σίγουρος στη θεωρία για τέτοιες προβλέψεις , οπότε ... ... θα ήθελα να μπορούσα να επαληθεύσω πειραματικά ... Δεν ξέρω πως αλλά θα προσπαθήσω να δω αν μπορούσα να φτιάξω αυτά σε Interactive Physics ... Πολύ απαιτητική διάταξη και πως να βάλω αυτή την αρμονική δύναμη; Ισως και να μην μπορώ εγώ ...  
Σχόλιο από τον/την Μιχαήλ Μιχαήλ στις 17 Αύγουστος 2016 στις 23:43
Διαγραφή σχολίου
Διονύση καλησπέρα και σ' ευχαριστώ. Δημήτρη έχεις δίκιο για τα μεταβατικά διακροτήματα που υπό προϋποθέσεις μπορεί να έχουν κάποια χρονική διάρκεια. Αν θέλεις δες εδώ από μια παλιότερη ανάλυση!
Σχόλιο από τον/την Γκενές Δημήτρης στις 17 Αύγουστος 2016 στις 23:55
Διαγραφή σχολίου
Ευχαριστώ Μιχαήλ. Θα το μελετήσω και θα προσπαθήσω να δω αν μπορώ να υπολογίσω τον χρόνο 5τ του εκθετικού όρου των μεταβατικών φαινομένων.
Σχόλιο από τον/την Γκενές Δημήτρης στις 18 Αύγουστος 2016 στις 12:21
Διαγραφή σχολίου
Μιχαήλ έχεις απόλυτο δίκιο ... και για την "στάσιμη κατάσταση" και ότι σύντομα ( σε 1-2 λεπτά ) η κίνηση γίνεται περιοδική ... Πολύ καλή η παλιότερη ανάλυση αν και πολύ πυκνή ... Συγχαρητήρια και σε ευχαριστώ.
Σχόλιο από τον/την Διονύσης Μάργαρης στις 18 Αύγουστος 2016 στις 15:00
Διαγραφή σχολίου
Ένα αρχείο i.p. στο οποίο αναπαρίσταται η παραπάνω εξαναγκασμένη ταλάντωση, με κλικ εδώ.
Σχόλιο από τον/την Μιχαήλ Μιχαήλ στις 18 Αύγουστος 2016 στις 17:31
Διαγραφή σχολίου
Δημήτρη σ' ευχαριστώ για το σχόλιό σου! Τελικά το i.p του Διονύση σε επιβεβαιώνει και παράγονται αρχικά διακροτήματα. Όμως σε 1,5 min περίπου η κίνηση γίνεται αρμονική ταλάντωση. Διονύση σ' ευχαριστώ πολύ για το i.p αρχείο.
Σχόλιο από τον/την Λεβέτας Στάθης στις 20 Σεπτέμβριος 2016 στις 10:37
Διαγραφή σχολίου
Συνάδελφοι, υπάρχει κάποια ασάφεια στην πρόταση ότι μία εξαναγκασμένη ταλάντωση αποκτά τελικά σταθερό πλάτος και η κίνηση γίνεται απλή αρμονική ταλάντωση (Α.Α.Τ)... Το χαρακτηριστικό των Α.Α.Τ. είναι το ότι διατηρούν την συνολική τους ενέργεια σταθερή, η οποία εξαρτάται μόνον από τις αρχικές συνθήκες, άμεσο αποτέλεσμα του ότι το σύστημα είναι συντηρητικό. Στις εξαναγκασμένης ταλαντώσεις (Ε.Τ.) με περιοδική διέγερση και υποκρίσιμη απόσβεση, τόσο η δύναμη απόσβεσης, όσο και η εξωτερική διεγείρουσα δύναμη είναι μη συντηρητικές και συνεχώς παράγουν έργο (οι στιγμιαίες τιμές των οποίων δεν συμπίπτουν εκτός της ειδικής περίπτωσης όπου η συχνότητα του διεγέρτη ισούται με την ιδιοσυχνότητα, οπότε τα δύο έργα είναι συνεχώς αντίθετα). Συνεπώς το σύστημα του Ε.Τ. είναι μη συντηρητικό. Άμεσο αποτέλεσμα είναι το ότι δεν διατηρείται η μηχανική ενέργεια της Ε.Τ., η οποία μετά τα μεταβατικά φαινόμενα προκύπτει περιοδική συνάρτηση του χρόνου (με περίοδο ίση με την ημιπερίοδο της διεγείρουσας δύναμης). Μία ακόμη διαφορά με τις Α.Α.Τ. είναι ότι η ποσότητα Α που χαρακτηρίζουμε ως "πλάτος" είναι ανεξάρτητη των αρχικών συνθηκών του Ε.Τ., εξαρτάται δε μόνον από τα σταθερά χαρακτηριστικά του (συχνότητα και πλάτος διεγέρτη, μάζα και σταθερά επαναφοράς του υλικού σημείου και συντελεστού απόσβεσης). Συνοψίζοντας, στις Ε.Τ. μετά την πάροδο των μεταβατικών φαινομένων, το σύστημα καταλήγει σε μία περιοδική κίνηση μεταξύ δύο ακραίων απομακρύνσεων -Α και +Α τις οποίες, κατ' απόλυτη τιμή, συμφωνούμε να αποκαλούμε πλάτος, αλλά η ενέργειά του δεν είναι σταθερή και η κίνηση δεν μπορεί να χαρακτηριστεί ως Α.Α.Τ..
206. Μέγιστη προσφερόμενη ενέργεια ανά περίοδο σε μια εξαναγκασμένη ταλάντωση

Το αρχείο εδώ

208. Μέση ταχύτητα στην α.α.τ

Σώμα πραγματοποιεί την α.α.τ   x=Aημωt. Να υπολογιστεί η μέση ταχύτητα του σώματος σε χρόνο:

α) t=T/4

β) t= T

γ) t=kT+T/4 

Συνοπτική λύση:

ή στο προσωπικό μου blog

Σχόλια
Σχόλιο από τον/την Βαγγέλης Κουντούρης στις 22 Οκτώβριος 2016 στις 19:50
Διαγραφή σχολίου
καλησπέρα Μιχαήλ σωστά η εκφώνηση έπρεπε να ήταν: Να υπολογιστεί η μέση διανυσματική ταχύτητα του σώματος στο χρονικό διάστημα: α. 0-Τ/4 β. 0-Τ γ. 0-(kΤ+Τ/4)  
Σχόλιο από τον/την Γκενές Δημήτρης στις 22 Οκτώβριος 2016 στις 20:25
Διαγραφή σχολίου
Καλησπέρα Μιχαήλ Καλή σκέψη αλλά να διευκρινίσουμε ότι μιλάμε για διανυσμτική μέση ταχύτητα ( όπως και στην σχετική παρατήρηση του σχολικού βιβλίου ) Και σωστά περιορίζεις στην αατ χωρίς αρχική φάση διότι αν ξεφύγουμε από εκεί τότε ... ρίξε μια διαγώνια ματιά στο αρχείο Ελάσσονες και Μείζονες Αρμονικές , να δεις τα μπερδέματα ... Εκτός αν μιλάμε για χρόνο Δt= kΤ/2 οπότε πάντα Δs=2kA και άρα η μέση αριθμητική ταχύτητα είναι πάντα 4Α/Τ ... Πολύ σωστά περιορίζεις ... και κάνεις όλα προσιτά και για μαθητές.
Σχόλιο από τον/την Δημήτρης Φ. στις 22 Οκτώβριος 2016 στις 22:25
Διαγραφή σχολίου
Εκτός και αν θέλεις να παγιδέψεις τους μαθητές και η απάντηση είναι σε όλες τις περιπτώσεις 4Α/Τ....!!
Σχόλιο από τον/την Κορφιάτης Ευάγγελος στις 22 Οκτώβριος 2016 στις 22:45
Διαγραφή σχολίου
Καλησπέρα και από εμένα. Θα συμφωνήσω απόλυτα με τον έτερο Βαγγέλη, δίνοντας έμφαση στα άκρα του εκάστοτε διαστήματος. Η μέση ταχύτητα σε χρονικό διάστημα 2h είναι διαφορετική από 0 έως 2h και διαφορετική από 3h έως 5h. Έσω την αίσθηση ότι η μέση διανυσματική ταχύτητα είναι ένα μέγεθος άχρηστο. Δεν έχω δει άλλο βιβλίο εκτός από το σχολικό που να αναφέρεται στην μέση διανυσματική ταχύτητα. Πάντως, η αναφορά στην μέση ταχύτητα χωρίς άλλον επιθετικό προσδιορισμό, υπονοεί την μέση αριθμητική ταχύτητα.
Σχόλιο από τον/την Κυριακόπουλος Γιάννης στις 22 Οκτώβριος 2016 στις 23:42
Διαγραφή σχολίου
Αντιγράφω από το βιβλίο των Δαπόντε, Κασσέτα, Μουρίκη, Σκιαθίτη: "Η μέση ταχύτητά του ορίζεται ως το πηλίκο της μετατόπισης προς την αντίστοιχη χρονική διάρκεια. Είναι μέγεθος διανυσματικό και έχει την κατεύθυνση της μετατόπισης. Μπορούμε να γράψουμε Το βιβλίο δεν αναφέρει αριθμητική ή διανυσματική ταχύτητα. Σε προηγούμενη σελίδα το βιβλίο γράφει στην παράγραφο "μέση ταχύτητα": .... η ταχύτητα οφείλει να είναι μέγεθος "διανυσματικό". Βάζει και μπλολντ όπου έβαλα και εγώ. Ήταν τόσο προσεκτικοί που είμαι σχεδόν σίγουρος ότι η απαλοιφή της αριθμητικής μέσης ταχύτητας δεν ήταν τυχαία. Ενστερνίζομαι την στάση αυτήν διότι το μέγεθος speed μάλλον δεν ενδιαφέρει την Φυσική όσο το velocity. Η στιγμιαία ταχύτητα είναι όριο της μέσης ταχύτητας. Η αριθμητική τι ρόλο παίζει. Δεν εννοώ στην καθημερινότητα αλλά στην Φυσική.
Σχόλιο από τον/την Εμμανουήλ Λαμπράκης στις 22 Οκτώβριος 2016 στις 23:42
Διαγραφή σχολίου
Καλησπέρα Ενδιαφέρον θέμα Μιχαήλ. Θα συμφωνήσω με το Βαγγέλη Κουντούρη στο ότι για τον προσδιορισμό της μέσης ταχύτητας θα πρέπει να καθορίζεται το χρονικό διάστημα. Τώρα όσον αφορά στον ορισμό της μέσης ταχύτητας θεωρώ ότι όπως όταν λέμε ταχύτητα αναφερόμαστε σε διανυσματικό μέγεθος το ίδιο θα πρέπει να συμβαίνει και όταν λέμε μέση ταχύτητα, διαφορετικά θα πρέπει να λέμε μέσο μέτρο ταχύτητας.
Σχόλιο από τον/την Κυριακόπουλος Γιάννης στις 23 Οκτώβριος 2016 στις 0:19
Διαγραφή σχολίου
Ο Διονύσης είχε ξεκινήσει ενδιαφέροντα διάλογο: http://blogs.sch.gr/dmargaris/archives/459 Ούτε στους Haliday-Resnick βρίσκω κάτι περί αριθμητικής ταχύτητας. Γράφουν ότι η μέση ταχύτητα είναι το διανυσματικό μέγεθος: Παραθέτουν το σχήμα:
Σχόλιο από τον/την Γιάννης Μπατσαούρας στις 23 Οκτώβριος 2016 στις 8:48
Διαγραφή σχολίου
Μια ερώτηση..Τα πηλίκα Μετατόπιση/χρονική διάρκεια (Δr/Δt) Διάστημα/Χρονική διάρκεια(Δs/Δt) Πως ονομάζονται 1. όταν Δt παίρνει καθορισμένη τιμή 2. όταν το Δt τείνει προς το μηδέν
Σχόλιο από τον/την Μιχαήλ Μιχαήλ στις 23 Οκτώβριος 2016 στις 9:00
Διαγραφή σχολίου
Βαγγέλη, καλημέρα. Θεωρώ πως όταν λέμε μέση ταχύτητα αναφερόμαστε στη μέση διανυσματική ταχύτητα. Για τη μέση αριθμητική ταχύτητα θα πρέπει να μας δίνεται διευκρίνηση (το αντίθετο από το βιβλίο της Α΄Λυκείου). Βαγγέλη, σωστά για το χρονικό διάστημα. Αν και αυτό που ήθελα να τονίσω είναι πως για την ίδια μετατόπιση στην αατ έχουμε κάθε φορά διαφορετική μέση ταχύτητα (μετρώντας από t0=0). Δημήτρη Γκενέ σε ευχαριστώ. Δημήτρη Φ. δεν είναι αυτός ο σκοπός μου. Γιάννη (Κυρ.) Μανώλη (Λαμπ.) σας ευχαριστώ για το σχολιασμό.
Σχόλιο από τον/την Εμμανουήλ Λαμπράκης στις 23 Οκτώβριος 2016 στις 9:13
Διαγραφή σχολίου
Καλημέρα Γιάννη Μ. Μέση ταχύτητα, μέσο μέτρο ταχύτητας , στιγμιαία ταχύτητα, μέτρο στιγμιαίας ταχύτητας.
Σχόλιο από τον/την Κυριακόπουλος Γιάννης στις 23 Οκτώβριος 2016 στις 10:44
Διαγραφή σχολίου
Καλημέρα παιδιά. Άλλο το μέσο μέτρο ταχύτητας και άλλο το μέτρο της μέσης ταχύτητας. Το μέσο μέτρο ταχύτητας είναι αυτό που συμπίπτει με την "αριθμητική" μέση ταχύτητα.
Σχόλιο από τον/την Εμμανουήλ Λαμπράκης στις 23 Οκτώβριος 2016 στις 11:18
Διαγραφή σχολίου
Ακριβώς Γιάννη.
Σχόλιο από τον/την Διονύσης Μάργαρης στις 23 Οκτώβριος 2016 στις 11:21
Διαγραφή σχολίου
Καλημέρα συνάδελφοι. Στη διεύθυνση που ανέφερε παραπάνω ο Γιάννης, έχω πάρει θέση πάνω στο θέμα. Θα πρέπει να ξεκαθαρίσουμε αν μιλάμε για την καθημερινή ζωή ή για φυσική. Αν μιλάμε για φυσική η velocity είτε μέση είτε στιγμιαία είναι διάνυσμα. Εξάλλου πώς μπορείς να ξεκινάς από μονόμετρο μέγεθος και πηγαίνοντας στο όριο να εμφανίζεται διάνυσμα; Η αριθμητική ταχύτητα της καθημερινής ζωής, το speed των άλλων, είναι πολύ χρήσιμο μέγεθος αν μιλάμε για αυτοκίνητα που πάνε από Αθήνα-Θεσ/νίκη...
Σχόλιο από τον/την Κυριακόπουλος Γιάννης στις 23 Οκτώβριος 2016 στις 11:38
Διαγραφή σχολίου
Καλημέρα Διονύση. Ήτανε 1997 στο ΠΕΚ Περιστερίου. Ο Ανδρέας δάσκαλος και εμείς από κάτω. -Γιατί δεν χρησιμοποιούμε την "βραδύτητα" Δt/Δx ; Η απάντηση που συζητήθηκε εκτενώς ήταν: -Διότι δεν είναι διάνυσμα!
Σχόλιο από τον/την Κορφιάτης Ευάγγελος στις 23 Οκτώβριος 2016 στις 12:03
Διαγραφή σχολίου
Καλημέρα και από εμένα. Χρησιμοποιώντας τον μετασχηματισμό που λέγεται αντιστροφή το μέγεθος βραδύτητα καθίσταται διάνυσμα.
Σχόλιο από τον/την Κυριακόπουλος Γιάννης στις 23 Οκτώβριος 2016 στις 12:26
Διαγραφή σχολίου
Καθίσταται Βαγγέλη αλλά.... Όταν βλέπω έναν παρατηρητή να κινείται και αυτός βλέπει μπάλα κινούμενη τότε προσθέτω τα διανύσματα μετατοπίσεων, μέσων ταχυτήτων, στιγμιαίων ταχυτήτων,επιταχύνσεων. Προσθέτω "βραδύτητες"; Δηλαδή ορίζουμε ένα διάνυσμα αλλά η Φυσική πως θα το χρησιμοποιήσει; Πιο απλά στη ευθύγραμμη κίνηση: Τι θα γίνει στην δισδιάστατη περίπτωση;
Σχόλιο από τον/την Κυριακόπουλος Γιάννης στις 23 Οκτώβριος 2016 στις 12:36
Διαγραφή σχολίου
Οι μέσες αριθμητικές ταχύτητες είναι παντελώς άχρηστες σε προβλήματα σχετικών κινήσεων. Οι μέσες ταχύτητες όμως προστίθενται.
209. Στάσιμο κύμα. x=0 κοιλία ή δεσμός;

Κατά μήκος γραμμικού ομογενούς ελαστικού μέσου, το οποίο εκτείνεται κατά τη διεύθυνση x΄x, δημιουργείται στάσιμο κύμα. Oι εξισώσεις των δυο τρεχόντων κυμάτων  που με τη συμβολή τους δημιούργησαν το στάσιμο κύμα είναι,

y1=A ημ2π(t/T-x/λ+φ0/2π)   και y2= A ημ2π(t/T+x/λ). Ποια είναι τότε η μικρότερη κατά απόλυτη τιμή αρχική φάση φ0, ώστε μετά τη συμβολή των δυο κυμάτων κατά μήκος της χορδής το σημείο x=0 να έχει πλάτος:

i)  2Α,  δηλαδή να είναι κοιλία του στάσιμου κύματος

ii)  0,  δηλαδή να είναι δεσμός του στάσιμου κύματος

iii)  Α;

 

Συνοπτική λύση:

Σχόλια
Σχόλιο από τον/την Κωστας Ψυλακος στις 30 Οκτώβριος 2016 στις 18:10
Διαγραφή σχολίου
Καλησπερα ! Μιχαλη με την δυσκολια στην προσβαση λογω των γνωστων προβληματων του δικτυου δεν μου δοθηκε η δυνατοτητα να γραψω κατι που ηθελα για την αναρτηση σου . Ετσι αναρτηθηκαν και αλλες στο ενδιαμεσο και καπου περασε... Ας ειναι ομως ! Παρατηρησα λοιπον οτι ο σκοπο σου ηταν να μεθοδεύσεις το πως θα διαχειριστουμε την εξισωση του πλατους για την θεση χ=0 αναλογα με τον αν θελουμε να εχουμε ΚΟΙΛΙΑ Ή ΔΕΣΜΟ Ή καποιο αλλο πλατος, εχοντας σαν βαση βεβαια τις γενικες εξισωσεις που εχεις δωσει. Ολα αυτα που εχουν ενα τετοιο μαθηματικο περιεχομενο δυστυχως δυσκολευουν παντα τους μαθητες ακομα και τους πιο καλους . Οποτε θεωρω χρησιμη την αναλυση που εκανες ! Στην ΠΑΡΑΤΗΡΗΣΗ που εχεις στο τελος πριν το (1) αυτο το οποιο γραφεις για το τυχαιο σημειο μπορει να μπλεξει λιγο στην μελετη διοτι αν αυτο απο ΚΟΙΛΙΑ γινει ΔΕΣΜΟΣ η εξισωση η γενικη θα ειναι η ιδια δηλαδη με ΚΟΙΛΙΑ στο χ=0 . Οποτε επειδη στην συνεχεια ο στοχος σου ειναι διαφορετικος καλο ειναι , νομιζω , να το αφαιρέσεις . Να προσθεσω τελος οτι μια πιο γενικη εξισωση του στασιμου κυματος μπορει να εχει την πιο κατω μορφη: y=2Aσυν[(2πχ/λ)+θ] ημ(ωt+φ) οπου θ,φ σταθερες γωνιες οι οποιες θα καθοριστουν απο τις συνθηκες ταλαντωσης του σχοινιου στις ακρες του.
Σχόλιο από τον/την Μιχαήλ Μιχαήλ στις 30 Οκτώβριος 2016 στις 19:59
Διαγραφή σχολίου
Κώστα σε ευχαριστώ για το σχόλιο. Οι παρατηρήσεις σου είναι πάντα χρήσιμες. Μπορείς αν θέλεις να αναρτήσεις και τη δική σου εκδοχή όπως άλλωστε το είχαμε συζητήσει και τηλεφωνικά! Σε ευχαριστώ
Σχόλιο από τον/την Κωστας Ψυλακος στις 30 Οκτώβριος 2016 στις 21:42
Διαγραφή σχολίου
Επεσαν πολλα Μιχαηλ μαζεμενα και "χαθηκα" λιγο ! Θα το ανεβασω. Να ενημερωσω οτι εχω διαχειριστει καπως διαφορετικα τις πραξεις και προχωραει πιο βατα η αναλυση.Επισης εξεταζω στην παρατηρηση τι συμβαινει με το τυχαιο σημειο και απο ΚΟΙΛΙΑ γινεται ΔΕΣΜΟΣ ποια η σχεση του λ με το λ' .Και καταληγω στο να συνοψισω τα συμπερασματα . Ε Δ Ω
210. Μέγιστη ορμή

Σώμα μάζας m1 που κινείται με ταχύτητα υ1  συγκρούεται μετωπικά (κεντρικά) και ελαστικά με αρχικά ακίνητο σώμα m2.

Για ποια αναλογία μαζών, και για τη δεδομένη αρχική ορμή, η ορμή της ακίνητης μάζας m2 μετά την κρούση γίνεται μέγιστη;

 

Συνοπτική λύση:

ή στο blog εδώ

Σχόλια
Σχόλιο από τον/την Γιάννης Μπατσαούρας στις 1 Νοέμβριος 2016 στις 22:40
Διαγραφή σχολίου
Συγχαρητήρια Μιχαήλ ...μη αναμενόμενο αποτέλεσμα ..πολύ καλή η ανάλυση σου και η παρατήρηση
Σχόλιο από τον/την Γκενές Δημήτρης στις 1 Νοέμβριος 2016 στις 22:55
Διαγραφή σχολίου
Καλησπέρα Μιχαήλ. Δυσκολεύομαι να καταλάβω την διατύπωση του προβλήματος και διαφωνώ με την διερευνησή σου. Εξηγούμαι Πρώτον ρωτάς στην εκφώνηση αλλά δεν απαντάς για ποιον λόγο m2/m1 έχουμε μέγιστη p2 ; Είναι αλήθεια ότι για την ακίνητη μάζα υπάρχει μέγιστη ορμή που μπορεί να αποκτήσει ( p2max=-Δp1max =-2p1max ) αλλά σε αυτήν τείνει ασυμπτωτικά όταν ο λόγος m2/m1 τείνει στο άπειρο . Και αυτό προκύπτει από δυο προκείμενες α) ελαστική κρούση άρα Κ'=Κ και β)για μηδενική Κινητική Ενέργεια της m2 (ως άπειρης μάζας ) ( Αυτά συνυπάρχουν ως παραδοχές στον τύπο εύρεσης της ορμής που χρησιμοποίησες ... και άρα χρησιμοποίησες παραδοχή που αντιφάσκει στην αναζήτηση μέγιστης ορμής p2 ) Δηλαδή αν απαντάς εμμέσως ότι ο λόγος m2/m1 πρέπει να τείνει στο άπειρο τότε είναι λάθος διότι αυτή οδηγεί σε μηδενική p2 Ίσως κάνω λάθος αλλά ας το συζητήσουμε και θα καταλάβω πιθανόν το λάθος μου.
Σχόλιο από τον/την Κορκίζογλου Πρόδρομος στις 1 Νοέμβριος 2016 στις 22:57
Διαγραφή σχολίου
Καλησπέρα Μιχάλη. Ωραίο θέμα βρήκες να αναδείξεις!! Το είχα πρωτοακούσει πριν από 11-12 χρόνια από τον Χρήστο Τρικαλινό στην Ε.Ε.Φ. ,όταν συγκεντρωνόμασταν για τα θέματα του διαγωνισμού Φυσικής. Μάλιστα το είχε θέσει ως εξής: Σε μια κρούση μιας μπάλας με έναν κατακόρυφο τοίχο, κάθετα προς αυτόν, αν ισχύει η Α.Δ.Ο στο σύστημα μπάλα-Γη. Είναι το ίδιο περίπου που έκανες κι εσύ. Όταν η ακίνητη μάζα είναι πολύ μεγαλύτερη από αυτή που θα προσκρούσει πάνω της, τότε η μεγάλη παίρνει τη μέγιστη δυνατή ορμή. Μπράβο!!
Σχόλιο από τον/την Γκενές Δημήτρης στις 1 Νοέμβριος 2016 στις 23:10
Διαγραφή σχολίου
Καλησπέρα Πρόδρομε Θέλεις να πεις ότι ο Τρικαλινός είπε ότι η μπάλα ανακρούει με την αντίθετη ορμή ( δηλαδή με την ίδια Κιν. Ενέρεγεια) και μετά με τα μαγικά μας προέκυψε εν ονόματι της διατήρησης της ελαστικότητας και περίσσευμα ενέργειας αφού η γη έχει μέγιστη ορμή ή μήπως μέγιστη ορμή δεν σημαίνει και λιγάκι ενέργεια ... ; Η ΑΔΕ είναι αρχή ... Και όταν χρησιμοποιούμε τύπους τους οποίους για να βρούμε διαιρέσαμε δια m2 δεν μπορούμε μετά να λέμε ότι το m2 είναι άπειρο ( για να διαιρέσουμε υποθέσαμε ότι δεν είναι )
Σχόλιο από τον/την Διονύσης Μάργαρης στις 1 Νοέμβριος 2016 στις 23:38
Διαγραφή σχολίου
Καλησπέρα παιδιά. Προτείνω μια ματιά στην ανάρτηση:

Μερικές «αντιφάσεις» στην ελαστική κρούση.

Σχόλιο από τον/την Γκενές Δημήτρης στις 2 Νοέμβριος 2016 στις 1:14
Διαγραφή σχολίου
Αν και δεν καταλαβαίνω την διερεύνήση αυτή του Μιχαήλ Τελικά στο συμπέρασμα του βλέπω να έχει δίκιο με δεδομένη ορμή p1 για να αλλάξει ο λόγος των μαζών m2/m1 πρέπει να αλλάζει η m2 και για όλο και μεγαλύτερες m2 , η p2 αυξάνεται ασυμπτωτικά στην 2p1 ενώ η κινητική K2' τείνει στο μηδέν Δείτε excelx
Σχόλιο από τον/την Βαγγέλης Κουντούρης στις 2 Νοέμβριος 2016 στις 1:30
Διαγραφή σχολίου
Ακριβώς όπως τα γράφει ο Διονύσης! (πάντως και αλλού έχουμε ασχοληθεί με το ίδιο θέμα, διότι θυμάμαι μια "ατάκα" μου: "τα όρια κάνουν άγαρμπα αστεία"...) τελικά η ning ηρέμησε;
Σχόλιο από τον/την Κορκίζογλου Πρόδρομος στις 2 Νοέμβριος 2016 στις 8:42
Διαγραφή σχολίου
Καλημέρα σε όλους. Μήτσο το πρόβλημα που έθεσα έχει ως εξής: όταν ρίχνεις μια μπάλα προς ένα τοίχο, στο σύστημα εσύ, η μπάλα και η Γη, που είναι μονωμένο, η ολική ορμή διατηρείται.Δίνοντας στην μπάλα μια ορμή mu, η Γη κι εσύ αποκτάτε αντίθετη και ισόποση ορμή, κι όταν η μπάλα ανακλασθεί στον τοίχο, πάλι το ισοζύγιο ορμής στο σύστημα εσύ, η μπάλα και Γη, μένει αμετάβλητο. Τι μεμπτό είχε ο λόγος μου; Απλώς η ορμή που αποκτά η Γη από το ρίξιμο της μπάλας είναι απειροελάχιστη, αφου το σύστημα που προανέφερα είναι ή θεωρείται μονωμένο.
Σχόλιο από τον/την Μιχαήλ Μιχαήλ στις 2 Νοέμβριος 2016 στις 9:18
Διαγραφή σχολίου
Καλημέρα σε όλους! Γιάννη, Πρόδρομε σας ευχαριστώ για το σχόλιο και για τα καλά σας λόγια. Δημήτρη, στο παράδειγμα που δίνω χρησιμοποιώ , m1=1Kg και m2=100Kg τη μια φορά και την επόμενη , m1=1Kg και m2=4,05Kg. Απέφυγα να χρησιμοποιήσω τις λέξεις άπειρο και μηδέν. Η απάντηση που δίνω είναι "για δεδομένη αρχική ορμή p1=m1υ1 όσο μεγαλύτερος είναι ο λόγος των μαζών τόσο μεγαλύτερη είναι η ορμή p2΄ της μάζας m2 αμέσως μετά την κρούση." Το όριο και στις δυο περιπτώσεις είναι τα 202 Κg∙m/s. Διονύση, Βαγγέλη σας ευχαριστώ για το σχόλιο. Διονύση δεν την είχα υπόψιν μου την πολύ ολοκληρωμένη πράγματι ανάρτησή σου.
Σχόλιο από τον/την Βαγγέλης Κουντούρης στις 2 Νοέμβριος 2016 στις 10:22
Διαγραφή σχολίου
καλημέρα σε όλους Μήτσο και Πρόδρομε κάπου έχω γράψει εδώ (αλλά δεν μπορώ να το ξαναβρώ) ότι: αν δεχθούμε ότι σε τέλεια ελαστική κρούση σφαίρας με τοίχο (Γη) η σφαίρα επιστρέφει με ταχύτητα ίσου μέτρου, ο τοίχος δεν αποκτά απειροελάχιστη ορμή, αλλά ορμή ομόρροπη της αρχικής της σφαίρας και διπλάσιου μέτρου (απειροελάχιστη είναι η ταχύτητά του, όχι η ορμή του)

Ετικέτες: κρούσεις, τάξη-γ

Σχόλιο από τον/την Γκενές Δημήτρης στις 2 Νοέμβριος 2016 στις 11:18
Διαγραφή σχολίου
Εντάξει λάθος δικό μου. Απλά με παραξένεψε πολύ το απτέλεσμα και αντέδρασα Έχεις δίκιο Μιχαήλ σχεδόν σε όλα ( μόνο η ορμή της m1= δεν είναι ακριβώς p'1=-p1 ) Η ορμή της m2 προσεγγίζει ασυμτωτικά την μέγιστη τιμή της που είναι 2p1 όταν η m2 τείνει στο άπειρο και ταυτόχρονα η κινητική ενέργεια Κ'2 τείνει στο μηδέν. Μιχαήλ δεν το είχα συνειδητοποιήσει . Ευχαριστώ έμαθα κάτι.

Σχόλιο

214. Νερό και αέρας

Μέσα στο δοχείο του σχήματος εμβαδού εγκάρσιας διατομής Α=164cm2 υπάρχει νερό πυκνότητας ρ=103Κg/m3.

Τότε:

α) Να υπολογιστεί η πίεση στο σημείο Γ.

 

β) Να υπολογιστεί η δύναμη F που ασκείται στη βάση ΔΕ του δοχείου.

 

γ) Να υπολογιστούν τα mol του αέρα μέσα στο δοχείο (ιδανικό αέριο).

 

Δίνονται pατμ=105N/m2, h1=h3=0,5m, h1=2,5m, θ=270C, R=0,082 L∙atm/mol∙K.

 

 

Συνοπτική λύση:

215. Η ενέργεια στη σύνθετη ταλάντωση Σώμα εκτελεί αρμονική ταλάντωση και η εξίσωση της απομάκρυνσής του σε συνάρτηση με το χρόνο είναι x=Αημ(ωt+θ). Η προηγούμενη εξίσωση θεωρούμε ότι προκύπτει από την επαλληλία των εξισώσεων x1=Α1ημ(ωt) και x2=Α2ημ(ωt+φ) που αντιστοιχούν στις εξισώσεις των απομακρύνσεων δύο αρμονικών ταλαντώσεων της ίδιας συχνότητας, ίδιας διεύθυνσης και ίδιας θέσης ισορροπίας με 0≤θ<φ<2π. Αν Ε, Ε1 και Ε2 είναι οι ενέργειες των ταλαντώσεων, x, x1 και x2 αντίστοιχα, Κ, Κ1 και Κ2 είναι οι αντίστοιχες κινητικές ενέργειες των ταλαντώσεων και U, U1 και U2 είναι οι αντίστοιχες δυναμικές τους ενέργειες την ίδια χρονική στιγμή t, τότε να δείξετε πως ισχύει: Ε=Ε1+Ε2+2[(K1+K2)^0,5+(U1+U2)^0,5]=Ε1+Ε2+2(E1E2)^0,5 συνφ Συνοπτική λύση: Σχόλια